Você está na página 1de 300

CONTENTS

IN T R O D U C T IO N 19

S E C T IO N O N E : L IS T E N IN G C O M P R E H E N S IO N
D IA G N O S T IC P R E -T E S T 35
L IS T E N I N G C O M P R E H E N S IO N 42
T h e L is t e n in g P art A Q u e s tio n s 43
S tr a te g ie s
SKILL 1: Focus on the second line 44
SK ILL 2: C hoose answers with synonym s 46
SK ILL 3: Avoid similar sounds 48
T O E F L EXERCISE (Skills 1-3) 50
W h o , W h at, W h e r e
SK ILL 4: Draw conclusions about who,what, where 51
SK ILL 5: Listen for who and what in passives 54
SK ILL 6: Listen for who and what with m ultiple nou ns 56
T O E FL EXERCISE (Skills 4-6) 58
T O E FL REVIEW EXERCISE (Skills 1-6) 59
N e g a tiv e s
SKILL 7: Listen for negative expressions 60
SKILL 8: Listen for dou ble negative expressions 62
SK ILL 9: Listen for alm ost negative expressions 64
SKILL 10: Listen for negatives with com paratives 66
T O E FL EXERCISE (Skills 7-10) 68
T O EFL REVIEW EXERCISE (Skills 1-10) 69
F u n c tio n s
SKILL 11: Listen for expressions o f agreem en t 70
SK ILL 12: Listen for expressions o f uncertainty and su ggestion 72
SK ILL 13: Listen for em phatic expressions o f surprise 75
T O E FL EXERCISE (Skills 11-13) 77
T O E FL REVIEW EXERCISE (Skills 1-13) 78
C o n tra r y M e a n in g s
SK ILL 14: Listen for wishes 7g
SKILL 15: Listen for untrue conditions gj
T O EFL EXERCISE (Skills 14-15) 83
T O E FL REVIEW EXERCISE (Skills 1-15) H4
I d io m a tic L a n g u a g e
SKILL 16: Listen for two- and three-part verbs g5
SK ILL 17: Listen for idiom s gy
T O E F L EXERCISE (Skills 16-17) 89
T O E FL REVIEW EXERCISE (Skills 1-17) 89
12 CO NTEN TS

T h e L is te n in g P art B Q u e s tio n s 91

B e fo r e L is te n in g
SK ILL 18: Anticipate the topics 93
SKILL 19: Anticipate the questions 95

W h ile L is te n in g
SKILL 20: D eterm ine the topic 97
SKILL 21: Draw conclusions about
who, what, when, where 98
SKILL 22: Listen for answers in order 99
T O EFL REVIEW EXERCISE (Skills 18-22) 101

T h e L is t e n in g P art C Q u e s tio n s 102

B e fo r e L is te n in g
104
SKILL 23: Anticipate the topics
106
SKILL 24: Anticipate the questions

W h ile L is te n in g
108
SKILL 25: D eterm ine the topic
SK ILL 26: Draw conclusions about
109
who, what, when, where
110
SK ILL 27: Listen for answers in order
112
T O E F L REVIEW EXERCISE (Skills 23-27)
113
T O E F L P O S T -T E S T

S E C T IO N T W O : S T R U C T U R E A N D W R IT T E N E X P R E S S IO N
121
D IA G N O S T IC P R E -T E S T
128
S T R U C T U R E A N D W R IT T E N E X P R E S S IO N
129
T h e S tr u c tu r e Q u e s tio n s
S e n te n c e s w ith O n e C la u se
130
SK ILL 1: Be sure the sentence has a subject and a verb
131
SK ILL 2: Be careful o f objects o f prepositions
133
SK ILL 3: Be careful o f appositives
134
SKILL 4: Be careful o f present participles
136
SKILL 5: Be careful o f past participles
138
EXERCISE (Skills 1-5)
138
T O E FL EXERCISE (Skills 1-5)

S e n te n c e s w ith M u ltip le C la u s e s
139
SKILL 6: U se coordinate connectors correctly
141
SK ILL 7: U se adverb time and cause connectors correctly
142
SKILL 8: U se other adverb connectors correctly
144
EXERCISE (Skills 6-8)
144
T O E F L EXERCISE (Skills 6-8)
145
T O E F L REVIEW EXERCISE (Skills 1-8)
CO N TEN TS 13

M o re S e n te n c e s w ith M u ltip le C la u s e s
SK ILL 9: U se n ou n clause connectors correctly 146
SK ILL 10 : U se n ou n clause connector/subjects correctly 148
SK ILL 11 : U se adjective clause connectors correctly 149
SK ILL 12 : U se adjective clause connector/subjects correctly 151
EXERCISE (Skills 9-12) 153
T O E F L EXERCISE (Skills 9-12) 153
T O E F L REVIEW EXERCISE (Skills 1-12) 154
S e n te n c e s w ith R e d u c e d C la u s e s
SK ILL 13: U se red uced adjective clauses correctly 155
SK ILL 14: U se reduced adverb clauses correctly 158
EXERCISE (Skills 13-14) 160
T O E FL EXERCISE (Skills 13-14) 160
T O E F L REVIEW EXERCISE (Skills 1-14) 161
S e n te n c e s w ith I n v e r te d S u b je cts a n d V erb s
SK ILL 15 Invert the subject and verb with question words 163
SK ILL 16 Invert the subject and verb with place expressions 164
SK ILL 17 Invert the subject and verb with negatives 166
SK ILL 18 Invert the subject and verb with conditionals 168
SK ILL 19 Invert the subject and verb with com parisons 170
EXERCISE (Skills 15-19) 172
T O E F L EXERCISE (Skills 15-19) 172
T O E FL REVIEW EXERCISE (Skills 1-19) 173
T h e W r itte n E x p r e s s io n Q u e s tio n s 175
P r o b le m s w ith S u b je ct/V e rb A g r e e m e n t
SK ILL 20 Make verbs agree after prepositional phrases 176
SK ILL 21 M ake verbs agree after expressions o f quantity 177
SK ILL 22 Make inverted verbs agree 178
SKILL 23 M ake verbs agree after certain words 180
EXERCISE (Skills 20-23) 181
T O E FL EXERCISE (Skills 20-23) 181
T O E F L REVIEW EXERCISE (Skills 1-23) 182
P r o b le m s w ith P a r a lle l S tr u c tu r e
SK ILL 24 U se parallel structure with coordinate conjunctions 184
SKILL 25: U se parallel structure with paired conjunctions 185
SK ILL 26: U se parallel structure with com parisons 186
EXERCISE (Skills 24-26) 188
T O E F L EXERCISE (Skills 24-26) 188
T O E FL REVIEW EXERCISE (Skills 1-26) 189
P r o b le m s w ith C o m p a r a tiv e s a n d S u p e r la tiv e s
SK ILL 27: Form com paratives and superlatives correctly 190
SK ILL 28: U se com paratives and superlatives correcdy 191
SK ILL 29: U se the irregular -er, -er structure correcdy 192
EXERCISE (Skills 27-29) 194
T O E F L EXERCISE (Skills 27-29) 194
T O E F L REVIEW EXERCISE (Skills 1-29) 195
14 CO NTEN TS

P r o b le m s w ith th e F o rm o f th e V erb
SKILL 30: After have, use the past participle 196
SKILL 31: After be, use the present participle or the
past participle '97
SKILL 32: After will, would, or other m odals, use
the base form o f the verb 198
EXERCISE (Skills 30-32) >99
T O EFL EXERCISE (Skills 30-32) 199
T O E FL REVIEW EXERCISE (Skills 1-32) 200

P r o b le m s w ith th e U s e o f th e V erb
SKILL 33: Know w hen to use the past with the present 201
SKILL 34: Use have and had correctly 202
SKILL 35: U se the correct tense with tim e expressions 204
SKILL 36: U se the correct tense with will and would 205
EXERCISE (Skills 33-36) 207
TO EFL EXERCISE (Skills 33-36) 207
T O EFL REVIEW EXERCISE (Skills 1-36) 208

P r o b le m s w ith P a ssiv e V erb s


SKILL 37: U se the correct form o f the passive 2 10
SKILL 38: Recognize active and'passive m eanings 211
EXERCISE (Skills 37-38) 212
TO E FL EXERCISE (Skills 37-38) 2 >2
TO E FL REVIEW EXERCISE (Skills 1-38) 213

P r o b le m s w ith N ouns
SKILL 39: U s e the correct singular or plural noun 214-
SKILL 40: D istinguish countable and uncountable nouns 2 15
SKILL 41: R ecognize irregular plurals o f nouns 217
SKILL 42: Distinguish the person from the thing 218
EXERCISE (Skills 39-42) 2 19
T O E FL EXERCISE (Skills 39-42) 220
T O E FL REVIEW EXERCISE (Skills 1-42) 221

P r o b le m s w ith P r o n o u n s
SKILL 43: Distinguish subject and object pronouns 222
SKILL 44: D istinguish possessive adjectives and pronou ns 223
SKILL 45: Check pronou n reference for agreem ent 225
EXERCISE (Skills 43-45) 226
TO E FL EXERCISE (Skills 43-45) 226
TO E FL REVIEW EXERCISE (Skills 1-45) 227

P r o b le m s w ith A d je c tiv e s a n d A d v erb s


SKILL 46: U se basic adjectives and adverbs correctly 228
SKILL 47: U se adjectives after linking verbs 230
SKILL 48: Position adjectives and adverbs correctly 232
EXERCISE (Skills 46-48) 233
TO EFL EXERCISE (Skills 46-48) 234
TO EFL REVIEW EXERCISE (Skills 1-48) 235
CO NTEN TS 15

M o re P r o b le m s w ith A d je c tiv e s
SK ILL 49: R ecognize -ly adjectives 236
SK ILL 50 U se predicate adjectives correctly 237
SK ILL 51 U se -ed and -ing adjectives correctly 238
EXERCISE (Skills 49-51) 239
T O E FL EXERCISE (Skills 49-51) 240
T O E FL REVIEW EXERCISE (Skills 1-51) 241
P r o b le m s w ith A r tic le s
SK ILL 52:
Use articles with singular nouns 242
SKILL 53D istinguish a and an 243
SKILL 54M ake articles agree with nouns 244
SKILL 55Distinguish specific and general ideas 245
EXERCISE (Skills 52-55)
246
T O E F L EXERCISE (Skills 52-55) 247
TO EFL REVIEW EXERCISE (Skills 1-55) 248
P r o b le m s w ith P r e p o s itio n s
SK ILL 56: R ecognize incorrect prepositions 249
SK ILL 57: R ecognize when prepositions have been om itted 250
EXERCISE (Skills 56-57)
251
T O E F L EXERCISE (Skills 56-57) 251
T O EFL REVIEW EXERCISE (Skills 1-57)
252
P r o b le m s w ith U s a g e
SKILL 58: Distinguish make and do
253
SKILL 59: Distinguish like, alike, unlike, and dislike 254
SKILL 60: Distinguish other, another, and others
255
EXERCISE (Skills 58-60)
256
T O E FL EXERCISE (Skills 58-60)
257
T O E FL REVIEW EXERCISE (Skills 1-60)
258
T O E F L P O S T -T E S T
259

S E C T IO N T H R E E : R E A D IN G C O M P R E H E N S IO N
D IA G N O S T IC P R E -T E S T
269
R E A D IN G C O M P R E H E N S IO N
281
T h e R e a d in g C o m p r e h e n s io n Q u e s tio n s
282
Q u e s tio n s a b o u t th e I d e a s o f th e P a ssa g e
3 J T .I . 1 Ancm or m n in U n n .!___
Answer m ain idea questions correctly
283
SKILL 2: R ecognize the organization o f ideas
287
T O E FL EXERCISE (Skills 1-2)
290
D ir e c tly A n sw e r e d Q u e s tio n s
SK ILL 3 Answer stated detail questions correctly 291
SK ILL 4 Find u nstated details
296
SK ILL 5: Find pronou n referents
299
T O E FL EXERCISE (Skills 3-5)
301
T O E FL REVIEW EXERCISE (Skills 1-5)
303
16 CO NTEN TS

I n d ir e c tly A n sw e r e d Q u e s tio n s
SKILL 6: Answer im plied detail questions correctly 307
SKILL 7: Answer transition questions correctly 311
314
T O E FL EXERCISE (Skills 6-7)
T O E FL REVIEW EXERCISE (Skills 1-7) 316

V o c a b u la r y Q u e s tio n s
SKILL 8: Find definitions from structural clues 320
SKILL 9: D eterm ine m eanings from word parts 324
SK ILL 10: U se context to determ ine m eanings o f difficult words 326
SK ILL 11: U se context to determ ine m eanings o f sim ple words 328
330
T O E FL EXERCISE (Skills 8-11)
T O E FL REVIEW EXERCISE (Skills 1-11) 333

O v e r a ll R e v ie w Q u e s tio n s
SKILL 12: D eterm ine w here specific inform ation is found 338
SK ILL 13: D eterm ine the tone, purpose, or course 341
346
T O E FL EXERCISE (Skills 12-13)
349
T O E FL REVIEW EXERCISE (Skills 1-13)
355
T O E F L P O S T -T E S T

S E C T I O N F O U R : T e s t o f W r itte n E n g lis h (T W E )
370
T h e Score
370
S a m p le E ssa y s
B e fo r e W r itin g
376
SK ILL 1: D ecod e the topic
379
SK ILL 2: D evelop su pp orting ideas

W h ile W r itin g
382
SKILL 3: W rite the introductory paragraph
385
SKILL 4: Write unified su pp orting paragraphs
388
SKILL 5: Write the conclu ding paragraph
SKILL 6: C onnect the su pp orting paragraphs in the essay 391

A fte r W r itin g
394
SK ILL 7: Edit sentence structure
394
7A Sim ple sentence structure
7B C om p oun d sentence structure 396
7C C om p lex sentence structure 398
401
SK ILL 8: Edit written expression
8A Inversions and agreem en t 401
8B Parallel, com parative, and superlative structures 402
403
8C Verbs
404
8D N ou n s and p ronou ns
8E Adjectives and adverbs 405
406
8F Prepositions and usage
407
P r a c tic e T e s ts
CO N TEN TS 17

CO M PLETE TESTS 409


CO M PLETE TEST ONE 411
CO M PLETE TEST TW O 437
CO M PLETE TEST T H R EE 463
CO M PLETE TEST F O U R .. 489
CO M PLETE TEST FIVE 515

A P P E N D IX E S 541
A P P E N D IX A: Similar Sounds 543
A P P E N D IX B: T w o- and T hree-P art Verbs 544
A P P E N D IX C: Idiom s 548
A P P E N D IX D: Prepositions 552
A P P E N D IX E: W ord Endings 554
A P P E N D IX F: Irregular Verb Forms 560
A P P E N D IX G: Form ation o f the Passive 562
A P P E N D IX H: Irregular Plurals 564
A P P E N D IX I: W ord Parts 565

SCOR ES A N D C H A R TS 579
Scoring Inform ation 581
D iagnostic Charts 583
Progress Chart 591
T O E F L A nswer Sheets 593

R E C O R D IN G S C R IP T '609

A N SW E R KEY 651
m a n

18
SECTION ONE.

LISTENING
COMPREHENSION

33
34
l D l D l D l D l l l n l

DIAGNOSTIC PRE-TEST
S E C T IO N 1
L IS T E N IN G C O M P R E H E N S IO N
T im e a p p r o x im a t e ly 3 5 m in u te s
(in c lu d in g t h e r e a d in g o f t h e d ir e c t io n s fo r e a c h p a r t)

In this section of the test, you will have an opportunity to demonstrate your ability to understand
conversations and talks in English. There are three parts to this section, with special directions for each
part. Answer all the questions on the basis of what is stated or im plied by the speakers you hear. Do
not take notes or write in your test book at any time. Do not turn the pages until you are told to do so.

Part A

Directions: In Part A you will hear short conversations between two people. After each conversation,
you will hear a question about the conversation. The conversations and questions will not be
repeated. After you hear a question, read the Four possible answers in your test book and choose the
best answer. Then, on your answer sheet, find the number of the question and fill in the space that
corresponds to the letter of the answer you have chosen.

Listen to an exam ple. Sam ple Answer

On the recording, you w ill hear:

(m an) That exam was just awful.


(w om an) Oh, it could have been worse.
(narrator) What does the w om an m ean?

In your test book, you w ill read: (A) The exam was really awful.
(B) It was the worst exam she had ever seen.
(C) It couldn't have been more difficult.
(D) It wasn't that hard.

You learn from the conversation that the man thought the exam was very difficult and that the
woman disagreed with the man. The best answer to the question, What does the woman mean?" is
(D), "It wasnt that hard. Therefore, the correct choice is (D).

TOEFL* test directions and form at are reprinted by perm ission


of ETS, the copyright owner. However, all exam ples and test
questions are provided by P earson Education. Inc. LISTENING COMPREHENSION PRE-TEST 35
l D l D l D l D l D l D l D l

1. (A) The coffee is much better this 8. (A) Housing within his budget is hard to
morning. locate.
(B) Hes feeling bitter this morning. (B) Its hard to find his house in New
(C) The coffee isnt very good. York.
(D) He cannot taste the butter. (C) He cant afford to move his house to
New York.
2. (A) The two classes meet in an hour and a (D) Housing in New York is unavailable.
half.
(B) The class meets three hours per week. 9. (A) The boss was working on the reports.
(C) Each half of the class is an hour long. (B) He would have to finish the reports
(D) Two times a week the class meets for before the end of next month.
an hour. (C) He was directed to stay late and finish
som e work.
3. (A) A few minutes ago, the flight (D) He could finish the reports at home.
departed.
(B) The fight will start in a while. 10. (A) The boisterous students made the
(C) They are frightened about the teacher mad.
departure. (B) The teacher angered the students with
(D) The plane is going to take off soon. the exam results.
(C) The students were angry that the
4. (A) He hasnt yet begun his project. teacher was around.
(B) Hes supposed to do his science (D) The angered students complained to
project next week. the teacher.
(C) He needs to start working on
changing the due date. 11. (A) The prices are reasonable.
(D) Hes been working steadily on his (B) The store is too far out of town.
science project. (C) He would like the woman to repeat
what she said.
5. (A) At the post office. (D) He agrees with the woman.
(B) In a florist shop.
(C) In a restaurant. 12. (A) It has rarely rained this much.
(D) In a hospital delivery room. (B) It hardly rained this year.
(C) It is barely raining this year.
6. (A) The professor drowned the cells in a (D) It seldom rains so little.
lab.
(B) The topic was presented in a boring 13. (A) He needs to do a better job writing
way. questions.
(C) The professor divided the lecture into (B) His writing must certainly be
parts. improved.
(D) The biologist tried to sell the results (C) Without the questions, he cannot
o f the experiment. write the answers.
(D) He needs to understand the written
7. (A) She needs to get a driver's license. questions better.
(B) It is impossible to cash a check
without two pieces of identification.
(C) The man should check to see if he
needs credit.
(D) A credit card can be used to get a
drivers license.

GO ON TO THE NEXT PAGE

36 LISTENING COMPREHENSION PRE-TEST


l D l D l D l D l D l D l D l

14. (A) The agent was standing in line with 20. (A) Shed like som e pie.
his passport. (B) Its easy to buy it.
(B) The line to get new passports is very (C) The task the mans working on isnt
long. difficult.
(C) The woman must wait her turn to get (D) Its easier to prepare pie than do what
her passport checked. the man is doing.
(D) He can check her passport instead of
the agent. 21. (A) He reported that the tim e for the
budget meeting had been set.
15. (A) He couldnt finish closing the library (B) He is always late in submitting his
book. accounting figures.
(B) He hadnt finished the library (C) He never manages to budget his time
assignment, but he was close. well.
(C) He was working on the assignment (D) He is never too late in turning in his
when the library closed. reports.
(D) His homework was incomplete
because the library wasnt open. 22. (A) The repairs would require an
extension.
16. (A) He wishes the hard work had had a (B) The car is going to need a lot of
better result. repairs.
(B) He thinks the lawyer hardly prepared. (C) Buying a new car would be quite
(C) He wishes the lawyer had prepared. expensive.
(D) He thinks the lawyer worked for free. (D) The mechanic extended the repair
warranty.
17. (A) The history class begins next week.
(B) He thinks the papers should be turned 23. (A) Betty wrote the letter as directed.
in next week. (B) The directions were given to Betty in
(C) He has already done the paper for a letter.
next week. (C) Betty will follow the instructions later.
(D) The papers are not due next week. (D) Betty worked exactly as instructed.

18. (A) He's not really happy. 24. (A) Walter's had a lack of success with his
(B) The contractors work was business.
satisfactory. (B) Walters failed in business.
(C) He would rather work with the (C) Walter's new company is doing rather
contractor himself. well.
(D) He was already contacted about the (D) Walter hoped to succeed in business.
work.
25. (A) He should put the organ in the closet.
19. (A) The man should try another type of (B) The closet has already been
paper. organized.
(B) The man should locate a typist (C) He needs to rearrange the closet.
tomorrow morning. (D) He wishes the closet were closer.
(C) The man should make a tape in the
morning. 26. (A) She didnt do the work.
(D) The man should complete the paper (B) She gave the assignment her best
without help. effort.
(C) She finished the assignm ent even
though it was difficult.
(D) She gave the man a signal.

LISTENING COMPREHENSION PRE-TEST 37


l D l D l D l D l D l D l D l
27. (A) She said some terrible things. 29. (A) The woman is more than a week late.
(B) She didnt say anything nice. (B) The children would have wrecked the
(C) She didn't have any nice things. house later.
(D) She said really wonderful things. (C) The woman was so late that she was a
wreck.
28. (A) New employees are rarely initiated (D) Hes glad that she was not any later.
into the company.
(B) New workers dont generally 30. (A) He had not gone to the store.
undertake actions on their own. (B) He was still at the market.
(C) New employees are initially rated. (C) He was going to take care of the
(D) Its rare for employees to make new shopping.
suggestions. (D) He always went to the market.

38 LISTENING COMPREHENSION PRE-TEST


l n l D l D l n l l n l D l
Part B

Directions: In this part of the test, you will hear longer conversations. After each conversation, you
will hear several questions. The conversations and questions will not be repeated.
After you hear a question, read the four possible answers in your test book and choose the best
answer. Then, on your answer sheet, find the number of the question and fill in the space that
corresponds to the letter of the answer you have chosen.
Remember, you are not allowed to take notes or write in your test book.

31. (A) She's a senior. 35. (A) In a book.


(B) Shes a junior. (B) From a television program.
(C) Shes a transfer student. (C) During a trip that she took.
(D) Shes a graduate student. (D) From a lecture.

32. (A) How to transfer to a junior college. 36. (A) To communicate with other dolphins.
(B) How to find his way around campus. (B) To recognize objects in the water.
(C) The course requirements for a (C) To learn human language.
literature major. (D) To express fear.
(D) Who won the campus election.
37. (A) Five.
33. (A) Three. (B) Fifteen.
(B) Five. (C) Fifty.
(C) Eight. (D) Five hundred.
(D) Ten.
38. (A) It is limited.
34. (A) American literature. (B) It is greater than human intelligence.
(B) World literature. (C) It is less than previously thought.
(C) Literary analysis. (D) We are beginning to learn how m uch
(D) Surveying. they have.

iu im 4 n u u |
TOEFL* test directions and form at are reprinted by perm ission
of ETS, the copyright owner. However, all exam ples and test
questions are provided by P earson E ducation, Inc. LISTENING COMPREHENSION PRE-TEST 39
l D l D l n l n l D l n l a l
Part C

D irections: In this part of the test, you will hear several talks. After each talk, you will hear some
questions. The talks and questions will not be repeated.

After you hear a question, you will read the four possible answers in your test book and choose the
best answer. Then, on your answer sheet, find the number of the question and fill in the space that
corresponds to the letter of the answer you have chosen.

Here is an exam ple.

On the recording, you w ill hear:

(narrator) Listen to an instructor talk to his class about painting.

(m an) Artist Grant Wood was a guiding force in the school o f painting know n as American
regionalist, a style reflecting the distinctive characteristics o f art from rural areas o f
the United States. Wood began drawing anim als on the family farm at the age o f
three, and when he was thirty-eight one o f his paintings received a remarkable
am ount o f public notice and acclaim. This painting, called American Gothic," is a
starkly simple depiction o f a serious couple staring directly o u t at the viewer.

Now listen to a sample question. Sam ple Answer

(narrator) What style o f painting is known as American regionalist? (A) 9

In your test book, you w ill read: (A) Art from Americas inner cities.
(B) Art from the central region of the U.S.
(C) Art from various urban areas in the U.S.
(D) Art from rural sections of America.

The best answer to the question, "What style of painting is known as American regionalist? is (D),
"Art from rural sections of America." Therefore, the correct choice is (D).

Now listen to another sample question. Sam ple Answer

(narrator) W hat is the name o f Wood's m ost successful painting? (A )

In your test book, you w ill read: (A)' "American Regionalist.


(B) "The Family Farm in Iowa."
(C) "American Gothic.
(D) "A Serious Couple.

The best answer to the question, "What is the name of Wood's most successful painting ? is (C).
American Gothic. Therefore, the correct choice is (C).
Remember, you are n ot allowed to take notes or write in your test book.

TOEFL* test directions and form at arc reprinted by perm ission


o f ETS, the copyright owner. However, all examples and test
40 LISTENING COMPREHENSION PRE-TEST q uestions are provided by Pearson Education, Inc.
l D l D l D l D l D l D l D l
39. (A) To protect its members. 45. (A) To the tennis courts.
(B) To save the natural environment. (B) To the arena.
(C) To honor the memory o f John Muir. (C) To the gymnasium.
(D) To improve San Francisco's natural (D) To the Athletic Department office.
beauty.
46. (A) Go to the Art Center.
40. (A) For less than a year. (B) Sign up for sports classes.
(B) Only for a decade. (C) Visit the exercise room.
(C) For more than a century. (D) Watch a football game.
(D) For at least two centuries.
47. (A) Science.
41. (A) San Francisco. (B) Art.
(B) All fifty states. (C) Literature.
(C) The Sierra Nevadas. (D) Music.
(D) The eastern U.S.
48. (A) They are completely different.
42. (A) All over the world. (B) They are somewhat similar but have
(B) In the entire United States. an essential difference.
(C) Only in California. (C) They are exactly the sam e in all
(D) Only in the Sierra Nevadas. respects.
(D) They are unrelated.
43. (A) Students signing up for athletic
teams. 49. (A) Objective.
(B) Students going on a tour of a (B) Idealistic.
university campus. (C) Philosophical.
(C) Students playing various sports. (D) Environmental.
(D) Students attending a university
dedication ceremony. 50. (A) Heredity.
(B) Environment.
44. (A) Membership on an athletic team. (C) Idealism.
(B) Enrollment in an exercise class. (D) Natural laws.
(C) A valid student identification card.
(D) Permission from a faculty member.

This is the end of the Listening Comprehension Pre-Test.


Turn off the recording.

(st o p) (st o p) (s t o p ) ^ ST Q p j (s t o p ) (s t o p) (s t o p )

W h e n y o u fin ish th e test, y o u m ay d o th e fo llo w in g :


T u rn to th e D ia g n o s tic C h art o n p a g e s 585$- '>84, a n d
c ir cle th e n u m b e r s o f th e q u e s tio n s th at y o u m isse d .
T u rn to S c o r in g I n fo r m a tio n o n p a g e s 5 8 1 - 5 8 2 , a n d
d e te r m in e y o u r T O E F L sc o r e.
T u rn to th e P r o g r e s s C h art o n p a g e 5 9 1 , a n d a d d you r
sc o r e to th e ch art.

LISTENING COMPREHENSION PRE-TEST 4I


LISTENING
COMPREHENSION

T h e first se c tio n o f th e T O E F L test is th e L iste n in g C o m p r e h e n s io n se c tio n . T h is se c tio n


c o n sists o f fifty q u e s tio n s (so m e tests m ay b e lo n g e r ). You w ill listen to r e c o r d e d m aterials
a n d r e s p o n d to q u e s tio n s a b o u t th e m aterial. You m u st lis te n carefully, b e c a u se y o u will
h e a r th e r e c o r d in g p rogram o n e tim e only, a n d th e m aterial o n th e r e c o r d in g is n o t writ
te n in y o u r test b o o k .
T h e r e are th r e e parts in th e L iste n in g C o m p r e h e n s io n se c tio n o f th e T O E F L test:

1. P a rt A c o n sists o f thirty sh o r t co n v e rsa tio n s, e a c h fo llo w e d by a q u e s tio n . You m u st


c h o o s e th e b e st answ er to e a c h q u e s tio n fr o m th e fo u r c h o ic e s in y o u r te st b ook .
2. P a rt B c o n sists o f tw o lo n g c o n v e rsa tio n s, e a c h fo llo w e d by a n u m b e r o f q u estio n s.
Y ou m u st c h o o s e th e b e st answ er to e a c h q u e s tio n from th e fo u r c h o ic e s in your
te st b o o k .
3. P art C c o n sists o f th r e e talks, e a c h fo llo w e d by a n u m b e r o f q u estio n s. You m u st
c h o o s e th e b e st an sw er to e a c h q u e s tio n fro m th e fo u r c h o ic e s in y o u r test b o o k .

GENERAL STRATEGIES

1. Be fam iliar w ith th e d irection s. The directions on every TOEFL test are the same, so it is
not necessary to listen carefully to them each time. You should be completely familiar with
the directions before the day of the test.
2. Listen carefully to th e con versation s and talk s. You should concentrate fully on what
the speakers are saying in the recording program, because you will hear it one time only.
3. Know w h er e th e e a sier and m ore difficult q u estio n s are generally found. Within
each part of the Listening Comprehension section, the questions generally progress from
easy to difficult.
4. N e v er leave any answ ers blank on your an sw er sh e e t. Even if you are unsure of the
correct response, you should answer each question. There is no penalty for guessing.
5. U s e any rem aining tim e to look ahead at th e an sw ers to th e q u estio n s th a t
follow . When you finish with one question, you may have time to look ahead at the answers
to the next question.

42
LISTENING PARTA

T HE L IS T E N IN G PA R TA Q U E S T IO N S

For e a c h o f th e thirty q u e s tio n s in P art A o f th e L iste n in g C o m p r e h e n s io n s e c tio n o f th e


T O E F L test, y o u w ill h e a r a sh o r t c o n v e r s a tio n b e tw e e n two sp ea k er s fo llo w e d by a q u e s
tio n . A fter y o u liste n to th e c o n v e r s a tio n a n d q u e s tio n , y o u m u st c h o o s e th e b e s t an sw er
to th e q u e s tio n from y o u r test b o o k .

Exam ple

On the recording, you hear:


(man) I've always wanted to visit Hawaii w ith you.
(woman) Why not next m onth?
(narrator) What does the w om an mean?

In your test book, you read:


(A) Next month isn't a good time for the trip.
(B) She doesnt want to go to Hawaii.
(C) She suggests taking the trip next month.
(D) Shes curious about why he doesn't want to go.

A n sw er (C ) is th e b e st answ er to th e q u e s tio n . W hy not n e xt m o n th ? is a s u g g e s tio n th a t th e y


take th e trip n e x t m o n th .

STRATEGIES FOR THE LISTENING PARTA QUESTIONS

1. A s you listen t o each sh o r t c o n v e rsa tio n , focu s on th e se c o n d line o f th e


con v ersa tio n . The answer to the question is generally found in the second line.
2. K eep in m ind th a t th e c o r r e c t a n sw er is probably a r e s ta te m e n t o f a key w ord or
Idea in th e se c o n d line o f th e co n v e rsa tio n . Think of possible restatements.
3. K eep in m ind th a t c er ta in str u c tu r es and e x p r essio n s are te s te d regularly in
L istening P art A . Listen for these structures and expressions:
structures (passives, negatives, wishes, conditions)
functional expressions (agreement, uncertainty, suggestion, surprise)
idiomatic expressions ( two-part verbs, three-part verbs, idioms)
4. K eep in m ind th a t th e s e q u e stio n s gen erally progress from easy to difficult. This
means that questions I through S will be the easiest, and questions 26 through 30 will be the
hardest.
5. Read th e an sw ers and c h o o se th e b e s t an sw er to each q u estio n . Remember to
answer each question even if you are not sure of the correct response. Never leave any
answers blank.
6. Even if you do n o t u n derstand t h e c o m p le te con v ersa tio n , you can find th e
c o r r e c t answer.
If you only understood a few words or ideas in the second line, choose the answer
that contains a restatement of those words or ideas.
If you did not understand anything at all in the second line of the conversation, choose
the answer that sounds the most different from what you heard.
Never choose an answer because it sounds like what you heard in the conversation.
LISTENING COMPREHENSION

T h e fo llo w in g skills will h e lp you to im p le m e n t th e se str a teg ie s in th e L iste n in g Part A


s e c tio n o f th e T O E FL test. 5

STRATEGIES

S k ill I: FO C U S O N T H E SEC O N D LINE

In L iste n in g Part A you w ill h ea r a sh o rt c o n v e r sa tio n in v o lv in g two p e o p le ; th is con versa


tio n is fo llo w e d by a q u estio n . It is im p o r ta n t to u n d e r sta n d th at th e answ er to this type o f
q u e s tio n is m o st o fte n (b u t n o t always!) fo u n d in th e s e c o n d lin e o f th e c o n v e rsa tio n .

Exam ple

On the recording, you hear:


(man) Billy really made a big m istake this time.
(woman) Yes, he forgot to turn in his research paper.
(narrator) What does the w om an say about Billy?

In your test book, you read:


(A) It was the first time he made a mistake.
(B) He forgot to write his paper.
(C) He turned in the paper in the wrong place.
(D) He didnt remember to submit his assignment.

T h e s e c o n d lin e o f th is co n v e rsa tio n in d ic a te s th a t B illy forgot to tu r n in his paper, a n d this


m e a n s th a t h e d id not remember to subm it it. T h e b e st an sw er is th e r e fo r e answ er ( D ) .

T h e fo llo w in g ch art o u tlin e s th e m o st im p o r ta n t strategy for L iste n in g P art A:

STRATEGY#!: FOCUS ONTHE SECOND LINE

I. The second line of the conversation probably contains the answer to the question.
2. Listen to the first line of the conversation. If you understand it, thats good. If you dont
understand it, dont worry because it probably does not contain the'answer.
LISTENING PARTA

E X E R C ISE 1: In th is e x e r c ise , y o u s h o u ld fo c u s o n th e s e c o n d lin e o f th e c o n v e r sa tio n ,


r e a d th e q u e s tio n , a n d th e n c h o o s e th e b e st an sw er to th a t q u e s tio n . R e m e m b e r th a t y o u
c a n p rob ab ly an sw er th e q u e s tio n ea sily w ith o n ly th e s e c o n d lin e.

(man) Can you tell me i f today's (A) She has strong ideas about movies.
matinee is a comedy, romance, (B) She prefers com ed ies over westerns
or western ? and romances.
(woman) I have no idea. (C) She d o esn t like todays m atinee.
(narrator) What does the woman mean ? (D) She does not know.

2.(wom an) Was anyone at home at Barbs (A) Barb answered the bell.
house when you went there to (B) T h e house was probably empty.
deliver the package? (C) T he bell wasnt in the house.
(m an) / rang the bell, but no one (D) T he house d o esn t have a bell.
answered.
(narrator) What does the man imply?

3.(woman) You ju st got back from the (A)Its unlikely that h e ll go to the
interview fo r the internship. interview.
How do you think it went ? (B) H e thinks h e ll be recom m en ded
(m an) I think i t s highly unlikely that I for a high-level job .
got the job. (C) T he interview was apparently quite
(narrator) What does the m an suggest ? unsuccessful.
(D) H e had an excellen t interview.

T O E F L E X E R C ISE 1: In th is e x e r c is e , lis te n ca re fu lly to th e sh o r t c o n v e r s a tio n a n d q u e s


tio n in th e r e c o r d in g p ro g ra m , a n d t h e n c h o o s e th e b e st answ er to th e q u e s tio n . You
s h o u ld fo c u s ca refu lly o n th e s e c o n d lin e .

N ow BEGIN THE RECORDING PROGRAM AT TOEFL EXERCISE I.

1. (A) H e is leaving now. 5. (A) H er eyes hurt.


(B) H e has to go out o f his way. (B) She thought the lecture was great.
(C) H e will n ot be leaving soon. (C) T he class was boring.
(D) H e will d o it his own way. (D) She did nt want to watch Professor
2. (A) H e locked die door. Martin.
(B) H e tried unsuccessfully to g e t into 6. (A) N ot all the bills have b een paid.
the house. (B) They d o n t have en ou gh credit to
(C) H e was able to op en the door. pay the bills.
(D) H e left the house without locking (C) What she said on the p h on e was
the door. not credible.
3. (A) (D) H e used a credit card to pay som e
She d oesn t like to listen to turkeys.
(B) o f the bills.
She thinks the d in ner sounds
special. 7. (A) S h ell call back quickly.
(C) She especially likes the roast turkey. (B) S h ell definitely b e back by 4:00.
(D) Sh ed prefer a different dinner. (C) S h ell give it back by 4:00.
4. (A) H e ll be busy with her hom ework (D) S h ell try to return fast.
tonight. 8. (A) She hasnt seen Tim.
(B) H e cant help her tonight. (B) Tim was there only for a m om ent.
(C) H e s sorry he cant ever h elp her. (C) Tim was around a short tim e ago.
(D) H e ll help her with her physics. (D) Tim will return in a m inute.
LISTENING COMPREHENSION

(A) She doesnt like the place he chose. 10. (A) T heres plenty to eat.
(B) She doesnt want to get into the car. (B) T he refrigerators broken.
(C) S h es glad the spot is reserved. (C) The food isnt in the refrigerator.
(D) They cant park the car there. (D) H e s n ot sure if theres enough.

S k i l l 2: C H O O S E ANSW ERS W IT H S Y N O N Y M S
O fte n th e c o r r e c t an sw er in L iste n in g Part A is an an sw er th a t c o n ta in s syn on ym s (w ords
w ith sim ilar m e a n in g s b u t d iffe r e n t so u n d s) fo r k e y w o r d s in th e c o n v e rsa tio n .

Exam ple

On the recording, you hear:


(woman) Why is Barbara feeling so happy?
(man) She just started working in a real estate agency.
(narrator) What does the man say about Barbara?

In your test book, you read:


(A) She always liked her work in real estate.
(B) She began a new job.
(C) She just bought some real estate.
(D) She bought a real estate agency.

In th is c o n v e rsa tio n , th e key w ord started m e a n s began, a n d th e key w ord w orking refers to
job. T h e b e st answ er to th is q u e s tio n is th e r e fo r e an sw er ( B ) .

T h e fo llo w in g ch a rt o u tlin e s a very im p o r ta n t strategy for L iste n in g Part A:

STRATEGY #2: CHOOSE ANSWERS WITH SYNONYMS

1. As you listen to the second line of the conversation,focus on key words In that line.
2. If you see any synonyms for key words in a particular answer, then you have probably found
the correct answer.

EX E R C ISE 2: In th is e x e r c ise , u n d e r lin e key w ord s in th e se c o n d lin e o f e a c h sh o rt c o n


v e rsa tio n . T h e n u n d e r lin e syn on ym s for th e se key w ord s in th e answ ers, a n d c h o o s e th e
b e st answ er to e a ch q u e s tio n . R e m em b er th a t th e b e s t answ er is p rob ab ly th e answ er th a t
c o n ta in s syn on ym s fo r th e key w ord s in th e s e c o n d lin e o f th e co n v ersa tio n .

1. (wom an) Did you see the manager about (A) H e got a job as bookstore manager.
the job in the bookstore? (B) The bookstore was not accepting
(m an) Yes, and I also had to Jill out an applications.
application. (C) H e saw a book about how to apply
(narrator) What does the m an mean? for jobs.
(D) It was necessary to com plete a form.
LISTENING PART A

2. (man) We 're planning to leave fo r the (A) If they could leave at n oon .
trip at about 2:00. (B) If it is possible to go by 12:00.
(woman) Couldnt we leave before noon ? (C) Why they cant leave at noon .
(narrator) What does the woman ask ? (D) If they could leave the room.

3. (man) Was the concert well received ? (A) The perform ance went on for a
(woman) The audience applauded fo r a long time.
long time after the performance. (B) There was applause throughout the
(narrator) What does the woman say about perform ance.
the concert? (C) The p eople clapped o n and on
after the concert.
(D) The aud ien ce waited for a long
tim e for the concert to begin.

T O E F L E X E R C ISE 2: In th is e x e r c ise , liste n carefu lly to th e sh o r t c o n v e r s a tio n a n d q u e s


tio n in th e r e c o r d in g p r o g r a m , a n d th e n c h o o s e th e b est answ er to th e q u e s tio n . You
s h o u ld lo o k fo r syn on ym s fo r key w o r d s in th e s e c o n d lin e.

N O W BEGIN THE RECORDING PROGRAM AT TOEFL EXERCISE 2.

(A) T h e final exam was harder than the 6. (A) T he firefighters saved the hom es
others. for last.
(B) T here were two exam s rather than (B) A firefighter saved the hillside last
one. night.
(C) H e thought the exam would be (C) The hom es on the hillside were
easier. burned.
(D) T h e exam was not very difficult. (D) The houses w erent destroyed.
(A) H e s not feelin g very well. 7. (A) T heres en ou gh soup.
(B) H e s rather sick o f working. (B) T he spices are adequate.
(C) H e s feelin g better today than (C) She thinks the sou ps too salty.
yesterday. (D) The man should add m ore salt and
(D) H e d really rather n ot answer the pepper.
question.
8. (A) He was lucky to receive a grant for
(A) T he com pany was fou n d ed about a his studies.
year ago. (B) He used his fortune to pay his fees.
(B) It was just established that he could (C) He is a scholar at a college with low
go into business. fees.
(C) T he family is well established. (D) He paid to get a scholarship.
(D) T he business only lasted a year.
9. (A) It profited from previous mistakes.
(A) H e did not look at the right (B) It earned a lot o f money.
schedule. (C) This was the last year that it would
(B) T h e plane landed in the right place. make a profit.
(C) T h e plane arrived on time. (D) It was not so successful.
(D) H e had to wait for the plane to land.
10. (A) Chucks bank account has too
(A) Sh ed rather go running. m uch m oney in it.
(B) She d oesn t want to g o into the (B) H e thinks Chuck has the wrong
pool. kind o f bank account.
(C) Sh ell change cloth es quickly and (C) H e thinks that Chuck is on his way
go swimming. h om e from the bank.
(D) She needs a sweatsuit to go (D) There isnt enough m oney in
running. Chucks account.
LISTENING COMPREHENSION

S k i l l 3: A V O ID SIM ILAR SO U N D S

O fte n th e in c o r r e c t answ ers in L iste n in g P art A are answ ers th a t c o n ta in w ord s w ith sim i
lar s o u n d s b u t very d iffe r e n t m e a n in g s fro m w h a t y o u h e a r in th e r e c o r d in g p rogram .
You sh o u ld d e fin ite ly a void th e se answers.

Exam ple

On the recording, you hear:


(man) Why couldn't Mark com e w ith us?
(woman) He was searching for a new apartment.
(narrator) What does the w om an say about Mark?

In your test book, you read:


(A) He was in the department office.
(B) He was looking for a place to live.
(C) He was working on his research project.
(D) He had an appointment at church.

T h e k ey w o rd s in th e s e c o n d lin e o f th e c o n v e r sa tio n are searching a n d apartm ent. In a n


sw ers (C ) a n d (D ) th e w ord s research a n d church s o u n d lik e search, so th e se answ ers are in
c o r r e c t. In answ ers (A ) a n d (D ), th e w ord s departm ent a n d appointm ent so u n d like
apartm ent, so th e se answ ers are in c o r r e c t. T h e b e st an sw er is th e r e fo r e answ er (B ).

T h e fo llo w in g ch a rt o u tlin e s a very im p o r ta n t strategy fo r L iste n in g Part A:

STRATEGY #3: AVOID SIMILAR SOUNDS

1. Identify key words in the second line of the conversation.


2. Identify words in the answers that contain similar sounds, and do not choose these answers.

NOTE: InAppendix A there are drills to practice distinguishing similar sounds. You may
want to complete these practice drills before trying the following exercises.

E X E R C ISE 3: In this e x e r c ise , u n d e r lin e key w ord s in th e s e c o n d lin e o f e a c h sh o rt c o n


v e rsa tio n . T h e n u n d e r lin e w ord s w ith so u n d s sim ila r to th e se key w ord s in th e answ ers,
an d c h o o s e th e b e st an sw er to e a c h q u e s tio n . R e m e m b e r th a t th e b e st an sw er is probab ly
th e an sw er th a t d o e s n o t c o n ta in w ord s w ith so u n d s th a t are sim ilar to th e so u n d s o f th e
key w o rd s in th e s e c o n d lin e o f th e c o n v e rsa tio n .

1. (wom an) I heard that Sally jiust moved (A) Sally has no sense o f responsibility.
into a new, big house near the (B) Sally sent her friend to the house.
beach. (C) Sally has n o money.
(m an) But Sally doesn '1 have a cent! (D) Sally is on the set with her.
(narrator) What does the man mean?
LISTENING PARTA

(woman) Did they get the new car they (A) They locked the map in a car.
wanted ? (B) They looked many times in the car.
(man) No, they lacked the money. (C) It cost a lot o f m oney when the car
(narrator) What does the man mean ? leaked oil.
(D) They d id n t have en ou gh m oney to
buy another car.
(man) Have you finished packing yet? (A) Its im portant to pack the suitcases.
(woman) You should call the porter to get (B) They n eed help carrying their bags.
the suitcases. (C) T he man should pack his suit in
(narrator) What does the woman mean ? case he needs it.
(D) The suitcases are quite portable.

T O E F L E X E R C ISE 3: In th is e x e r c ise , liste n ca re fu lly to th e sh o r t c o n v e r sa tio n a n d q u e s


tio n in th e r e c o r d in g p r o g r a m , a n d th e n c h o o s e th e b e st answ er to th e q u e s tio n . You
sh o u ld b e c a refu l to a void answ ers w ith sim ila r so u n d s.

N O W BEGIN THE RECORDING PROGRAM AT TOEFL EXERCISE 3.

1. (A) She has to wait for som e cash. 6. (A) Twenty pairs o f sh oes are on sale.
(B) T he waiter is bringing a glass o f (B) The sh oe salesclerk spent twenty
water. dollars on pears.
(C) The lawn is too dry. (C) The shoes cost twenty dollars.
(D) She n eed s to watch out for a crash. (D) The shoes could be repaired for
2. twenty dollars.
(A) The sweaters the wrong size.
(B) The m an s feet aren t sweating. 7. (A) Tom tended to dislike biology lab.
(C) The sweater makes the m an seem (B) Attendance wasnt necessary at
fat. biology lab.
(D) The sweet girl d o e sn t feel right. (C) Tom w ent to biology lab.
3. (A) He has been regularly using a (D) There was a tendency to require
biology lab.
computer.
(B) He com m unicates with a Boston 8. (A) The m eal will be served at noon.
company. (B) The m ales should be driven there
(C) He regularly g oes to com m unities by noon.
around Boston. (C) H e s expecting the ice to m elt
(D) H e has been traveling back and before noon.
forth to Boston. (D) The letters ough t to be delivered at
12:00.
4. (A) He thought the lesson d id n t
matter. 9. (A) T he w eather will probably get
(B) H e c o u ld n t learn the lesson. worse later.
(C) H e learned a massive num ber o f (B) The newspaper headlines
details. described a bad storm.
(D) He d id n t like m ost o f the lesson. (C) There was news about a headstrong
man.
5. (A) Som e animals started the first fire.
(D) He had a new bed.
(B) Anim als are killed by forest fires.
(C) In the first frost, animals die. 10. (A) If she could do the grocery shopping.
(D) Frost can kill animals. (B) If she prefers cooked vegetables or
salad.
(C) If she could help prepare the salad.
(D) If she minds sh opp in g for
vegetables.
LISTENING COMPREHENSION

T O E F L E X E R C ISE (S k ills 1 -3 ): In th is e x e r c is e , listen carefu lly to th e sh o rt c o n v ersa tio n


a n d q u e s tio n in th e r e c o r d in g p ro g ra m , a n d t h e n c h o o s e th e b e st answ er to th e q u estio n .

D N O W BEGIN THE RECORDING PROGRAM AT TOEFL EXERCISE (SKILLS 1-3).

(A) He would like som e iced coffee. 7. (A) The managers will take the train to
(B) H e wants to stop drinking coffee. the program.
(C) A drink seem s like a good idea. (B) A program to develop new
(D) H e needs to drink som ething to managers will com m ence soon.
stop his coughing. (C) The new m anagem ent program is
very weak.
2. (A) She would prefer a sunny day.
(D) The program will be maintained to
(B) The park is too crowded.
the letter.
(C) She would like a place that is not so
loud. 8. (A) The fire started to attack the
(D) She cannot walk because sh es too building.
old. (B) T he firefighter stared at the
attacker.
(A) H e should open an account.
(C) The fire probably began at the top
(B) He should take a ride on a ship.
o f the building.
(C) H e should try to keep the cost
(D) The firefighter started to attack the
cheap.
fire.
(D) H e should try som ething
m onoton ous to get to sleep. 9. (A) H e assured the woman that he
knew the truth.
4. (A) The departm ent is not changing
(B) He is sure that it isnt new.
the requirements.
(C) He thought that the woman was
(B) H e hasnt heard anything about aware o f what happened.
the change.
(D) H e soon will know the truth.
(C) The changes are believable.
(D) What has happened is incredible to 10. (A) The art professor is not one of his
him. fans.
(B) His drawings were amazing.
5. (A) The wait has taken close to an
(C) The catches that h e made were
hour.
fantastic.
(B) They were stranded in their car.
(D) His sketches showed a fantasy
(C) Most o f the people have been in
world.
line for hours.
(D) They made a line in the sand.
6. (A) The instructor is selecting several
passages.
(B) T he conductor is fair to the
passengers.
(C) The stamp collector is conducting
his business.
(D) The riders are paying for the train
trip.
LISTENING PARTA

W H O ,W H A T , W H E R E ------------------------------------------------------------------

S k ill 4: D R A W C O N C L U S IO N S A B O U T WHO, WHAT, W H ER E

It is c o m m o n in L iste n in g P art A to ask y o u to draw s o m e k in d o f c o n c lu s io n . In th is type


o f q u e s tio n th e an sw er is n o t clea rly stated ; in ste a d y o u m u st draw a c o n c lu s io n b a sed
o n c lu e s g iv e n in th e c o n v e rsa tio n . O n e k in d o f c o n c lu s io n th a t is c o m m o n in this p art
o f th e test is to ask y o u to d e te r m in e who th e sp ea k er is, b a sed o n c lu e s g iv e n in th e
c o n v e rsa tio n .

Exam ple

On the recording, you hear:


(woman) Can you tell me w hat assignm ents I m issed when I was
absent from your class?
(man) You m issed one hom ework assignm ent and a quiz-
(narrator) Who is the man?

In your test book, you read:


(A) A newspaper editor.
(B) A police officer.
(C) A teacher.
(D) A student.

T h e c lu e s y o u r class, homework, a n d q u iz in th e c o n v e r sa tio n te ll y o u th a t th e m a n is p r o b a


bly a teach er. A n sw er (C ) is th e r e fo r e th e c o r r e c t answer.

A n o th e r type o f c o n c lu s io n th a t is c o m m o n in L iste n in g P art A is to d e te r m in e w h a t


w ill p rob ab ly h a p p e n n e x t, b a sed o n c lu e s g iv e n in th e c o n v e rsa tio n .

Exam ple

On the recording, you hear:


(woman) Are you going to read those books here in the library?
(man) I think I'd rather check them out now and take them
home.
(narrator) What w ill the m an probably do next?

In your test book, you read:


(A) Sit down in the library.
(B) Look for some more books.
(C) Return the books to the shelves.
(D) Go to the circulation desk.

T h e m an says th a t h e w o u ld lik e to check the books out now. S in c e th e circulation desk is w h e r e


y o u g o to c h e c k b o o k s o u t fr o m a library, th e m an w ill p ro b a b ly g o to th e c ir c u la tio n d e sk
n e x t. T h e c o r r e c t an sw er is th e r e fo r e an sw er (D ).
LISTENING COMPREHENSION

A fin a l type o f c o n c lu s io n th a t is c o m m o n in L iste n in g Part A is to d e te r m in e where


th e c o n v e r sa tio n p ro b a b ly takes p la c e , b a sed o n c lu e s g iv e n in th e c o n v e rsa tio n .

Exam ple

On the recording, you hear:


(woman) Are you going into the water, or are you just going to
lie there on the sand?
(man) I think I need to p u t on som e suntan lotion.
(narrator) Where does this conversation probably take place?

In your test book, you read:


(A) At a beauty salon.
(B) At the beach.
(C) In a sandbox.
(D) At an outdoor restaurant.

T h e c lu e s water, sa n d , a n d s u n ta n lotion in th e c o n v e r sa tio n tell y o u th at th is c o n v e rsa tio n


p r o b a b ly takes p la c e at th e beach. A n sw er (B ) is th e r e fo r e th e c o r r e c t answer.

T h e fo llo w in g ch a rt o u tlin e s th e key p o in t th a t y o u sh o u ld r e m e m b e r a b o u t th is type


o f q u estio n :

CONCLUSIONS ABOUT WHO, WHAT, WHERE

It is common for you to be asked to draw one of the following conclusions in Listening Part A:
1. WHO is probably talking?

2. WHAT will s/he probably do next?

3. W H ERE does the conversation probably take place?

E X E R C ISE 4: In th is e x e r c ise , r ea d th e sh o r t c o n v e r sa tio n an d q u e s tio n , u n d e r lin e the


c lu e s th a t h e lp y o u answ er th e q u e s tio n , a n d th e n c h o o s e th e b e st answer. You w ill have to
draw c o n c lu s io n s a b o u t who, what, a n d where.

1. (m an) I d like to deposit th check in (A) A store clerk.


my account, please. (B) A bank teller.
(wom an) Would you like any cash back ? (C) An accountant.
(narrator) Who is the woman? (D) A waitress.

(wom an) H ave you deposited your (A) Earn his paycheck.
paycheck yet ? (B) Write a check for a deposit on an
(m an) No, but thats next on my list of apartment.
errands. (C) Go to a bank.
(narrator) What will the m an probably do (D) Make a list o f errands to run.
next?
LISTENING PARTA

(man) Did you get the bread, eggs, (A) In a restaurant.


and milk? (B) At a bakery.
(woman) Now we need to stand in line at (C) On a farm.
the checkout counter. (D) In a market.
(narrator) Where does this conversation
probably take place?

T O E F L E X E R C ISE 4: In th is e x e r c is e , liste n ca refu lly to th e sh o r t c o n v e r s a tio n a n d q u e s


tio n in th e r e c o r d in g p ro g ra m a n d th e n c h o o s e th e b e s t an sw er to th e q u e s tio n . You w ill
have to draw c o n c lu s io n s a b o u t who, w hat, a n d where.

n N O W BEGIN THE RECORDING PROGRAM AT TOEFL EXERCISE 4.

1. (A) In a photography studio. 6. (A) On a playground.


(B) In a biology laboratory. (B) In a parking lot.
(C) In an office. (C) At a zoo.
(D) In the library. (D) In a ph oto studio.
2. (A) H e s a pilot. 7. (A) R espond to the mail.
(B) H e s a flight attendant. (B) Put the letters in a file.
(C) H e s a m em ber o f the grounds (C) It depend s o n where the file is.
crew. (D) File the answers she received to the
(D) H e works clearing land. letters.
3. (A) Wash the dishes immediately. 8. (A) In an airplane.
(B) Use as many dishes as possible. (B) In a police car.
(C) Wash the dishes for as lon g as (C) In a theater.
possible. (D) At a fireworks exhibit.
(D) Wait until later to clean up.
9. (A) Take care o f Bob.
4. (A) In a bank. (B) Invite Bob to dinner.
(B) In a restaurant. (C) Let Bob know that they accept his
(C) At a service station. invitation.
(D) In a beauty salon. (D) R espond to the w om ans question.
5. (A) A salesclerk in a shoe store. 10. (A) A pharmacist.
(B) A sh oe repairperson. (B) A dentist.
(C) A party caterer. (C) A teacher.
(D) A salesclerk in a fixtures (D) A business manager.
departm ent.
LISTENING COMPREHENSION

S k ill 5: LISTEN FOR W HO A N D WHAT IN PASSIVES

It is so m e tim e s d iffic u lt to u n d e rsta n d who or w h a t is d o in g th e a c tio n in a passive se n


te n c e . T h is p r o b le m is o fte n te sted in L iste n in g P art A.

Exam ple

On the recording, you hear:

(man) Did Sally go to the bank this morning?


(woman) Yes, she did. She got a new checking account.
(narrator) What does the w om an imply?

In your test book, you read:

(A) Sally wrote several checks.


(B) Sally wanted to check up on the bank.
(C) A new checking account was opened.
(D) Sally checked on the balance in her account.

In this co n v ersa tio n , th e w om an u ses an active sta te m en t th at m ea n s that Sally opened a check
in g account. T h e c o rrect answ er u ses th e passive stru ctu re th at a checking account was opened
to exp ress th e sam e idea. T h e r e fo r e , the b est answ er to th e q u estio n above is answ er (C ).

You sh o u ld n o te th e fo llo w in g a b o u t passive s e n t e n c e s in L iste n in g P art A:

PASSIVE STATEMENTS ___________________________

1. If the conversation contains a passive statement, the answer to the question Is often an oct/ve
statement.
2. If the conversation contains an active statement, the answer to the question is often a passive
statement.

NOTE: Check carefully who or what is doing the action in these questions.

E X E R C ISE 5: In th is e x e r c ise e a c h o f th e c o r r e c t answ ers is e ith e r a passive r e sta te m e n t


o f an active se n te n c e or a n active r e sta te m e n t o f a passive se n te n c e . R ead th e sh o r t c o n
v ersa tio n an d u n d e r lin e th e key active o r passive sta te m e n t. T h e n read th e q u e stio n and
c h o o s e th e b e st answ er to th e q u e stio n . B e c a re fu l a b o u t who an d w hat w ith th e se passives.

1. (wom an) Alice needs to pay her tuition (A) A lices education has paid off.
today. (B) A lices tuition needs to be paid.
(m an) But her tuition has alreadybeen (C) Alice has already paid her fees.
paid. (D) Alice has already received the
(narrator) What does the man imply T money.
LISTENING PARTA

(man) Have you been taking good (A) She drank som e water on the lawn
care o f the lawn ? this m orning.
(woman) I watered it only this morning. (B) She waited for him on the lawn this
(narrator) What does the woman mean ? m orning.
(C) T he lawn has already b een watered
today.
(D) She wanted a new lawn this morning.

(man)Did you hear the news about the (A) Som eone located the girl.
child who was lost in the park ? (B) She heard about the new park from
(woman) Yes, and I heard that she was ju st the child.
found! (C) T he child found her lost pet.
(narrator) What does the woman mean ? (D) The child was the last o n e in the
park.

T O E F L E X ER C ISE 5: In th is e x e r c is e , liste n c a re fu lly to th e sh o r t c o n v e r s a tio n a n d q u e s


tio n in th e r e c o r d in g p r o g r a m , a n d th e n c h o o s e th e b e st an sw er to th e q u e s tio n . You
sh o u ld be p a rticu la rly c a refu l o f p assives.

N ow BEGIN THE RECORDING PROGRAM AT TOEFL EXERCISE 5.

(A) If the restaurant is on the corner. 5. (A) She was broke from skiing.
(B) If the man would like to go to the (B) She w ent skiing in spite o f her
restaurant. accident.
(C) If the vegetables are fresh. (C) H er leg was hurt on a skiing trip.
(D) If vegetarian food can be obtained. (D) H er skis were broken in the
m ountains.
2. (A) H e admittfcd that h e wanted to go
to law school in the fall. 6. (A) T he road the horses took was long
(B) T h e law school accepted him as a and hard.
student. (B) It was hard to find the hidden
(C) T h e law professor adm itted that he houses.
would be a student in the fall (C) T he riders worked the horses too
semester. m uch.
(D) H e would be adm itted to law (D) It was hard for p eop le to ride the
school after the fall semester. horses for long.
3. (A) Marks plants were cared for in his 7. (A) H e d id n t want the coffee that the
absence. wom an ordered.
(B) Marks plan was to be out o f town. (B) H e wasnt sure if the wom an
(C) Mark was careful about his plans wanted coffee.
for the out-of-town trip. (C) H e assum ed the wom an had
(D) She was careful while Mark was ordered coffee.
gone. (D) H e was unaware that coffee had
already b een ordered.
4. (A) T h e lights in the trees were
destroyed in the storm. 8. (A) T he car was in the left parking lo t
(B) T h e storm dam aged the trees. at the airport.
(C) T h e falling trees destroyed a store. (B) T he friends parked their car at the
(D) In the light the destruction o f the airport.
storm could be seen. (C) The airport c o u ld n t hold a lot o f
cars.
(D) T here were a lot o f cars to the left
o f the parking lot.
LISTENING COMPREHENSION

9. (A) The students p ointed at Mac. 10. (A) After the earthquake, the
(B) Mac was present when the other insurance com pany came out to
students m ade the appointm ent. inspect the damage.
(C) The class representative suggested (B) The insurance company insisted
Mac to the other students. that the building be repaired to
(D) Mac was chosen by his classmates to m eet earthquake safety standards.
represent them. (C) The inhabitants paid their
premiums after the earthquake.
(D) T he insurance company paid for
the earthquake damage.

S k ill 6: LISTEN FOR W HO A N D WHAT W IT H MULTIPLE N O U N S

W h e n th e r e is m o r e th a n o n e n o u n in a s e n te n c e in L iste n in g P art A , it is c o m m o n for


th e answ ers to c o n fu s e w h ic h n o u n d o e s w hat.

Exam ple

On the recording, you hear:


(man) Do you know w ho is in the band now?
(woman) I heard that Mara replaced Robert in the band.
(narrator) What does the w om an say about the band?

In your test book, you read:


(A) Robert became a new member of the band.
(B) Robert took Mara's place in the band.
(C) Mara didnt have a place in the band.
(D) Mara took Roberts place in the band.

In th e w o m a n s r e s p o n se to th e m a n s q u e s tio n , sh e talks a b o u t tw o p e o p le (M ara


a n d Robert), a n d th e se tw o p e o p le are c o n fu s e d in th e answ ers. B e ca u se M ara replaced
Robert, th is m e a n s th at M ara is in th e b a n d an d R o b er t is n o t. T h e b e st answ er is th e r efo r e
an sw er (D ).

T h e fo llo w in g ch art o u tlin e s th e key p o in t th a t y o u sh o u ld r e m e m b e r a b o u t q u estio n s


w ith m u ltip le n ou n s:
LISTENING PARTA

EX E R C ISE 6: In th is e x e r c ise , u n d e r lin e th e c o n fu s in g n o u n s in e a c h sh o r t c o n v e rsa


tio n . T h e n read th e q u e s tio n a n d c h o o s e th e b e st an sw er to th a t q u e s tio n . R e m e m b e r to
th in k very ca re fu lly a b o u t w h o is d o in g w h at.

(m an) Why is Bill not at work th (A) T he doctor d ecid ed to take som e
week? rime o ff from work.
(wom an) H is doctor made him take a (B) The doctor told Bill he w asnt too
week off. weak to work.
(narrator) What does the woman mean ? (C) Bill was mad w hen the doctor took
som e time off.
(D) Bill took a vacation on his d octors
orders.
(m an) Why is P aul going back home (A) Paul is gettin g m arried this
this summer? summer.
(woman) H e s returning to Vermont fo r (B) Pauls sister is returning from
his sisters wedding. Verm ont to g e t married.
(narrator) What does the woman mean 1 (C) Paul will be there when his sister
gets m arried this summer.
(D) Pauls sister is com in g to his
wedding in Vermont.
3. (man) D id you hear that John's uncle (A) John received an inheritance when
died? his un cle died.
(woman) Yes, and John was named (B) Its a b enefit that J o h n s nam e is
beneficiary in his uncles will. the sam e as his u n cles.
(narrator) What does the woman mean ? (C) John knows that his un cle will
com e to the benefit.
(D) J o h n s uncle gave him a b eneficial
name.

T O E F L E X E R C ISE 6: In th is e x e r c is e , liste n ca refu lly to th e sh o r t c o n v e r s a tio n a n d q u e s


tio n in th e r e c o r d in g p r o g r a m , a n d th e n c h o o s e th e b e st an sw er to th e q u e s tio n . Y ou
s h o u ld b e particu larly c a re fu l o f w h o is d o in g w hat.

N ow BEGIN THE RECORDING PROGRAM AT T o e f l E x e r c is e 6.


n
(A) The passenger waited at the corner. 3. (A) The baby sister w ent to bed quite
(B) T he passenger looked for a taxi at early.
the corner. (B) The children w ere forced to go to
(C) T he cab driver waited for the bed early.
passenger. (C) The baby-sitter m ade the bed after
(D) T he passenger cornered the the children g o t up.
waiting taxi driver. (D) The baby-sitter did not stay up late.
2. (A) It was hard for her to hear Jane last 4. (A) The m an taught his son about
night. football.
(B) Jane gave a harp recital last night. (B) The boy is receiving the ball from
(C) Jane was playing hard while she was his dad.
hurt. (C) T he ball is being tossed in to the air
(D) She played the harp last night for by the boy.
Jane. (D) The m an is playing with the ball in
the sun.
LISTENING COMPREHENSION

(A) The students were told to go listen 8. (A) The chairman decid ed that Tony
to the speaker. ' would serve on the board for
(B) T he professor attended that another year.
even in gs lecture. (B) The chairman elected the board.
(C) The students were given directions (C) The board decided Tony could be
to the lecture. chairman after o n e year.
(D) The professor was directed to the (D) Tony becam e chairman for one
lecture hall. more year.

(A) T he m anager went to the supply 9. (A) The ju d ge defended the murderer.
room. (B) The ju d ge tried to protect the
(B) The clerk set supplies on the floor defendant from the murderer.
(C) The clerk went to the supply room (C) The ju d ge said that the defendant
at the m anagers request. was a criminal.
(D) T he clerk backed into the manager (D) The defense cou ld n t make a
in the supply room. jud gm en t about the criminal.

(A) T he librarian was quite reserved 10. (A) The woman should announce the
with the students for two days. nam es o f the com m ittee
(B) W ithin two days the librarian had members.
the books for the students. (B) H e is thankful to be appointed to
(C) T he librarian asked the students the com m ittee.
for the books. (C) H e is sure about the tim e o f the
(D) T he students put the books on appointm ent with the
hold for two days. com mittee.
(D) The woman will serve on the
com mittee.

T O E F L E X E R C ISE (S k ills 4 - 6 ) : In th is e x e r c ise , listen carefu lly to th e sh o r t co n v e rsa tio n


a n d q u e s tio n in th e r e c o r d in g p rogram , an d th e n c h o o s e th e b e st answ er to th e q u estio n .

N o w BEGIN THE RECORDING PROGRAM AT TOEFL EXERCISE (SKILLS 4-6).

(A) In a departm ent store. 4. (A) A delivery man.


(B) In a stationery store. (B) A famous chef.
(C) At the post office. (C) A clerk in a fast-food restaurant.
(D) At the airport. (D) An airline steward.

2. (A) T he teacher gave the students a 5. (A) They n eed new print for the
hand. additional copies.
(B) T he term papers were turned in. (B) They can make extra copies if
(C) The students got the papers from necessary.
the office. (C) Printers are n eeded for the
(D) T he teacher handed the papers to additional copies.
the students. (D) Additional copies are needed
immediately.
3. (A) The attendant checked the oil in
Marks car. 6. (A) The professor bought two books.
(B) Mark checked to see if he had (B) The students had to purchase two
en ou gh oil in his car. books.
(C) Mark checked with the servicc (C) T he students sold two books to the
station attendant. professor.
(D) Mark wrote a check to pay for the (D) The students were required to read
oil. two books by the professor.
LISTENING PARTA

7. (A) T he doctor returned to the office. 9. (A) Mary becam e the new class
(B) Jim asked the doctor to com e to president.
the office. (B) Sue took her place as class
(C) T he doctor will not return until president.
n ext week. (C) In place o f Mary, Sue becam e
(D) Jim was told to com e back. senior class president.
(D) T he senior class president replaced
8. (A) Go to work in the lab.
Sue and Mary.
(B) Sample the work from the lab.
(C) Have the sam ples delivered. 10. (A) The panel was analyzed on the
(D) Send a n ote to the lab. television program.
(B) A com m ittee evaluated recent
political events.
(C) T he program featured a
psychoanalyst.
(D) T he panel discussed the television
program.

T O E F L R EV IEW E X E R C ISE (S k ills 1 -6 ): In th is e x e r c ise , listen ca re fu lly to th e s h o r t


c o n v e r s a tio n a n d q u e s tio n in th e r e c o r d in g p r o g r a m , an d th e n c h o o s e th e b e s t a n sw er to
th e q u e s tio n .

N O W BEGIN THE RECORDING PROGRAM AT TOEFL REVIEW EXERCISE (SKILLS 1-6).

1. (A) H e seem ed to be rather hungry. 6. (A) T he car stalled on the road.


(B) She was quite angry at him. (B) Som eo.ie took the car.
(C) H e was trying to hang the posters. (C) Rob sold his car.
(D) She believes he was mad. (D) Rob heard som eon e steal his car.
2. (A) T h e parents are g oin g to stay up 7. (A) Buying the bigger container.
late. (B) Putting the milk in the cart.
(B) T h e parents have given H annah (C) Taking a carton that is smaller.
her allowance. (D) G etting the milk tom orrow instead.
(C) Lately, the parents have not been
8. (A) T he receptionist w elcom ed the
so loud.
businesspeople.
(D) H annah does n ot have to go to bed
(B) T he man created a shipping and
early.
receiving business.
3. (A) At a departm ent store. (C) T he businesspeople were rather
(B) At a service station. greedy.
(C) At a collection agency. (D) T he businesspeople greeted the
(D) In a delivery room. receptionist.
4. (A) She just broke som e eggs. 9. (A) T h e police officer was stationed
(B) They n eed to eat fast. near the tourist.
(C) She is serious about the boat. (B) T he tourist was forced to
(D) H e has a ch oice to make. accom pany the p olice officer.
(C) T he tourist becam e m ad at the
5. (A) It was urgent that Ellen do her best.
police station.
(B) H e really urged Ellen to do more.
(D) T h e tourist stated that the p olice
(C) H e was encouraged by Ellen to try
officer never cam e.
harder.
(D) Ellen told him that she was trying 10. (A) H e hasnt seen her ideas.
to do better. (B) It was a terrible deal.
(C) H e d oesn t like the idea.
(D) It sounds m agnificent to him.
LISTENING COMPREHENSION

N EGATIVES

S k i l l 7: LISTEN FOR NEG ATIVE EXPRESSIONS

N e g a tiv e e x p r e s sio n s are very c o m m o n in L iste n in g P art A, a n d th e m o st c o m m o n kin d


o f c o r r e c t r e s p o n s e to a n e g a tiv e sta te m e n t is a p o sitiv e sta te m e n t c o n ta in in g a w ord w ith
an o p p o s ite m e a n in g .

Exam ple

On the recording, you hear:


(man) How did they get to their grandmother's house in
Maine in only five hours?
(woman) They didn't drive slowly on the trip to Maine.
(narrator) What does the w om an say about the trip?

In your test book, you read:


(A) They drove rather quickly.
(B) They couldnt have driven more slowly.
(C) They wanted to travel slowly to Maine.
(D) They didnt drive to Maine.

T h e c o r r e c t an sw er is answ er (A ). I f th ey d id not d rive slowly to M ain e, th is m e a n s that th ey


d r o v e ra th er quickly. N o tic e th a t th e c o r r e c t an sw er u ses quickly, th e o p p o s ite o f slowly.
T h e answ ers th a t u se slowly are n o t co rr ec t.

T h e fo llo w in g c h a r t o u tlin e s th e types o f n e g a tiv e e x p r e ssio n s th a t y o u sh o u ld be


c a r e fu l of:

TYPES OF NEGATIVE EXPRESSIONS

Expression Example Correct Answer

Regular negative: not or n't Tom is not sad about the results. not sod = happy

Other negatives: nobody, none, Nobody arrived on time. nobody . . . on time = late
nothing, never Sal never works hard. never works hard = lazy

Negative prefixes: un-, in-, dis The patient was insane. insane = not sane = crazy
' . .. - ' ' - : . y

E X E R C ISE 7: In th is e x e r c ise , u n d e r lin e th e n e g a tiv e in th e se c o n d lin e o f e a c h sh o rt


c o n v e r sa tio n . T h e n read th e q u e stio n an d c h o o s e th e b e st answ er to th a t q u estio n .
R e m e m b e r th a t th e b e st answ er is o n e th at u se s an o p p o s ite m e a n in g .

(m an) I cant seem to get the door (A) The key in the drawer is on the right.
unlocked. (B) The man should write the message
(wom an) That isn't the right key fo r the on the door.
door. (C) The man has the wrong key.
(narrator) What does the woman mean ? (D) The right key isnt in the drawer.
LISTENING PARTA

2. (m an) Were you pleased with last (A) T he convention was disorganized.
week's convention ? (B) She d id n t plan to attend the
(woman) Nothing went as planned. convention.
(narrator) What does the woman mean ? (C) She planned the convention last
week.
(D) She wasnt pleased with the last
w eek o f the convention.

(woman) Are you planning to go to college (A) H e definitely wants to g o to


next year? college.
(m an) I m really unsure about the idea. (B) H e is certain about his plans.
(narrator) What does the m an mean ? (C) H e s hesitant about attending
college.
(D) His idea is to go to college.

T O E F L E X E R C ISE 7: In th is e x e r c is e , lis te n carefu lly to th e sh o r t c o n v e r s a tio n a n d


q u e s tio n in th e r e c o r d in g p r o g r a m , a n d th e n c h o o s e th e b e st an sw er to th e q u e s tio n . You
sh o u ld b e p articu larly c a refu l o f n e g a tiv e e x p r e s sio n s.

o N ow BEGIN THE RECORDING PROGRAM AT TOEFL EXERCISE 7.

(A) She is very busy. (A) The service satisfied her.


(B) She has lots o f free time. (B) T he food was worse than the
(C) It is not necessary to take ou t the service.
trash. (C) She thought the service was bad.
(D) She will do it if she has time. (D) N either the food nor the service
2. (A) T h e interview is very im portant. was satisfying.
(B) H e is w orried about the interview. (A) H e told his kids to leave.
(C) What h e s wearing to the interview (B) H e seriously wanted the woman to
is important. g-
(D) H e is not concerned about the (C) He was joking when he told the
interview. wom an to leave.
(A) H e has alm ost all the notes. (D) He left with the woman.
(B) H is attendance was perfect. (A) The project will take all their effort.
(C) H e went to all the lectures but one. (B) They have n o other work to do.
(D) H e missed m ore than one (C) Its im possible to finish.
psychology class. (D) They arent even close to finishing
4. (A) T hey passed the library at the project.
6 :00 . 9. (A) She d oesn t m ind an hou r m ore.
(B) T h e library opens at 6:00 in the (B) S h ed rather stay m ore than an
summer. hour.
(C) T h e library closes at 6:00. (C) Its better to stay than go.
(D) You cant check out m ore than six (D) She prefers to leave.
books in the summer.
10. (A) T he service at the hotel wasnt too
5. (A) Water the plants on ce a day. good.
(B) Give the plants no m ore water. (B) This hotel gav< excellen t service.
(C) Water the plants often while the (C) T he service at the h otel could have
m an is gone. b een improved.
(D) Give the plants a lim ited am ount o f (D) This h o tels service was the sam e as
water. the service at other hotels.
LISTENING COMPREHENSION

S k i l l 8: LISTE N FOR DOUBLE N EG A TIVE EXPRESSIONS

It is p o s sib le fo r tw o n e g a tiv e id e a s to a p p ea r in o n e s e n te n c e , an d th e r esu lt can b e q u ite


c o n fu s in g .

E x am p le

On the recording, you hear:

(man) I c a n t believe the news that I heard about the concert.


(woman) Well, it isn't impossible for the concert to take place.
(narrator) What does the wom an say about the concert?

In your test book, you read:

(A) Theres no possibility that the concert will take place.


(B) The concert will definitely not take place.
(C) The concert might take place.
(D) The concert can't take place.

T h e c o r r e c t an sw er to th is q u e s tio n is answ er (C ). I f it i s n t impossible fo r th e c o n c e r t to


take p la c e , th e n it is possible, a n d th e m o d a l m ight in d ic a te s possibility.

T h e fo llo w in g ch art o u tlin e s th e situ a tio n s w h e r e d o u b le n eg a tiv es c a n occu r:

DOUBLE NEGATIVES

Situation Example Meaning

negative word (e.g., not, no, none) and He didnt like the unclean did not like unclean office
a negative prefix (e.g., in-, un-, dis-) office. = liked clean office

two negative verbs It isn't snowing, so they aren't implies that they would go
going to the mountains. if it were snowing

neither or not... either Sue didnt like the movie, and both did not like the movie
neither did Mark.

E X E R C IS E 8: In this e x e r c ise , u n d e r lin e th e tw o n e g a tiv es in th e se c o n d lin e o f each


sh o r t c o n v e r s a tio n . T h e n read th e q u e s tio n a n d c h o o s e th e b est answ er to that q u estio n .
R e m e m b e r th a t tw o n e g a tiv e s m ak e th e s e n te n c e p o sitiv e.

(m an) Paula, you worked so hard (A) She hopes everyone will be
setting up the field trip. pleased.
(woman) I hope no ones unhappy with (B) She knows no on e is happy with
the arrangements. what she has done.
(narrator) What does Paula mean ? (C) Sh es arranged to take a trip
because shes unhappy.
(D) Everyones happy with the
condition o f the field.
LISTENING PARTA

2. (wom an) How was your history exam? (A) H e studied a lot and passed.
(m an) I d id n t study enough, so I (B) H e failed in spite o f his effort.
didn t do well. (C) H e got a g ood grade even though
(narrator) What does the man mean? he d id n t study.
(D) His grade was poor because o f
inadequate preparation.

3. (m an) Were your friends able to gel (A) A lthough Mark c o u ld n t get both
tickets fo r the concert ? tickets, Paul did.
(wom an) Mark couldn't gel tickets fo r the (B) Both were unable to obtain tickets.
concert, and neither could Paul. (C) N either Mark nor Paul wanted to
(narrator) What does the woman mean? go to the concert.
(D) Mark tried to get tickets, but Paul
didnt.

T O E F L E X E R C IS E 8: In th is e x e r c ise , lis te n carefu lly to th e sh o r t c o n v e r sa tio n a n d q u e s


tio n in th e r e c o r d in g p ro g ra m , a n d th e n c h o o s e th e b e st answ er to th e q u e s tio n . You
s h o u ld b e p articu larly c a refu l o f d o u b le n e g a tiv es.

N O W BEGIN THE RECORDING PROGRAM AT TOEFL EXERCISE 8.


n
1. (A) H e ll definitely be elected. 6. (A) Steve wanted to finish his paper,
(B) T he election is now com plete. and so did Paul.
(C) She has high hop es for his chances. (B) Both Steves and Pauls papers were
(D) It may happen. incom plete.
2. (A) Both parts o f his game were bad. (C) Steve and Paul were busy doin g
(B) H e served better than he volleyed. their term papers.
(C) Som e parts o f his gam e were better (D) W hen Steve wasnt able to finish his
than others. paper, Paul c o u ld n t help.
(D) He played rather well. 7. (A) It wasnt G eorges responsibility to
3. (A) It is a surprise that he was pay the bill.
prepared. (B) Bill was irresponsible about paying
(B) H e was not ready, as usual. G eorges rent.
(C) H e prepared a really big surprise. (C) G eorge acted carelessly by not
(D) His strong preparation cam e as no taking care o f the bill.
surprise. (D) George took responsibility for the
unpaid bill.
4. (A) She felt good enough to go out.
(B) She w ent out to get som e m edicine. 8. (A) Its fortunate that he was accepted.
(C) She felt like dancing, so she went (B) Its good that he wasnt adm itted.
out with everyone. (C) Fortunately, the university d id n t
(D) admit him.
She stayed hom e because she was
sick. (D) Its too bad he was rejected.

5. (A) She has problem s that others arent


aware of.
(B) O thers arent aware o f her
problem s.
(C) She knows sh es b een a problem .
(D) She d o e sn t have a care in the
world.
LISTENING COMPREHENSION

9. (A) T h e first essay was better than the 10. (A) Roger has been bothered.
second. (B) Roger wasnt the least bit
(B) T h e first and second drafts disturbed.
co u ld n t b e better. (C) T he problem s have had little i
(C) T h e second draft o f the essay was on Roger.
m uch better than the first. (D) Roger hasnt b een disturbed.
(D) Both versions were poorly written.

S k i l l 9: E XPRESSIONS
LISTE N FOR A L M O S T N E G A TIV E 1

C e rta in e x p r e s sio n s in E n g lish have a lm o st n e g a tiv e m e a n in g s. T h e s e e x p r e s sio n s are


c o m m o n o n th e T O E F L te st a n d n e e d to b e r ev iew ed .

Exam ple

On the recording, you hear:


(woman) Were you able to pay the electric bill?
(man) I had barely enough money.
(narrator) What does the m an imply?

In your test book, you read:


(A) He had plenty of money for the bill.
(B) He did not have enough money for the bill.
(C) He paid the bill but has no money left.
(D) He was unable to pay the bill.

In th e m a n s sta te m e n t, th e w ord enough in d ic a te s th a t th e r e was enough, so h e d id pay


the bill. H ow ever, it was barely e n o u g h , so h e a lm o st d id n o t have e n o u g h a n d certain ly
has no money left. T h e c o r r e c t answ er is th e r e fo r e a n sw er (C ) .

T h e fo llo w in g c h a r t o u tlin e s c o m m o n a lm o st n e g a tiv e ex p r essio n s:

COMMON "ALMOST NEGATIVE EXPRESSIONS

Meaning Expression Example

almost none hardly, barely, scarcely, only There is hardly any food in the refrigerator.

almost never rarely, seldom He rarely drives to work.


E X E R C ISE 9: In th is e x e r c ise , u n d e r lin e th e a lm o st n e g a tiv e e x p r e ssio n in th e se c o n d


lin e o f e a c h sh o r t c o n v e rsa tio n . T h e n read th e q u e s tio n a n d c h o o s e th e b e st answer. R e
m e m b e r th a t th e b e s t answ er is o n e th a t m e a n s th a t it is true b u t it is almost not tru e.

(m an) I hear that M onas been offered (A) M ona hasnt worked hard.
the managers job. (B) M onas experience has been hard.
(woman) B ut she has hardly any ivork (C) M onas job as manager is hard.
experience! (D) M ona hasnt worked for very long.
(narrator) What does the woman say about
M ona?
LISTENING PARTA

2. (w o m a n ) How much time did Sam spend (A) Sam usually spends this m uch tim e
on his paper for economics on his schoolwork.
class? (B) Sam has rarely worked so hard.
(m a n ) Sam has seldom taken so much (C) Sam took too m uch tim e on this
time on a research paper. paper.
(n a rra to r) What does the m an mean ? (D) Sam sh ouldve worked harder o n
this paper.

3. (woman) Does Steve usually park his car (A) H e parks his car there o n ce in a
there? while.
(man) Only once has he parked his car (B) H es parked his car there a lot.
in that lot. (C) H e only leaves his car there for
(narrator) What does the m an mean ? short periods o f time.
(D) H e left his car there on ju st one

T O E F L E X E R C ISE 9: In th is e x e r c is e , lis te n ca refu lly to th e sh o r t c o n v e r s a tio n a n d q u e s


tio n in th e r e c o r d in g p r o g r a m , a n d t h e n c h o o s e th e b e st an sw er to th e q u e s tio n . Y ou
sh o u ld b e p articu larly c a refu l o f a lm o st n eg a tiv e" e x p r e ssio n s.

N O W BEGIN THE RECORDING PROGRAM AT TOEFL EXERCISE 9.

1. (A) T h eres litde rain in July. 5. (A) Betty o ften tr.kes vacations in
(B) In July it never rains. winter.
(C) It rains hard in July. <B) Betty prefers to take vacations in
(D) W hen it rains in July, it rains hard. winter.
(C) Occasionally Betty works o n e w eek
2. (A) T h e university accepted three
during vacation.
students.
(D) A w inter vacation is unusual for
(B) N on e o f the students is goin g to
Betty.
the university.
(C) John was n ot accepted. 6. (A) H e rarely spends time o n his
(D ) Two were n ot adm itted. courses.
(B) H es an excellen t student.
3. (A) A lthough he did pass, Marks exam
(C) H e never studies.
grade wasnt too good .
(D) His books are always open.
(B) Mark failed his history exam.
(C) T he highest grade on the history 7- (A) H e finished the exam in plenty o f
exam w ent to Mark. time.
(D) Professor Franks d id n't pass Mark (B) H e was scared h e w ou ld nt finish.
on the history exam. (C) H used every possible m inute to
finish.
4. (A) H e often has lon g waits in Dr.
(D) H e was unable to com plete the
Roberts office.
exam .
(B ) He must wait p auendy for Robert.
(C) Dr. Roberts is generally punctual. 8. (A) This was a very long staff m eeting.
(D) He d o e sn t m ind waiting for Dr. (B) This was the only staff m eeting in a
Roberts. long time.
(C) T he m eeting lasted only until o n e
o'clock.
(D) T he o n e staff m eeting s h o u ld ve
lasted longer.
LISTENING COMPREHENSION

9. (A) M eat tastes delicious to him when 10. (A) H e broke his arm trying to move it.
its cooked rare. (B) He only hurt the broken arm.
(B) H e isnt sure if the m eal is (C) H e only tries to move the broken
delicious. arm.
(C) This m eat is the best h e s tasted in (D) T heres no pain if he rests quietly.
a long lime.
(D) H e d like to eat som e m eat from
this delicatessen.

S k ill 10: LISTEN FOR NEG ATIVES W IT H COMPARATIVES

N e g a tiv e s ca n b e u s e d w ith c o m p a r a tiv e s in L iste n in g Part A o f th e T O E F L test. A s e n


te n c e w ith a n e g a tiv e a n d a co m p a r a tiv e has a su p e rla tiv e, o r v ery stro n g , m e a n in g .

Exam ple

On the recording, you hear:


(woman) W hat do you think o f the new student in math class?
(man) No one is more intelligent than she is.
(narrator) What does the m an say about the new student?

In your test book, you read:


(A) She is not very smart.
(B) He is smarter than she is.
(C) Other students are smarter than she is.
(D) She is the smartest student in the class.

T h e m an r e s p o n d s to th e w o m a n s q u e s tio n w ith th e n e g a tiv e no a n d th e c om p arative


more intelligent, a n d th is c o m b in a tio n h a s a su p e rla tiv e m e a n in g . T h e b e st answ er is th e r e
fo r e answ er (D ).

T h e fo llo w in g c h a r t o u tlin e s c o m p a r is o n s th a t y o u sh o u ld b e c a refu l o f w h e n th ey are


u s e d w ith n egatives:

- , V COMPARATIVES WITH NEGATIVES

Comparative Example ;, SF^.' Meaning

more No one Is more beautiful than she is. She is the most beautiful.
..... " / . V ;; " '
He couldnt be happier. He is extremely happy.

LISTENING PARTA

EX E R C ISE 10: In this e x e r c ise , u n d e r lin e th e n e g a tiv e a n d th e co m p a r a tiv e in th e s e c


o n d lin e o f e a c h sh o r t c o n v e r sa tio n . T h e n read th e q u e s tio n a n d c h o o s e th e b e s t an sw er
to th a t q u e s tio n . R e m e m b e r th a t th e b e s t an sw er is o n e th at e x p r e s se s a su p e rla tiv e, o r
very str o n g , id e a .

1. (woman) Have you gotten over your cold (A) H es feelin g terrific.
yet? (B) He felt a lot worse today.
(man) I couldn't befeeling any better (C) H e's not feelin g to o well today.
today. (D) H e s a bit better today.
(narrator) What does the man mean ?

2. (woman) . What did you think o f Mike (A) Mike was extrem ely friendly when
when you first met him ? he m et him.
(man) He couldnt have been more (B) Mike could have m et him sooner.
unfriendly. (C) Mike d id n t seem to like him at all.
(narrator) What does the man mean ? (D) W hen he m et Mike, he did n't have
a friend.

( man) Did you see Theresa's grade on (A) Theresa couldve gotten a higher
the math exam ? It was grade.
unbelievable! (B) Anyone could get a good grade.
(woman) No one else could have done (C) Theresa got the high est grade.
better. (D) A high grade is im possible for
(narrator) What does the woman mean? anyone.

T O E F L E X E R C ISE 10: In th is e x e r c ise , liste n ca refu lly to th e sh o r t c o n v e r s a tio n a n d


q u e stio n in th e r e c o r d in g p ro g ra m , a n d th e n c h o o s e th e b e st an sw er to th e q u e s tio n . You
sh o u ld b e p articu larly c a refu l o f c o m p a r a tiv e s w ith n eg a tiv es.

N O W BEGIN THE RECORDING PROGRAM AT TOEFL EXERCISE 10.

(A) Shes n ot very happy. 5. (A) T he patient absolutely d id n t need


(B) She d id n t do very well on the the surgery.
exam. (B) T he necessity for the surgery was
(C) She could be som ewhat happier. unquestionable.
(D) Shes delighted with the results. (C) T he surgeon felt that the operation
was necessary.
(A) Paula is always lazy.
(D) It was essential that the surgery be
(B) Paula did nt work very hard this
perform ed immediately.
semester.
(C) Paula m ade a strong effort. 6. (A) They were not very lucky.
(D) Paula could have worked harder. (B) N o on e was hurt.
(C) The accident was unfortunate.
(A) T he prices were great!
(D) She w anted to have better luck.
(B) T he prices were too high.
(C) She d id n t buy m uch because o f 7. (A) N othing was very difficult.
the prices. (B) T he exam wasnt at all easy.
(D) T he prices could have b een lower. (C) T he exam co u ld n t have been
easier.
(A) She is not very sm art (D) T h e exam had noth in g difficult on it.
(B) She always tells him everything.
(C) He d o e sn t know her very well.
(>) Shes extrem ely intelligent.
LISTENING COMPREHENSION

8. (A) She wants that jo b very m uch. 10. (A) This math project was extrem ely
(B) N o o n e is goin g to get the job . com plex.
(C) Everybody else wants that jo b as (B) This math project was less
m uch as she does. com plicated than the last
(D) She is n ot sure about taking the (C) They seldom com plete their math
job . projects.
(D) Com plicated m ath projects are
9. (A) She was second in the race.
often assigned.
(B) She was alm ost the slowest person
in the race.
(C) She w on the race.
(D) She was n ot faster than anyone
else.

T O E F L EX ER C ISE (Sk ills 7 - 1 0 ) : In this e x e r c ise , liste n carefu lly to th e sh o rt c on versation


a n d q u e stio n in th e r ec o r d in g p rogram , a n d th e n c h o o s e th e b e st answ er to th e q u estio n .

N O W BEGIN THE RECORDING PROGRAM A T T O ER EXERCISE (S k IILS 7-10).

1. (A) She can try a litde harder. 5. (A) She handed the paper in on time.
(B) T here is a lot m ore that she can do. (B) She was able to com plete the
(C) S h es d oin g the best that she can. paper, but she d id n t turn it in.
(D) It is im possible for her to do (C) T he paper was a com plete mess, so
anything. she didnt turn it in.
(D) T he paper was unfinished.
2. (A) S h es always been late for the bus.
(B) T h e bus has always b een late. 6. (A) N either Tim nor Sylvia is taking
(C) T h e bus only left on time once. care o f Art.
(D) Only on this trip has the bus been (B) Sylvia likes m odern art even less
on time. than Tim does.
(C) Sylvia d oesn t care for anything
3. (A) T here wasnt enough soup to go
Tim does.
around.
(D) Sylvia and Tim agree in their
(B) We had so m uch soup that we
opin ion o f m odern art.
co u ld n t finish it.
(C) Everyone g ot one serving o f soup, 7. (A) They always work hard in the
but there wasn't enough for afternoon.
seconds. (B) They d o n t d o m uch after lunch.
(D) Everyone around the table had a (C) After noon they never work.
lot o f soup. (D) Its never hard for them to work in
the afternoon.
4. (A) She does want to see the movie.
(B) Its extrem ely im portant to her to 8. (A) It's hard for him to work when it
g- gets warm.
(C) She d oesn t want to go there (B) W henever it gets warm, he turns on
anymore. the air conditioner.
(D) She really cou ld n t m ove there. (C) T he air conditioner only works
when it isnt n eeded.
(D) H e likes to use the air conditioner
when it is warm.
LISTENING PARTA

9. (A) H e did really poorly. 10. (A) With so many m em bers present,
(B) H e s felt worse before. the com m ittee co u ld n t reach a
(C) The results could not have b een decision.
better. (B) The com m ittee sh ouldve waited
(D) H e s not too unhappy with the until more m em bers were
results. present.
(C) The issue sh ou ld n t have been
decid ed by all the com m itted
members.
(D) The issue wasnt d ecid ed because
so many m em bers were absent.

TO EFL REVIEW EXERCISE (Skills 1 -1 0 ): In this exercise, listen carefully to the sh ort conver
sation and q u estion in th e record in g program , an d then c h o o se th e best answer to th e q u estion .

C ) N ow BEGIN THE RECORDING PROGRAM AT TOEFL REVIEW EXERCISE (SKILLS I 10).

1. (A In a doctors office. 7. (A) The waitress was sitting in the back


(B At a bar. o f the restaurant.
(C In a travel agency. (B) They were waiting for a seat in the
(D In a business office. restaurant.
(C) The custom ers had a table in the
2. (A She bou ght som e sheets.
back.
(B She g o t a new piece o f clothing.
(D) The waitress sat down b ehin d the
(C She c o u ld n t find anything because
table.
sh es too short.
(D She was sure to greet her boss. 8. (A) Its hard for the market to sell its
fruit.
3. (A The hotel was all right, excep t for
(B) All o f the fresh fruit at the market
the poor view.
is hard.
(B T he view from the hotel room was
(C) She hardly ever g oes to the m arket
spectacular.
to buy fresh fruit.
(C She would have preferred a better
(D) There was a scarcity o f fresh fruit at
hotel.
the market.
(D O nly a few hotels would have been
better. 9. (A) The man sh ould never be late for
school.
4. (A Take a nap.
(B) The man can always return to
(B Try the rest o f the work.
school.
(C See a doctor.
(C) The man should never go back to
(D Have a bite to eat.
school.
5. (A Shes an exacting person. (D) If the m ans late to school, he
(B She cant be expected to give you should go through the back
four o f them. door.
(C She generally forgives others.
10. (A) She cant bear to try.
(D She isn t exact about what she gives
(B) She is a daring person.
to others.
(C) She d o esn t want the man even to
6. (A She's unable to take her vacation try.
this year. (D) She is challenging the m an to make
(B H er vacation next w eek has b een the effort.
postponed.
(C Shell go on vacation next week.
(D She'll return from vacation in a
week.
LISTENING COMPREHENSION

F U N C T IO N S _______________________________________________

S k ill I I : LISTEN FOR EXPRESSIONS OF A G REEM ENT

E x p r e ssio n s o f a g r e e m e n t are c o m m o n in L iste n in g Part A , so y o u s h o u ld b e c o m e fam il


iar w ith th e m . T h e fo llo w in g e x a m p le sh ow s a g r e e m e n t w ith a p o sitive sta te m en t.

E xam ple

On the recording, you hear:


(man) I think that the hypothesis is indefensible.
(woman) So do I.
(narrator) What does the w om an mean?

In your test book, you read:


(A) She is unsure about the hypothesis.
(B) The hippopotamus is behind the fence.
(C) She thinks that the hypothesis can be defended.
(D) She agrees with the man.

' T h e e x p r e ssio n so do 1 is an e x p r e ssio n th a t sh ow s a g r e e m e n t w ith a p o sitiv e sta te m en t, so


th e w o m a n m e a n s th at sh e agrees w ith th e m a n . T h e b e st answ er is th e r e fo r e answ er (D ).
O th e r e x p r e ssio n s are u se d to sh ow a g r e e m e n t w ith n e g a tiv e sta te m en ts.

E xam ple

On the recording, you hear


(woman) I d o n t think that our history teacher is very interesting.
(man) Neither do I.
(narrator) What does the m an mean?

In your test book, you read:


(A) He disagrees with the woman.
(B) He thinks the history teacher is interesting.
(C) He shares the woman's opinion.
(D) He doesnt think the w om ans idea is good.

T h e e x p r e s sio n neither do I is an e x p r e s sio n th a t sh ow s a g r e e m e n t w ith a n e g a tiv e state


m e n t, so th e m an agrees w ith th e w o m a n . T h e b e s t answ er is th e r e fo r e an sw er (C ).

T h e fo llo w in g ch art lists c o m m o n e x p r e s s io n s th at sh o w a g r e e m e n t. You sh o u ld b e


c o m e fam iliar w ith th e se ex p ressio n s:
LISTENING PARTA

EXPRESSIONS OF AGREEMENT
'
Agreement with Positive Statements Agreement with Negative Statements

So do/. Neither do 1.
Me. too. 1don't either.
I'll say! - v y .-;.- ,1:.--;- . V " a
Isn't id -:;y
You can say that again! .

E X ER C ISE 11: In this e x e r c ise , u n d e r lin e th e e x p r e s sio n o f a g r e e m e n t in e a c h sh o r t


c o n v e rsa tio n . T h e n read th e q u e s tio n a n d c h o o s e th e b e s t an sw er to th a t q u e s tio n .
R e m em b er th at th e b e st an sw er is o n e th a t sh ow s a g r e e m e n t.

(woman) These paintings are really (A) T hese paintings arent very
fascinating! interesting.
(man) Aren '1 they! (B) H e isnt fascinated by these
(narrator) What does the man mean ? paintings.
(C) H e isnt sure how he feels.
(D) H e finds these paintings quite
interesting.

(woman) I don't really carefo r the way the (A) H e thinks the building was n ot
building was renovated. renovated.
(man) I don '1 either. (B) H e has the same op in ion o f the
(narrator) What does the man mean ? building as the woman.
(C) H e d o e sn t care about the
renovation o f the building.
(D) H e suggests being careful in the
renovated building.

(man) I think that both candidates for (A) She agrees with the man.
county supervisor are (B) She thinks h e should b ecom e
unqualified. county supervisor.
(woman) Me, too. (C) She thinks the candidates are
(narrator) What does the woman mean ? qualified.
(D) She has no op in ion about the
candidates for county supervisor.

T O E F L E X E R C ISE 11: In th is e x e r c ise , listen c a re fu lly to th e sh o r t c o n v e r s a tio n a n d


q u e stio n in th e r e c o r d in g p r o g r a m , a n d th e n c h o o s e th e b e s t an sw er to th e q u e s tio n . You
sh o u ld pay a tte n tio n to e x p r e s sio n s o f a g r e e m e n t.

n N O W BEGIN THE RECORDING PROGRAM AT TOEFL EXERCISE I I.

(A) T he trip would cost too m uch. 2. (A) H e w ould like to see the elections
(B) She d o esn t think that a trip would for town council.
be a good idea. (B) H e agrees that Matt sh ould be
(C) She would like to take two trips elected.
rather than one. (C) H e thinks the election s sh ould take
(D) She would also like to take a trip. place next m onth.
(D) H e disagrees with the wom an.
USTENING COMPREHENSION

3. (A) She is not sure which course she 7. (A) H e thinks the p am es arent oud.
should take. (B) H e says that the neighbors d o n t
(B) Sh es not sure if she should take a have many parties.
trip to France. (C) H e agrees that the upstairs
(C) She knows that she is not ready for neighbors are noisy.
interm ediate French. (D) T he loud parties d o n t bother him.
(D) She wants to take neither
8. (A) She doesnt like this m eal too much.
beginn in g nor interm ediate
(B) This food tastes wonderful to her.
French.
(C) Shes not sure if she likes it.
4. (A) T he man should repeat what he (D) She cant stand this meal.
said.
9. (A) She agrees that gettin g the car was
(B) The man said som ething foolish.
not a good idea.
(C) She thinks that the food is the best
(B) She im agines that she would like to
she has ever tasted.
have a similar car.
(D) She agrees that the food is pretty
(C) She thinks that the man is mistaken
bad.
about the car.
5. (A) This party hasnt b een any fun at all. (D) She thinks the man has no
(B) H e wonders if the woman enjoyed im agination.
herself.
10. (A) H e would like the woman to repeat
(C) H e wants to know what she said.
what she said.
(D) H e s enjoyed him self
(B) He thinks that on e sem ester is
tremendously.
enough time for the course.
6. (A) She condon es what happened. (C) He also thinks that the course
(B) She does not like what the m an said. should be extended.
(C) She agrees with the man about (D) H e would like to take the course
what happened. two semesters from now.
(D) She says that she did not d o it.

S k i l l 12: LISTEN FOR EXPRESSIONS OF U N C E R T A IN TY


A N D S U G G E S TIO N

E x p r e ssio n s o f u n c e r ta in ty an d s u g g e s tio n are c o m m o n in L iste n in g P art A , so y o u sh o u ld


b e c o m e fam iliar w ith th e m . T h e fo llo w in g e x a m p le sh ow s an e x p r e s sio n o f un certain ty.

E xam ple

On the recording, you hear:


(man) Do you know anything about the final exam in
Physics?
(woman) It's going to be rather difficult, isn't it?
(narrator) W hat does the woman mean?

In your test book, you read:


(A) The exam is not going to be too difficult.
(B) Shes positive that its going to be hard.
(C) She thinks that it might be hard.
(D) She has no idea about the exam.
LISTENING PARTA

T h e tag q u e s tio n is n 't it c h a n g e s a d e fin ite s ta te m e n t in to a s ta te m e n t th a t sh o w s u n c e r


tainty, so th e b e st an sw er is o n e th a t e x p r e s se s un certain ty. T h e b e st an sw er to th is q u e s
tio n is an sw er (C ) b e c a u se th e w o rd s th in k s a n d m ight e x p r ess un certain ty.
O th e r e x p r e s sio n s th at are c o m m o n in L iste n in g Part A are e x p r e s sio n s o f s u g g e s tio n .

E xam ple

On the recording, you hear:

(man) I'll never have time to type m y paper tomorrow.


(woman) Why not do it now?
(narrator) W hat does the wom an suggest?

In your test book, you read:

(A) Finishing the paper today.


(B) Not working on the paper now.
(C) Never typing the paper.
(D) Taking time out from the paper now.

In this e x a m p le , th e e x p r e s sio n why not is an e x p r e s sio n o f s u g g e s tio n , so th e w o m a n su g


g e sts doin g it now. In th is s u g g e s tio n , th e w o m a n is r e fe r r in g to th e p a p e r th a t th e m an
n e e d s to type, so th e b e st an sw er is an sw er (A ).

T h e fo llo w in g c h a r t lists c o m m o n e x p r e s sio n s th a t sh o w u n ce r ta in ty a n d su g g estio n :-

EXPRESSIONS OF UNCERTAINTY AND SUGGESTION

Uncertainty c .
Suggestion

...isn 't it ( tag)? Why not. ..?


As far as 1know. Let's
.
As far as 1can tell.

EX E R C ISE 12: In th is e x e r c ise , u n d e r lin e th e e x p r e s sio n o f u n ce r ta in ty o r s u g g e s tio n in


e a c h sh o r t c o n v e rsa tio n . T h e n r ea d th e q u e s tio n a n d c h o o s e th e b e s t a n sw er to th a t q u e s
tio n . R e m e m b e r th a t th e b e st a n sw er is o n e th at sh ow s u n ce r ta in ty o r s u g g e s tio n .

(man) Do you know what time they 're (A) She's not com pletely sure w hen
leavingfor the city? they are leaving.
(woman) They have to leave at fo u r (B) They are returning from the city at
o clock, don't they ? about 4:00.
(narrator) What does the woman mean ? (C) She knows when they are leaving.
(D) She d oesn t have any idea w hen
they are leaving.

(woman) I m so thirsty from all this (A) They should stop drinking.
walking. (B) They should go for a walk.
(man) Let's stop and get a drink. (C) They should walk thirty m iles.
(narrator) What does the man suggest ? (D) They should take a break a nd have
a drink.
LISTENING COMPREHENSION

3. (m an) Is the exam still scheduled for (A) The exam is far away.
3:00 on Thursday ? (B) She knows that the exam schedule
(wom an) As fa r as I know. has b een changed.
(narrator) What does the woman mean f (C) She is sure that the exam is set for
Thursday.
(D) She thinks she knows w hen the
test is.

T O E F L E X ER C ISE 12: In th is e x e r c ise , liste n ca refu lly to th e sh o r t c o n v e rsa tio n a n d


q u e s tio n ir. th e r e c o r d in g p rogram , a n d th e n c h o o s e th e b e st answ er to th e q u e s tio n . You
s h o u ld b e p articu larly carefu l o f e x p r e ssio n s o f u n ce r ta in ty a n d su g g e stio n .

N O W BEGIN THE RECORDING PROGRAM AT TOEFL EXERCISE I 2.


n
1. (A) H e s sure about which chapters 5. (A) Shes told Matt h e ll go far.
they are to read. (B) Matt has far from enough talent.
(B) H e thinks he knows what the (C) She told Matt to roll farther.
assignm ent is. (D) She believes Matt has the ability for
(C) He has to tell her how far she the part.
should go.
6. (A) They should go to the hospital.
(D) T he professor told them to read
(B) Mary should visit the man.
the chapters after the exam.
(C) T he woman should try n ot to break
2. (A) The man should take the pie out. her leg.
(B) The man should try som ething (D) They should go on a trip with Mary.
else. (A) She knows where the children are.
(C) The man sh ouldnt try cherry pie.
(B) The children have finished playing
(D) T he man should feel sorry.
ball.
3. (A) H e knows the movie starts at 8:00. (C) Shes going to the park to find the
(B) H e is n ot quite sure when the children.
m ovie begins. (D) She believes that the children are
(C) H e thinks the start o f the movie has in the park.
been changed. (A) The man should try to borrow
8.
(D) H e will start the movie him self at
som e from a neighbor.
8 : 00 .
(B) The man should take a check to
4. (A) N ot d oin g the dishes now. Tom.
(B) Leaving the house with the, dishes. (C) T he man should work on his math
(C) Leaving later so that they can do assignm ent with Tom.
the dishes now. (D) The man should check behind the
(D) Washing the dishes before they door.
leave.
LISTENING PARTA

9. (A) H e thinks the bill is due in the 10. (A) They should p ostp on e their
m iddle o f the m onth. decision until m orning.
(B) T he bill is approxim ately fifteen (B) They should g o to sleep in the new
dollars. house.
(C) H e knows w hen they should pay (C) They should n o t buy such a big
the bill. house.
(D) T he bill is goin g to be fifteen days (D) They should d ecid e w here to go to
late. sleep.

S k i l l 13: LISTEN FOR E M P H A TIC EXPRESSIONS OF SURPRISE

E m p h a tic e x p r e s sio n s o f su rp rise are c o m m o n in L iste n in g P art A , so y o u s h o u ld b e c o m e


fam iliar w ith th e m . W h e n su rp rise is e x p r e s se d , it im p lie s th a t th e sp e a k e r d id n o t e x p e c t
s o m e th in g to b e tru e.

E xam ple

On the recording, you hear:


(woman) Did you see Paul driving around in his M ustang?
(man) Then, he DID get a new car.
(narrator) What had the m an thought?

In your test book, you read:


(A) Paul would definitely get a Mustang.
(B) Paul did not know how to drive.
(C) Paul did not like Mustangs.
(D) Paul would not get a new car.

In th is co n v e rsa tio n th e e m p h a tic fo r m he d id get is u se d to sh o w th e m a n s su rp rise th a t


P au l g o t a n ew car. It m e a n s th a t th e m an e x p e c te d th a t P aul w ould not get a n e w car, so th e
b e st answ er is an sw er ( D ) .

T h e fo llo w in g c h a r t o u tlin e s v a rio u s w ays to e x p r e ss e m p h a tic su rp rise:

... v7-V-' .;- EXPRESSIONS OF EMPHATIC SURPRISE

Verb Emphatic Form Example Meaning

be be, with emphasis Then, he is here! 1 thought he was not here.


modal modal, with emphasis Then, you can go! 1thought you could not go.
present tense do(es), with emphasis Then, you do play tennis! 1thought you did not play
- ' ... T y - > ;' tennis.
past tense did, with emphasis Then, she did read it. 1 thought she had not read it.
perfect tense have, with emphasis Then, he has gone there. 1thought he had not gone there.
...........
LISTENING COMPREHENSION

E X E R C ISE 13: In th is e x e r c ise , u n d e r lin e th e e x p r e s sio n o f e m p h a tic su rp rise in e a c h


sh o r t c o n v e r s a tio n . T h e n read th e q u e s tio n a n d c h o o s e th e b e st answ er to th a t q u e stio n .
R e m e m b e r th a t th e b e st answ er is o n e th a t sh ow s su rp rise.

(man) I ju st got 600 on the TOEFL (A) The man had n ot passed.
test! (B) T he man would pass easily.
(wom an) Then you did pass. (C) The man had already passed.
(narrator) What had the woman assumed? (D) The man got the score he was
expected to get.

2. (wom an) Would you like to go skiing this (A) The woman was a good skier.
weekend ? (B) T he woman was going skiing this
(man) So you can ski! weekend.
(narrator) What had the man assumed? (C) The woman did n ot know how to
ski.
(D) T he woman did n ot intend to go
skiing.

(man) I ju st got this letter from my (A) The m ans sister never wrote to
sister. him.
(wom an) So the mail has come already. (B) T he mail had not yet arrived.
(narrator) What had the woman assumed? (C) The mail always cam e early.
(D) The mail had already arrived.

T O E F L E X ER C ISE 13: In th is e x e r c ise , listen carefu lly to th e sh o rt c o n v e rsa tio n a n d


q u e s tio n in th e r e c o r d in g p rogram , an d th e n c h o o s e th e b e st answ er to th e q u e s tio n . You
sh o u ld b e p articu larly carefu l o f e x p r e s sio n s o f e m p h a tic su rp rise.

o N O W BEGIN THE RECORDING PROGRAM AT TOEFL EXERCISE I 3.

(A) Greg always com es to parties. 4. (A) The woman had run more than
(B) Greg would com e to the party later. three miles.
(C) Greg was unable to attend the (B) T he woman always got lots o f
party. exercise.
(D) Greg would stay at the party for (C) The woman ran for three hours in
only a m om ent. the morning.
(D) The woman had not gotten much
(A) T he woman always rode her
exercise.
motorcycle to school.
(B) The woman was not com ing to 5. (A) H e had been som ewhere else.
school today. (B) He had been in the library.
(C) T he woman was an expert (C) H e had been working on his
m otorcycle rider. research project.
(D) T he w oman did not know how to (D) H e would start working on his
ride a motorcycle. project in five hours.

(A) T he man was n ot a very good cook. 6. (A) He had changed apartments.
(B) The man never invited friends over (B) He did not like his new apartment.
for dinner. (C) H e was still in his old apartment.
(C) T he man would never invite him (D) H e had moved from a house to an
over for dinner. apartm ent
(D) T he man was an excellen t cook.
LISTENING PARTA

(A) T he wom an did not like desserts. 9. (A) She had registered in physics.
(B) T he woman ate sweets regularly. (B) She would go to physics class later.
(C) T he w om an would n ot share her (C) She had already taken a physics
chocolate cake. class.
(D) The woman had eaten his p iece o f (D) She had not enrolled in physics.
cake.
10. (A) T he pipes were n ot clear.
(A) T he man was goin g to study hard. (B) The plum ber w ould be late.
(B) T h e man already had a drivers (C) T he plum ber had already cleared
license. the pipes.
(C) T he man would n ot take the test. (D) T he pipes did n o t n eed to be
(D) The man had already taken the test cleared.

T O E F L EXERCISE (Sk ills 1 1 -1 3 ): In th is ex er cise , listen carefully to th e sh o rt co n v e rsa tio n


a n d q u estio n in th e r ec o r d in g p rogram , a n d th e n c h o o s e th e b est answ er to th e q u estio n .

N O W BEGIN THE RECORDING PROGRAM AT TOEFL EXERCISE (SKILLS I 1-13).


n
(A) She plans to talk a lot this m onth. 6. (A) He would be glad to say it over again.
(B) She has a lo t to say about the (B) H e would like the woman to repeat
p h on e bill. what she said.
(C) T he bill is high because she has a (C) H e says that h e w ould like to take
lot to say. the class again.
(D) She agrees with the m an. (D) H e s happy the class is over, too.
(A) Bill had never really b een sick. 7. (A) H e finifned all the problem s.
(B) Bill was too sick to com e to class. (B) H e d o esn t believe what the wom an
(C) Bill was sick o f calculus class. said.
(D) Bill had forgotten about the (C) H e was able to finish som e o f the
calculus class that m orning. problem s.
3. Both he and the w oman were
(A) T he man sh ould go o u t tonight.
unsuccessful o n the math
(B) T he man sh ould stay h om e and
problem s.
relax.
(C) T h e man should work on the paper 8. (A) T he man had m ailed the package.
tonight. (B) T he man had forgotten to go to the
(D) T h e man should go ou t M onday post office.
instead. (C) T he m an had given the package to
4. (A) T he cafeteria was op en in the the wom an to mail.
m o r n in g . '
T he m an rem em bered the package
(B) T h e cafeteria did not serve after he went to the p ost office.
breakfast. 9. (A) They should take both cars.
(C) T h e breakfast in the cafeteria was (B) T he woman sh ould try n o t to be
n ot very tasty. afraid.
(D) T he woman never ate breakfast in (C) The woman should buy a bigger car.
the cafeteria. (D) They should go together in his car.

5. (A) H e believes that it is acceptable to 10. (A) H e wants to know if the m uffins
park there. taste good.
(B) T h e parking lot is too far from (B) H e thinks the m uffins were
their destination. recently prepared.
(C) H e knows that they w on t g et a (C) T he m uffins are not really fresh.
ticket. (D) H es sure that the m uffins were ju st
(D) H e knows w here the parking lo t is. m ade.
LISTENING COMPREHENSION

T O E F L REVIEW EXERCISE (Skills 1-13): In this exercise, listen carefully to the short conver
sation an d q u estion in the record in g program , a n d th e n c h o o se the b est answer to the question.

N O W BEGIN THE RECORDING PROGRAM AT TOEFL R e v ie w E x e r c is e (S k ills I -

1. (A) Write a message to the man. 7. (A) H e couldn't find Paulas ph one
(B) Make som e p h one calls. number, so he d id n t call her.
(C) R espond to the m ans questions. (B) H e couldnt give Paula the list
(D) Get a new phone installed. over the phone.
(C) W hen he went to call Paula, he
2. (A) S hes not sure if sh es free.
cou ld n t find the list.
(B) S h es marked it on her calendar.
(D) H e co u ld n t recollect the number
(C) Shell write a check for the
that was on the list.
calendar.
(D) H er calendar says she has to have a 8. (A) She co u ld n t take her luggage to
m eeting at 3:00. the store.
(B) She stored her luggage at the train
3. (A) H e barely rode the bicycle.
station.
(B) H e d id n t have enough money.
(C) She carried her luggage from the
(C) The bicycle did nt need to be paid
train station to the store.
for.
(D) There were no lockers for her bags.
(D) H e paid for the bicycle.
9. (A) T he woman had taken a different
4. (A) She fixed the television.
major.
(B) Bob m ade the television work.
(B) T he woman had chosen psychology
(C) The woman looked at Bob on
as a major.
television.
(C) T he woman was uninform ed.
(D) Bob works for the woman.
(D) The woman n eeded to see a
5. (A) H e helped her say what she psychiatrist
co u ld n t say.
10. (A) She would like the man to repeat
(B) She was unable to say anything
what he said.
about him.
(B) She thinks the exam could have
(C) He hasn't helped her very much.
been a litde m ore difficult.
(D) What he said was very helpful.
(C) She shares the same opinion o f the
6. (A) The man should spend more time exam as the man.
on registration. (D) She believes that the exam was easy.
(B) The man should walk m ore quickly
through registration.
(C) The m an should send in his
registration materials.
(D) The man should try to avoid
registering next semester.

C O N TR A R Y M E A N IN G S _____

S k ill 14: LISTEN FOR W ISH ES

C o n v er sa tio n s a b o u t w ish es can a p p ear in L iste n in g P art A. T h e im p o r ta n t id e a to re


m e m b e r a b o u t w ish es is th at a w ish im p lie s h at the opposite o f the w ish is true.
LISTENING PARTA

E x am p le

On the recording, you hear:


(woman) It's too bad that y o u have to stay here and work during
the school break.
(man) I really w ish I could go w ith you and the others to
Palm Springs.
(narrator) What does the m an mean?

In your test book, you read:


(A) Maybe he will go with the others on the trip.
(B) He is unable to go on the trip.
(C) He's happy to be going on the trip.
(D) Hes going on the trip, but not with the others.

In th is c o n v e r sa tio n th e m a n wishes th a t h e could go w ith th e o th e r s o n th e trip , so th e im


p lie d m e a n in g is th a t h e is unable to go. T h e c o r r e c t an sw er is th e r e fo r e an sw er (B ).

T h e fo llo w in g c h a r t o u tlin e s th e key p o in ts th at y o u sh o u ld k n o w a b o u t w ishes:

KEY INFORMATION ABOUT WISHES

Point Example Meaning

An affirmative wish implies a .M.K.-.-.&ir.:,


1 wish 1 had time to help. = no time to help
negative reality. .
: - ' v v
A negative wish implies an 1wish 1 did not have time to help. ' =^ time
' to ii".
help
affirmative reality.
. ... . .
' - .
A past tense verb implies a present 1 wish he were at hom e* = is not at home
reality.
A . . : ..v. -v.. .
A past perfect tense verb implies a 1 wish he had been at home. = was not at home
past reality. ; \ : -J . ^.. .'
; . - ... .ft: '

Remember that were is used instead of was in wishes: I wish I were going."

E X E R C ISE 14: In th is e x e r c is e , u n d e r lin e th e w ish in e a ch sh o r t c o n v e r sa tio n . T h e n read


th e q u e s tio n a n d c h o o s e th e b est an sw er to th a t q u e s tio n . R e m e m b e r th a t th e b e st an sw er
is o n e th at im p lie s th e o p p o s ite o f w h a t is sa id .

(man) Do you think weU be able to (A) T he sky is n ot very cloudy.


have the picnic today ? (B) T he sky yesterday was clou dier than
(woman) / wish the sky weren 't so cloudy. it is today.
(narrator) What does the woman mean ? (C) T he sky is too cloudy.
(D) T h e sky is rather clear.

(woman) Did you enjoy the Thanksgiving (A) He d id n t eat very m uch.
dinner? (B) He plans on eating a lot.
(m an) I wish I h a d n t eaten so much. (C) He thinks he is eating a lot.
(narrator) What does the man mean ? (D) He ate too much.
LISTENING COMPREHENSION

3. (man) Are you coming to the party (A) She is com ing to the party.
tonight? (B) She m ight com e to the party.
(wom an) I wish I could. (C) She will try to com e to the party.
(narrator) What does the woman mean? (D) She is n o t com in g to the party.

T O E F L E X ER C ISE 14: In th is e x e r c ise , listen carefu lly to th e sh o r t c o n v e r s a tio n an d


q u e s tio n in th e r e c o r d in g p rogram , a n d th e n c h o o s e th e b e st a n sw er to th e q u e s tio n . You
s h o u ld r e m e m b e r th a t a w ish im p lie s a n o p p o s ite m e a n in g .

N O W BEGIN THE RECORDING PROGRAM A T TOEFL EXERCISE 14.

1. (A) The line is short. 6. (A) Harry did not prepare e n ou gh for
(B) There are not very many p eople in the exam.
front o f them. (B) Harry studied hard for the exam.
(C) The line in front o f them is too (C) He has n ot heard anything about
long. Harry.
(D) N ot many p eople want to get (D) He had a bet with Harry.
tickets to the concert. 7. (A) The algebra course that she is
2. (A) The woman told him about the taking is n ot her favorite.
ticket. (B) She d oesn t n eed to take the
(B) He wanted the woman to get a algebra course.
ticket. (C) She has a good schedule o f courses
(C) H e was happy to find ou t about the this semester.
ticket. (D) Shes good at m ath, but sh e s
(D) The woman did not tell him about taking the algebra course anyway.
the ticket. 8. (A) H e was able to find a cheap
3. (A) She is not working too many hours apartment.
next week. (B) His apartment is too expensive.
(B) She d oesn t have enough hours (C) H e d oesn t like th e apartm ents
n ext week. location.
(C) She is working too many hours (D) The apartm ent is cheap because o f
n ext week. its location.
(D) She likes working so m uch. He arrived early at the auditorium.
9. (A)
4. (A) T he departm ent did n ot change (B) He got o n e o f the best seats in the
the requirements. auditorium.
(B) She likes the new requirem ents. (C) H e was n ot early en ou gh to get a
(C) She changed her apartment just seat at the front.
b e f o r e g ra d u a tio n . (D) H e prefers sitting at the back.
(D ) S h e d o e s n o t lik e th e c h a n g e s th a t
10. (A) H ed like to work on his social skills
th e d e p a r t m e n t m a d e .
at the game.
5. (A) H e is going to the theater. (B) He wishes he could work on his
(B) He d o esn t have enough money. term paper for sociology.
(C) He isnt afraid to go. (C) H e cant attend the gam e because
(D) H e d o esn t want to spend the o f his schoolwork.
money. (D) Sociology is less im portant to him
than football this weekend.
LISTENING PARTA

S k ill 15: LIS TE N FOR U N T R U E C O N D IT IO N S

C o n v er sa tio n s c o n ta in in g c o n d itio n s c a n a p p e a r in L iste n in g P art A. T h e im p o r ta n t id e a


to r e m e m b e r a b o u t c o n d itio n s is th a t a c o n d itio n im p lie s th a t th e opposite o f the condition
is true.

E x am p le

On the recording, you hear:


(man) Do you think that you '11be able to go to the party?
(woman) I f I had time, I w ould go.
(narrator) What does the w om an say about the party?

In your test book, you read:


(A) Maybe she'll go.
(B) She has time, so she'll go.
(C) She is goiag even if she doesnt have time.
(D) It's impossible to go.

_________________________________________________ l

In th is q u e s tio n , th e c o n d itio n i f I h a d tim e im p lie s th a t th e o p p o s ite is true: T h e w o m a n


d o e s n o t h ave tim e fo r th e party, so sh e can n o t go. T h e r e fo r e , th e b e st an sw er to this q u e s
tio n is an sw er (D ).

T h e fo llo w in g b o x o u tlin e s th e key p o in ts th a t y o u sh o u ld k n ow a b o u t u n tr u e


c o n d itio n s:

KEY INFORMATION ABOUT UNTRUE CONDITIONS

. *. Point r Example Meaning

An affirmative condition implies If she were at home, she = not at home


a negative reality. could do it.* ,
A negative condition implies an If she weren't h om e, she = at home
affirmative reality. could do it.

* - : " , ' - " .... ;.....


A past tense implies a present . If 1 had money, 1would buy it. = do not have money
reality. .
A past perfect verb implies a If 1 had had money, 1 would = did noc have money
past reality. ' have bought it ;
. ~ : - : : ...
jv . .. .
Had can be used without if Had 1hod money, 1 would = did not have money
: 'Z v > r ' have bought it.**

Remember that were is used instead of was in untrue conditions: "If I were there, I would help.
" T h is has the same meaning as If I had had m o n e y .. .." Note that the subject and "had" are
inverted.
LISTENING COMPREHENSION

E X E R C IS E 15: In th is e x e r c ise , u n d e r lin e th e c o n d itio n in e a c h sh o r t c o n v e rsa tio n .


T h e n , r e a d th e q u e s tio n a n d c h o o s e th e b e st an sw er to th a t q u e s tio n . R e m e m b e r th a t th e
b e st an sw er is o n e th a t im p lie s th e o p p o s ite o f w h at is said.

1. (m an)
Are you going to have something (A) She is not going to cat.
to eat ? (B) T he food looks fresh.
(wom an) I f the food looked fresh, I would (C) She d oesn t like fresh food.
eat some. (D) She already ate som ething.
(narrator) What does the woman mean ?

2. (wom an) The flight must have taken (A) It arrived early.
longer than usual. (B) It was unusually short.
(m an) H ad the flight left on time, we (C) It left on time.
would not have arrived so late. (D) It departed late.
(narrator) What does the man say about
the flight ?

3. (m an)
Are you sure you want to go (A) She really is feelin g fine.
o u t? You do not seem to be (B) There is plenty o f aspirin in the
feeling very well. m edicine cabinet.
(wom an) I f there were some aspirin in the (C) It is necessary to get som e aspirin.
medicine cabinet, I would not (D) She does not n eed to go out.
need to go to the drugstore.
(narrator) What does the woman mean?

T O E F L E X E R C ISE 15: In th is e x e r c ise , listen carefu lly to th e sh o rt c o n v e rsa tio n an d


q u e s tio n in th e r e c o r d in g p rogram , an d th e n c h o o s e th e b e st answ er to th e q u e s tio n . You
s h o u ld b e particu larly carefu l o f u n tr u e c o n d itio n s .

N O W BEGIN THE RECORDING PROGRAM AT TOEFL EXERCISE 15.

(A) T he woman did not need to call him. 4. (A) She did not put en ou gh postage on
(B) T he woman called to let him know the letter.
about the m eeting. (B) T he letter arrived last week.
(C) H e s n ot glad that the woman called. (C) T he letter did not n eed more
(D) H e already knew about the m eeting postage.
w hen the woman called. (D) She did not put any postage on the
letter.
2. (A) T he m an often drives too quickly.
(B) T he police do not stop the man too 5. (A) H e has a dog.
m uch. (B) H e d oesn t pay attention to dogs.
(C) T he man drove rather slowly. (C) H e wishes he had a dog.
(D) T he police should not stop the (D) D ogs do not n eed m uch attention.
man so often.
6. (A) They knew they had to prepare for
3. (A) Shes so happy they d o n t have to the exam.
work on Friday. (B) They did nt prepare for the exam.
(B) It would be nice if they could finish (C) As soon as they knew about the
their work on Friday. exam, they began to prepare for
(C) She wonders if the man would be it.
nice enough to com e in to work (D) They knew that the preparation for
in her place on Friday. the exam would take a lot o f
(D) Its too bad they must work on time.
Friday.
LISTENING PARTA

7. (A) It costs too m uch for him to go. 9. (A) T h e woman d id n t com e.
(B) H e agrees to go with them. (B) T h e woman wanted to be there.
(C) H e is unworried about the cost o f (C) T h e woman was g oin g to leave
the restaurant. immediately.
(D) T h e restaurant is rather (D) T h e woman was n ot really there.
inexpensive.
10. (A) Kathy d id n t work as hard as
(A) W hen J o e saw the car com ing, he possible because she d id n t know
tried to get out o f the way. what the reward was.
(B) Joe was able to get out o f the way (B) Kathy c o u ld n t have put m ore
because he saw the car com ing. effort into the project to win the
(C) Joe ju m p ed out o f the way o f the prize.
o n com in g car. (C) Kathy won first prize because o f her
(D) Because J o e did nt see the car hard work on the art project.
com ing, he co u ld n t get ou t o f (D) Kathy worked so hard that she
the way. knew first prize was hers.

T O E F L E X ER C ISE (Skills 1 4 -1 5 ): In this ex er cise , listen carefully to th e sh o r t con versation


an d q u estio n in th e r ec o r d in g program , a n d th e n c h o o s e th e b est answ er to th e q u estio n .

n N ow BEGIN THE RECORDING PROGRAM AT TOEFL EXERCISE (SKILLS 14-15).

1. (A) She enjoys violent movies. 6. (A) T hey filled up the gas tank at the
(B) She would have preferred a more last service station.
violent movie. (B) A lthough they filled up the tank,
(C) She thinks the film was too violent. they still ran out o f gas.
(D) She enjoyed the movie. (C) Even thou gh they d id n t stop at the
service station, they d id n t run
2. (A) H e left the windows open.
out o f gas.
(B) The rain did not get in.
(D) They ran ou t o f gas because they
(C) H e forgot to close the windows.
d id n t stop at the gas station.
(D) T he rain got into the house.
(A) H is schedule is not really heavy.
3. (A) Her family is unable to com e to
(B) H e needs to add a few more
graduation.
courses.
(B) It is possible that her family will
(C) H e enrolled in m ore courses than
com e.
he really wants.
(C) H er parents are com ing to the
(D) H e will register for a lot o f courses
cerem onies.
next semester.
(D) She is n o t graduating this year.
8. (A) She never took the bus to work.
4. (A) He is g o in g to miss the conference.
(B) She regularly takes the bus.
(B) He will take his vacation n ext week.
(C) She d oesn t know how to get to
(C) He will attend the conference.
work.
(D) He w on t miss his vacation.
(D) She gets lost on the bus.
5. (A) H e enjoys chem istry lab.
(B) H e d o e sn t have chem istry lab this
afternoon.
(C) H e isnt taking chem istry class.
(D) H e has to go to the lab.
LISTENING COMPREHENSION

9. (A) She bought som e eggs at the store. 10. (A) Teresa is feeling a lot better.
(B) She doesn't have any eggs to lend (B) The doctor did nt prescribe the
him. m edicine.
(C) H e can borrow som e eggs. (C) Teresa d id n t follow the doctors
(D) She did nt go to the store. orders.
(D) Teresa did exacdy what the doctor
said.

T O E F L REVIEW EXERCISE (Skills 1-15): In this exercise, listen carefully to the sh ort conver
sation a n d q u estion in the record in g program , a n d th en c h o o se th e b est answer to th e question.

N ow BEGIN THE RECORDING PROGRAM AT TOEFL REVIEW EXERCISE (SKILLS IIS).


o
1. (A) Drinking the hot tea. 6. (A) H e went to the office every
(B) Making more tea in a few minutes. m orning.
(C) Letting the tea cool o ff a bit. .(B) H e was not working.
(D) Having the tea immediately. (C) He had to arrive at work earlier
than 8 o clock.
2. (A) In a bus station.
(D) H e had a job.
(B) In a store.
(C) In a restaurant. 7. (A) H e did not enjoy his vacation as
(D) In a theater. much as possible.
(B) H e got lost on his vacation.
3. (A) H es unhappy to end the semester.
(C) The vacation was really enjoyable.
(B) H es glad to be finishing .chool.
(D) H e did not really lose his passport.
(C) H e co u ld n t be happier to begin
the semester. 8. (A) It will take eight hours to get to
(D) The end o f the sem ester is making Riverdale on the bus.
him feel sad. (B) He believes he knows the correct
bus.
4. (A) The storm destroyed the houss. (C) H e d o e sn t know where Riverdale
(B) The house blocked the trees.
is.
(C) The stormy weather caused the
(D) H e assures the woman that he
trees to fall.
knows the way to Riverdale.
(D) During the storm, som eone
knocked on the door o f the 9. (A) The laboratory assistant com pleted
house. one experim ent.
(B) The laboratory assistant cou ld n t
5. (A) The team hasnt won often.
finish one experim ent.
(B) H e usually d o esn t pay attention to
(C) The laboratory assistant did nt
the football team.
want to do m ore experim ents.
(C) Its out o f the ordinary for the team
(D) N on e o f the experim ents could be
to lose.
com pleted.
(D) H e usually hears about the football
gam es. 10. (A) She would like the man to repeat
what h e said.
(B) The sem ester is really over!
(C) The sem ester will never end.
(D) She has the same wish as the man.
LISTENING PARTA

ID IO M A T IC LA N G U A G E _____________________________________

S k il l 16: LISTEN F O R T W O -A N D THREE-PART VERBS

Two- a n d th ree-p art verb s a p p ea r in s o m e q u e s tio n s in L iste n in g Part A. T h e s e verb s are


e x p r essio n s th a t in c lu d e a verb aijd o n e o r m o r e p a r ticles (su ch as in, on, o r a t) ; th e parti
c le c h a n g e s th e m e a n in g o f th e verb. Q u e s tio n s in v o lv in g two- a n d th r e e-p a r t v e rb s can b e
d iffic u lt for stu d e n ts b e c a u se th e a d d itio n o f th e p a r ticle c h a n g e s th e m e a n in g o f th e
verb in an id io m a tic way.

E xam ple

On the recording, you hear:


(man) What tim e does the meeting start?
(woman) - Didn't you hear that it was called o ff by the director?
(narrator) What does the w om an say about the meeting?

In your test book, you read:


(A) The director called a meeting.
(B) The director phoned her about the meeting.
(C) The director called the meeting to order.
(D) The director canceled the meeting.

In this q u e s tio n , th e tw o-part verb called o f f h a s a d iffe r e n t m e a n in g th a n th e verb call,


w h ich m ea n s phone. T h e tw o-part v e rb call o f f m e a n s cancel, so th e b e st an sw er is
answ er (D ).

NOTE: A list of common two- and three-part verbs and exercises using these verbs
appear in Appendix B. You may want to study these two- and three-part verbs before you
try the following exercises.

EX ER C ISE 16: In th is e x e r c ise , u n d e r lin e th e two- o r th r e e-p a r t verb in e a c h s h o r t c o n


v e rsa tio n . T h e n read th e q u e s tio n a n d c h o o s e th e b e st an sw er to th a t q u e s tio n . R e m e m
b e r th a t the b e st answ er is o n e th a t is r e la te d to th e m e a n in g o f th e two- o r th ree-p a rt verb
an d m ig h t n o t se e m to b e r e la te d to th e m e a n in g o f th e verb w ith o u t t h e p a r ticle.

1. (man) Did you have your history exam (A) She would like to pu t it o u t o f her
today f mind.
(woman) No, the professor p u t it offfor (B) The professor canceled it.
another week. (C) It was m oved to anoth er location.
(narrator) What does the woman say about (D) It was delayed.
the exam ?

2. (woman) Do we have any more soap ? (A) H e will run to the store.
(man) Weve run out o f it. Someone (B) H e n eed s soap to wash him self
will have to go to the store. after running.
(narrator) What does the man mean ? (C) There is no m ore soap.
(D) They have a store o f soap at hom e.
LISTENING COMPREHENSION

3. (man) 1 need to take the written test to (A) Reapply for his drivers license.
renew my driver's license. (B) Sweep around the lawn.
(woman) Then, you '11 have to brush up (C) Learn the laws for the first time.
on the laws. (D) Review the inform ation that will be
(narrator) What does the man need to do ? on the test.

T O E F L E X ER C ISE 16: In th is e x e r c ise , liste n carefu lly to th e sh o rt c o n v e rsa tio n a n d


q u e s tio n in th e r e c o r d in g p rogram , a n d th e n c h o o s e th e b e st answ er to th e q u e stio n . You
sh o u ld b e p articu larly carefu l o f two- a n d th ree-p art verbs.

N O W BEGIN THE RECORDING PROGRAM AT TOEFL EXERCISE I 6.


o
1. (A) P hone their neighbors. 6. (A) She gets along with lots o f people.
(B) Call to their neighbors over the (B) She gets back at people who cross
fence. her.
(C) H elp the neighbors m ove in. (C) She gets rid o f people she doesnt
(D) Visit their neighbors. want to spend time with.
(D) She tries to get ahead o f everyone
2. (A) The course is becom ing more else.
interesting.
(B) T he course used to be more 7. (A) He must try to find the children.
interesting. (B) It is necessary for him to clean up
(C) T he course is about the same as it after the children.
was. (C) The children n eed to be watched.
(D) H e s not as bored in the class as the (D) H es going to see what the children
woman. have done.

3. (A) H er headache is getting worse. 8. (A) They are going on strike.


(B) She felt better this m orning than (B) They are lying down on the job.
now. (C) They are being released from their
(C) She seem s to be feelin g better now. jobs.
(D) She is just getting another (D) They are relaxing too m uch at the
headache now. factory.

4. (A) The man should stop breaking his 9. (A) H e is betting that the football team
cigarettes in half. will win.
(B) The man should decrease the( (B) He really wants to succeed.
num ber o f cigarettes he smokes. (C) It is n ot so difficult to play on the
(C) T he man should cut the ends o ff football team.
his cigarettes. (D) He pulled a m uscle w hile playing
(D) The man should stop smoking football.
com pletely. 10. (A) Shes unsure why she tolerates the
5. (A) T he client presented his case to the man.
lawyer. (B) She doesn't know where she put
(B) T he client was upset about the her keys.
lawyers rejection. (C) She is actually the on e who put the
(C) The client was annoyed because keys in the car.
the lawyer returned the suitcase. (D ) She can't understand why the man
( D ) T he client m ade the lawyer did what he did.
unhappy about the case.
LISTENING PARTA 87

S k i l l 17: LISTE N FOR ID IO M S

Id io m s a p p e a r in s o m e q u e s tio n s in L iste n in g P art A. Id io m s are sp e c ia l e x p r e s sio n s in a


la n g u a g e th a t all sp ea k er s o f th e la n g u a g e kn ow ; th e se sp ec ia l e x p r e s s io n s d e sc r ib e o n e
situ a tio n in life b u t a r e a p p lie d to m a n y d if fe r e n t areas o f life . Id io m q u e s tio n s c a n b e d if
fic u lt fo r stu d e n ts b e c a u se th e y s e e m to b e d e sc r ib in g o n e situ a tio n w h e n th e y are really
d e sc r ib in g a d iffe r e n t situ a tio n .

E x am p le
On the recording, you hear:
(man) Tom is a full-tim e student and is holding down a full
tim e job.
(woman) He's really burning the candle at both ends.
(narrator) What does the w om an say about Tom?
In your test book, you read:
(A) He's lighting a candle.
(B) Hes holding the candle at the top and the bottom.
(C) Hes doing too much.
(D) Hes working as a firefighter.

In th is q u e s tio n , th e id io m b u rn in g the candle a t both ends h as n o th in g to d o w ith c a n d le s


a n d n o th in g to d o w ith b u r n in g o r fires, so an sw ers (A ), (B ), a n d (D ) a r e n o t c o r r e c t. In
stead , th is id io m is a n e x p r e s sio n th a t is u s e d in a situ a tio n w h e n s o m e o n e is tr y in g to d o
m o r e than h e o r sh e really c a n do; a fter all, a c a n d le u su ally o n ly b u r n s at o n e e n d , s o a-
c a n d le th a t b u r n s at tw o e n d s is d o in g more th a n it can. T h e r e fo r e , th e b e st a n sw er to th e
q u e s tio n a b o v e is a n sw er ( C ) .

NOTE: A list of common idioms and exercises using chese idioms appear in Appendix C.
You may want to study these idioms before you try the following exercises.

E X E R C ISE 17: In th is e x e r c ise , u n d e r lin e th e id io m in e a c h sh o r t c o n v e r s a tio n . T h e n


r e a d th e q u e s tio n a n d c h o o s e th e b e s t a n sw er to th a t q u e s tio n . R e m e m b e r th a t th e b e st
an sw er is o n e th a t m ig h t n o t s e e m to b e r e la te d to th e id io m in th e s e c o n d lin e .

(man) I have to take Advanced Biology (A) T he m an sh ould try a p iece o f


from Professor Stanton next cake.
semester. (B) T he m an sh ould worry about the
(woman) D o n t worry about it. It's a piece course.
o f cake. (C) T he man sh o u ld n t take part in the
(narrator) What does the woman mean ? course.
(D) T he course is easy.

(wom an) Thanks fo r changing the oil (A) It will take him a w hole day to do
AND putting air in the tires. the job.
(man) I t s all in a day's work. (B) This is a regular part o f his job .
(narrator) What does the m an mean ? (C) H e can do the work at the en d o f
the day.
(D) H es too busy today to d o the work.
LISTENING COMPREHENSION

3. (m an) What was it like while the (A) T he president dropped his pen.
president was giving his speech ? (B) T he audience was very quiet.
(wom an) You could hear a pin drop. (C) T he speech contained several puns.
(narrator) What does the woman mean ? (D) T he president discussed dropping
a bomb.

T O E F L E X E R C ISE 17: In this e x e r c ise , liste n ca refu lly to th e sh o r t c o n v e rsa tio n and
q u e s tio n in th e r e c o r d in g p rogram , a n d th e n c h o o s e th e b e st an sw er to th e q u e stio n . You
sh o u ld b e particu larly carefu l o f id iom s.

N ow BEGIN THE RECORDING PROGRAM AT TOEFL EXERCISE 17.


n
(A) T h e m ans never late. 6. (A) Sh ell do it immediately.
(B) Its good that the man was fifteen (B) It is not possible to do it.
m inutes late. (C) T he man should have told her
(C) Its never good to be late for class. sooner.
(D) Its good that the man went to (D) She would have d on e it if the man
class, on time or not. had asked.
(A) T he w om ans work is all in her 7. (A) Abbie used a feather in his art
head. project.
(B) T h e woman has to do two (B) H e was knocked down.
experim ents rather than one. (C) H e was really surprised.
(C) Its a good idea to work together. (D) A bbies father knocked on the
(D) T h e biology experim ent concerns door.
two-headed animals.
8. (A) They are taking a boat trip
(A) She has n o time to work now. together.
(B) She d oesn t want to work on the (B) T he six chapters are all about the
report either. boat.
(C) Its best to get it over with now. (C) Everyone has to do the same thing.
(D) T heres no time to present the (D) T he man will read while h es on
report now. the boat.
(A) S h es very lucky to get the last 9. (A) She is taller than die others.
book. (B) She put her science project on top
(B) S h es sorry she can t get the book o f the others.
today. (C) She has a really good head on her
(C) She always has good luck with shoulders.
books. (D) S h es the best o f them all.
(L>) She just wanted to look at the book.
10. (A) T h e man needs to improve his
(A) T he man d oesn t like eating in penm anship.
restaurants. (B) T he man d o esn t really n eed to
(B) She d oesn t really like that apply for the scholarship.
restaurant. (C) T he man needs to fill out the
(C) Each o f them has his own application with dots and crosses.
restaurant. (D) T he man needs to pay attention to
(D) Everyone has different tastes. every detail.
LISTENING PARTA

T O E F L EX ER C ISE (Skills 1 6 -1 7 ): In this e x er cise , listen carefully to th e sh o r t co n v ersa tio n


a n d q u estio n in th e r ec o r d in g p rogram , a n d th e n c h o o se th e b est answ er to th e q u estio n .

n N O W BEGIN THE RECORDING PROGRAM A T T O E R EXERCISE (SKILLS 16-17).

1. (A) She gets lots o f take-out dinners. 6. (A) T h e man m ight start a fire in the
(B) She and her room m ate alternate park.
cooking responsibilities. (B) T he man parked his car near the
(C) H er room m ate cooks m ore often fire.
than she does. (C) T h e m ans thinking o f doin g
(D) H er room m ate does the cooking som ething dangerous.
while she does other chores. (D) T he m ans playing a gam e in the
park.
2. (A) H e resem bles his father.
(B) H e has a chippcd tooth. 7. (A) T h e m achines do n o t act very well.
(C) H e lives o n e block from his father. (B) T h e m achines d o n t really bother
(D) H e and his father were playing a her.
gam e with blocks. (C) She would like them to stop the
3. (A) Sh es go in g som ewhere else. noise.
(B) She does n ot like football. (D) She wishes the m achines would
(C) She has a lot o f work to do. cut the wood.
(D) She is gettin g sick. 8. (A) Fred has a d og that barks a lot.
4. (A) H e put his foot w here h e should (B) Fred has hidden the m oney in a
tree.
n ot have.
(B) H e put the food that the teacher (C) Fred has backed into a tree.
gave him into his m outh. (D) Fred has m ade a mistake.
(C) H e said som ething embarrassing. 9. (A) She will give him any help he
(D) H e told the teacher that his foot needs.
was hurt. (B) H e has to give away what he d o e sn t
5. (A) S h ed like the man to delay his trip. need.
(B) She prefers that the man leave a (C) H e should not give up.
few m inutes earlier than he (D) H e should give back what he
planned. borrowed.
(C) She wants to know if the m an will 10. (A) She'd rather go swimm ing than do
stay in the market for only a few the homework.
m inutes. (B) T he chem istry hom ework is really
(D) Sh ed like to talk to the m an for a difficult.
few m inutes. (C) S h es d oin g the hom ework by the
swimming pool.
(D) T h e stream is drying up.

TO EFL REVIEW EXERCISE (Skills 1-17): In this exercise, listen carefully to th e sh ort conver
sation a n d q u estion in th e r eco rd in g program , an d th en c h o o se the b est answer to the q u estio n

o N ow BEGIN THE RECORDING PROGRAM At TOEFI REVIEW EXERCISE (SKILLS 1-17).

(A) T heres no m ore w ood inside. 2. (A) She worked late at a conference.
(B) The w ood in the fireplace should (B) H er m eeting was canceled.
be pu t outside. (C) She called a con ference at work.
(C) T heres a fire outside. (D) She was late to a con ference.
(D) H e n eed s to bring som e wood
outside.
LISTENING COMPREHENSION

3. (A) In a hospital. 8. (A) Inform ation about the problem is


(B) At a police station. unavailable.
(C) At the beach. (B) N o o n e has been inform ed.
(D) In a locker room . (C) Everybody knows what is going on.
(D) Nobody is aware that the problem
4. (A) There was too m uch room on the
is serious.
dance floor.
(B) H e enjoyed the room w here they 9. (A) H e did not sleep well.
w ent dancing. (B) H e never woke up this morning.
(C) T he dance floor was too crowded. (C) The alarm failed to go off.
(D) The club n eeded more rooms for (D) He n eed ed a loud alarm to wake
dancing. up-
5. (A) H e could not understand the fax 10. (A) The pilot made an em ergency
m achine. landing.
(B) H e wrote the letter that was sent. (B) T he pilot was forced to leave the
(C) T h e fax m achine was easy for him plane in a hurry.
to use. (C) The pilot fielded questions about
(D) H e was not very good with figures. the forced landing.
(D) The plane was dam aged when it
6. (A) The woman hit her head on a nail.
landed forcefully.
(B) T he woman hit his new car.
(C) The woman was exactly right.
(D) T he woman bought the new car.
7. (A) H e would like the woman to help
him find his paper.
(B) H e wants the woman to pat the
paper away.
(C) H e needs the woman to review the
paper.
(D) H e would like the woman to write
the paper for him.
LISTENING PART B

-------------T H E L IS T E N IN G PART B Q U E S T IO N S -------------


Part B o f th e L iste n in g C o m p r e h e n s io n se c tio n o f th e T O E F L test c o n sists o f two lo n g c o n
versa tio n s, e a c h fo llo w e d by a n u m b e r o f q u estio n s. You will h e a r th e c o n v e r sa tio n s an d
th e q u e s tio n s o n th e r eco rd in g ; th ey are n o t w ritten in you r te st b o o k . Y ou m u st c h o o s e
th e b e st an sw er to e a c h q u e s tio n from t h e fo u r c h o ic e s th a t are w ritten in y o u r te st b o o k .
T h e c o n v e r sa tio n s a r e o ft e n a b o u t s o m e a sp e c t o f s c h o o l life (h o w d iffic u lt a class is,
h o w to w rite a r esea r ch p ap er, h o w to r e g iste r fo r a c o u r s e ) . T h e c o n v e r s a tio n s c a n also
b e a b o u t to p ic s c u r re n tly in th e new s in th e U n ite d S tates (d e s a lin a tio n o f th e w ater
su pp ly, r ec y c lin g o f u s e d p r o d u cts, d a m a g e fr o m a sto rm o r s o m e o th e r type o f n atu ral
p h en o m en o n ).

E xam ple

On the recording, you hear:


(narrator) Q u estio n s I th ro u g h 4. Listen to a conversation
between a professor and a student.
(man) Hello, Professor Denton. Are you free for a m om ent?
Could I have a word w ith you?
(woman) Come on in, Michael. O f course I have som e time.
These are m y office hours, and this b the right tim e for
you to come and ask questions. Now, how can I help
you?
(man) Well, I have a quick question fo r you about the
hom ework assignm ent for tomorrow. I thought the
assignm ent was to answer the first three questions at
the top o f page 67 in the text, but when I looked, there
weren't any questions there. I'm confused.
(woman) The assignm ent was to answer the first three questions
at the top o f page 76, not 67.
(man) Oh, now I understand. I m glad I came in to check.
Thanks for yo u r help.
(woman) No problem. See you tomorrow.

Questions:

1. On the recording, you hear:


(narrator) Who is the man?
In your test book, you read: (A) A professor.
(B) An office worker.
(C) Professor Dentons assistant.
(D) A student.

2. On the recording, you hear:


(narrator) When does the m an come to see
Professor Denton?
In your test book, you read: (A) During regular class hours.
(B) Just before class time.
(C) As soon as class is finished.
(D) During office hours.
(continued on next page)
LISTENING COMPREHENSION

3. On the recording, you hear:


(narrator) Why does the man come to see Professor
Denton?
In your test book, you read: (A) To turn in an assignment.
(B) To ask a question.
(C) To pick up a completed test.
(D) To explain why he did not attend
class.

4. On the recording, you hear:


(narrator) What incorrect information did the man
have?
In your test book, you read: (A) The date the assignment was due.
(B) The page number of the
assignment.
(C) The length of the assignment.
(D) The numbers of the assignment
questions.

T h e first q u e s tio n asks y o u to d e te r m in e w h o th e m an is. S in ce th e m an o p e n s th e c o n


v e rsa tio n w ith Professor D enton a n d h e asks a b o u t th e p a g e n u m b e r o f an a ssig n m e n t fo r to
m orrow , h e is p rob ab ly a stu d e n t. T h e b e st answ er to th is q u e s tio n is th e r e fo r e answ er
( D ). T h e s e c o n d q u e stio n asks a b o u t w h en th e m an c o m e s to se e th e professor. T h e p ro
fe sso r says th a t these are my office hours, so th e b est answ er to this q u e s tio n is answ er (D ).
T h e th ird q u e s tio n asks w h y th e m a n c o m e s to se e th e p rofessor. S in c e th e m a n says I have
a quick question f o r you, th e b e st an sw er to th is q u e s tio n is answ er (B ). T h e last q u estio n
asks w h a t in c o r r e c t in fo r m a tio n th e m an h a d . T h e m an th o u g h t th a t th e a ssig n m e n t was
o n p a g e 6 7 a n d n o t o n p a g e 76, so h e was m ista k en a b o u t th e page num ber o f th e assign
m e n t. T h e b e st answ er to th is q u e s tio n is answ er (B ).

_________________ STRATEGIES FORTHE LISTENING PART B QUESTIONS_________________

1. If you have tim e , p review th e answ ers t o th e L istening Part B q u estio n s. While you
are looking at the answers, you should try to do the following:
Anticipate the to p ic s of the conversations you will hear.
Anticipate the q u estio n s for each of the groups of answers.
2. L isten carefully to th e first line o f th e con versation . The first line of the conversation
often contains the main idea, subject, or topic of the conversation, and you will often be
asked to answer such questions.
3. A s you listen to th e con versation , draw co n clu sion s a b o u t th e situation o f the
con versation : w h o is talking, w h er e th e con versation ta k es p lace, o r w h en it tak es
p lace. You will often be asked to make such inferences about the conversation.
4. A s you listen to the conversation, follow along w ith th e answers in your te st book and
try t o determ ine th e correct answers. Detail questions are generally answered In order in the
conversation, and the answers often sound the same as what is said in the recording program.
5. You should g u ess even if you are n ot su re. Never leave any answers blank.
6. U se any rem aining tim e to look ahead a t th e answ ers to th e q u estio n s th a t follow.
LISTENING PART B

T h e fo llo w in g sk ills w ill h e lp y o u to im p le m e n t th e s e stra teg ies in P art B o f th e L iste n in g


C o m p r e h e n s io n s e c tio n o f th e T O E F L test.

BEFORE LIS T E N IN G

S k il l 18: A N T IC IP A T E T H E TO P IC S

It is very h e lp fu l to y o u r o v era ll c o m p r e h e n s io n i f y o u k n ow w h at to p ic s to e x p e c t in Lis


te n in g P art B. You sh o u ld th e r e fo r e try to a n tic ip a te th e to p ic s y o u w ill b e h e a r in g . For
e x a m p le , are th e c o n v e r sa tio n s a b o u t s o m e a sp e c t o f s c h o o l life, o r s o m e typ e o f so c ia l is
su e , o r a trip s o m e o n e is p la n n in g ? A h e lp fu l strategy is th e r e fo r e to l o o k b riefly at th e a n
sw ers in th e test b o o k , b e fo r e y o u actu a lly h e a r th e c o n v e r sa tio n s in th e r e c o r d in g
p ro g ra m , a n d try to d e te r m in e th e to p ic s o f th e c o n v e r sa tio n s th a t y o u w ill hear.

E X E R C ISE 18: L o o k at th e an sw ers to th e five q u e s tio n s to g e th er , a n d try to a n tic ip a te


th e to p ic o f th e c o n v e r s a tio n fo r th o s e five q u e s tio n s. ( O f co u r se , y o u c a n n o t always d e
te r m in e e x a ctly w h a t th e to p ic is, b u t y o u o fte n ca n g e t a g e n e r a l id e a .) Q u e s tio n s 1
th r o u g h 5 h ave b e e n a n sw e re d fo r y ou .
1. (A) Find work on campus. 4. (A) Every m orning.
(B) Work in the employment office. (B) A fternoons and w eekends.
(C) H elp students find jobs. (C) W hen h e s in class.
(D) Ask the woman questions. (D) Weekdays.
2. (A) In the library. 5. (A) Fill out a form.
(B) In a classroom. (B) Give her som e additional
(C) In a cam pus office. inform ation.
(D) In an apartment. (C) Tell her som e news.
3. (A) N o m ore than ten. (D) P hone her.
(B) At least twenty.
(C) N ot m ore than twenty.
(D) U p to ten.

W hat is th e to p ic o f th e c o n v e r s a tio n fo r q u e s tio n s 1 th r o u g h 5?

looking for a job on campus________________

You can g u e ss th is b e c a u se o f th e fo llo w in g clu es:

work on cam pus


em ploym ent office
stu d en ts
jobs
LISTENING COMPREHENSION

6. (A) Just before a vacation. 9. (A) Sleeping outside on the ground.


(B) Just after the end o f a school (B) Spending time in a sauna or hot
semester. tub.
(C) At the end o f the summer. (C) Relaxing at the lodge.
(D) Just after a break from school. (D) Enjoying excellent food.

7. (A) A trip to visit the Eskimos. 10. (A) Shed be scared, but shed like to
(B) A trip the woman is planning to try.
take. (B) She cant wait.
(C) A trip the man has already taken. (C) It would be quite exciting for her.
(D) A cam ping trip the man and (D) Sh ed prefer not to try.
woman took.
8. (A) T hree hours.
(B) Three com plete days.
(C) T hree classes.
(D) T hree weeks.

W h at is th e to p ic o f th e c o n v e rsa tio n fo r q u e s tio n s 6 th r o u g h 10?

(A) All kinds o f pollution. 14. (A) Only in North America.


(B) How acid rain has harm ed the (B) At the North and South Poles.
earth. (C) In parts o f several northern
(C) P ollution from cars and factories. continents.
(D) T he causes and possible effects o f (D) In equatorial areas.
acid rain. She should protect herself from
15. (A)
(A) N uclear power. the rain.
(B) Electricity. (B) She should clean up the water
(C) Burning coal and oil. supply.
(D) Solar power. (C) She should read a novel.
(D) She should get m ore information
(A) From sulfur dioxide and water
about acid rain.
vapor.
(B) From sulfur dioxide and nitrogen
oxide.
(C) From nitric acid and sulfur
dioxide.
(D) From water vapor and nitric acid.

W hat is th e to p ic o f th e c o n v e rsa tio n fo r q u e s tio n s 11 th r o u g h 15?


LISTENING PART B

S k il l 19: A N T IC IP A T E T H E Q U E S T IO N S

It is very h e lp fu l to you r ab ility to an sw er in d iv id u a l q u e s tio n s in L iste n in g P art B if y ou


c a n a n tic ip a te w h a t th e q u e s tio n s w ill b e a n d lis te n sp ec ific a lly fo r th e a n sw ers to th o se
q u e s tio n s.

E x am p le

In your test book, you read:


(A) In the airport.
(B) In the library.
(C) In the dormitory.
(D) In the travel agent's office.

You try to anticipate the question:


Where does the conversation probably take place?

In th is e x a m p le , y o u can b e q u ite c e r ta in th a t o n e o f th e q u e s tio n s w ill b e a b o u t w h er e


th e c o n v e r sa tio n takes p la c e. S in c e y o u are su re th a t th is is o n e o f th e q u e s tio n s , y o u can
liste n carefu lly fo r c lu e s th a t w ill give y o u th e answ er. T h is e x a m p le sh o w s th a t a h e lp fu l
strategy is th e r e fo r e to lo o k b riefly at th e answ ers in th e test b o o k , b e fo r e y o u actu ally
h e a r th e c o n v e r sa tio n s in th e r e c o r d in g p ro g ra m , a n d try to d e te r m in e th e q u e s tio n s that
y o u w ill b e a sk ed to answer.

E X E R C ISE 19: Stu dy th e fo llo w in g answ ers an d try to d e te r m in e w h at th e q u e s tio n s w ill


b e . (You sh o u ld n o te th at p e r h a p s y ou will o n ly b e a b le to p r e d ic t p art o f a q u e s tio n , rather
th an th e c o m p le te q u e s tio n .) I f y o u c a n n o t p r e d ic t th e q u e stio n in a sh o r t p e r io d o f tim e,
th e n m o v e o n to th e n e x t g r o u p o f answ ers. Q u e stio n 1 has b e e n a n sw ered fo r y ou.

1. Question: What does (someone) want to do?___________________________________________


(A) Find work on campus.
(B) Work in the em ploym ent office.
(C) H elp students find jobs.
(D) Ask the woman questions.
2. Q uestion:__________________________________________________ _____________________________
(A) In the library.
(B) In a classroom.
(C) In a cam pus office.
(D) In an apartment.
3. Question: _____________________________________________________ __________ ________________
(A) N o m ore than ten.
(B) At least twenty.
(C) N ot m ore than twenty.
(D) U p to ten.
4. Q u estion :______ ________________________________________________ ______________ __________
(A) Every m orning.
(B) A fternoons and weekends.
(C) W hen h e s in class.
(D) Weekdays.
LISTENING COMPREHENSION

5. Q u estio n :______________________________________
(A) Fill ou t a form.
(B) Give her som e additional inform ation.
(C) Tell h er som e news.
(D) P hone her.
6. Q u e stio n :__________________________ --------------
(A) Just before a vacation.
(B) Just after the end o f a school semester.
(C) At the end o f the summer.
(D) Just after a break from school.

7. Q u e stio n :______________________________________
(A) A trip to visit the Eskimos.
(B) A trip the woman is planning to take.
(C) A trip the man has already taken.
(D ) A cam ping trip the man and woman took.

8. Q u estio n :______________________________________
(A) Three hours.
(B) Three com plete days.
(C) T hree classes.
(D) Three weeks.
9. Q u estio n :_____________________________________
(A) Sleep in g outside on the ground.
(B) Sp en ding tim e in a sauna or h ot tub.
(C) R elaxing at the lodge.
(D) Enjoying excellen t food.

10. Q u e stio n :___________________ _________________ _


(A) S h ed be scared, but sh ed like to try.
(B) She ca n t wait.
(C) It w ould be quite exciting for her.
(D ) Sh ed prefer n ot to try.
11. Q u estio n :___________________________ -_________
(A) All kinds o f pollution.
(B) How acid rain has harm ed the earth.
(C) P ollution from cars and factories.
(D ) The causes and possible effects o f acid rain.

12. Q u estio n :_____________________________ ________


(A) N uclear power.
(B) Electricity.
(C) B urning coal and oil.
(D) Solar power.
13. Q u e stio n :__________________________________
(A) From sulfur dioxide and water vapor.
(B) From sulfur dioxide and nitrogen oxide.
(C) From nitric acid and sulfur dioxide.
(D ) From water vapor and nitric acid.
14. Q u estio n :___________________________________
(A) Only in North America.
(B) At the North and South Poles.
(C) In parts o f several northern continents.
(D) In equatorial areas.
LISTENING iART B

15. Q u estion :___________________________ --------------------


(A) She should protect herself from the rain.
(B) She should clean up the water supply.
(C) She sh ould read a novel.
(D) She should get m ore inform ation about acid rain.

W H IL E LIS T E N IN G

S k il l 20: D E TER M IN E T H E T O P IC

As y o u listen to e a c h c o n v e r sa tio n in L iste n in g Part B, y o u s h o u ld b e th in k in g a b o u t th e


to p ic (su b je ct) o r m a in id e a fo r e a c h c o n v e r sa tio n . S in c e th e first o n e o r tw o s e n t e n c e s
g e n e r a lly give th e to p ic , y o u sh o u ld b e ask in g y o u r se lf w h at th e to p ic is w h ile y o u are
lis te n in g ca refu lly to th e first p art o f th e co n v e rsa tio n .

E x am p le

On the recording, you hear:


(man) You can't believe w hat I ju st got!
(woman) I bet you got that new car you've always wanted.
(man) Now, how in the world did you figure that out?

You think:
The topic of the conversation is the new car that the man just got.

EX ER C ISE 20: L iste n to th e first p a r t o f e a c h o f th e co n v e rsa tio n s, a n d d e c id e o n th e


to p ic o f e a c h c o n v e r sa tio n .

N O W BEGIN THE RECORDING PROGRAM AT EXERCISE 20.

1. What is the topic o f Conversation 1?

2. What is the topic o f Conversation 2?

3. What is the topic o f Conversation 3?


LISTENING COMPREHENSION

S k il l 21: D R A W C O N C L U S IO N S A B O U T W HO , WHAT, W H EN , W H ERE

A s y o u listen to e a c h c o n v e r sa tio n in L iste n in g P art B, y o u sh o u ld b e tryin g to se t th e situ


a tio n in you r m in d . You sh o u ld b e th in k in g th e fo llo w in g th o u g h ts:
W ho is talking?
W hen does the conversation probably take place?
W here does the conversation probably take place?
W h a t is the source o f info rm a tio n fo r the conversation ?

E xam ple

On the recording, you hear:


(man) Why do you have so m any books?
(woman) I need them for my paper on George Washington. Do
you know how I can check them out?
(man) Yes, you should go downstairs to the circulation desk
and fill out a card for each book.

You think:
Who is probably talking? (two students)
Where are they? (in the library)
What course are they discussing? (American History)

E X E R C ISE 21: L isten to th e first part o f e a c h o f th e c o n v e rsa tio n s an d try to im a g in e th e


situ a tio n . T h e n answ er th e q u e s tio n s in th e te x t.

N ow BEGIN THE RECORDING PROGRAM AT EXERCISE 2 1.

Conversation 1
1. W ho is probably talking?

2. W here does the conversation take place?

Conversation 2
1. W ho is probably talking?

2. W hen does the conversation take place?

3. What is the source o f the m ans information?

Conversation 3
1. W ho is probably talking?

2. W hen does the conversadon take place?

3. What is the source o f the information?


LISTENING PART B

S k i l l 22: LISTEN FOR A N S W E R S IN ORDER

T h e r e are two p o ssib le m e th o d s to u se w h ile y o u listen to a c o n v e r sa tio n in th e L iste n in g


Part B o f th e T O E F L test.

You can j u s t listen to the conversation (a n d ignore the answers).


You can fo llo w along w ith the answers w hile you listen.

S o m e stu d e n ts p r e fe r to ju s t liste n to th e c o n v e r sa tio n w h ile it is b e in g s p o k e n , a n d i f th a t


m e th o d w orks w ell fo r y ou , th e n th at is w h at y o u sh o u ld d o . O th e r stu d e n ts fin d that th ey
can answ er m o r e q u e s tio n s c o rr ec tly i f th e y r e a d a lo n g w ith th e answ ers w h ile th e c o n v e r
sation is b e in g sp o k e n . B e ca u se th e d e ta il q u e s tio n s are a n sw e re d in ord er, it is p o s sib le to
read a lo n g w h ile y o u listen to th e c o n v e r sa tio n in th e r e c o r d in g p ro g ra m .

Exam ple

On the recording, you hear: In your test book, you read (same time):
(man) Can I help you? 1. (A) A checking account.
(woman) I m Interested in opening an account. (B) A savings account.
(man) Well, we have several different types o f (C) A money market account.
accounts: checking accounts, savings (D) A time deposit account.
accounts, money market accounts,
time deposit accounts.
(woman) I t s a checking account that I am
interested in.
(man) I can help you w ith that. First, you 2. (A) A form.
have to fill out a form, and then I need (B) An account.
to see som e identification. That's (C) A piece of identification.
about all there is to it. (D) A check.
(woman) That sounds easy enough. Thanks for
your help.

On the recording, you hear:


(narrator) 1. What type o f account does the
w om an want?
2. What does the m an need for her
to show him?

W h en y o u read th e an sw ers to th e first q u e s tio n , you can a n tic ip a te th a t th e first q u e s tio n


is: W hat type o f account ? As y o u listen , y o u d e te r m in e th a t th e w o m a n w an ts a checking ac
count. T h e r e fo r e , y o u c a n a n tic ip a te th a t th e b e st answ er to th e first q u e s tio n is (A ) .
W h e n y o u r ea d th e an sw ers to th e s e c o n d q u e s tio n , y o u c a n .a n ticip a te th a t th e se c
o n d q u e s tio n is g o in g to ask W hat th in g . . . ? In th e c o n v e r sa tio n , th e m an asks h e r to fill
o u t a. fo r m a n d sh o w s o m e identification, so as y o u are lis te n in g y o u can a n tic ip a te th at th e
c o r r e c t answ er to th e s e c o n d q u e s tio n is e ith e r (A) o r (C ). W h e n y o u h e a r th e q u e s tio n ,
y o u can d e te r m in e th a t th e b e st an sw er is an sw er (C ).
LISTENING COMPREHENSION

T O E F L E X E R C IS E 22: L isten to e a c h c o m p le te c o n v e r sa tio n a n d an sw er th e q u e s tio n s


th a t follow .

N O W BEGIN THE RECORDING PROGRAM AT TOEFL EXERCISE 22.


o
1. (A) Find work on campus. 9. (A) Sleeping outside on the ground.
(B) Work in the em ploym ent office. (B) Spending time in a h ot tub.
(C) H elp students find jobs. (C) Relaxing at the lodge.
(D) Ask the woman questions. (D) Enjoying excellent food.

2. (A) In the library. 10. (A) S h ed b e scared, but shed like to


(B) In a classroom. try.
(C) In a cam pus office. (B) She can t wait.
(D) In an apartment. (C) It w ould be quite exciting for her.
(D) Shed prefer not to try.
3. (A) N o m ore than ten.
(B) At least twenty. 11. (A) All kinds o f pollution.
(C) N ot m ore than twenty. (B) How acid rain has harm ed the
(D) U p to ten. earth.
(C) Pollution from cars and factories.
4. (A) Every m orning. (D) T he causes and possible effects o f
(B) A fternoons and weekends. acid rain.
(C) W hen h e s in class.
(D) Weekdays. 12. (A) Nuclear power.
(B) Electricity.
5. (A) Fill out a form.
(C) Burning coal and oil.
(B) Give her som e additional
(D) Solar power.
inform ation.
(C) Tell her som e news. 13. (A) From sulfur dioxide and water
(D) Phone her. vapor.
(B) From sulfur dioxide and nitrogen
6. (A) Just before a vacation. oxide.
(B) Just after the end o f a school (C) From nitric acid and sulfur
semester. dioxide.
(C) At the end o f the summer. (D) From water vapor and nitric acid.
(D) Just after a break from school.
14. (A) Only in North America.
7. (A) A trip to visit the Eskimos. At the N orth and South Poles.
(B)
(B) A trip the woman is planning to In parts o f several northern
(C)
take. continents.
(C) A trip the man has already taken. (D) In equatorial areas.
(D) A cam ping trip the man and
woman took. 15. (A) She should protect herself from
the rain.
8. (A) T hree hours.
(B) She should clean up the water
(B) T hree com plete days.
supply.
(C) T hree classes.
(C) She should read a novel.
(D) T hree weeks.
(D) She should get m ore information
about acid rain.
LISTENING PART B

T O E F L REV IEW E X E R C ISE (S k ills 1 8 -2 2 ): In this e x e r c ise , y o u will u s e all o f th e in fo r


m a tio n th at y o u le a r n e d in S k ills 18 th r o u g h 22.

B e fo re th e r e c o r d in g p ro g ra m b e g in s, y o u sh o u ld read over th e an sw ers to q u e s tio n s 1


th r o u g h 15 a n d d o th e fo llow in g:
A n ticip a te the topics you w ill hear.
A n ticip a te the questions.

W h ile y o u are lis te n in g to th e c o n v e r sa tio n s, y o u sh o u ld d o th e follo w in g :


L isten fo r the topic in the fir s t lines.
D ra w conclusions about the situ a tio n (w h o, w hat, w h e n , w h er e).
L isten fo r the answers in order.

N O W BEGIN THE RECORDING PROGRAM AT TOEFL REVIEW EXERCISE SKILLS ( 18 -22).


n
(A) To a concert. (A) Laramie.
(B) To a rehearsal. (B) Devils Tower N ational M onum ent.
(C) To a lecture. (C) Old Faithful.
(D) To the library. (D) Wyoming.
2. (A) O ne. 9. (A) Hear again about Yellowstone.
(B) Two. (B) Take a trip to Yellowstone.
(C) Three. (C) Get a jo b in a national park.
(D) Four. (D) Move to Yellowstone.

3. (A) T he bus does not go directly to the 10. (A) How and when we celebrate
Music Building. Thanksgiving.
(B) T he bus goes very slowly to the (B) The traditional Thanksgiving
Music Building. dinner.
(C) The bus som etim es does not com e. (C) W hen Thanksgiving began.
(D) T he bus will not arrive for a while. (D) Abraham Lincoln.

4. (A) Walk. 11. (A) With colonists in Massachusetts.


(B) Wait for the bus. (B) Alone and thinking abou t how
(C) Miss the lecture. Thanksgiving developed.
(D) T hink o f another plan. (C) With a big Thanksgiving dinner.
(D) In an untraditional manner.
5. (A) Boring.
(B) Fantastic. 12 . (A) The terrible winter.
(C) Lengthy. (B) The corn harvest.
(D) Faithful. (C) The develop m en t o f Thanksgiving
Day.
6. (A) By car.
(D) For getting the w hole family
(B) By plane. together.
(C) By train.
(D) By bicycle. 13. (A) At many different times.
(B) In July.
7. (A) She went directly to Yellowstone.
(C) Any tim e in November.
(B) She spent a few weeks in Laramie.
(D) On a Thursday in Novem ber.
(C) She stopped at the Devil's Tower
National M onum ent.
(D) She m ade a few stops before going
on to Yellowstone.
LISTENING COMPREHENSION

T H E LIS TE N IN G PART C Q U E S T IO N S

P art C o f th e L iste n in g C o m p r e h e n s io n s e c tio n o f th e T O E F L test c o n sists o f th r e e talks,


e a c h fo llo w e d by a n u m b e r o f q u estio n s. You w ill h e a r th e talks a n d th e q u e s tio n s o n a
rec o r d in g ; th ey a re n o t w ritten in y o u r test b o o k . Y ou m u st c h o o s e th e b e st a nsw er to each
q u e s tio n fro m th e fo u r c h o ic e s th a t are w ritten in y o u r te st b o o k . L ike th e c o n v e rsa tio n s
in L iste n in g Part B, th e talks are o fte n a b o u t so m e a sp e c t o f s c h o o l life o r to p ic s cu rren tly
in th e new s. It is a lso very c o m m o n fo r th e talks to b e s h o r te n e d v ersio n s o f le c tu r e s from
c o u r se s ta u g h t in A m e ric a n c o lle g e s an d u n iv er sitie s.

Exam ple

On the recording, you hear:


(narrator) Q uestions I th ro u g h 4. Listen to a talk about the
settlement o f America.
(woman) The settling o f the vast farm lands in central North
America was delayed at least partly because o f an
error by one man. In the early nineteenth century,
Lieutenant Zebulon Pike o f the U.S. Army w as'sent
out to explore and chart the huge expanses o f land in
the center o f the continent. When he returned from
his explorations, he wrote a report in which he
erroneously stated that the vast plains in the central
part o f the continent were desertlike, comparable to
the Sahara in Africa. In reality, however, these vast
plains contained some o f the m ost fertile farmland in
the world. Because o f Pikes mistake, the m aps o f the
day depicted the central part o f what is today the
United States as a vast desert rather than the excellent
and available farm land that it was. This mistaken
belief about the nature o f those lands caused settlers
to avoid the central plains for years.

Questions:

1. On the recording, you hear:


(narrator) What is the topic o f this talk?

In your test book, you read: (A) Zebulon Pikes career.


(B) A mistake that influenced the
settlement of America.
(C) A report for the army.
(D) The farmlands.

2. On the recording, you hear:


(narrator) How did Pike describe the area that he
explored?

In your test book, you read: (A) As a desert.


(B) As usable for army purposes.
(C) As located in the Sahara.
(D) As available for farmland.
(continued on next page)
LISTENING PART C

3. On the recording, you hear:


(narrator) W hat was this area really like?

In your test book, you read: (A) It was a vast desert.


(B) It was covered with farms.
(C) It was excellent farmland.
(D) It was similar to the Sahara.
4. On the recording, you hear:
(narrator) This talk w ould probably be given in
which o f the following courses?

In your test book, you read: (A) Agricultural Science.


(B) American History.
(C) Geology of the United States.
(D) Military Science.

T h e first q u e s tio n asks a b o u t th e to p ic o f th e talk. T h e to p ic o f th e talk is f o u n d in th e


first s e n te n c e o f th e talk: T he settling o f the vast fa r m la n d s in central N o rth A m erica was delayed
at least partly because o f a n error by one m an. T h e r e fo r e , th e b e st an sw er to th e q u e s tio n is
(B ). T h e s e c o n d q u e s tio n is a d e ta il q u e s tio n th at asks h o w P ike d e sc r ib e d th is area. It is
sta te d in th e talk th a t P ike wrote a report in w hich he erroneously stated th a t the v a st p la in s in the
central p a r t o f the continent were desertlike. . . . T h e r e fo r e , th e b e st an sw er to th is q u e s tio n is
( A ) . T h e th ird q u e s tio n is a n a d d itio n a l d e ta il q u e s tio n th a t asks w h a t th e a r ea was really
lik e. B e ca u se th e talk in d ic a te s th a t in reality . . . these vast p la in s contained some o f the most
fertile fa r m la n d in the world, th e b e st an sw er to th is q u e s tio n is (C ) . T h e fo u r th q u e s tio n is
an in fe r e n c e q u e s tio n . It asks in w h ic h c o u r se this le c tu r e w o u ld p ro b a b ly b e g iv e n . T h e
w o r d probably in d ic a te s to y o u th at th e q u e s tio n is n o t a n sw ered d ir ec tly in th e talk. You
m u s t draw a c o n c lu s io n fro m th e in fo r m a tio n in th e talk to an sw er th is q u e s tio n . B e ca u se
th is talk r efe rs to the early nineteenth century a n d d isc u sse s th e settling o f the v a st fa r m la n d s in
central N o rth A m erica, it w o u ld p ro b a b ly b e g iv e n in a n A m e ric a n H is to r y c o u r se . T h e b e st
an sw er to th is q u e s tio n is ( B ) .

_________________ STRATEGIES FORTHE LISTENING PART C QUESTIONS_________________

1. If you have tim e , p review th e an sw ers t o th e L istening Part C q u estio n s. While you
are looking at the answers, you should try to do the following:
Anticipate the to p ic s of the talks you will hear.
Anticipate the q u estio n s for each of the groups of answers.
2. Listen carefully to th e first lin e o f th e talk. The first line of the talk often contains the
main idea, subject, or topic of the talk, and you will often be asked this type of question.
3. A s you listen to th e talk, draw c o n c lu sio n s a b o u t th e situ a tio n o f th e talk: w h o is
talking, w h ere o r w h en th e talk ta k es p la ce, w h ich c o u r se th is lec tu r e m ig h t be
given in. You will often be asked to make such inferences about the talk.
4. A s you listen to th e talk, follow alon g w ith th e an sw ers in your t e s t b o o k r.nd try
to d e te r m in e th e c o r r e c t an sw ers. Detail questions are generally answered in order In
the talk, and the answers often sound the same as what Is said on the recording.
5. You sh ould g u e ss even if you are n o t su re . Never leave any answers blank.
6. U se any rem aining tim e to look ahead a t th e answ ers to th e q u estio n s th a t follow .
LISTENING COMPREHENSION

T h e fo llo w in g sk ills w ill h e lp y o u to im p le m e n t th e s e str a teg ie s in P art C o f th e L iste n in g


C o m p r e h e n s io n se c tio n o f th e T O E F L test.

BEFORE L IS T E N IN G ________________________________________

S k i l l 23: A N TIC IPA TE TH E TOPICS

It is very h e lp fu l to you r overall c o m p r e h e n sio n if y o u k n ow w h at to p ic s to e x p e c t in Lis


te n in g P art C. You sh o u ld th e r efo r e try to a n tic ip a te th e to p ics th a t you w ill be h e a rin g
(as y o u d id in L iste n in g P art B ). F or e x a m p le , are th e talks a b o u t A m erica n history, o r lit
era tu re, o r so m e a sp ec t o f sc h o o l life? A h e lp fu l strategy is th e r efo r e to lo o k b riefly at the
answ ers in th e test b o o k , b e fo r e y o u a ctu ally h ea r th e talks o n th e r e c o r d in g , a n d try to
d e te r m in e th e to p ic s o f th e talks th a t you w ill hear.

EX ER C ISE 23: L o o k at th e answ ers to th e five q u e stio n s to g e th er , a n d try to a n ticip a te


th e to p ic o f th e talk for th o s e five q u estio n s. (O f c o u r se , y o u c a n n o t always d e te r m in e e x
actly w h a t th e to p ic is, b u t y o u o fte n can g e t a g e n e r a l id e a .) Q u e stio n s 1 th r o u g h 5 have
b e e n a n sw ered fo r you.

(A) During a biology laboratory 4. (A) Room assignments.


session. (B) Exam topics.
(B) In a biology study group. (C) Reading assignments.
(C) On the first day o f class. (D) T he first lecture.
(D) Just before the final exam. 5. (A) Exams and lab work.
(A) O nce a week. (B) Reading and writing assignments.
(B) Two times a week. (C) Class participation and grades on
(C) T hree times a week. examinations.
(D) For fifteen hours. (D) Lecture and laboratory attendance.

(A) To do the first laboratory


assignment.
(B) To take the first exam.
(C) To study the laboratory manual.
(D) To read o n e chapter o f the text.

W h a t is th e to p ic o f th e talk for q u e s tio n s 1 th r o u g h 5?

the requirements of a biology class

You can g u e ss this b e c a u se o f th e fo llo w in g clues:


biology
fir s t day o f class
reading assignm ents
exams
lab work
LISTENING PART C

6. (A) What caused the Ring o f Fire. 9. (A) They are not so violent.
(B) The volcanoes o f the Ring o f Fire. (B) They are located alon g the Ring o f
(C) Hawaiian volcanoes. Fire.
(D) D ifferent types o f volcanoes. (C) They contain a lo t o f gas.
(D) They contain thick lava.
7. (A) The Ring o f Fire.
(B) The characteristics o f volcanoes in 10. (A) A volcano on the Ring o f Fire.
the Ring o f Fire. (B) An island in Hawaii.
(C) The volcanoes o f Hawaii. (C) A long, low volcanic m ountain.
(D) Mauna Loa. (D) An explosive volcano.
8. (A) In Hawaii.
(B) In the U nited States.
(C) A long the Ring o f Fire.
(D) Within the Ring o f Fire.

W hat is th e to p ic o f th e talk fo r q u e s tio n s 6 th r o u g h 10?

11. (A) An artist. 14. (A) T he American H istory M useum.


(B) A tour guide. (B) T he Sm ithsonian Arts and
(C) An Indian. Industries Building.
(D) Orville Wright. (C) T he W ashington M useum.
(D) The National Air and Space
12. (A) Several.
Museum.
(B) Sixty thousand.
(C) Sixteen m illion. 15. (A) To the White H ouse.
(D) M illions and millions. (B) To the Sm ithsonian.
(C) To the mall.
13. (A) The National Air and Space
(D) To various other m useum s.
Museum.
(B) The Museum o f Natural History.
(C) The American History Museum.
(D) The Sm ithsonian Arts and
Industries Building.

W hat is th e to p ic o f th e talk for q u e s tio n s 11 th r o u g h 15?


LISTENING COMPREHENSION

S k il l 24: A N TIC IP A TE T H E Q U E S T IO N S

It is very h e lp fu l to y o u r ability to an sw er in d iv id u a l q u e stio n s in L iste n in g P art C i f y ou


ca n a n tic ip a te w h at th e q u e s tio n s w ill b e a n d listen sp ec ific a lly fo r th e an sw ers to th o s e
q u e s tio n s (as you d id in L iste n in g P art B ).

Exam ple

In your test book, you read:


(A) For three weeks.
(B) For three days.
(C) For three months.
(D) For three hours.

You try to anticipate the question:


How long does (something) last?

In th is e x a m p le, y o u can b e q u ite c e r ta in th a t o n e o f th e q u e stio n s w ill b e a b o u t h o w lo n g


so m e th in g lasts. S in c e you are su re th a t th is is o n e o f th e q u e s tio n s, y o u c a n liste n ca re
fully for c lu e s th a t w ill give y o u th e answ er. T h is e x a m p le sh ow s th a t a h e lp fu l strategy is
th e r e fo r e to lo o k b riefly at th e an sw ers in th e test b o o k , b e fo r e y o u actu ally h e a r th e talks
o n th e rec o rd in g , a n d try to d e te r m in e th e q u e s tio n s that y o u will b e ask ed to answer.

EXER CISE 24: Study th e fo llo w in g answ ers a n d try to d e te r m in e w h a t th e q u e s tio n s will
be. (You sh o u ld n o te th at p e r h a p s y o u w ill o n ly be ab le to p r e d ict p art o f a q u e stio n , rather
th an th e c o m p le te q u e s tio n .) If y o u c a n n o t p r e d ict th e q u estio n in a sh o rt p e r io d o f tim e,
th e n m ove o n to th e n e x t g r o u p o f answ ers. Q u e stio n 1 has b e e n an sw ered fo r you.

1 rw stin n - When does the talk probably take place?_________________________________


(A) During a biology laboratory session.
(B) In a biology study group.
(C) O n the first day o f class.
(D) Just before the final exam.
2. Q uestion:_______________________________ ________________________________________________
(A) O nce a week.
(B) Two times a week.
(C) Three tim es a week.
(D) For fifteen hours.
3. Q uestion:______________________________ _________________________________________________
(A) To do the first laboratory assignment.
(B) To take the first exam.
(C) To study the laboratory manual.
(D) To read o n e chapter o f the text.
4. Q uestion:______ ___________________________ ______________________________________________
(A) Room assignments.
(B) Exam topics.
(C) Reading assignments.
(D) T he first lecture.
LISTENING PART C 107

5. Q u estion :_____________________________________________
(A) Exams and lab work.
(B) Reading and writing assignm ents.
(C) Class participation and grades on exam inations.
(D) Lecture and laboratory attendance.
6. Q uestion:_____________________________________________
(A) What caused the Ring o f Fire.
(B) The volcanoes o f the Ring o f Fire.
(C) Hawaiian volcanoes.
(D) D ifferent types o f volcanoes.
7. Q uestion:_____________________________________________
(A) The Ring o f Fire.
(B) The characteristics o f volcanoes in the Ring o f Fire.
(C) The volcanoes o f Hawaii.
(D) Mauna Loa.
8. Q u estion :_____________________________________________
(A) In Hawaii.
(B) In the U nited States.
(C) A long the Ring o f Fire.
(D) Within the Ring o f Fire.
9. Q uestion:_____________________________________________
(A) They are not so violent.
(B) They are located along the Ring o f Fire.
(C) They contain a lot o f gas.
(D) They contain thick lava.
10. Q u estion :_____________________________________________
(A) A volcano on the Ring o f Fire.
(B) An island in Hawaii.
(C) A long, low volcanic mountain.
(D) An explosive volcano.
11. Q u estion :_____________________________________________
(A) An artist.
(B) A tour guide.
(C) An Indian.
(D) Orville Wright.
12. Q uestion:_____________________________________________
(A) Several.
(B) Sixty thousand.
(C) Sixteen million.
(D) M illions and m illions.
13. Q u estion :_____________________________________________
(A) T he National Air and Space Museum.
(B) The M useum o f Natural History.
(C) T he American History M useum.
(D) T he Sm ithsonian Arts and Industries Building.
LISTENING COMPREHENSION

14. Q u estion :_________________________________ ______


(A) The American History Museum.
(B) The Sm ithsonian Arts and Industries Building.
(C) The Washington Museum.
(D) The National Air and Space Museum.
15. Q u estion :________________________________________
(A) To the White House.
(B) To the Smithsonian.
(C) To the mall.
(D) To various other museums.

W H IL E LIS TE N IN G

S k i l l 25: DETERM INE TH E TO PIC


As you listen to ea ch talk in L isten in g Part C, y o u sh o u ld b e th in k in g a b o u t th e to p ic (su b
je c t) o r m a in id e a fo r th e talk (as you d id in L iste n in g Part B ). S in ce th e first se n te n c e is
g e n e ra lly a to p ic se n te n c e , y o u sh o u ld b e ask in g y o u r se lf w hat th e to p ic is w h ile you are
lis te n in g ca refu lly to th e first p art o f th e talk.

Exam ple

On the recording, you hear:


(man) The major earthquake that occurred east o f Los Angeles in
1971 is still affecting the economy o f the area today.

You think:
The topic of the talk is the effect of the 1971 earthquake on
Los Angeles today.

E X ER C ISE 25: L isten to th e first part o f e a c h o f th e talks, a n d d e c id e o n th e to p ic o f


e a ch talk.

N O W BEGIN THE RECORDING PROGRAM AT EXERCISE 25.

1. What is the topic o f Talk 1?

2. What is the topic o f Talk 2?

3. What is the topic o f Talk 3?


LISTENING PART c

S k i l l 26: D R A W C O N C L U S IO N S A B O U T W HO , WHAT, W H EN , W H ER E
As y o u listen to e a c h talk in L iste n in g Part C , y o u sh o u ld b e tr y in g to se t th e situ a tio n in
you r m in d (a s y o u d id in L iste n in g Part B ). You s h o u ld b e th in k in g th e fo llo w in g
th ou gh ts:

W ho is talkin g ?
W hen does the talk probably take place?
W here does the talk probably take place?
W h a t course is the talk concerned w ith?
W h a t is the source o f in fo rm a tio n fo r the talk?

Exam ple

On the recording, you hear:


(woman) The next stop on our tour o f Atlanta will be the original
hom e o f Coca-Cola, at 107 Marietta Street. Coca-Cola
was manufactured at this location un til early in
September o f 1888.

You think:
Who is probably talking? (a tour guide)
Where are they? (in Atlanta)
When does the talk take place? (in the middle of a tour)

EX ER C ISE 26: L iste n to th e first p art o f e a c h o f th e talks a n d try to im a g in e th e situ a


tio n . T h e n an sw er th e q u e s tio n s in th e text.

o N O W BEGIN THE RECORDING PROGRAM AT EXERCISE 26.


Talk 1
1. W ho is probably talking? _____________________________________________

2. W here does the talk probably take p l a c e ? _____________________________________________

3. W hen does the talk probably take place? _____________________________________________

4. What course is b ein g discussed? _____________________________________________


Talk 2
1. W ho is probably talking? _____________________________________________

2. W here does the talk probably take place? _____________________________________________

3. W hen does the talk probably take place? _____________________________________________

4. What course is b ein g discussed? ---------------------------------------------------------------------


Talk 3
1. W ho is probably talking? ---------------------------------------------------------------------

2. W here does the talk take place? _____________________________________________

3. W hen d oes the talk take place? _____________________________________________


LISTENING COMPREHENSION

S k i l l 27: LISTEN FOR ANSW ERS IN ORDER

T h e r e are tw o p o ssib le m e th o d s to u se w h ile y o u liste n to th e talks in L iste n in g Part C.


You can j u s t listen to the talk (a n d ignore the answers).
* You can follow along w ith the answers w hile you listen.
S o m e stu d e n ts p refer to ju s t listen to th e talk w h ile it is b e in g sp o k e n , a n d i f th a t m e th o d
w orks w ell fo r y o u , th e n th at is w h at y o u sh o u ld d o . O th e r stu d e n ts fin d th a t th ey can a n
sw er m o r e q u e s tio n s co rrectly i f they read a lo n g w ith th e answ ers w h ile th e talk is b e in g
g iv en . B e ca u se th e d etail q u e stio n s are a n sw ered in ord er, it is p o ssib le to read a lo n g
w h ile y o u lis te n to th e talk in th e r e c o r d in g p rogram .

Exam ple

On the recording, you hear: In your test book, you read (same time):
(woman) The Great Chicago Fire began on 1. (A) In a bam.
October 8, 1871, and, according to (B) In Mrs. OLearys home.
legend, began when a cow knocked (C) In a cow pasture.
over a lantern in Mrs. O Learys (D) In a lantern factory.
bam . No matter how it began, it
was a disastrous fire. The preceding 2. (A) The dry weather prior to the fire
sum m er had been exceedingly dry made it worse.
in the Chicago area, and the extreme (B) It happened during the summer.
dryness accompanied by Chicagos (C) Chicagos winds made it worse.
infam ous winds created an inferno (D) It killed many people.
that destroyed 18,000 buildings and
killed more than 300 people before
it was extinguished the following day.

On the recording, you hear:


(narrator) . According to legend, where did
the Great Chicago Fire begin ?
2. Which o f the following is not
true about the Great Chicago Fire?

W h en you read th e answ ers to th e first q u e stio n , y o u can an ticip a te th a t th e first q u estio n
is: Where did som ething happen'? As you listen , y o u d e te r m in e th at th e fire b e g a n in M rs.
O 'Leary's barn. T h e r e fo r e , y o u can a n tic ip a te that th e b est answ er to th e first q u estio n
is (A ).
I f you rea d th e answ ers to th e se c o n d q u e s tio n w h ile y o u listen to th e talk, y o u can d e
te r m in e th at answ ers (A ), (C ), an d (D ) are tru e. A n sw er (B ) is n o t true: th e fire d id n o t
b e g in in th e su m m er, it b e g a n in October, w h ich is in th e a u tu m n . T h e r e fo r e , answ er (B ) is
th e b e st answ er to th e q u e stio n Which o f the fo llo w in g is n o t true about the Great Chicago F ire?
LISTENING PART C

T O E F L EXER CISE 27: L isten to e a c h c o m p le te talk a n d answ er th e q u e s tio n s th a t follow ,

n N O W BEGIN THE RECORDING PROGRAM AT TOEFL EXERCISE 27.

1. (A) During a biology laboratory 9. (A) They are not so violent.


session. (B) They are located along the Ring o f
(B) In a biology study group. Fire.
(C) On the first day o f class. (C) They contain a lot o f gas.
(D) Just before the final exam. (D) They contain thick lava.
2. (A) O nce a week. 10. (A) A volcano on the Ring o f Fire.
(B) Two times a week. (B) An island in Hawaii.
(C) Three times a week. (C) A long, low volcanic m ountain.
(D) For fifteen hours. (D) An explosive volcano.
3. (A) To do the first laboratory 11. (A) An artist.
assignment. (B) A tour guide.
(B) To take the first exam. (C) An Indian.
(C) To study the laboratory m anual. (D) Orville Wright.
(D) To read o n e chapter o f the text.
12. (A) Several.
4. (A) Room assignments. (B) Sixty thousand.
(B) Exam topics. (C) Sixteen m illion.
(C) Reading assignments. (D) M illions and m illions.
(D) T he first lecture.
13. (A) T he National Air and Space
5. (A) Exams and lab work. Museum.
(B) Reading and writing assignments. (B) T he Museum o f Natural History.
(C) Class participation and grades on (C) T he Am erican H istory M useum.
exam inations. (D) T he Sm ithsonian Arts and
(D) Lecture and laboratory attendance. Industries Building.
6. (A) What caused the Ring o f Fire. 14. (A) T he Am erican H istory M useum.
(B) T h e volcanoes o f the Ring o f Fire. (B) The Sm ithsonian Arts and
(C) Hawaiian volcanoes. Industries Building.
(D) Different types o f volcanoes. (C) The W ashington M useum.
(D) T he National Air and Space
7. (A) T h e Ring o f Fire.
Museum.
(B) T h e characteristics o f volcanoes in
the Ring o f Fire. 15. (A) To the W hite H o u .
(C) T he volcanoes o f Hawaii. (B) To the Sm ithsonian.
(D) Mauna Loa. (C) To the mall.
(D) To various other m useum s.
8. (A) In Hawaii.
(B) In the U n ited States.
(C) A long the Ring o f Fire.
(D) Within the Ring o f Fire.
LISTENING COMPREHENSION

T O E F L REV IEW E X E R C ISE (S k ills 2 3 -2 7 ): In th is e x e r c ise , y o u w ill u se all o f th e in fo r


m a tio n th a t y o u le a r n e d in Skills 2 3 th r o u g h 27.

B e fo r e th e r e c o r d in g p rogram b e g in s, y o u s h o u ld read over th e answ ers to q u e stio n s 1


th r o u g h 12 a n d d o th e follo w in g :
A n tic ip a te the topics you w ill hear.
A n tic ip a te the questions.

W h ile y o u are lis te n in g to th e talks, you sh o u ld d o th e follow in g:

L isten fo r the topic in the fir s t sentence.


D raw conclusions about the situ a tio n (w ho, w h at, w h e n , w h er e).
Listen f o r the answers in order.

n N O W BEGIN THE RECORDING PROGRAM AT TOEFL REVIEW EXERCISE (SKILLS 23-27).

(A) O ther librarians. 7. (A) Alcohol.


(B) Undergraduate students. (B) Nicotine.
(C) Students who are not in the (C) Caffeine.
business department. (D) A reduced supply o f blood.
(D) Graduate business students. g (A) It increases the flow o f blood to the
2. (A) It opens at 7:00 A.M. skin.
(B) It closes at 7:00 P.M. (B) It causes increased consum ption of
(C) It closes at midnight. alcohol.
(D) It is always open. (C) It prevents the skin from receiving
enough nourishm ent.
3. (A) Com puter area and business
(D) It causes stress.
materials.
(B) Magazines and newspapers. 9. (A) Before the Civil War.
(C) Business departm ent and library (B) At the end o f the Civil War.
staff offices. (C) At the beginning o f the twentieth
(D) First and second floors o f the century.
library. (D) W ithin the last decade.

4. (A) Go hom e. 10. (A) The Civil War ended.


(B) Return to class. (B) T he U.S. governm ent issued a large
(C) Work on the computers. am ount o f paper currency.
(D) Tour the library. (C) T he price o f gold plum m eted.
(D) T he value o f gold becam e inflated.
5. (A) A student in health services.
(B) A drug abuse lecturer. 11. (A) The president.
(C) A derm atologist. (B) The presidents brother.
(D) A representative o f the tobacco (C) The presidents brother-in-law.
industry. (D) T he presidents wife.

6. (A) How to reduce nicotine and other 12. (A) Issue greenbacks.
addictions. (B) Sell gold.
(B) How stress affects the skin. (C) Corner the gold markeL
(C) The effects o f alcohol on health. (D) H old its gold reserves.
(D) How to achieve optim al health.
l D l D l D l D l D l D l D l
TOEFL POST-TEST
SECTION 1
LISTENING COMPREHENSION
T im e a p p r o x im a te ly 3 5 m in u te s
( in c lu d in g th e r e a d in g o f t h e d ir e c t io n s fo r e a c h p a r t)

In this section of the test, you will have an opportunity to demonstrate your ability to understand
conversations and talks in English. There are three parts to this section, with special direotions for each
part. Answer all the questions on the basis of what is stated or im plied by the speakers you hear. Do
not take notes or write in your test book at any time. Do not turn the pages until you are told to do so.

P art A

D irections: In Part A you will hear short conversations between two people. After each conversation,
you will hear a question about the conversation. The conversations and questions will not be
repeated. After you hear a question, read the four possible answers in your test book and choose the
best answer. Then, on your answer sheet, find the number o f the question and fill in the space that
corresponds to the letter of the answer you have chosen.

Listen to an exam ple. Sam p le Answer


On the recording, you w ill hear: CD

(m an) That exam was just awful.
(w om an) Oh, it could have been worse.
(narrator) What does the w om an mean?

In your test book, you w ill read: (A) The exam was really awful.
(B) It was the worst exam she had ever seen.
(C) It couldn't have been more difficult.
(D) It wasn't that hard.

You learn from the conversation that the man thought the exam was very difficult and that the
woman disagreed with the man. The best answer to the question, "What does the wom an mean?" is
(D), "It wasnt that hard. Therefore, the correct choice is (D).

TOEFL* test directions an d form at are rep rin ted by perm ission
o f ETS, the copyright owner. However, all exam ples and test
questions are provided by P earson Education, Inc. LISTENING COMPREHENSION POST-TEST
I d I d I d I d I d I d
1. (A) Hell correct the exams this afternoon. 8. (A) Shes found a new ring.
(B) The exam will be at noon. (B) She would like a hug.
(C) He will collect the exams at 12:00. (C) Shes shopping for a carpet.
(D) The tests have not yet been graded. (D) Shes thankful she has a rag.

2. (A) Martha applied for a visa last month. 9. (A) In a department store.
(B) Martha's visa will last for only a (B) In a bank.
month. (C) In an accounting firm.
(C) Martha arrived last month without (D) In a checkout line.
her visa.
(D) Marthas visa was already delivered. 10. (A) Jane usually visits San Francisco for
her vacations.
3. (A) The professor described what the (B) Janes cousin often visits San
students should do. Francisco.
(B) There was a long line to register for (C) Whenever theres a holiday, Jane's
the required class. cousin goes to San Francisco.
(C) The professor required an outline. (D) Whenever theres a holiday, Jane
(D) The professor lined up for retirement. leaves San Francisco.

4. (A) Chuck had improved. 11. (A) He wishes he had something tc eat.
(B) This visit was better than the last. (B) He hopes he wont eat for weeks.
(C) Chuck looked at him in the hospital. (C) He wishes he hadn't eaten so much.
(D) Chuck didn't seem to be doing very (D) He wishes he werent eating.
well.
12. (A) Traffic should not be allowed.
5. (A) She thinks the tuition should be (B) She thinks that the traffic should stay
increased. outside.
(B) The semesters tuition is quite (C) She agrees that the traffic is noisy.
affordable. (D) Shell stay outside with the man.
(C) It costs too much.
(D) She has more than enough for tuition. 13. (A) The headings for today's reading
assignment.
6. (A) He thinks he got a good grade. (B) The chance to make the headlines.
(B) The history grades were all C or (C) Her reading ability.
above. (D) The daily newspaper.
(C) No one got history grades.
(D) All the grades were C or lower. 14. (A) The bus trip is only five minutes long.
(B) The man missed the bus by five
7. (A) The parking lots were full before minutes.
10:00. (C) The man doesnt have time to waste.
(B) It was impossible to start class by (D) The bus was five minutes late.
10:00.
(C) He parked the car before class at 15. (A) Its not possible to pass the class.
10:00. (B) She'll definitely fail.
(D) The possibility of finding a place to (C) It's always possible.
park increased. (D) She shouldn't say anything about the
class.

GO ON TO THE NEXT PAGE

I 14 LISTENING COMPREHENSION POST-TEST


lDlDlnlnlnlnlnl
16. (A) She gave Tom money to pay the rent. 24. (A) He agrees with the wom ans
(B) She was given money for the rent. suggestion.
(C) Tom borrowed money for the rent. (B) Parking is not free on the weekend.
(D) She had som e money to lend. (C) It is not necessary for them to park.
(D) He thinks they dont have to pay.
17. (A) The cake is extremely good.
(B) He never tasted the cake. 25. (A) He is eager to leave his job.
(C) He wished he hadn't tasted the cake. (B) He is unhappy at the thought of
(D) The cake has never been very good. retiring.
(C) He couldnt be unhappier about
18. (A) At the com er she ran into another car. retiring.
(B) She ran to Carl because she cared. (D) He is retiring too soon.
(C) She unexpectedly met one of her
relatives. 26. (A) He got the car he really wanted.
(D) Carl was running from place to place. (B) He didn't get a new car.
(C) The car that he got was not his first
19. (A) She shouldn't leave her purse here. choice.
(B) Shes probably in the apartment. (D) He didnt really want a new car.
(C) Her purse must not be in the
apartment. 27. (A) Mr. Drew pointedly asked the
(D) She left without taking her purse. president about the committee.
(B) The president pointed to Mr. Drews
20. (A) The landlord failed to collect rent on head.
the first of last month. (C) Mr. Drew became head of the new
(B) The tenants absolutely must pay rent commission.
by the first o f the month. (D) Mr. Drew was committed to the
(C) The landlord will not fail to collect presidents appointments.
your rent on the first o f next month.
(D) It is important to call the landlord 28. (A) She felt inferior.
about rent on the first of the month. (B) She wasnt furious.
(C) She felt there should have been more
21. (A) Taking the car out for a test drive. fairness.
(B) Listening to the noises. (D) She was extremely angry.
(C) Fixing the car herself.
(D) Getting the car repaired. 29. (A) The man would do the dishes.
(B) The plates did not need to be washed.
22. (A) Marthas jobs are easy. (C) The man would not be ready to go.
(B) Its easy to hold two jobs. (D) The dishes would not be done.
(C) Its better for Martha to have two jobs.
(D) Martha should slow down. 30. (A) He knew that grapes were cheaper
than cherries.
23. (A) The plane took off just after he (B) He didn't know that grapes were
arrived. cheaper than cherries.
(B) He arrived just after the plane took (C) He bought grapes because they were
off. cheaper than cherries.
(C) He wasnt in time to catch the plane. (D) He didn't buy either grapes or
(D) He arrived too late to catch the plane. cherries because of the price.

LISTENING COMPREHENSION POST-TEST I 15


i D l D l D l D l l l l
Part B

D ire ctio n s: In this part of the test, you will hear longer conversations. After each conversation, you
will hear several questions. The conversations and questions will not be repeated.
After you hear a question, read the four possible answers in your test book and choose the best
answer. Then, on your answer sheet, find the number o f the question and fill in the space that
corresponds to the letter of the answer you have chosen.
Remember, you are not allowed to take notes or write in your test book.

31. (A) Attend a football game alone. 35. (A) Trash orbiting Earth.
(B) Go to a sporting event. (B) A trip by an astronaut to the Moon.
(C) Eat in the cafeteria and study. (C) The overabundance of garbage on
(D) See a play. Earth.
(D) Becoming space scientists.
32. (A) Its the final game of the season.
(B) It's better than the drama 36. (A) From a lecture.
departments play. (B) In a magazine article.
(C) Its a very important game. (C) In a book.
(D) Its close to the cafeteria. (D) On a television program.

33. (A) A play. 37. (A) 17,000 pounds.


(B) A game. (B) 3,000 tons.
(C) A study group meeting. (C) 3,000 pounds.
(D) Dinner in the cafeteria. (D) 300 tons.

34. (A) Saturday night. 38. (A) She will be able to travel in space.
(B) After dinner in the cafeteria. (B) The problem will take care of itself.
(C) Sunday afternoon. (C) Scientists will find solutions to the
(D) Maybe next weekend. problem.
(D) The junk will fall to Earth.

GO ONTO THE NEXT PAGE

TOEFL test directions and form at are reprinted by perm ission


of ETS, the copyright owner. However, all exam ples and test
I 16 LISTENING COMPREHENSION POST-TEST questions are provided by Pearson Education, Inc.
i D l D l D l D l D l D l D l
Part C

D ire ctio n s: In this part of the test, you will hear several talks. After each talk, you will hear some
questions. The talks and questions will not be repeated.
After you hear a question, you will read the four possible answers in your test book and choose the
best answer. Then, on your answer sheet, find the number of the question and fill in the space that
corresponds to the letter of the answer you have chosen.

H e re is a n ex am p le.

O n th e re c o rd in g , yo u w ill h e ar:

(n a rr a to r) Listen to an instructor talk to his class about painting.

(m an ) Artist Grant Wood was a guiding force in the school o f painting know n as American
regionalist, a style reflecting the distinctive characteristics o f art from rural areas o f
the United States. Wood began drawing animals on the family farm at the age o f
three, and w hen he was thirty-eight one o f his paintings received a remarkable
a m ount o f public notice and acclaim. This painting, caller. American Gothic, " is a
starkly simple depiction o f a serious couple staring directly cut at the viewer.

Now listen to a sample question. Sam p le Answer

(n a rra to r) What style o f painting is know n as American regionalist?


(B)

I n y o u r te s t b o o k , you w ill re a d : (A) Art from America s inner cities.
(B) Art from the central region of the U.S.
(C) Art from various urban areas in the U.S.
(D) Art from rural sections of America.

The best answer to the question, "What style o f painting is known as American regionalist? is (D),
"Art from rural sections of America. Therefore, the correct choice is (D).

Now listen to another sample question. Sam p le Answer

(n a rra to r) What is the nam e o f Wood's m ost successful painting?

I n y o u r te st b o o k , you w ill re a d : (A) American Regionalist.



(B)
(B) "The Family Farm in Iowa.
(C) "American Gothic."
(D) "A Serious Couple.

The best answer to the question, "What is the name of Wood's most successful painting? is (C),
"American Gothic. Therefore, the correct choice is (C).
Remember, you are n o t allowed to take notes or write in your test book.

(w a it)
TOEFL test directions an d form at are reprinted by perm ission
of ETS, th e copyright owner. However, all exam ples and test
questions are provided by P earson Education, Inc. LISTENING COMPREHENSION POST-TEST I 17
i D l D l D l D l D l D l D l
39. (A) On the first day of class. 46. (A) Preparing for a trip.
(B) In the middle of the semester. (B) Writing a report about the weather.
(C) At the end o f class. (C) Beginning a study of the weather.
(D) In the final week of the semester. (D) Buying warm clothes for a trip.

40. (A) Later today. 47. (A) Modem American Authors.


(B) By Friday o f this week. (B) United States History.
(C) In two weeks. (C) American Democracy.
(D) In three weeks. (D) Nineteenth-Century American
Literature.
41. (A) Journal and magazine articles.
(B) Books from outside the library. 48. (A) The death o f Abraham Lincoln.
(C) Books listed in student journals. (B) The beauty of American democracy.
(D) Both books and journals. (C) The raising of plants.
(D) The maturity of poetry.
42. (A) TVvo.
(B) Three. 49. (A) Its a poem about the author.
(C) Five. (B) Its a poem about Abraham Lincoln.
(D) Seven. (C) Its a collection of twelve poems that
remained unchanged.
43. (A) In winter. (D) Its a volume of poetry that grew with
(B) In spring. its author.
(C) In summer.
(D) In fall. 50. (A) Leaves of Grass."
(B) "Song of Myself.
44. (A) Seasonable, with warm summers and (C) When Lilacs Last in the Dooryard
cold winters. Bloomed.
(B) Fairly constant and moderate. (D) American Democracy."
(C) Very humid.
(D) Extremely hot year-round.

45. (A) They come from the southwest.


(B) They come most days of the year.
(C) They are the hardest during the night.
(D) They increase the humidity.

This is the end of Section 1. Stop work on Section 1.


Turn off the recording.

[st o p] [st o p ] [s t o p ] STOP [s t o p ] [sto pJ Isto pJ

W h e n y o u fin ish th e test, y o u m ay d o th e fo llow in g:


T u rn to th e D ia g n o stic C h art o n p a g e s 5 8 3 -5 8 4 , a n d
cir cle th e n u m b e rs o f th e q u e s tio n s th a t y o u m issed .
T u rn to S c o r in g I n fo r m a tio n o n p a g e s 5 8 1 -5 8 2 , an d
d e te r m in e your T O E FL sc o r e.
T u rn to th e P ro g r ess C hart o n p a g e 5 9 1 , a n d a d d y o u r
sc o r e to th e charL

I 18 LISTENING COMPREHENSION POST-TEST


SECTION TW O

STRUCTURE AND
WRITTEN
EXPRESSION
120
2 2 * 2 * 2 * 2 2 2 2

DIAGNOSTIC PRE-TEST
SECTION 2
STRUCTURE AND WRITTEN EXPRESSION
T im e 2 5 m in u t e s
(in c lu d in g th e r e a d in g o f th e d ir e c t io n s )
N o w s e t y o u r c lo c k fo r 2 5 m in u te s .

This section is designed to measure your ability to recognize language that is appropriate for standard
written English. There are two types of questions in this section, with special directions for each type.

S tru c tu re

D irections: These questions are incomplete sentences. Beneath each sentence you will see four
words or phrases, marked (A), (B), (C), and (D). Choose the o n e word or phrase that best completes
the sentence. Then, on your answer sheet, find the number o f the question and fill in the space that
corresponds to the letter of the answer you have chosen.
Look at the following examples.

E x am p le I
S a m p le A nsw er
The president. . the election by a landslide. ()
(A) won
(B) he won
(C) yesterday
(D) fortunately

The sentence should read, "The president won the election by a landslide. Therefore you should
choose answer (A).

E xam ple II
S a m p le A nsw er
When _ . the conference? ( g )
(A) the doctor attended
(B) did the doctor attend
(C) the doctor will attend
(D) the doctors attendance

The sentence should read, "When did the doctor attend the conference? Therefore you should
choose answer (B).

GO ON TO THE NEXT PAGE


TOEFL test directions and form at are reprinted by perm ission
of ETS. the copyright owner. However, all exam ples and test
f
questions are provided by Pearson E ducation. Inc.
STRUCTURE AND WRITTEN EXPRESSION PRE-TEST 12 1
2 * 2 * 2 * 2 * 2 * 2 * 2 * 2

1. The North P ole _____ a latitude of 90 7. _____ off the Hawaiian coastline are living,
degrees north. others are dead.
(A) has (A) Coral reefs
(B) is having (B) Some types o f coral reefs
(C) which is having (C) There are many types o f coral reefs
(D) it has (D) While some types of coral reefs

2. _____ greyhound, can achieve speeds up to 8. People who reverse the letters of words
thirty-six miles per hour. to read suffer from dyslexia.

(A) The (A) if they tried


(B) The fastest (B) when trying
(C) The fastest dog (C) when tried
(D) The fastest dog, the (D) if he tries

3. The Mayflower was bound for Virginia, but 9. Featured at the Henry Ford M useum --------
a hurricane_____ off course. of antique cars dating from 1865.

(A) blew it (A) an exhibit is


(B) to blow it (B) an exhibit
(C) it blew (C) is an exhibit
(D) blowing it (D) which is an exhibit

4. The greenhouse effect occu rs--------heat 10. Rubber_____ from vulcanized silicones
radiating from the Sun. with a high molecular weight is difficult to
distinguish from natural rubber.
(A) when does the Earths atmosphere
trap (A) is produced
(B) does the Earths atmosphere trap (B) producing
(C) when the Earths atmosphere traps (C) that produces
(D) the Earth's atmosphere traps (D) produced

5. The Rose B ow l,______ place on New Years 11. _____ in scope, romanticism was a
Day, is the oldest postseason collegiate reaction against neoclassical principles.
football game in the United States.
(A) Mainly literary
(A) takes (B) It was mainly literary
(B) which takes (C) The main literature was
(C) it takes (D) The literature was mainly
(D) took
12. The Central Intelligence Agency (CIA)
6. Experim ents_______represent a giant step _____ came about as a result of the
into the medicine of the future. National Security Act of 1947.

(A) using gene therapy (A) what


(B) use gene therapy (B) it was
(C) they use (C) was what
(D) gene therapy uses (D) it was what

I 22 STRUCTURE AND WRITTEN EXPRESSION PRE-TEST


2 2 * 2 * 2 2 2 2 2

13. Oil shale is a soft, fine-grained 15. According to the World Health
sedimentary rock-------- oil and natural gas Organization,_____ there to be an
are obtained. outbreak of any o f the six m ost dangerous
(A) from diseases, this could be cause for
(B) is from quarantine.
(C) is which (A) were
(D) from which (B) they were
(C) there were
14. --------- appears considerably larger at the (D) were they
horizon than it does overhead is merely an
optical illusion.
(A) The Moon
(B) That the Moon
(C) When the Moon
(D) The Moon which

STRUCTURE AND WRITTEN EXPRESSION PRE-TEST 123


2 * 2 * 2 * 2 * 2 * 2 * 2 2

W r it t e n E x p r e s s io n

D irections: In these questions, each sentence has four underlined words or phrases. The four
underlined parts of the sentence are marked (A), (B), (C), and (D). Identify the o n e underlined word
or phrase that must be changed in order for the sentence to be correct. Then, on your answer sheet,
find the number of the question and fill in the space that corresponds to the letter of the answer you
have chosen.
Look at the following examples.

Exam ple I Sam ple Answer

The four string on a violin are tuned


A B C D
in fifths.

The sentence should read, "The four strings on a violin are tuned in fifths. Therefore, you should
choose answer (B).

Exam ple II Sam ple Answer

The research for the book Roots taking d ) #


A B C
Alex Haley twelve years.
D

The sentence should read, "The research for the book Roots took Alex Haley twelve years." Therefore,
you should choose answer (C).

TOEFL* test directions an d form at are reprinted by perm ission


o f ETS, the copyright owner. However, all examples an d test
124 STRUCTURE AND WRITTEN EXPRESSION PRE-TEST questions arc provided by Pearson Education, Inc.
2 * 2 * 2 * 2 * 2 * 2 * 2 * 2

16. Segregation in public schools was declare unconstitutionalby the Supreme Courtin
~A B C D
1954.

17. Sirius, the Dog Star, is the most brightest star in the sky with an absolute magnitude
A B
about twenty-three times that of the Sun.
C D~

18. Killer whales tend to wander in family clusters that hunt, play, and resting together.
A B C D

19. Some of the m ost useful resistor material are carbon, metals, and metallic alloys.
A B C D

20. The community of Bethesda, Maryland, was previous known as Darcys Store.
A B C D

21. J. H. Pratt used group therapy early in the past century when he brought
A B C
tuberculosis patients together to discuss its disease.
TT

22. Alloys of gold and copper have been widely using in various types of coins.
~~K~ B C D

23. The United States has import all carpet w ools in recent years because dom estic
A B C
wools are too fine and soft for carpets.
D

24. Banks are rushing to merge because consolidations enable them to slash theirs costs
A B C~
and expand.
D

25. That water has a very high specific heat means that, without a large temperature
~~A~ B
change, water can add or lose a large number of heat.
C D

26. Benny Goodman was equally talented as both a jazz performer as well ac a classical
A B C
musician.

b s s b }
STRUCTURE AND WRITTEN EXPRESSION PRE-TEST I25
2 2 * 2 2 * 2 * 2 2 * 2

27. No longer satisfied with the emphasis of the Denishawn School, Martha Graham
A B C
is moving to the staff of the Eastman School in 1925.
D

28. Irving Berlin wrote "Oh, How I Hate to Get Up in the Morning" while serving in a
A E C
U.S. Army during World War I.
D

29. Shortly before the Allied invasion of Normandy, Ernest Hemingway has gone to
A B C
London as a war correspondent for Collier's.
D

30. During the 1960s, the Berkeley campus of the University o f California came to
A
national attention as a result its radical political activity.
I T - C D

31. Because of the flourish with which John Hancock signed the Declaration of
A
Independence, his name become synonymous with signature.
IT ' C D

32. On the floor of the Pacific Ocean is hundreds of flat-topped mountains more than a
~~A~ B C D
mile beneath sea level.

33. William Hart was an act best known for his roles as western heros in silent films.
A B ~C 17

34. Prior to an extermination program early in the last century, alive wolves roamed
A B C
across nearly all of North America.
D

35. The state seal still used in Massachusetts designed by Paul Revere, who also
~~A~ B ' C-
designed the first Continental currency.
D

36. Artist Gutzon Borglum designed the Mount Rushmore Memorial and worked on
A
project from 1925 until his death in 1941.
B C~~ TT

126 STRUCTURE AND WRITTEN EXPRESSION PRE-TEST


2 2 * 2 * 2 2 2 2 * 2

37. It is proving less costly and more profitably for drugmakers to market directly to
A B C D
patients.

38. Sapphires weighing as much as two pounds have on occasion mined.


A B C D

39. Like snakes, lizards can be found on all others continents except Antarctica.
A B C D

40. Banks, savings and loans, and finance companies have recently been doing hom e
A B
equity loans with greater frequency than ever before.
C D

This is the end of the Structure and Written Expression Pre-Test.

W h e n y o u fin ish th e test, y o u m ay d o th e fo llow in g:


T u r n to th e D ia g n o s tic C h art o n p a g e s 5 8 5 5 8 7 , an d
c ir c le th e n u m b e r s o f th e q u e s tio n s th a t y o u m issed .
T u r n to S c o r in g I n fo r m a tio n o n p a g e s 5 8 1 -5 8 2 , an d
d e te r m in e y o u r T O E F L sco re.
T u r n to th e P r o g r e s s C h art o n p a g e 5 9 1 , a n d a d d y o u r
sc o r e to th e chart.

STRUCTURE AND WRITTEN EXPRESSION PRE-TEST I 27


STRUCTURE AND
WRITTEN
EXPRESSION

T h e s e c o n d se c tio n o f th e T O E FL te st is th e S tru ctu re a n d W ritten E x p r e ssio n se c tio n .


T h is se c tio n c o n sists o f forty q u e stio n s (so m e tests m ay b e lo n g e r ). You have twenty-five
m in u te s to c o m p le te th e forty q u e s tio n s in th is se c tio n .
T h e r e are tw o types o f q u e stio n s in th e S tru ctu re a n d W ritten E x p r e ssio n se c tio n o f
th e T O E F L test:
1. S tru ctu re (q u e s tio n s 1 -1 5 ) c o n sists o f fifte e n s e n te n c e s in w h ic h part o f th e s e n
te n c e h a s b e e n r e p la c e d w ith a blan k. E ach s e n te n c e is fo llo w e d by fo u r answ er
c h o ic e s . You m u st c h o o s e th e answ er th a t c o m p le te s th e s e n te n c e in a gram m ati
cally c o r r e c t way.
2. W ritten E x p r e ssio n (q u e stio n s 1 6 -4 0 ) c o n sists o f tw enty-five s e n te n c e s in w h ich
fo u r w ord s o r g r o u p s o f w ords have b e e n u n d e r lin e d . You m u st c h o o s e th e u n d e r
lin e d w ord o r g r o u p o f w ords th a t is not co rr ec t.

GENERAL STRATEGIES

1. Be familiar with the directions. The directions on everyTOEFL test are the same, so it is
not necessary to spend time reading the directions carefully when you take the test.You
should be completely familiar with the directions before the day of the test.
2. Begin with questions I through 15. Anticipate that questions I through S will be the
easiest. Anticipate that questions I I through 15 will be the most difficult. D o not spend too
much time on questions I I through 15. There will be easier questions that come later.
3. Continue with questions 16 through 40. Anticipate that questions 16 through 20 will be
the easiest. Anticipate that questions 36 through 40 will be the most difficult. Do not spend
too much time on questions 36 through 40.
4. If you have tim e, return to questions I I through 15. You should spend extra time on
questions I I through 15 only after you spend all the time that you want on the easier
questions.
5. Never leave any answers blank on your answer sh eet. Even if you are not sure of the
correct response, you should answer each question. There is no penalty for guessing.

128
STRUCTURE

T H E S TR U C TU R E Q U E S T IO N S

In th e T O E F L test, q u e s tio n s 1 th r o u g h 15 o f th e S tru ctu re a n d W ritten E x p r e ssio n se c


tio n test you r k n o w le d g e o f th e c o r r e c t str u c tu r e o f E n g lish se n te n c e s . T h e q u e s tio n s in
this se c tio n are m u ltip le -c h o ic e q u e s tio n s in w h ic h you m u st c h o o s e th e le tte r o f th e
answ er that b e st c o m p le te s th e s e n te n c e .

Exam ple

is taking a trip to N ew York.


(A) They
(B) When
(C) The woman
(D) Her

In th is e x a m p le , y o u sh o u ld n o tic e im m e d ia te ly th a t th e s e n te n c e h a s a verb (is ta k in g ),


a n d th a t th e verb n e e d s a su b ject. A n sw ers (B ) a n d (D ) are in c o r r e c t b e c a u se w hen a n d
her are n o t su b jects. In an sw er (A ), they is a su b je ct, b u t they is p lu ral a n d th e v e rb is ta k in g
is sin gu lar. T h e c o r r e c t an sw er is a n sw e r ( C ) ; the w o m a n is a sin g u la r su b je ct. You sh o u ld
th e r e fo r e c h o o s e a n sw er ( C ) .

____________________ STRATEGIES FOR THE STRUCTURE QUESTIONS___________________

1. First study th e sen ten ce. Your purpose is to determine what is needed to complete the
sentence correctly.

2. Then study each answer based on how well it co m p letes th e sen ten ce. Eliminate
answers that do not complete the sentence correctly.
3. Do not try to elim inate incorrect answers by looking only at the an sw ers. The
incorrect answers are generally correct by themselves.The incorrect answers are generally
incorrect only when used to complete the sentence.
4. N ever leave any answers blank. Be sure to answer each question even if you are unsure
of the correct response.

5. Do not spend to o much tim e on th e Structure questions. Be sure to leave adequate


time for the Written Expression questions.

T h e fo llo w in g sk ills w ill h e lp y o u to im p le m e n t th e se str a teg ie s in th e S tr u c tu r e s e c tio n o f


th e T O E F L test.
STRUCTURE AND WRITTEN EXPRESSION

SENTENCES W IT H O N E CLAUSE___________________________

S o m e s e n te n c e s in E n g lish have ju s t o n e su b ject a n d verb , a n d it is v ery im p o r ta n t fo r you


to fin d th e su b je ct an d verb in th e se se n te n c e s . In so m e s e n te n c e s it is easy to fin d th e
su b je ct a n d verb . H ow ever, c e r ta in stru ctu res, su ch as o b je c ts o f p r e p o sitio n s, a p p o sitiv es,
a n d p a r ticip le s, can c a u se c o n fu s io n in lo c a tin g th e su b je ct a n d verb b e c a u s e e a c h o f
th e se str u c tu r es can lo o k lik e a su b je c t o r verb. T h e o b je c t o f th e p r e p o s itio n c a n b e m is
ta k en fo r a su b ject.
T h e r e fo r e , y o u sh o u ld b e a b le to d o th e fo llo w in g in s e n te n c e s w ith o n e su b je c t a n d
verb: (1 ) b e su re th e s e n te n c e h a s a su b ject a n d a verb , (2 ) b e c a refu l o f o b je c ts o f p r e p o
sitio n s a n d a p p o sitiv es w h e n y o u are lo o k in g for th e su b ject, a n d (3 ) b e c a r e fu l o f p r e se n t
p a r ticip le s an d p ast p a r ticip le s w h e n y o u are lo o k in g fo r th e verb .

S k ill I: BE SURE T H E SEN TE N C E H A S A SUBJECT A N D AVERB

You k n o w th at a s e n te n c e in E n g lish s h o u ld have a su b je ct an d a verb . T h e m o st c o m m o n


types o f p r o b le m s th at y o u will e n c o u n te r in th e S tru ctu re se c tio n o f th e T O E F L te st have
to d o w ith su b jects an d verbs: p e r h a p s th e s e n te n c e is m issin g e ith e r th e su b je ct o r th e
verb o r b o th , o r p e r h a p s th e s e n te n c e h a s an ex tra su b je ct o r verb .

Exam ple I
_ was backed up for miles on the freeway.

(A) Yesterday
(B) In the morning
(C) Traffic
(D) Cars

In th is e x a m p le y o u s h o u ld n o tic e im m e d ia te ly th at th e r e is a verb (w a s), b u t th e r e is n o


su b ject. A n sw er (C ) is th e b est answ er b e c a u se it is a sin g u la r su b je c t th a t a g ie e s w ith th e
sin g u la r verb was. A n sw er (A ) , yesterday, a n d an sw er ( B ) , in the m orning, are n o t su bjects,
so th e y are n o t c o rr ec t. A lth o u g h answ er ( D ), cars, c o u ld b e a su b ject, it is n o t c o r r e c t b e
cau se cars is p lu ral an d it d o e s n o t a g r ee w ith th e sin g u la r verb was.

Exam ple II
E ngineers_____ for work on the new space program.
(A) necessary
(B) are needed
(C) hopefully
(D) next month

In th is e x a m p le y o u s h o u ld n o t ic e im m ed ia te ly th a t th e s e n te n c e h as a su b je c t (engineers),
a n d th a t th e r e is n o verb . B e c a u se an sw er (B ), are needed, is a verb , it is th e b e st answer.
A n sw ers (A ), ( C ) , a n d (D ) are n o t verb s, so th e y are n o t co rr ec t.
STRUCTURE

E xam ple III


The b o y _____ going to the movies with a friend.
(A) he is
(B) he always was
(C) is relaxing
(D) will be

T h is s e n te n c e h a s a su b je c t (boy) a n d h as p a rt o f a verb (going) ; to b e c o r r e c t, s o m e fo rm


o f th e verb be is n e e d e d to m ak e th e s e n te n c e c o m p le te . A n sw ers (A) a n d (B ) are in c o r
rect b e c a u se th e s e n te n c e alread y h a s a su b je c t (boy) a n d d o e s n o t n e e d th e e x tra su b ject
he. A n sw er (C ) is in c o r r e c t b e c a u se relaxing is an ex tra verb part th a t is u n n e c e ss a r y b e
c a u se o f going. A n sw er (D ) is th e b e st answ er; w ill be to g e th e r w ith g o in g is a c o m p le te verb.

T h e fo llo w in g c h a r t o u tlin e s w h at y o u sh o u ld r e m e m b e r a b o u t su b je cts a n d verbs:

SUBJECTS ANDVERBS
A sentence in English must have at least one subject and one verb.

E X E R C ISE 1: U n d e r lin e th e su b je cts o n c e a n d th e verbs tw ice in e a c h o f th e fo llo w in g


se n te n c e s . T h e n in d ic a te i f th e s e n te n c e s are c o r r e c t (C ) o r in c o r r e c t (I ).

1 1 Last w eek went fishing for trout at the nearby m ountain lake.
C 9 A schedu le o f the days events can be obtained at the front desk.

3. A jo b o n the day shift or the night shift at the plant available.

4 T he new com puter program has provides a variety o f helpful applications.

5. T he b ox can be op en ed only with a special screwdriver.

6. T he assigned text for history class it contains m ore than twenty chapters.

7. T he papers in the wastebasket should be em ptied into the trash can outside.

8. Departure before dawn on a boat in the m iddle o f the harbor.

9. Yesterday found an interesting article on pollution.

10. T he new m achine is processes 50 percent m ore than the previous m achine.

S k ill 2: BE C A R EFU L OF OBJECTS OF P R EPO SITIO N S

A n o b je c t o f a p r e p o s itio n is a n o u n o r a p r o n o u n th a t c o m e s a fter a p r e p o s itio n , su c h as


in , at, of, to, by, behind, c n, a n d so o n , to fo r m a p r e p o sitio n a l p h ra se.
(/ fter his exams) Tom will take a trip (by boat).

T h is s e n t e n c e c o n ta in s tw o o b je c ts o f p r e p o sitio n s. E xa m s is th e o b je c t o f th e p r e p o sitio n
after a n d boat is th e o b je c t o f th e p r e p o s itio n by.
STRUCTUREANDWRITTEN EXPRESSION

A n o b je c t o f a p r e p o sitio n can c a u se c o n fu s io n in th e S tru ctu re se c tio n o f th e T O E F L


te st b e c a u se it can b e m ista k en fo r th e su b ject o f a s e n te n c e .

Exam ple
With his friend found the movie theater.
(A) has
(B) he
(C) later
(D) when

In th is e x a m p le y o u s h o u ld lo o k first for th e su b ject a n d th e verb. You sh o u ld n o tic e th e


verb fo u n d a n d sh o u ld also n o tic e th a t th e r e is n o su bject. D o n o t th in k th a t fr ie n d is th e
subject; frie n d is th e o b je c t o f th e p r e p o sitio n with, a n d o n e n o u n c a n n o t b e b o th a su b je ct
a n d an o b je c t at th e sa m e tim e. B e ca u se a su b ject is n e e d e d in this se n te n c e , an sw er ( B ) ,
he, is th e b e st answer. A n sw ers (A ), (C ), a n d (D ) are n o t c o r r e c t b e c a u se th ey c a n n o t be
su bjects.

T h e fo llo w in g ch art o u tlin e s th e key in fo r m a tio n th a t y o u sh o u ld r e m e m b e r a b o u t


o b je c ts o f p rep o sitio n s:

OBJECTS OF PREPOSITIONS
A preposition is followed by a noun or pronoun that is called an object o f the preposition.
If a word is an object o f a preposition, it is not th e subject.

NOTE; A lengthy list of prepositions and practice in recognizing prepositions can be


found in Appendix D at the back of the text. You may want to complete these exercises
before continuing with Exercise 2.

E X ER C ISE 2: E ach o f th e fo llo w in g s e n te n c e s c o n ta in s o n e o r m o r e p r e p o sitio n a l


p h rases. U n d e r lin e th e su b jects o n c e a n d th e verbs tw ice. C ircle th e p r e p o sitio n a l
p h ra ses th a t c o m e b e fo r e th e verb . T h e n in d ic a te i f th e s e n te n c e s are c o r r e c t (C ) o r in
c o r r e c t (I).
C 1. The interviews (by radio broadcasters) were carried live by the station.

^ 2. (in the last possible m om ent)(before takeoff) took his seat in the airplane.

_____ 3. At the neighb orh ood flower shop, flowers in quantities o f a dozen or a h a lf dozen
can be delivered for free.
_____ 4. The progressive reading m ethods at this school are given credit for the im proved
test scores.
_____ 5. For the last three years at various hospitals in the county has b een practicing
m edicine.

_____ 6. In the past a career in politics was n ot considered acceptable in som e circles.

_____ 7. Shopping in the downtown area o f the city it has improved a lot in recent years.

_____ 8. At the building site the carpenters with the m ost experience were given the m ost
intricate work.
STRUCTURE

-------- 9. For the fever and headache took two aspirin tablets.

-------- 10. T h e report with com plete docum entation was delivered at the conference.

S k i l l 3: BE CAREFUL OF APPO SITIVES

A p p o sitiv es c a n c a u se c o n fu s io n in th e S tru ctu re s e c tio n o f th e T O E F L te st b e c a u se an


a p p o sitiv e c a n b e m ista k e n fo r th e su b je ct o f a s e n te n c e . A n a p p o sitiv e is a n o u n th at
c o m e s b e fo r e o r a fte r a n o th e r n o u n a n d h as th e sa m e m e a n in g .
I I
Sally, the best student in the class, got an A on the exam.

In th is e x a m p le S ally is th e su b ject o f th e s e n te n c e a n d the best stu d e n t in the class c a n easily


b e r e c o g n iz e d as a n a p p o sitiv e p h ra se b e c a u se o f th e n o u n stu d e n t a n d b e c a u se o f th e
c o m m a s. T h e s e n t e n c e says th at Sally a n d the best stu d e n t in the class are th e sa m e p e r so n .
N o te th at i f y o u le a v e o u t th e a p p o sitiv e p h ra se, th e s e n t e n c e still m a k es se n s e (Sally got
a n A on the exam ).
T h e fo llo w in g e x a m p le sh ow s h o w an a p p o sitiv e c a n b e c o n fu s e d w ith th e su b je c t o f a
se n te n c e in th e S tr u c tu r e se c tio n o f th e T O E F L test.

Exam ple I
, George, is attending the lecture.
(A) Right now
(B) Happily
(C) Because of the time
(D) My friend

In this e x a m p le y o u sh o u ld r e c o g n iz e from th e c o m m a s that George is n o t th e su bject o f th e


se n te n c e . George is a n ap p ositive. B eca u se this s e n te n c e still n e e d s a su bject, th e b est answ er
is ( D ) , my frien d . A n sw ers (A ) , (B ) , a n d (C) are in c o r r e c t b e c a u se th ey are n o t subjects.
T h e n e x t e x a m p le sh ow s th at an a p p o sitiv e d o e s n o t always c o m e a fter th e su b ject; an
a p p o sitiv e c a n a lso c o m e at th e b e g in n in g o f th e s e n te n c e .

Exam ple II
, Sarah rarely misses her basketball shots.
(A) An excellent basketball player
(B) An excellent basketball player is
(C) Sarah is an excellent basketball player
(D) Her excellent basketball play

In th is e x a m p le y o u c a n te ll th a t Sarah is th e su b je ct a n d misses is th e verb b e c a u se th e r e is


n o c o m m a se p a r a tin g th e m . In th e sp a ce y o u s h o u ld p u t a n a p p o sitiv e fo r S arah, an d
Sarah is a n excellent basketball player, so an sw er (A) is th e b e st answ er. A n sw ers (B ) a n d (C)
are n o t c o r r e c t b e c a u s e th e y e a ch c o n ta in th e verb is, a n d a n a p p o sitiv e d o e s n o t n e e d a
verb . A n sw e r (D ) c o n ta in s a n o u n , play, th a t c o u ld p o ssib ly be an a p p o sitiv e, b u t play is
n o t th e sa m e as Sarah, so th is an sw er is n o t co rr ec t.
STRUCTURE AND WRITTEN EXPRESSION

T h e fo llo w in g ch art o u tlin e s th e key in fo r m a tio n th a t y o u sh o u ld r e m e m b e r a b o u t


a p p ositives:

APPOSITIVES

An ap Positive is a noun that comes before or after another noun and is generally set off from the
noun ivith commas. If a word is an appositive, it is not th e subjcct. The following appositive
struct jres are both possible in English:

S, APP. v
Tom, a really good mechanic. Is fixing the car.

APP, S V
A really good mechanic, Tom Is fixing the car.

E X E R C ISE 3: E a c h o f th e fo llo w in g s e n te n c e s c o n ta in s an a p p ositive. U n d e r lin e th e su b


j e c t s o n c e a n d th e verb s tw ice. C ircle th e a p p o sitiv e p h rases. T h e n , in d ic a te if th e se n
te n c e s are c o r r e c t (C ) o r in c o r r e c t ( I ) .
C 1. (The son o f the previous owner,) the new owner is undertaking som e fairly broad
changes in m anagem ent policy.

I 2. Last semester, (a friend,) graduated cum laude from the university.


_____ 3. Valentine's Day, February 14, is a special holiday for sweethearts.

_____ 4. At long last, the ch ief executive officer, has decided to step down.

_____ 5. T onights supper, leftovers from last night, did not taste any better tonight than
last night.
_____ 6. T he only entrance to the closet, the door was kept locked at all times.

_____ 7. In the cold o f winter, a wall heating unit, would not turn on.

_____ 8. The new tile pattern, yellow flowers on a w hite background, really brightens up
the room.
_____ 9. The high-powered com puter the m ost powerful m achine o f its type, was finally
readied for use.
_____ 10. A longtim e friend and confident, the psychologist was often invited over for
Sunday dinner.

S kill 4: BE CAREFUL OF PRESENT PARTICIPLES


A p r e s e n t p a r ticip le is th e -ing fo r m o f th e verb ( talkin g , p la yin g ). In th e S tru ctu re se c tio n
o f th e T O E F L te st a p r e se n t p a r ticip le can ca u se c o n fu s io n b e c a u se it can b e e ith e r a part
o f th e verb or an ad jective. It is p art o f th e verb w h e n it is p r e c e d e d by so m e form o f the
v e r b be.

T h e m an is talking to his friend.


- VERB

In th is s e n te n c e ta lk in g is part o f th e verb b e c a u se it is a c c o m p a n ie d by is.


STRUCTURE

A p r e s e n t p a r tic ip le is an a d jectiv e w h e n it is n o t a c c o m p a n ie d by s o m e fo r m o f th e
v e rb be.
T h e man talking to his friend has a beard.
ADJECTIVE

In th is s e n te n c e ta lk in g is an a d jectiv e a n d n o t p art o f th e verb b e c a u se it is n o t a c c o m p a


n ie d by s o m e fo r m o f be. T h e verb in th is s e n t e n c e is has.
T h e fo llo w in g e x a m p le sh ow s h o w a p r e se n t p a r ticip le can b e c o n fu s e d w ith th e verb
in th e S tr u c tu r e s e c tio n o f th e T O E F L test.

Exam ple
The child playing in the yard is my son.
(A) now
(B) is
(C) he
(D) was

In th is e x a m p le , i f y o u lo o k at o n ly th e first w ord s o f th e s e n te n c e , it a p p ea r s th a t child is


th e su b je c t an d p la y in g is p a rt o f th e verb . If y o u th in k th at p la y in g is p art o f th e verb , you
m ig h t c h o o s e a n sw er (B ), is, o r an sw er ( D ) , was, to c o m p le te th e verb . H ow ever, th e s e two
an sw ers are in c o r r e c t b e c a u se p la y in g is n o t p art o f th e verb . You s h o u ld r e c o g n iz e that
p la y in g is a p a r ticip ia l a d jectiv e ra th e r th a n a verb b e c a u se th e r e is a n o th e r verb in th e
s e n t e n c e (is). In th is s e n te n c e th e r e is a c o m p le te su b ject (child) a n d a c o m p le t e v e rb (is),
so th is s e n te n c e d o e s n o t n e e d a n o th e r su b je c t o r verb. T h e b e st an sw er h e r e is ( A ) .

T h e fo llo w in g c h a r t o u tlin e s w h a t y o u s h o u ld r e m e m b e r a b o u t p r e s e n t p a rticip les:

- ! " .. .'.
PRESENT PARTICIPLES

A present participle is the -ing form of the verb. T h e present particip le can be ( I) p art o f th e
verb o r (2 ) an adjective. It is part of the verb when it is accompanied by some form of the verb
be. It is an adjective when it is not accompanied by some form of the verb be.
.. - . . . . . . . . _. * *
1. The boy is standing in the comer.
2. The boy standing in the comer was naughty.

E X E R C ISE 4: E a c h o f th e fo llo w in g s e n t e n c e s c o n ta in s o n e o r m o r e p r e s e n t p a r ticip le s.


U n d e r lin e th e su b je cts o n c e a n d th e verb s tw ice. C ircle th e p r e s e n t p a r tic ip le s an d
la b e l th e m as a d je c tiv e s o r verb s. T h e n in d ic a te i f th e s e n te n c e s are c o r r e c t (C ) or
in c o r r e c t (I ).

1. The com panies (offering) the lowest prices will have the m ost customers.
ADJ.

I 2. Those travelers are (completing") their trip on Delta should report to Gate T hree.
VERB

_____ 3. T he artisans were dem onstrating various handicrafts at booths throughout the fair.
_____ 4. The fraternities are giving the wildest parties attract the m ost new pledges.
_____ 5. The first team winning four games is awarded the championship.
STRUCTURE AND WRITTEN EXPRESSION

6. T he speaker was trying to make his point was often interrupted vociferously.
7. T he fruits were rotting because o f the m oisture in the crates carrying them to
market.

8. Any students desiring official transcripts should com plete the appropriate form.
9. The advertisem ents were an n ou n cin g the half-day sale received a lot o f attention.
10. T he spices flavoring the meal were quite distinctive.

S k i l l 5: BE CAREFUL OF PAST PARTICIPLES


P ast p a r tic ip le s can cau se c o n fu s io n in th e S tru ctu re se c tio n o f th e T O E F L test b e c a u se
a p ast p a r tic ip le can b e e ith e r an ad jective or a p art o f a verb . T h e p a st p a rticip le is
th e fo r m o f th e verb th a t ap p ea rs w ith have o r be. It o fte n e n d s in -ed, b u t th e r e are also
m a n y irr eg u la r past p a r ticip le s in E n glish . (S e e A p p e n d ix F fo r a list o f irreg u la r past
p a r tic ip le s.)

The family has purchased a television.


VERB
The poem was written by Paul.
v e rb .

In th e first s e n te n c e th e p ast p a rticip le purchased is part o f th e verb b e c a u se it is a c co m p a


n ie d by has. In th e s e c o n d s e n te n c e th e past p a r ticip le w ritten is p art o f th e verb b e c a u se it
is a c c o m p a n ie d by was.
A p a st p a r tic ip le is an ad jective w h e n it is n o t a c c o m p a n ie d by so m e fo r m o f be o r have.

T he television purchased yesterday was expensive.


a d je c tiv e
The poem urritten by Paul appeared in the magazine.
a d je c tiv e

In th e first s e n t e n c e purchased is an ad jective ra th er than a verb b e c a u se it is n o t a c co m p a


n ie d by a fo r m o f be o r have (a n d th e r e is a verb , was, later in th e s e n te n c e ). In th e se c o n d
s e n t e n c e w ritten is an ad jective rath er th an a v erb b e c a u se it is n o t a c c o m p a n ie d by a form
o f be o r have, (a n d th e r e is a verb, appeared, later in th e s e n t e n c e ).
T h e fo llo w in g e x a m p le sh ow s h o w a past p a r ticip le c a n b e c o n fu s e d w ith th e verb in
th e S tr u c tu r e se c tio n o f th e T O E F L test.

Exam ple

The packages mailed at the post office will arrive Monday.


(A) have
(B) were
(C) them
(D) just
STRUCTURE

In th is e x a m p le , i f y o u lo o k o n ly a t th e first few w ord s o f th e s e n te n c e , it a p p ea r s th a t p ack


ages is th e su b je ct a n d m ailed is e ith e r a c o m p le te verb o r a past p a r tic ip le th a t n e e d s a
h e lp in g verb . B u t i f y o u lo o k fu r th e r in th e s e n te n c e , y o u w ill se e th a t th e verb is w ill ar
rive. You w ill th e n r e c o g n iz e th at m ailed is a p a rticip ia l ad jective a n d is th e r e fo r e n o t part
o f th e verb . A n sw ers (A ) a n d (B ) are in c o r r e c t b e c a u se m ailed is an a d jectiv e an d d o e s n o t
n e e d a h e lp in g verb su c h as have o r were. A n sw er (C ) is in c o r r e c t b e c a u se th e r e is n o n e e d
fo r th e o b je c t them. A n sw e r (D ) is th e b e st an sw er to th is q u e s tio n .

T h e fo llo w in g c h a r t o u tlin e s w h a t y o u sh o u ld r e m e m b e r a b o u t p ast p articip les:

PAST PARTICIPLES

A past participle often ends In -ed, but there are also many Irregular past participles. For many
verbs, including -ed verbs, the simple past and the past participle are the same and can be easily
confused. The -ed form o f the verb can be ( I) the sim ple past, (2 ) the past p articip le of a
verb, o r (3) an adjective.

1. She pain ted this picture.


2. She has painted this picture.
3. The picture pain ted by Karen is now in a museum.

E X E R C ISE 5: E ach o f th e fo llo w in g s e n te n c e s c o n ta in s o n e o r m o r e p ast p a r ticip le s. U n


d e r lin e th e su b je cts o n c e a n d th e verb s tw ice. C ircle th e p ast p a r ticip le s a n d la b e l th e m as
a d jectiv es o r verb s. T h e n in d ic a te i f th e s e n te n c e s are c o r r e c t (C ) o r in c o r r e c t (I ).

! 1. T he m oney was (offered) by the client was not (accepted)


VERB VERB

2. T he car (jisteti) in the advertisem ent had already (stalled)


a d j. v erb

-------- 3. T he chapters were taught by the professor this m orning will be on next week's
exam.

-------- 4. The loaves o f bread were baked in a brick oven at a low temperature for many
hours.

-------- 5. T he ports were reached by the sailors were under the control o f a foreign
nation.

-------- 6. T hose suspected in the string o f robberies were arrested by the police.

-------- 7. T he pizza is served in this restaurant is the tastiest in the county.

-------- 8. The courses are listed on the second page o f the brochure have several
prerequisites.

-------- 9. All the tenants were invited to the In d epend en ce Day barbecue at the
apartm ent com plex.

-------- 10. Any bills paid by the first o f the m onth will be credited to your account by the
next day.
STRUCTURE AND WRITTEN EXPRESSION

E X E R C IS E (S k ills 1 -5 ): U n d e r lin e th e su b jects o n c e an d th e verbs tw ice in e a c h o f th e


fo llo w in g s e n te n c e s . T h e n in d ic a te if th e s e n te n c e s are c o r r e c t (C) o r in c o r r e c t (1).

_____ 1. For three weeks at the beginn in g o f the sem ester students with fewer than the
m axim um num ber o f units can add additional courses.
_____ 2. O n her lunch hour went to a nearby departm ent store to purchase a w edding
gift.
_____ 3. T h e fir trees were grown for the holiday season were harvested in November.
_____ 4. In the grove the overripe oranges were falling on the ground.

_____ 5. T h e papers being delivered at 4:00 will contain the announcem ent o f the
presidents resignation.
_____ 6. A specialty shop with various blends from around the world in the shopping
mall.
_____ 7. T h e portraits exhibited in the H ouston M useum last m onth are now o n display
in Dallas.
_____ 8. W ith a sudden jerk o f his hand threw the ball across the field to one o f the
other players.
_____ 9. Construction o f the housing developm ent it will be underway by the first o f the
m onth.
_____ 10. T hose applicants returning their com pleted forms at the earliest date have the
highest priority.

T O E F L E X E R C ISE (S k ills 1 -5 ): C h o o se th e le tte r o f th e w o rd o r g r o u p o f w ord s th a t b est


c o m p le te s th e se n te n c e .

1. The North Platte River_ .from . tea plant are small and white.
Wyoming into Nebraska. (A) The
(A) it flowed (B) On the
(B) flows (C) Having flowers the
(C) flowing (D)' The flowers of the
(D) with flowing water
5. The tetracyclines. . antibiotics, are
. Biloxi received its name from a used to treat infections.
Sioux word meaning "first people.
(A) are a family of
(A) The city of (B) being a family
(B) Located in (C) a family of
(C) It is in (D) their family is
(D) The tour included
6. Any possible academic assistance from
3. A pride of lions . _up to forty lions. taking stim ulants_____ marginal at best.
including one to three males, several
(A) it is
females, and cubs. there is
(B)
(A) can contain (C) is
(B) it contains (D) as
(C) contain
(D) containing
STRUCTURE I

7. Henry Adams, b om in B oston ,-------- 9. Still a novelty in the late nineteenth


famous as a historian and novelist. century,_____ limited to the rich.

(A) became (A) was


(B) and became (B) was photography
(C) he was (C) it was photography
(D) and he became (D) photography was

8. The major c a u se_____ the pull of the 10. A computerized map of the freeways using
Moon on the Earth. information gathered by sensors
embedded in the pavem ent--------on a
(A) the ocean tides are local cable channel during rush hours.
(B) of ocean tides is
(C) of the tides in the ocean (A) airs
(D) the oceans tides (B) airing
(C) air
(D) to air

SEN TEN CES W IT H M ULTIPLE CLAUSES-----------------------------------

M any se n te n c e s in E n g lish h a v e m o r e than o n e c la u se. (A c la u se is a g r o u p o f w ords c o n


ta in in g a su b je ct a n d a verb .) W h e n e v e r you fin d a s e n t e n c e o n th e T O E F L test w ith m o r e
th an o n e c la u se , y o u n e e d to m ak e su re that ev ery su b je ct h a s a verb a n d ev er y verb h a s a
su b ject. N e x t y o u n e e d to c h e c k th a t th e variou s cla u se s in th e se n te n c e are co rrectly
j o in e d . T h e r e are v ariou s ways to j o in cla u se s in E n g lish . C erta in p a tte rn s a p p ea r
fr eq u e n tly in E n g lish a n d o n th e T O E F L test. You sh o u ld b e very fa m ilia r w ith th e se
p a ttern s.

S kill 6: USE C O O R D IN A T E C O N N E C TO R S CORRECTLY

W h e n y o u h a v e tw o cla u se s in an E n g lish s e n te n c e , y o u m u st c o n n e c t th e tw o cla u se s c o r


rectly. O n e way to c o n n e c t tw o c la u ses is to u se and, but, or, so, or yet b e tw e e n th e cla u ses.

Tom is singing, and Paul is dancing.

Tom is tall, but Paul is short.

Tom must write the letter, or Paul will do it.

Tom told a jo k e, so Paul laughed.

Tom is tired, yet he is n ot going to sleep.

In e a c h o f th e se e x a m p le s , th e r e are two cla u se s th a t are c o rr ec tly j o i n e d w ith a c o o r d i


n a te c o n ju n c tio n a n d , but, or, so, o r yet, a n d a c o m m a ( ,) .
STRUCTURE AND WRITTEN EXPRESSION

T h e fo llo w in g e x a m p le sh ow s h o w th is s e n te n c e p a ttern c o u ld b e te ste d in th e Struc


ture se c tio n o f th e T O E F L test.

Exam ple
A power failure occurred, the lamps went out.
(A) then
(B) so
(C) later
(D) next

In th is e x a m p le y o u sh o u ld n o tic e q u ick ly th at th e r e are two c la u se s, a power fa ilu r e oc


curred a n d the lam ps w ent out. T h is s e n te n c e n e e d s a c o n n e c to r to j o in th e tw o clau ses.
Then, later, an d next are n o t c o n n e c to r s, so answ ers (A ), (C ), an d (D ) are n o t c o rr ec t. T h e
b e st an sw er is an sw er (B ) b e c a u se so can c o n n e c t two clau ses.

T h e fo llo w in g ch a rt lists th e c o o r d in a te c o n n e c to r s an d th e s e n te n c e p a tte r n u se d


w ith th em :

COORDINATE CONNECTORS

and but or . so yet

S V, (coordinate connector) S V

She laughed, but she w anted to cry.

E X E R C ISE 6: E ach o f t h e fo llo w in g s e n te n c e s c o n ta in s m o r e th a n o n e c la u se . U n d e r lin e


th e su b jects o n c e an d th e verb s tw ice. C ircle th e c o n n e c to r s. T h e n in d ic a te i f th e se n
te n c e s are c o r r e c t (C ) o r in c o r r e c t (I).

C 1. T he software should be used on an IBM computer, (and) this com puter is an


IBM. = ^ = ^ ==

T he rain clouds can be seen in the distance, (b u t)n o has fallen.

They are trying to sell their house, it has been on the market for two months.
So the quality o f the print was not good, I changed the toner cartridge.
T h e lifeguard will warn you about the riptides, or she may require you to get
out o f the water.
6. You should have finished the work yesterday, yet is not close to being finished
today.
7. The ph on e rang again and again, so the receptionist was n ot able to get much
work done.
8. T he missing wallet was found, but the cash and credit cards had been rem oved.
9. Or you can drive your car for another 2,000 miles, you can get it fixed.
10. T h e chem ist was awarded the N obel Prize, he flew to Europe to accept it.
STRUCTURE

S kill 7: USE ADVERB TIM E A N D CAUSE C O N N E C T O R S CO RRECTLY

S e n te n c e s w ith ad verb c la u se s h ave two b asic p a tte rn s in E n g lish . S tu d y th e c la u se s a n d


c o n n e c to r s in th e fo llo w in g se n te n c e s:

I will sign the check before you leave.

In ea ch o f th e se e x a m p le s , th e r e are two clau ses: y o u leave a n d I m i l sign the check, a n d th e


c la u se you leave is an ad verb tim e c la u se b e c a u se it is in tr o d u c e d w ith th e c o n n e c t o r before.
In th e first e x a m p le th e c o n n e c to r before c o m e s in th e m id d le o f th e s e n te n c e , a n d n o
c o m m a (,) is u sed . In th e s e c o n d e x a m p le th e c o n n e c to r before c o m e s at th e b e g in n in g o f
th e se n te n c e . In th is p a tte rn , w h e n th e c o n n e c to r c o m e s at th e b e g in n in g o f th e s e n
te n c e , a c o m m a (,) is r e q u ir e d in th e m id d le o f th e se n te n c e .
T h e fo llo w in g e x a m p le sh ow s h o w th is s e n te n c e p a tte rn c o u ld b e te s te d in th e S tru c
tu re se c tio n o f th e T O E F L test.

E x am p le

--------was late, I missed the appointment.


(A) I
(B) Because
(C) The train
(D) Since he

In this e x a m p le y o u sh o u ld r e c o g n iz e ea sily th at th e r e is a verb , was, th a t n e e d s a su b ject.


T h e r e is a lso a n o th e r c la u se , I m issed the appointm ent. If y o u c h o o s e an sw er (A ) o r an sw er
(C ), y o u w ill h a v e a su b je c t for th e verb rvas, b u t y o u w ill n o t have a c o n n e c t o r to j o i n th e
two cla u ses. B e c a u se y o u n e e d a c o n n e c t o r to j o in two c la u se s, a n sw ers (A ) a n d (C ) are
in c o r r e c t. A n sw er (B ) is in c o r r e c t b e c a u se th e r e is n o su b je ct fo r th e v e r b was. A n sw er
(D ) is th e b e st a n sw er b e c a u se th e r e is a su b ject, he, for th e verb was, a n d th e r e is a c o n
n ecto r, since, to j o i n th e tw o cla u ses.

T h e fo llo w in g c h a r t lists ad verb tim e a n d cause c o n n e c to r s a n d th e s e n te n c e p a tte r n s


u sed w ith them :
STRUCTURE AND WRITTEN EXPRESSION

E X E R C ISE 7: E ach o f th e fo llo w in g s e n te n c e s c o n ta in s m o r e th an o n e c la u se . U n d e r lin e


th e su b jects o n c e an d th e verbs tw ice. C ircle th e c o n n e c to r s. T h e n in d ic a te if th e se n
te n c e s are c o r r e c t (C ) o r in c o r r e c t (I).

C 1. (Since) the bank closes in less than an hour, the deposits need to be tallied
immediately.
I 2. Their backgrounds are thoroughly investigated (before) are adm itted to the
organization.
_____ 3. The citizens are becom ing m ore and m ore incen sed about traffic accidents
whenever the accidents occur at that intersection.
_____ 4. T he ground had been prepared, the seedlings were carefully planted.
_____ 5. We can start the conference now that all the participants have arrived.
_____ 6. The building quite vulnerable to damage until the storm windows are installed.
____ 7. O nce the address label for the package is typed, can be sent to the mail room.
_____ 8. Because the recent change in work shifts was not posted, several workers missed
their shifts.
_____ 9. The m other is going to be quite upset with her son as long he misbehaves so much.
_____ 10. Inasmuch as all the votes have n ot yet been counted the outcom e o f the election
cannot be announced.

S kill 8: USE OTH ER ADVERB C O N N E C T O R S CORRECTLY

A d verb c la u se s ca n e x p r ess th e id e a s o f tim e a n d c a u se , as y o u saw in Skill 7; adverb


c la u se s c a n also ex p r ess a n u m b e r o f o th e r id ea s, su ch as c o n tra st, c o n d itio n , m an n er,
a n d p la c e. B e ca u se th e se cla u ses are adverb c la u se s, th ey have th e sam e stru ctu re as th e
tim e a n d ca u se cla u ses in Skill 7. Study th e fo llo w in g ex a m p les:

I will leave at 7:00 i f I am ready.

Although I was late, I managed to catch the train.

In e a c h o f th e se e x a m p le s, th e r e are two cla u ses th a t are c o rr ec tly j o in e d w ith adverb c o n


n e c to rs. In th e first se n te n c e th e adverb c o n d itio n c o n n e c to r i f c o m e s in th e m id d le o f
th e se n te n c e . In th e s e c o n d s e n te n c e th e adverb c o n tr a st c o n n e c to r although c o m e s at th e
b e g in n in g o f th e se n te n c e , an d a c o m m a (,) is u s e d in th e m id d le o f th e s e n te n c e .
T h e fo llo w in g e x a m p le sh ow s a way th at th is s e n te n c e p a tte rn can b e te sted in th e
S tru ctu re se c tio n o f th e T O E F L test.

E xam ple
You will get a good grade on the exam provided
(A) studying
(B) study
(C) to study
(D) you study
STRUCTURE

In th is e x a m p le y o u sh o u ld q u ick ly n o tic e th e adverb c o n d itio n c o n n e c to r provided. T h is


c o n n e c to r c o m e s in th e m id d le o f th e s e n te n c e ; b e c a u se it is a c o n n e c to r , it m u st be fo l
low ed by a su b je ct a n d a verb. T h e b e s t an sw er to th is q u e s tio n is an sw er ( D ) , w h ic h c o n
tains th e su b je c t a n d verb yo u study.

T h e fo llo w in g c h a r t lists th e a d v e r b c o n tr a st, c o n d it io n , m a n n er , a n d p la c e c o n n e c


tors a n d th e s e n t e n c e p a tte rn s u s e d w ith th em :

OTHER ADVERB CONNECTORS

CONDITION CONTRAST MANNER PLACE

If although as where
in case even though in that wherever
provided though
providing while
unless whereas
whether

S V (adverb connector) S V

Bob w e n t to school even th o u g h h e fe lt sick.

(adverb connector) S V, S V

Even th o u g h Bob fe lt sick, he w e n t to school.

NOTE: A comma is often used in the middle of the sentence with a contrast connector.
The Smith family arrived at 2:00, while the Jones family arrived an hour later.

E X ER C ISE 8: E ach o f th e fo llo w in g s e n t e n c e s c o n ta in s m o r e th an o n e c la u se . U n d e r lin e


th e su b jects o n c e a n d th e verb s tw ice. C ircle th e c o n n e c to r s . T h e n in d ic a te i f th e s e n
te n c e s are c o r r e c t (C ) o r in c o r r e c t (I ).

__ 0 1. It is im possible to enter that program (Tf)you lack experien ce as a teacher.

__ !__ 2. T he commandant left strict orders about the passes, several soldiers left the post anyway.

_____ 3. N o on e is adm itted to the academy unless he or she the education requirem ents.

_____ 4. W hile m ost students turned the assignm ent in on tme, a few asked for an extension.
_____ 5. I will take you wherever n e e d to go to com plete the registration procedures.
_____ 6. I will wait here in the airport with you w hether the plane leaves on time or not.
_____ 7. Providing the envelope is postm arked by this Friday, your application still
acceptable.

_____ 8. As the nurse already explain ed all visitors m ust leave the hospital room now.
_____ 9. This exam will be m ore difficult than usual in that it covers two chapters instead
o f one.
_____ 10. T hough snow had b een falling all day long, everyone g o t to the church on time for
the wedding.
STRUCTURE AND WRITTEN EXPRESSION

EX ER C ISE (S k ills 6 - 8 ) : U n d e r lin e th e su b jects o n c e a n d th e verbs tw ice in e a c h o f th e


fo llo w in g se n te n c e s. C ircle th e c o n n e c to r s . T h e n in d ic a te if th e s e n te n c e s are c o r r e c t (C )
or in c o r r e c t (I).

1. Until the registrar makes a decision about your status, you must stay in an
unclassified category.
2. Or the bills can be paid by mail by the first o f the month.
T he parents left a p h on e num ber with the baby-sitter in case a problem with the
children.
4. The furniture will be delivered as soon it is paid for.
5. W henever you want to hold the m eeting, we will schedule it.
6. T he governm ent was overthrown in a revolution, the king has not returned to his
hom eland.
7. Whereas m ost o f the docum ents are com plete, this form still needs to be notarized.
8. Trash will be collected in the m orning, so you should put the trash cans o u t tonight.
9. It is im possible for the airplane to take o ff while is snowing so hard.
10. We did not go out to dinner tonight eventhough I would have preferred n ot to cook.

T O E F L EXERCISE (S k ills 6 -8 ) : C h o o se th e le tte r o f th e w ord o r g r o u p o f w ord s th a t b e st


c o m p le te s th e s e n te n c e .

1. The president of the U. S. appoints the . sucked groundwater from below,


cabinet m embers,_____ appointments are some parts of the city have begun to sink
subject to Senate approval. as much as ten inches annually.
(A) their (A) Pumps have
(B) with their (B) As pumps have
(C) because their (C) So pumps have
(D) but their (D) With pumps

2. The prisoners were prevented from 5. Case studies are the target o f much
speaking to reporters because_____ skepticism in the scientific community,
_____ used extensively by numerous
(A) not wanting the story in the papers, researchers.
(B) the story in the papers the
superintendent did not want (A) they are
(C) the public to hear the story (B) are
(D) the superintendent did not want the (C) yet they
story in the papers (D) yet they are

Like Thomas Bergers fictional character 6. According to the hypothesis in the study,
Little Big Man, Lauderdale managed to find the monarchs pick up the magnetic field of
himself w here_____ of important events th e _____ migrate by following magnetic
took place. fields.
(A) it was an extraordinary number (A) target monarchs
(B) there was an extraordinary number (B) target since monarchs
(C) an extraordinary number (C) target since monarchs are
(D) an extraordinary number existed (D) target
STRUCTURE

7. _____ show the relations among neurons, 9. If ultraviolet radiation enters the Earth's
they do not preclude the possibility that atm osphere,_____ generally blocked by the
other aspects are important. ozone concentrated in the atmosphere.
(A) Neural theories (A) it
(B) A neural theory (B) it is
(C) Although neural theories (C) so it is
(D) However neural theories (D) then it

8. _____or refinanced, the lender will 10. Among human chromosomes, the Y
generally require setting up an escrow chromosome is un usual_____ most of the
account to ensure the payment of property chromosome does not participate in m eiotic
taxes and homeowners insurance. recombination.
(A) A home is (A) in
(B) A home is bought (B) so
(C) When a home (C) and
(D) When a hom e is bought (D) in that

T O E F L R EV IEW E X E R C ISE (S k ills 1 -8 ): th e le tte r o f th e w o rd o r g r o u p o f w ords


th a t b e st c o m p le te s th e s e n te n c e .

1. The three basic chords i n _____ the tonic, 5. While in reality Alpha Centauri is a triple
the dominant, and the subdominant. star,--------to the naked eye to be a single
star.
(A) functional harmony
(B) functional harmony is (A) it appears
(C) functional harmony are (B) but it appears
(D) functional harmony they are (C) appears
(D) despite it
2. --------Hale Telescope, at the Palomar
Observatory in southern California, 6. The Suns gravity severely distorted the
scientists can photograph objects several path of the c o m et_____ entered its wildly
billion light years away. erratic orbit around Jupiter.
(A) The (A) it
(B) With the (B) when
(C) They use the (C) after the com et came into it
(D) It is the (D) once the comet

3. Without the proper card installed inside 7. Each object-------- Jupiter's magnetic field
the com puter,_____ impossible to run a is deluged with electrical charges.
graphical program.
(A) enters
(A) is definitely (B) it enters
(B) because o f (C) entering
(C) it is (D) enter
(D) is
8. As its name suggests, the Prairie Wetlands
4. The charter for the Louisiana lottery was Resource Center_____ the protection of
coming up for renew al,_____ spared no wetlands on the prairies o f the Dakotas,
expense in the fight to win renewal. Montana, Minnesota, and Nebraska.
(A) the lottery committee (A) it focuses
(B) so the lottery committee and (B) focuses on
(C) so the lottery committee (C) focusing
(D) the lottery committee made (D) to focus on
STRUCTURE AND WRITTEN EXPRESSION

9. One of the largest and most powerful birds 10 . creation of such a community was a
of prey in the w orld,_____ a six-foot desirable step, the requisite political
wingspan and legs and talons roughly the upheaval had to be accepted.
size of a mans arms and legs.
(A) Since the
(A) so the harpy has (B) The
(B) the harpy having (C) Later, the
(C) with the harpy having (D) It was the
(D) the harpy has

MORE SENTENCES W IT H M ULTIPLE CLAUSES-----------------------

A s w c saw in S k ills 6 th r o u g h 8 , m an y s e n te n c e s in E n g lish have m o r e th a n o n e c la u se . In


Sk ills 9 th r o u g h 12, w e w ill se e m o r e p a tte rn s for c o n n e c tin g th e cla u ses in s e n te n c e s with
m u ltip le cla u ses. B e ca u se th ese p attern s a p p ear fr eq u e n tly in E n g lish a n d o n th e T O EFL
test, y o u s h o u ld b e very fam iliar w ith th e m .

S k ill 9: USE N O U N CLAUSE C O N N E C T O R S CORRECTLY

A n o u n c la u se is a clau se th at fu n c tio n s as a n o u n ; b e c a u se th e n o u n c la u se is a n o u n , it is
u s e d in a s e n te n c e as e ith e r an o b je c t o f a verb , an o b je c t o f a p r e p o sitio n , or th e su b ject
o f th e se n te n c e .

I know when he will arrive.


NOU N CLAUSE AS OBJECT OF VERB

I am concerned about when he will arrive.


N O U N CLAUSE AS OBJECT O F PREPOSITION

When he will arrive is not important.

N OU N CLAUSE AS SUBJECT

In th e first e x a m p le th e r e are tw o cla u se s, I know an d he w ill arrive. T h e se two cla u ses are
j o in e d w ith th e c o n n e c to r when. W hen c h a n g e s th e c la u se he w ill arrive in to a n o u n clau se
th a t fu n c tio n s as th e o b je c t o f th e verb know.
In th e s e c o n d e x a m p le th e tw o c la u se s I am concerned an d he w ill arrive are also j o in e d
by th e c o n n e c to r when. W hen c h a n g e s th e c la u se he w ill arrive in to a n o u n c la u se th a t fu n c
tio n s as th e o b je c t o f th e p r e p o sitio n about.
T h e th ird e x a m p le is m o r e d ifficu lt. In th is e x a m p le th e r e are two clau ses, b u t they
are a little h a r d e r to r e c o g n iz e . H e w ill arrive is o n e o f th e cla u ses, a n d th e c o n n e c to r when
c h a n g e s it in to a n o u n clau se th at fu n c tio n s as th e su b je ct o f th e se n te n c e . T h e o th e r
c la u se h a s th e n o u n cla u se when he w ill arrive as its su b ject an d is as its verb.
T h e fo llo w in g e x a m p le sh ow s h o w th e se s e n te n c e p a tte rn s c o u ld b e te sted in th e
S tr u c tu r e se c tio n o f th e T O E F L test.
STRUCTURE

E x am p le
was late caused many problems.
(A) That he
(B) The driver
(C) There
(D) Because

In th is e x a m p le th e r e are tw o verb s ( was a n d caused) , a n d e a c h o f th e s e v e rb s n e e d s a su b


j e c t. A n sw er (B ) is w r o n g b e c a u se the driver is o n e su b ject, a n d tw o su b je cts are n e e d e d .
A n sw ers (C ) a n d (D ) are in c o r r e c t b e c a u se there a n d because a re n o t su b jects. T h e b e st an
sw er is an sw er (A ). I f y o u c h o o s e an sw er (A ), th e c o m p le te d s e n te n c e w o u ld be: T h a t he
was late caused m a n y problems. In th is s e n te n c e he is th e su b je c t o f th e verb was, a n d th e
n o u n c la u se that he was late is th e su b je ct o f th e verb caused.
T h e fo llo w in g c h a r t lists th e n o u n c la u se c o n n e c to r s a n d th e s e n te n c e p a tte r n s u se d
w ith them :

NOUN CLAUSE CONNECTORS

what, when, where, why, how whatever, whenever whether, if that

N O U N CLAUSE AS OBJECT

S V (noun connector) S V

/ know what you did.

N O U N CLAUSE AS SUBJECT

(noun connector) S V V . ' / "

W h at you did was wrong.

EX E R C ISE 9: E a ch o f th e fo llo w in g s e n te n c e s c o n ta in s m o r e th an o n e c la u se . U n d e r lin e


th e su b je cts o n c e a n d th e verb s tw ice. C ircle th e c o n n e c to r s . P u t b o x e s a r o u n d th e n o u n
cla u ses. T h e n in d ic a te i f t h e s e n t e n c e s are c o r r e c t (C ) o r in c o r r e c t (I).
C (W hen)the season starts is determ ined by the weather.

2. T he manual (how)the device should be built.

3. The schedule indicated if the teams would be playing in the final gam e.

4. He refused to en ter a plea could not be determ ined by the lawyer.

5. Talked about where we should go for lunch.

6. Why the condition o f the patient deteriorated so rapidly it was n ot explained.

7. W hether or n o t the new office w ould be built was to be determ ined at the m eeting.

8. That the professor has n o t yet decided when the paper is due.

9. T he contract will be awarded is the question to be answered at the m eeting.

10. H e always talked with w hom ever he pleased and did whatever he wanted.
STRUCTURE AND WRITTEN EXPRESSION

S k il l 10: USE N O U N CLAUSE CO NNECTO R/SUBJECTS CORRECTLY

In Sk ill 9 w e saw th a t n o u n c la u se c o n n e c to r s w ere u s e d to in tr o d u c e n o u n su b ject cla u ses


o r n o u n o b je c t clau ses. In Skill 10 w e will se e th a t in so m e c a se s a n o u n cla u se c o n n e c to r
is n o t ju s t a c o n n e c to r ; a n o u n clau se c o n n e c to r can also b e th e su b je ct o f th e cla u se at
th e sa m e tim e.

N OU N CLAUSE AS SUBJECT

In th e first e x a m p le th e r e are two clauses: I do not kn o w an d w hat is in the box. T h e se two


c la u se s are j o in e d by th e c o n n e c to r w hal. It is im p o r ta n t to u n d e r sta n d that in this se n
te n c e th e w o rd w hat se rv es two fu n c tio n s. It is b o th th e su b je ct o f th e verb is a n d th e c o n
n e c to r that j o in s th e two clau ses.
In th e s e c o n d e x a m p le th e r e are two cla u ses. In th e first c la u se we is th e su b ject o f are.
In th e s e c o n d cla u se who is th e su b ject o f w ill do. W ho also se rv es as th e c o n n e c to r th a t
jo in s th e tw o clau ses. T h e n o u n c la u se who w ill do the work fu n c tio n s as th e o b je c t o f th e
p r e p o sitio n about.
In th e last e x a m p le th ere are also two clau ses: whoever is th e su b ject o f th e verb is com
ing, a n d th e n o u n cla u se whoever is com ing to the party is th e su b ject o f m ust bring. T h e w ord
whoever se rv es two fu n c tio n s in the se n te n c e : It is th e su b ject o f th e verb is coming, an d it is
th e c o n n e c t o r th at jo in s th e two clau ses.
T h e fo llo w in g e x a m p le sh ow s h o w this s e n te n c e p a ttern c o u ld b e te sted in th e Struc
tu re se c tio n o f th e T O E F L test.

E xam ple
was on television made me angry.
(A) It
(B) The story
(C) What
(D) When

In th is e x a m p le y o u sh o u ld n o tic e im m e d ia te ly th at th e r e are two verb s, was a n d made,


a n d e a c h o f th o s e verb s n e e d s a su bject. A n sw ers (A) a n d (B) are in c o r r e c t b e c a u se it an d
the story c a n n o t b e th e su b ject fo r b o th was a n d m ade a t th e sa m e tim e. A n sw er (D ) is in
c o r r e c t b e c a u se w hen is n o t a su bject. In answ er (C ) w hal s e r v es as b o th th e su b ject o f th e
verb w as a n d th e c o n n e c to r th a t jo in s th e tw o c la u se s to g e th er ; th e n o u n cla u se w hat was
on television is th e su b ject o f th e verb made. A n sw er (C ) is th e r e fo r e th e b e st answer.
STRUCTURE

T h e fo llo w in g c h a r t lists th e n o u n c la u se c o n n e c to r /s u b j e c ts a n d th e s e n te n c e pat


te rn s u sed w ith th em :

NOUN CLAUSE CONNECTOR/SUBJECTS

who what which


'
whoever whatever whichever
............. .

N O U N CLAUSE AS OBJECT

S V | (noun connector/subject) V

/ know w hat happened.

NO U N CLAUSE AS SUBJECT

| (noun connector/subject) V] V

W hat happened was great.

EX E R C ISE 10: E ach o f th e fo llo w in g s e n te n c e s c o n ta in s m o r e th a n o n e c la u se . U n d e r


lin e th e su b jects o n c e a n d th e verb s tw ice. C ircle th e c o n n e c to r s. P u t b o x e s a r o u n d th e
n o u n cla u ses. T h e n in d ic a te i f th e s e n te n c e s are c o r r e c t (C ) o r in c o r r e c t (I ).

_____ 3. The em ployee was unhappy about what was added to his jo b description.

_____ 4. W hoever wants to take the desert tour during spring break signing up at the office.

_____ 5. The m otorist was unable to discover who he had struck his car.

_____ 6. The voters should elect w hichever o f the candidates seem s best to them .

_____ 7. It was difficult to distinguish what was on.sale and what was m erely o n display.

-------- 8. You sh ould buy whatever the cheapest and m ost durable.

_____ 9. What was written in the letter angered him beyond belief.

--------- 10. You can spend your time with whoever im portant to you.

S kill 11: USE ADJECTIVE CLAUSE C O N N E C TO R S CORRECTLY


A n a d jectiv e cla u se is a cla u se th a t d e sc r ib e s a n o u n . B e ca u se th e cla u se is a n a d jectiv e, it
is p o s itio n e d d irectly a fter th e n o u n th a t it d escrib es.

T he woman is filling the glass that sh e put on the table.


ADJECTIVE CLAUSE

r
T he glass that she put on the table contains milk.
ADJECTIVE CLAUSE
STRUCTURE AND WRITTEN EXPRESSION

In th e first e x a m p le th e r e are tw o clau ses: w om an is th e su b je ct o f th e v e rb is fillin g , an d


she is th e su b ject o f th e verb pu t. T h a t is th e ad jective c la u se c o n n e c to r th a t j o in s th e se two
c la u se s, a n d th e a d jectiv e c la u se that she p u t on the table d e sc r ib e s th e n o u n glass.
In th e s e c o n d e x a m p le th e r e are also two clauses: glass is th e su b je ct o f th e verb con
tains, a n d she is th e su b je ct o f th e verb p u t. In th is s e n te n c e a lso , th a t is th e a d jectiv e cla u se
c o n n e c to r th at j o in s th e se two c la u se s, a n d th e ad jective c la u se th a t she p u t on the table d e
scrib es th e n o u n glass.
T h e fo llo w in g e x a m p le sh ow s h o w th e se s e n te n c e p a tte rn s c o u ld b e te ste d in th e
S tru ctu re s e c d o n o f th e T O E F L test.

E xam ple
The g ift_____ selected for the bride was rather expensive.

(A) because
(B) was
(C) since
(D) which we

In th is e x a m p le y o u s h o u ld n o tic e q u ick ly th at th e r e are two c la u se s, g ift is th e su b je ct o f


th e verb was, a n d th e verb selected n e e d s a su b ject. B e ca u se th e r e are tw o c la u se s, a c o n
n e c to r is also n e e d e d . A nsw ers (A ) a n d (C ) have c o n n e c to r s, b u t th e r e are n o su b jects, so
th e se answ ers are n o t c o rr ec t. A n sw er (B ) c h a n g e s selected in to a passive verb; in th is case
th e s e n te n c e w o u ld have o n e su b je ct a n d two verbs, so answ er (B ) is n o t c o r r e c t. T h e b est
answ er to th is q u e s tio n is answ er ( D ) . T h e c o r r e c t s e n te n c e sh o u ld say: T he g ift w hich we se
lected fo r the bride was rather expensive. In th is se n te n c e g ift is th e su b ject o f th e verb was, we is
th e su b ject o f th e verb selected, a n d th e c o n n e c to r which j o in s th e s e two c la u se s.

T h e fo llo w in g ch a rt lists th e ad jective c la u se c o n n e c to r s a n d th e s e n te n c e p attern s


u se d w ith them :

ADJECTIVE CLAUSE CONNECTORS

whom which that


(for people) (for things) (for people o r things)

S V |(agiective^ gggggg>T) S V

I liked the book which you recommended.

S |(adjective connector) S V |V

The book which you recommended wot Interesting.

NOTE: The adjective connectors can be omitted.This omission Is very common In spoken English or in casual
written English. It is not as common in formal English or in the Structure section of the TOEFL test.
STRUCTURE

E X ER C ISE 11: E ach o f th e fo llo w in g s e n te n c e s c o n ta in s m o r e th an o n e c la u se . U n d e r


lin e th e su b je cts o n c e a n d th e verb s tw ice. C ircle th e c o n n e c to r s. P u t b o x e s a r o u n d th e
ad jective cla u se s. T h e n in d ic a te if th e s e n t e n c e s are c o r r e c t (C ) o r in c o r r e c t (I).

3. I ju st finished reading the novel whom the professor suggested for my book report.

4. T h e plane that h e was schedu led to take to Hawaii was delayed.

5. T h e movie which we watched on cable last nigh t it was really frightening.

6. I m ade an appointm ent with the doctor whom you recom m ended.

7. T he enthusiasm with which he greeted m e m ade me fe el welcom e.

8. T he story that you told m e about Bob.

9. T h e m en with w hom were having the discussion did n ot seem very friendly.

10. Im not really sure about taking part in the plans that we m ade last night.

S k i l l 12: USE ADJECTIVE CLAUSE CONNECTOR/SUBJECTS CORRECTLY

In Sk ill 11 w e saw th a t a d je c tiv e c la u se c o n n e c to r s w e re u sed to in tr o d u c e c la u se s th a t d e


scrib e n o u n s . In Sk ill 12 w e w ill se e th a t in s o m e c a ses a n a d jectiv e c la u se c o n n e c to r is n o t
ju s t a c o n n e c to r ; an ad je c tiv e c la u se c o n n e c t o r can a lso b e th e su b je c t o f th e c la u se a t th e
sa m e tim e.

ADJECTIVE CLAUSE

In th e first e x a m p le th e r e a re tw o clau ses: w om an is th e su b je c t o f th e verb is fillin g , a n d


that is th e su b je c t o f th e v e rb is. T h e s e tw o c la u se s are j o in e d w ith th e c o n n e c t o r that. N o
tice th a t in th is e x a m p le th e w o rd th a t se r v es tw o fu n c tio n s a t th e sa m e tim e: it is th e su b
j e c t o f th e verb is, a n d it is th e c o n n e c to r th a t j o in s th e two c la u se s. T h e a d je c tiv e c la u se
th a t is on the table d e sc r ib e s th e n o u n glass.
In th e s e c o n d e x a m p le , th e r e a re also tw o clau ses: glass is th e su b je c t o f th e verb con
tains, a n d th a t is th e su b je c t o f th e v e rb is. In th is e x a m p le th a t also se r v e s tw o fu n c tio n s : it
is th e su b je c t o f th e v e rb is, a n d it is th e c o n n e c to r th a t jo in s th e two c la u se s. B e c a u se th a t
is on the table is an a d je c tiv e c la u se d e sc r ib in g th e n o u n glass, it d irectly fo llo w s glass.
STRUCTURE AND WRITTEN EXPRESSION

T h e fo llo w in g e x a m p le sh ow s h o w th e se s e n te n c e p a ttern s c o u ld b e te sted in th e


S tru ctu re s e c tio n o f th e T O E F L test.

E xam ple
is on the table has four sections.
(A) The notebook
(B) The notebook which
(C) Because the notebook
(D) In the notebook

In th is e x a m p le you sh o u ld n o tic e im m e d ia te ly that th e s e n te n c e has two verbs, is an d has,


a n d e a c h o f th e m n e e d s a su b ject. (You k n ow th a t table is n o t a su b ject b e c a u se it follow s
th e p r e p o sitio n on; table is th e o b je c t o f th e p r e p o sitio n .) T h e on ly answ er th a t has two
su b jects is an sw er (B ), so answ er (B ) is th e c o r r e c t answer. T h e c o r r e c t s e n te n c e sh o u ld
say: T he notebook which is on the table has fo u r sections. In this se n te n c e notebook is th e su b ject
o f th e verb has, a n d which is th e su b ject o f th e verb is. W hich is also th e c o n n e c to r that
jo in s th e two clau ses.

T h e fo llo w in g ch a rt lists th e a d jecu v e c la u se c o n n e c t o r /s u b je c ts an d th e s e n te n c e


p a tte rn s u se d w ith them :

ADJECTIVE CLAUSE CONNECTOR/SUBJECTS

who which that


(for people) (for things) (for people or things)

S V |(adiective connector/subject) VI

She needs a secretary who types fast.

S |(adjective connector/subiect) V I V

A secretary who types fast is invaluable.

EX ER C ISE 12: E ach o f th e fo llo w in g s e n te n c e s c o n ta in s m o r e than, o n e c la u se. U n d e r


lin e th e su b jects o n c e a n d th e verbs tw ice. C ircle th e c o n n e c to r s. P u t b o x e s a r o u n d th e
ad je c u v e cla u ses. T h e n in d ic a te if th e s e n te n c e s are c o r r e c t (C ) o r in c o r r e c t (I).

C 1. The ice cream (that) is served in the restaurant has a sm ooth, creamy texture.

2. T he cars are trying to enter the freeway system are lined up for blocks.

3. I have great respect for everyone who on the D eans List.

4. It is going to be very difficult to work with the man which just began working here.

5. The door that leads to the vault it was tightly locked.

6. T he neighbors reported the man who was trying to break into the car to the police.
STRUCTURE

7. T hese plants can only survive in an environm ent is extrem ely hum id.

8. T he boss m eets with any production workers w ho they have surpassed their quotas.

9. The salesclerk ran after the woman w ho had left her credit card in the store.

10. T h e shoes which m atched the dress that was on sale.

EX ER C ISE (S k ills 9 - 1 2 ) : E ach o f th e fo llo w in g s e n te n c e s c o n ta in s m o r e th a n o n e c la u se .


U n d e r lin e th e su b je cts o n c e a n d th e verb s tw ice. C ircle th e c o n n e c to r s . P u t b o x e s a r o u n d
th e cla u ses. T h e n in d ic a te if th e s e n te n c e s are c o r r e c t (C ) o r in c o r r e c t (I).

-------- 1. N o on e explained to m e w hether was com ing or not.

-------- 2. T he part o f the structure that has already b een built needs to be torn down.

-------- 3. T h e girl who she ju st jo in e d the softball team is a great shortstop.

-------- 4. I have no idea about when the m eeting is supposed to start.

-------- 5. We have b een told that we can leave w henever want.

-------- 6. T h e racquet with w hom I was playing was too big and too heavy for me.

-------- 7. I will never understand that he did.

-------- 8. H e was still sick was obvious to the entire m edical staff.

-------- 9. W'hat is m ost im portant in this situation it is to finish on time.

-------- 10. T h e newspapers that were piled up on the front porch were an indication that the
residents had not b een h om e in som e time.

T O E F L E X E R C ISE (S k ills 912): C h o o s e th e lette r o f th e w o rd o r g r o u p o f w ord s th a t


b e st c o m p le te s th e s e n te n c e .

1. Dolphins form extremely complicated 3. The Apollo I I astronauts_____ of the


allegiances a n d -------- continually change. Earths inhabitants witnessed on the
,., ... , famous first moonwalk on July 20, 1969,
(A) enmities that .. .. ..
. ... were Neil Armstrong and Buzz Aldnn.
(B) that are enm ities
(C) enmities that are (A) whom
(D) that enmities (B) whom millions
(C) were some
2. Scientists are now beginning to conduct (D) whom some were
experiments o n _____ trigger different
sorts of health risks. 4. At the end of the nineteenth century, Alfred
,., . .... Binet developed a test for measuring
(A) noise pollution can . . , . 6 r
in
(B)\ .u
that. noise
pollution
li . intelligence---------
, . served as the basis of
m odem IQ tests.
(C ) how noise pollution
(D) how noise pollution can (A) has
(B) it has
(C) and
(D) which has
STRUCTURE AND WRITTEN EXPRESSION

8. _____ will be carried in the next space


5 . _____ have at least four hours of hazardous
materials response training is mandated by shuttle payload has not yet been
announced to the public.
federal law.
(A) All police officers (A) It
(B) All police officers must (B) What
(C) That all police officers (C) When
(D) For all police officers (D) That

6. A clouds reservoir of negative charge 9. During free fa ll,_____ up to a full minute,


extends upward from the altitude a t -------- a skydiver will fall at a constant speed of
the freezing point. 120 m.p.h.

(A) temperatures hit (A) it is


(B) hit temperatures (B) which is
(C) which temperatures hit (C) being
(D) which hit temperatures (D) is

7. In a 1988 advanced officers' training 10. The fa ct_____ the most important ratings
program, Sampson developed a plan to period is about to begin has caused all
incorporate police in enforcing three networks to shore up their schedules.
environmental protection laws whenever is that
(A)
feasible. of
(B)
(A) it is (C) that
(B) is (D) what
(C) has
(D) it has

T O E F L REVIEW EX ER C ISE (S k ills 1 -1 2 ): C h o o se th e lette r o f th e w o rd o r g r o u p o f


w ord s th a t b e st c o m p le te s th e s e n t e n c e .

1 . _____ loom high above the north and 4. The benefit_____ the study is that it
northeastern boundaries of the expanding provides necessary information to anyone
city of Tucson. who needs it.

(A) The Santa Catalina mountains (A) of


(B) Because the Santa Catalina (B) which
mountains (C) that
(C) The Santa Catalina mountains are (D) because
(D) That the Santa Catalina mountains
5. The same symptoms that occu r--------
2. Radioactive_____ provides a powerful way occur with cocaine.
to measure geologic time. (A) amphetamines can
(A) it (B) with amphetamines can
(B) dates (C) so amphetamines
(C) dating (D) with amphetamines they
(D) can
6. Many companies across the country have
3. _____ contained in the chromosomes, and molded the concepts_____ describes into
they are thought of as the units of heredity. an integrated strategy for preventing stress.

(A) Genes which are (A) and Wolf


(B) Genes are (B) that Wolf
(C) When genes (C) what Wolf
(D) Because of genes (D) so Wolf
STRUCTURE

7. in the first draft of the budget will 9. A need for space law to include
not necessarily be in the final draft. commercial concerns has been recognized
inasm uch_____ been expanding
(A) Although it appears drastically in recent years.
(B) It appears
(C) What appears (A) the commercial launch industry
(D) Despite its appearance (B) the commercial launch industry has
(C) as has the commercial launch
8. If a food label indicates that a food is industry
mostly carbohydrate, it does not mean (D) as the commercial launch industry has
is a good food to eat.
10. The report on the nuclear power plant
(A) and it indicated that when the plant had gone on
(B) and line ___ _ unsafe.
(C) that it
(D) when (A) and it had been
(B) it had been
(C) had been
(D) that it had been

SENTENCES W IT H R ED U C ED CLAUSES.

It is p o ssib le in E n g lish fo r a c la u se to a p p e a r in a c o m p le te fo r m o r in a r e d u c e d form .

My friend sh ould be o n the train lafych ^ arriving at the station now.


Although w^s not realty difficult, the exam took a lot o f tirae.

T h e first s e n t e n c e sh o w s an a d je c tiv e c la u se in its c o m p le te fo r m , w hich is a rr iv in g at the


station now, a n d in its r e d u c e d fo r m , a r r iv in g a t the sta tio n now. T h e s e c o n d s e n t e n c e sh ow s
an adverb c la u se in its c o m p le te fo r m , although it was n o t really difficu lt, a n d its r e d u c e d
fo r m , although n o t really difficu lt.
T h e two typ es o f c la u se s th a t c a n r e d u c e in E n g lish are: (1) a d je c tiv e c la u se s a n d (2)
adverb c la u se s. It is im p o r ta n t to b e c o m e fa m ilia r w ith th e se r e d u c e d c la u se s b e c a u s e
th ey a p p e a r fr e q u e n tly o n th e T O E F L test.

S k i l l 13: USE RED U C ED AD JEC TIVE CLAUSES CORRECTLY

A d jective c la u se s c a n a p p e a r in a r e d u c e d fo r m . In th e r e d u c e d fo r m , th e a d je c tiv e c la u se
c o n n e c to r a n d th e -verb th a t d ir e c d y fo llo w it are o m itte d .

T h e wom an ntyo if waving to us is the tour guide.


T h e letter whfch likis written last week arrived today.
T h e pitcher t\a t on the table is full o f iced tea.

E ach o f th e se s e n t e n c e s m ay b e u s e d in th e c o m p le te fo r m o r in th e r e d u c e d fo r m . In th e
r e d u c e d fo r m th e c o n n e c to r who, which, o r th a t is o m itte d a lo n g w ith th e >e-verb is o r was.
If th e r e is n o be-\erb in th e a d je c tiv e c la u se , it is still p o ssib le to h ave a r e d u c e d fo r m .
W h en th e r e is n o e-verb in th e a d je c tiv e c la u se , th e c o n n e c to r is o m itte d a n d th e verb is
c h a n g e d in to th e -n gform .
STRUCTURE AND WRITTEN EXPRESSION

appearing
I d o n t understand the article wftych apfcars in todays paper.

In th is e x a m p le th e r e is n o fte-verb in th e ad jective c la u se which appears in today's paper,


so th e c o n n e c to r w hich is o m itte d a n d th e m ain verb appears is c h a n g e d to th e -in g form
appearing.
It sh o u ld b e n o te d th a t n o t all ad jective c la u se s can a p p ea r in a r e d u c e d fo r m . A n ad
je c tiv e c la u se ca n a p p ea r in a r e d u c e d form o n ly i f th e ad jective c la u se c o n n e c to r is fo l
lo w e d d irectly by a verb . In o th e r w ords, an ad jective cla u se c a n o n ly b e r e d u c e d if th e
c o n n e c to r is a lso a su b ject.

T he woman that I ju st met is the tour guide. (does not reduce)


T he letter which you sent me arrived yesterday. (does not reduce)

In th e s e two e x a m p le s th e ad jective cla u ses c a n n o t b e r e d u c e d b e c a u se th e adjective


c la u se c o n n e c to r s th a t a n d which are n o t d irectly fo llo w e d by verbs; th a t is directly fo l
lo w ed by th e su b ject I, a n d which is d irectly fo llo w e d by th e su b ject you.
A fin al p o in t to n o te is th at so m e ad jective cla u ses are set o f f from th e rest o f th e s e n
te n c e w ith c o m m a s, a n d th e se ad jective cla u ses can also b e r e d u c e d . In a d d itio n , w h en an
ad jective c la u se is se t o f f w ith c o m m a s, th e r e d u c e d ad jective cla u se can a p p ea r at th e
fr o n t o f th e s e n te n c e .

T he W hite H ouse, which is located in Washington, is the hom e o f the president.


T he W hite H ouse, heated in Washington, is the hom e o f the president.
Located in Washington, the W hite House is the hom e o f the president.

T he president, who is now preparing to give a speech, is m eeting with his advisors.
T he president, now preparing to give a speech, is m eeting with his advisors.
Now prefmring to give a speech, the president is m eeting with his advisors.

In th e s e two e x a m p le s , th e ad jective cla u ses are set o f f from th e rest o f th e s e n te n c e w ith


c o m m a s, so e a c h s e n te n c e can b e str u c tu r ed in th r e e d iffe r e n t ways: ( 1 ) w ith th e c o m
p le te c la u se , (2 ) w ith th e r ed u c e d c la u se fo llo w in g th e n o u n th at it d e sc r ib es, an d (3)
w ith th e r e d u c e d c la u se at th e b e g in n in g o f th e s e n te n c e .
T h e fo llo w in g e x a m p le sh ow s h o w r e d u c e d ad jective c la u se s c o u ld b e te sted in the
S tru ctu re se c tio n o f th e T O E F L test.

Exam ple
_____ on several different television programs, the witness gave
conflicting accounts of what had happened.
(A) He appeared
(B) Who appeared
(C) Appearing
(D) Appears

In th is e x a m p le , answ er (A) is in c o r r e c t b e c a u se th ere are two clau ses, H e appeared. . . an d


the w itness g a v e . . . , a n d th ere is n o c o n n e c to r to jo in th e m . A nsw er (B) is in c o r r e c t b ecau se
STRUCTURE

an ad jective c la u se su c h as w ho appeared . . . c a n n o t a p p ea r at th e b e g in n in g o f a s e n te n c e
(u n le ss it is in a r e d u c e d fo r m ). A n sw er (C ) is th e c o r r e c t an sw er b e c a u se it is th e re
d u c e d fo r m o f th e c la u se w ho appeared, a n d th is r e d u c e d fo r m can a p p e a r at th e fr o n t o f
th e s e n te n c e . A n sw er (D ) is n o t th e r e d u c e d fo rm o f a verb; it is m e r e ly a verb in th e p r e
s e n t ten se; a verb su c h as appears n e e d s a su b je c t a n d a c o n n e c to r to b e c o r r e c t.
T h e fo llo w in g c h a r t lists th e str u c tu r e fo r r e d u c e d ad jective c la u se s a n d r u le s fo r h o w
a n d w h e n r e d u c e d fo r m s c a n be u sed :

REDUCED ADJECTIVE CLAUSES

with a be-verb in the (ADJECTIvrtrONNECOR/SyBiEGT) "


adjective clause __ __fwfio which tfrat)~

with no be-verb in the (AOJECTTvTtTONNtCIgR^yBJECT)---^'''^ (VERB + INC)


adjective clause - (wfio which Vrai)'

To reduce an adjective clause, omit the adjective clause connector/subject and the be-verb.
If there is no be-verb, omit the connector/subject and change the main verb to the -ing form.
Only reduce an adjective clause if the connector/subject is directly followed by the verb.
If an adjective clause is set off with commas, the reduced clause can be moved to the front of the
sentence.

E X E R C ISE 13: E ach o f th e fo llo w in g s e n te n c e s c o n ta in s a n a d je c tiv e c la u se , in a c o m


p le te o r r e d u c e d fo r m . U n d e r lin e th e ad jective cla u se s. T h e n in d ic a te i f th e s e n te n c e s
are c o r r e c t (C ) o r in c o r r e c t (I).

__ _ 1. We will have to return the m erchandise purchased yesterday at the Broadway.

__ L___ 2. T he children sat in the fancy restaurant found it difficult to behave.

_____ 3. Serving a term o f four years, the mayor o f the town will face reelection n ext year.

_____ 4. T h e brand new Cadillac, purchasing less than two weeks ago, was destroyed in the
accident.

_____ 5. T he fans who supporting their team always com e out to the gam es in large num bers.

___ 6. T he suspect can be seen in the photographs were just released by the police.

_____ 7. T h e food placing on the picnic table attracted a large num ber o f flies.

_____ 8. Impressed with everything she had heard about the course, Marie sign ed her
children up for it.

_____ 9. T h e passengers in the airport waiting room , heard the an n ou n cem en t o f the
canceled flight, groaned audibly.

_____ 10. Dissatisfied with the service at the restaurant, the meal really war not enjoyable.
STRUCTUREANDWRITTEN EXPRESSION

S kill 14: USE REDUCED ADVERB CLAUSES CORRECTLY

A d verb c la u se s can also ap p ea r in a r e d u c e d fo r m . In th e r e d u c e d fo r m , th e adverb c o n


n e c to r rem a in s, b u t th e su b ject a n d e-verb are o m itte d .
Although hp k rather unwell, the speaker will take part in the seminar.

W'hen \\m ready, you can begin your speech.

T h e s e tw o e x a m p le s m ay b e u sed in e ith e r th e c o m p le te o r r e d u c e d fo r m . In th e r ed u c ed
fo r m , th e ad verb c o n n e c to r s although a n d when rem ain ; th e su b jects he a n d you as w ell as
th e e-verbs is a n d are are o m itted .
I f th e r e is n o e-verb in th e adverb c la u se , it is still p o s sib le to h ave a r e d u c e d form .
W h e n th e r e is n o e-verb in th e adverb c la u se, th e su b je ct is o m itte d a n d th e m a in verb is
c h a n g e d in to th e -r e fo r m .

feeling
Although \e fetys rather sick, the speaker will take part in the seminar.
giving
When yhu gfyeyour speech, you should speak loudly and distinctly.

In th e first e x a m p le th e adverb c la u se although he feels rather sick d o e s n o t in c lu d e a everb;


to r e d u c e th is c la u se , th e su b ject he is o m itte d a n d th e m a in verb feels is c h a n g e d to feeling.
In th e s e c o n d e x a m p le th e adverb c la u se w hen you give y o u r speech also d o e s n o t in c lu d e a
e-verb; to r e d u c e th is c la u se , th e su b ject you is o m itte d a n d th e m a in verb give is c h a n g e d
to g iving.
T h e fo llo w in g e x a m p le sh ow s h ow this s e n te n c e p a tte r n c o u ld b e te sted in th e Struc
tu re s e c tio n o f th e T O E F L test.

E xam ple
W hen_____ , you are free to leave.
(A) the finished report
(B) finished with the report
(C) the report
(D) is the report finished

In th is e x a m p le you sh o u ld n o tic e th e adverb c o n n e c to r when, a n d y o u sh o u ld k n ow th at


th is tim e w ord c o u ld b e fo llo w e d by e ith e r a c o m p le te cla u se o r a r e d u c e d c la u se. A n
sw ers (A ) a n d (C ) c o n ta in th e su b jects the fin ish e d report an d the report a n d n o verb, so
th e se an sw ers are in c o r r e c t. In answ er (D ) th e su b je ct a n d verb are in v e rte d , a n d this is
n o t a q u e s tio n , so an sw er (D ) is in c o r r e c t. T h e c o r r e c t answ er is answ er (B ); th is answ er
is th e r e d u c e d fo r m o f th e cla u se when you are fin ish e d w ith the report.
It sh o u ld b e n o te d th at n o t all adverb c la u se s can a p p ea r in a r e d u c e d fo r m , an d a
n u m b e r o f ad verb c la u se s can o n ly b e r e d u c e d i f th e verb is in th e passive form .
Once you submit your thesis, you will graduate. Once ^ submitted., your thesis will be reviewed,
(active does not reduce) (passive does reduce)

In th e first e x a m p le , th e adverb clau se once you subm it y o u r thesis d o e s n o t r e d u c e b e c a u se


c la u se s in tr o d u c e d by once o n ly r e d u c e i f th e verb is passive, a n d th e verb subm it is active.
In th e s e c o n d e x a m p le , th e adverb cla u se once it is subm itted d o e s r e d u c e to once subm itted
b e c a u se th e c la u se is in tr o d u c e d by once an d th e verb is subm itted is passive.
STRUCTURE

T h e fo llo w in g c h a r t lists th e str u c tu r es for r e d u c e d adverb c la u se s a n d w h ic h adverb


c la u se c o n n e c to r s c a n b e u s e d in a r e d u c e d form :

- R E D U C E D A D V ER B C lA U S E S

w ith a be-verb in (a d verb c o n n ecto r) X


the ad verb clause

w ith no be-verb in (ADVERB CONNECTOR) (VERB + ING)


the adverb clause

T im e C o n d itio n C o n tra s t Place M a n n er

reduces in a c t iv e after although


before unless though
since whether
while
when

reduces in pa ssiv e once if although where as


until unless though wherever
when whether
whenever

To red u ce an ad verb clause, o m it th e subject and the be-verb fro m the ad verb clause.
If th e re is no be-verb, then o m it th e subject and change the ve rb to the -ing form .

E X ER C ISE 14: E ach o f th e fo llo w in g s e n te n c e s c o n ta in s a r e d u c e d ad verb c la u se . C ircle


th e ad verb c o n n e c to r s . U n d e r lin e th e r e d u c e d c la u se s. T h e n in d ic a te i f th e s e n t e n c e s are
c o r r e c t (C ) o r in c o r r e c t (I).

1 . (if) not com pletely satisfied, you can return the product to the manufacturer.

! 2. Steve has had to learn how to cook and clean (since) left hom e.

-------- 3. T h e ointm ent can be applied where needed.

-------- 4. Tom began to look for a job after com pleting his masters degree in engineering.

-------- 5. A lthough n e t selectin g for the team, he attends all o f the gam es as a fan.

-------- 6. W hen purchased at this store, the buyer gets a guarantee on all items.

-------- 7. T h e m edicine is not effective unless taken as directed.

-------- 8. You should negotiate a lot before buy a new car.

-------- 9. O n ce purchased, the swimsuits cannot be returned.

-------- 10. T h ou gh located near the coast, the town does not get m uch o f an ocean breeze.
STRUCTURE AND WRITTEN EXPRESSION

EX ER C ISE (Sk ills 1 3 -1 4 ): E ach o f th e fo llo w in g s e n te n c e s c o n ta in s a r e d u c e d c la u se.


U n d e r lin e th e r e d u c e d c la u se s. T h e n in d ic a te i f th e s e n te n c e s are c o r r e c t (C ) o r in c o r
r e c t (I).

-------- 1. T hough was surprised at the results, she was pleased with what she had don e.

_ 2. Wearing only a light sweater, she stepped out into the pouring rain.

-------- 3. T he family stopped to visit m any relatives while driving across the country.

_____ 4. T he com pany president, n eed ed a vacation, boarded a plane for the Bahamas.

_____ 5. When applying for the jo b , you should bring your letters o f reference.

_____ 6. She looked up into the dreary sky was filled with dark thunderclouds.

____ _ 7. Feeling weak after a long illness, Sally wanted to try to get back to work.

_____ 8. Before decided to have surgery, you should get a second opinion.

_____ 9. T he construction material, a rather grainy type o f wood, gave the room a rustic
feeling.

_____ 10. The application will at least be reviewed if submitted by the fifteenth o f the m onth.

T O E F L EXER CISE (S k ills 1 3 -1 4 ): C h o o s e th e le tte r o f th e w ord o r g r o u p o f w ord s th at


b e st c o m p le te s th e s e n te n c e .

1. W hen_____ nests during spring nesting 4. --------- benind government secrecy for
season, Canadian geese are fiercely nearly half a century, the Hanford plant in
territorial. central Washington produced plutonium
for the nuclear weapons of the Cold War.
(A) building
(B) are building (A) It is hidden
(C) built (B) Hidden
(D) are built (C) Which is hidden
(D) The plant is hiding
2. In 1870, Calvin, along with Adirondack
hunter Alvah Dunning, made the first 5. U n til--------incorrect, astronomers had
known ascent of Seward M ountain,_____ assumed that the insides of white dwarfs
far from roads or trails. were uniform.
(A) a remote peak (A) they
(B) it is a remote peak (B) their proof
(C) a remote peak is (C) the astronomers recently proven
(D) which a remote peak (D) recently proven

3. Kokanee salmon begin to deteriorate and 6. --------- artifacts from the early Chinese
die so o n _____ at the age o f four. dynasties, numerous archeologists have
explored the southern Silk Road.
(A) they spawn
(B) after spawning (A) They were searching for
(C) spawn (B) It was a search for
(D) spawned the salmon (C) Searched for
(D) Searching for
STRUCTURE

7. In Hailey, th e best-know n lectu rer was 9. in N orth Am erican w aterw ays less
w om ens rights activist Abigail Scott th an a decade ago, zebra m ussels have
Duniway o f P ortland, Oregon, w ho could already earned a nasty rep u tatio n for th eir
usually be persu ad ed to sp e a k _____ tow n expensive h abit of clogging w ater pipes in
visiting h e r son. the Great Lakes area.
(A) she w as in (A) The first sighting
(B) while in (B) Although first sighted
(C) why le she was (C) Zebra m ussels w ere first sighted
(D) was in (D) First sighting

8. The N ational R e sta u ra n t_____ 10. Sm all com panies m ay take th eir goods
W ashington, says th a t federal efforts to abroad for trad e show s w ithout paying
regulate w orkplace sm oking w ould lim it foreign value-added taxes by acquiring
re stau ra n ts ability to respond to the an ATA carnet.
desires of th e ir patrons.
(A) a docum ent calls
(A) Association in (B) a docum ent called
(B) Association is in (C) calls a docum ent
(C) A ssociation w hich is in (D) called a docum ent
(D) Association, based in

T O E F L R EV IEW E X E R C ISE (S k ills 1 -1 4 ): C h o o se th e lette r o f th e w o r d o r g r o u p o f


w ord s th a t b e st c o m p le t e s th e s e n te n c e .

1. In the United S tates_____ approximately 5. --------a cheese shop has since grown into a
four million miles of roads/streets, and small conglomerate consisting of a
highways. catering business and two retail stores.
(A )there (A) In the beginning of
(B) is (B) It began as
(C) they (C) Its beginning which was
(D) there are (D) What began as

2 . _____ twelve million immigrants entered 6. Primarily a government contractor,_____


the United States via Ellis Island. preferential treatment from government
agencies as both a minority-group member
(A )More than
and a woman.
(B) There were more than
(C) Of more than (A) receives Weber
(D) The report of (B) Weber receives
(C) the reception o f Weber
3. The television,_____ so long been a part o f (D) according to Weber's reception
our culture, has an enormous influence.
7. Because the project depends o n _____ at
(A )has
the federal level, the city and county may
(B) it has
have to wait until the budget cutting ends.
(C) which
(D) which has (A) it happens
(B) which happening
4. Psychologists have traditionally (C) what happens
maintained that infants cannot formulate (D) that it happens
long-term m emories u n til_____ the age of
eight or nine months.
(A )they
(B) they reach
(C) to reach
(D) reach
STRUCTURE AND WRITTEN EXPRESSION

8. definitive study of a w estern h a rd 10. _ early approaches for coping with


rock m ining com m unity cem etery appears w orkplace stress dealt w ith the problem
to have been done is in Silver City, Nevada. only after its sym ptom s had appeared.

(A) Most (A) Although well intending


(B) The m ost (B) Although it is a good intention
(C) W here m ost (C) Although a good intention
(D) W here the m ost (D) Although well intended

One of the areas of m ultim edia th a t is


growing quickly is sound.
(A) yet is easily overlooked
(B) is easily overlooked
(C) it is easily overlooked
(D) th at is easily overlooked

SENTENCES W IT H INVERTED SUBJECTS A N D VERBS-----------

S u b jects a n d verb s are in v e rte d in a variety o f situ a tio n s in E n g lish . In v erted su b jects an d
verb s o c c u r m o st o fte n in th e fo r m a tio n o f a q u e s tio n . T o fo r m a q u e s tio n w ith a h e lp in g
verb (be, have, can, could, will, would, e t c .) , th e su b ject a n d h e lp in g v erb are in verted .

H e can go to th e movies.
C an h e go to th e movies?

You w ould tell m e the tru th .


W ould you tell m e th e tru th ?

She was sick yesterday.


Was she sick yesterday?

To fo r m a q u e s tio n w h e n th e r e is n o h e lp in g verb in th e se n te n c e , th e h e lp in g verb do is


u sed .

H e goes to the movies.


D oes h e go to th e movies?

You to ld m e th e tru th .
D id you tell m e th e tru th ?

T h e r e are m a n y o th e r situ a tio n s in E n g lish w h e n su b je cts a n d verb s are in v e rte d , b u t if


y o u j u s t r e m e m b e r th is m e th o d o f in v e rtin g su b jects a n d verb s, y o u w ill b e a b le to h a n d le
th e o th e r situ a tio n s. T h e m o st c o m m o n p r o b le m s w ith in v e rte d su b jects an d verb s o n th e
T O E F L te st o c c u r in th e fo llo w in g situ ation s: (1 ) w ith q u e s tio n w ord s su ch as what, when,
where, why, a n d how; (2) a fter so m e p la c e e x p r essio n s; (3) a fter n e g a tiv e ex p ressio n s; (4)
in s o m e c o n d itio n a ls; an d (5 ) after s o m e c o m p a r iso n s.
STRUCTURE

S k ill 15: IN V E R T T H E SUBJECT A N D V E R B W IT H Q U E S T IO N W O R D S

T h e r e is s o m e c o n fu s io n a b o u t w h e n to in v e rt th e su b ject a n d verb a fter q u e s tio n w ord s


su c h as what, when, where, why, a n d how. T h e se w ords can have two v e ry d if fe r e n t fu n c tio n s
in a s e n te n c e . First, th e y c a n in tr o d u c e a q u e s tio n , a n d in th is case th e su b je c t a n d verb
th a t fo llo w a r e in v e rte d .
What is the hom ew ork?
When can I leave?
Where a re you g o in g ?

A lso, th e se w o rd s c a n j o in to g e th e r tw o cla u ses, a n d in th is c a se th e su b je c t a n d verb th a t


fo llo w are n o t in v e rte d .
I d o n o t know what the hom ew ork is.
When I can leave, I will take th e first train.
D o you know where you. a re g o in g ?

In e a c h o f th e s e e x a m p le s th e r e are tw o cla u ses j o in e d by a q u e s tio n w ord . N o t ic e th a t th e


su b jects a n d verb s th a t fo llo w th e q u e s tio n w ord s w hat, when, a n d where are n o t in v e r te d in
th is case.
T h e fo llo w in g e x a m p le sh ow s h o w this s e n te n c e p a tte rn c o u ld b e te s te d in th e S tru c
tu re se c tio n o f th e T O E F L test.

E x am p le
The law yer asked the client why it.

(A) did he do
(B) did he *
(C) he did
(D) did

In th is e x a m p le th e q u e s tio n w o rd why is u s e d to c o n n e c t th e two c la u se s, so a su b je c t a n d


verb are n e e d e d a fter th is c o n n e c to r ; this is n o t a q u e s tio n , so th e su b je c t a n d verb s h o u ld
n o t b e in v e r te d . T h e b e st an sw er is th e r e fo r e answ er (C ).
T h e fo llo w in g c h a r t lists th e q u e s tio n w ord s a n d th e ir s e n t e n c e p a ttern s:

INVERTED SUBJECTS AND VERBS WITH QUESTION WORDS

when
---------- T T - r r : :

When the questii introduces a question, the subject and verb are invei

mestion woi

W hen the questii :onnects tw o clauses, the subject and verb that follow are not ii

question won

I know
.. -
STRUCTURE AND WRITTEN EXPRESSION

E X E R C IS E 15: E ach o f th e fo llo w in g s e n te n c e s c o n ta in s a q u e s tio n w ord. C ircle th e q u es


tio n w ord s. U n d e r lin e th e su b jects o n c e a n d th e verb s tw ice. T h e n In d ic a te i f th e se n
te n c e s are c o r r e c t (C ) o r in c o r r e c t (I).

1. The p h on e company is n ot certain (When) will the new directories be ready.

C 2. The professor does not understand (why) so many students did poorly on the exam.

3. How new students can get inform ation about parking?

4. W here is it cheapest Ld get typeset copies printed?

5. Only the pilot can tell you how far can the plane go on one tank o f fuel.

6. What type o f security does he prefer for his investments?

7. N ot even the bank president knows when the vault will be opened.

8. H ow lon g it has been since you arrived in the U nited States?

9. T he jury doubts what the witness said under cross-examination.

10. Do you know why he wants to take an extended leave o f absence?

S k ill 16: INVERT TH E SUBJECT A N D VERB W IT H PLACE EXPRESSIONS

A fte r id e a s e x p r e s sin g p la c e , th e su b ject a n d th e verb so m e tim e s in v ert in E n g lish . T h is


ca n h a p p e n w ith sin g le w ord s e x p r e s sin g p la c e, su ch as here, there, o r nowhere.

Here is the book that you lent me.


There are the keys that I thought I lost.
Nowhere have I seen such beautiful weather.

In th e fir s t e x a m p le th e p la c e w ord here c a u ses th e su b ject book to c o m e a fter th e verb is.


In th e s e c o n d e x a m p le th e p la c e w ord there c a u ses th e su b je ct keys to c o m e after th e verb
are. In th e last e x a m p le th e p la c e w ord nowhere ca u se s th e su b ject I to c o m e after th e verb
have.
T h e su b je c t a n d verb can a lso b e in v erted a fter p r e p o sitio n a l p h ra ses e x p r e ssin g
p la c e.

In the closet are the cloth es that you want.


Around the comer is Sams house.
Beyond the mountains lies the town where you will live.

In th e first e x a m p le th e p r e p o sitio n a l p h ra se o f p la c e in the closet c a u se s th e su b je ct clothes


to c o m e a fter th e verb are. In th e s e c o n d e x a m p le th e p r e p o sitio n a l p h ra se o f p lace
a ro u n d the com er c a u se s th e su b ject house to c o m e after th e verb is. In th e la st e x a m p le th e
p r e p o sitio n a l p h r a se o f p la c e beyond the m o u n ta in s ca u se s th e su b je c t tow n to c o m e after
th e verb lies.
STRUCTURE

It is im p o r ta n t (a n d a b it d iffic u lt) to u n d e r sta n d th a t th e su b je ct a n d verb w ill in vert


a fter p la c e e x p r e s s io n s at th e b e g in n in g o f a s e n te n c e o n ly w h e n th e p la c e e x p r e s s io n is
necessary to c o m p le te th e s e n te n c e . S tu d y th e fo llo w in g ex a m p les:

In the forest are many exotic birds.


In the forest I walked for many hours.

In th e first e x a m p le th e su b je c t birds a n d verb are are in v e rte d b e c a u se th e p la c e e x p r e s


s io n in the forest is n e e d e d to c o m p le te th e id e a m a n y exotic birds are. . . . In th e s e c o n d e x
a m p le th e su b je ct / a n d th e verb w alked a r e n o t in v e r te d b e c a u se th e id e a I w alked f o r m any
hours is c o m p le t e w ith o u t th e p la c e e x p r e s sio n in the forest; th e p la c e e x p r e s sio n is th e r e
fo r e n o t n e e d e d to c o m p le te th e s e n te n c e .
T h e fo llo w in g e x a m p le sh ow s h o w th is s e n te n c e p a tte r n c o u ld b e te ste d in th e S tru c
tu re s e c tio n o f th e T O E F L test.

Exam ple
On the second level of the parking lot
(A) is empty
(B) are empty
(C) som e empty stalls are
(D) are some empty stalls

T h is e x a m p le b e g in s w ith th e p la c e e x p r e s sio n on the second level o f the p a rk in g lot, w h ich


c o n sists o f two p r e p o sitio n a l p h ra ses, on the second level a n d o f the p a rk in g lot. T h is s e n te n c e
n e e d s a su b je ct a n d a v e rb to b e c o m p le te , a n d th e two answ ers th a t c o n ta in b o th a su b
j e c t , stalls, a n d verb , are, are an sw ers (C ) a n d (D ). T h e su b ject a n d verb sh o u ld b e in
v e rte d b e c a u s e th e p la c e e x p r e s sio n is n e c e s sa r y to c o m p le te th e id e a some empty stalls
are. . . . T h e b e st an sw er is th e r e fo r e an sw er (D ).

T h e fo llo w in g ch art lists th e s e n t e n c e p a tte rn s u s e d w ith p la c e e x p r essio n s:

INVERTED SUBJECTS AND VERBS WITH PLACE EXPRESSIONS

W h e n a place exp ression at th e fro n t o f the sentence is necessary to co m p lete


th e sentence, th e subject and v e rb that fo llo w are inverted.

( PLACE (n e c e s s a ry )) V S

In the classroom were some old desks.

W h e n a place expression at th e fro n t o f th e sen ten ce contains extra inform ation th a t is not
needed to co m p le te th e sen ten ce, the subject and ve rb that fo llo w are not inverted.

( PLACE (extra) ) S V

In the classroom,I studied very hard.


STRUCTURE AND WRITTEN EXPRESSION

E X E R C ISE 16: E a ch o f th e fo llo w in g s e n te n c e s c o n ta in s an e x p r e s sio n o f p la c e at th e b e


g in n in g o f th e se n te n c e . C ircle th e e x p r e s sio n s o f p la c e. L o o k at th e cla u ses th a t im m e d i
a tely fo llo w th e p la c e e x p r e ssio n s a n d u n d e r lin e th e su b jects o n c e a n d th e verb s tw ice.
T h e n in d ic a te i f th e s e n te n c e s are c o r r e c t (C ) o r in c o r r e c t (I).

c 1 . (In front o f the house) were som e giant trees.

1 2. (There) a big house is on the corner.

3. In the cave was a vast treasure o f gem s and jewels.

4. To the north the stream is that the settlers will have to cross.

fi. Around the corner are the offices that you are trying to Find.

6. At the Italian restaurant was the food too spicy for my taste.

V. Now here in the world farmers can grow such delicious food.

8. In the backyard the two trees are that need to be pruned.

9. Around the recreation hall and down the path are the tents where we will be staying
this week.

j.___ 10. In the apartment next to m ine, a family was that had a lot o f pets.

S k ill 17: IN VER T T H E SUBJECT A N D VERB W IT H NEGATIVES

T h e su b je c t a n d verb can also b e in v e rte d a fter c er ta in n e g a tiv es a n d r ela ted ex p r essio n s.


W h e n n e g a tiv e e x p r e ssio n s, su ch as no, not, o r never, c o m e at th e b e g in n in g o f a se n te n c e ,
th e su b je ct a n d verb are in v erted .

Not once did I miss a question.


Never has Mr. Jones taken a vacation.
A t no time can the woman talk on the telephone.

In th e first e x a m p le th e n e g a tiv e e x p r e ssio n not once c a u se s th e su b ject I to c o m e after the


h e lp in g verb did. In th e s e c o n d e x a m p le th e n e g a tiv e w ord never ca u se s th e su b ject Mr.
Jones to c o m e after th e h e lp in g verb has. In th e last e x a m p le th e n e g a tiv e e x p r e ssio n at no
tim e ca u se s th e su b je ct w om an to c o m e after th e h e lp in g verb can.
C erta in w ord s in E n g lish , su ch as hardly, barely, scarcely, a n d only, act lik e n egatives.
I f o n e o f th e s e w o rd s c o m e s at th e b e g in n in g o f a s e n te n c e , th e su b je ct a n d verb are also
in v e rte d .

Hardly ever does he take tim e off.


(This means that he almost never takes time off.)

Only once did the manager issue overtim e paychecks.


(This means that the manager almost net;-issued overtime paychecks.)
STRUCTURE

I n th e first e x a m p le th e a lm o st n e g a tiv e e x p r e s sio n hardly ever c a u se s th e su b je c t he to


c o m e a fter th e h e lp in g verb does. In th e s e c o n d e x a m p le th e a lm o st n e g a tiv e e x p r e s sio n
only once c a u se s th e su b je ct m anager to c o m e after th e h e lp in g verb did.
W h e n a n e g a tiv e e x p r e s sio n a p p ea r s in fr o n t o f a su b je ct a n d verb in t h e m id d le o f a
s e n t e n c e , th e su b je c t an d verb are also in v e rte d . T h is h a p p e n s o fte n w ith th e n e g a tiv e
w o rd s neither a n d nor.

I do n o t want to go, and neither does Tom .


T he secretary is n ot attending the m eeting, nor is her boss.

In th e first e x a m p le th e n e g a tiv e neither ca u se s th e su b je ct Tom to c o m e a fte r th e h e lp in g


v e rb does. In th e s e c o n d e x a m p le th e n e g a tiv e nor c a u se s th e su b ject boss to c o m e a fter th e
v erb is.
T h e fo llo w in g e x a m p le sh ow s h o w th is s e n te n c e p a tte rn c o u ld b e te ste d in th e S tru c
tu re s e c tio n o f th e T O E F L test.

Exam ple
Only in extremely dangerous situations stopped.
(A) will be the printing presses
(B) the printing presses will be
(C) that the printing presses will be
(D) will the printing presses be

In th is e x a m p le y o u s h o u ld n o tic e th a t th e s e n t e n c e b e g in s w ith th e n e g a tiv e only, so an


in v e r te d su b je c t a n d verb are n e e d e d . A n sw er (D ) c o n ta in s a c o r r e c tly in v e r te d su b je c t
a n d verb, w ith th e h e lp in g verb w ill, th e su b ject p r in tin g presses, a n d th e m a in v e rb be, so
an sw er (D ) is th e b e st answ er.

T h e f o llo w in g c h a r t lists th e n e g a tiv e e x p r e s sio n s a n d th e s e n te n c e p a tte r n u s e d w ith


th em :

IN V ER T ED S U B JE C T S A N D V E R B S W IT H N EG A T IVES

no not never neither nor


barely hardly only rarely scarcely seldom
'______ ______;_____ ____________;______ . . ._____ - _____ -____________
W hen a negative expression appears in front of a subject and verb (at the
beginning of a sentence or in the middle of a sentence), the subject and verb
ore inverted.

(negative expression) _ V S

Rarely were they so happy.


. ' ' ' . . _____ . - . - ' . -
STRUCTURE AND WRITTEN EXPRESSION

E X E R C ISE 17: E ach o f th e fo llo w in g s e n te n c e s c o n ta in s a n e g a tiv e o r a lm o st negative"


e x p r e s s io n . C ircle th e n e g a tiv e e x p r essio n s. L o o k at th e c la u se s th a t fo llo w a n d u n d e r lin e
th e su b je cts o n c e a n d th e verb s tw ice. T h e n in d ic a te i f th e s e n te n c e s a r e c o r r e c t (C) o r in
c o r r e c t (I).

! 1. (N ever) the boy wrote to his sisters.

2. (O n no occasion) did they say that to me.

-------- 3. Steve did not win the prize, nor did he expect to do so.

-------- 4. Only once in my life gone I have to New York City.

_____ 5. Did he go out o f the house at no time.

_____ 6. Seldom their secretary has made such mistakes.

-------- 7. N o sooner had she hung up the ph one than it rang again.

-------- 8. Sheila did not arrive late for work, nor she left early.

_____ 9. Barely had he finished the exam when the graduate assistant collected the papers.

-------- 10. T he police did not arrive in time to save the girl, and neither did the paramedics.

S k ill 18: IN V E R T T H E SUBJECT A N D VERB W IT H C O N D IT IO N A L S

In c e r ta in c o n d itio n a l stru ctu res, th e su b ject a n d verb m ay a lso be in v e rte d . T h is can o c


cu r w h e n th e h e lp in g verb in th e c o n d itio n a l c la u se is had, should, o r were, an d th e c o n d i
tio n a l c o n n e c t o r i f is o m itte d .

I f he had taken more time, the results would have been better.
Had he taken more tim e, the results would have been better.

I would help you i f I were in a position to help.


I would help you were 1 in a position to help.

I f you should arrive before 6:00, just give me a call.


Should you arrive before 6:00, just give m e a call.

In e a c h o f th e s e e x a m p le s y o u can s e e th at w h e n i f is in c lu d e d , th e su b ject a n d verb are


in t h e r eg u la r o r d e r ( i f he h a d taken, i f I were, i f you should arrive). It is also p o ssib le to o m it
if, in this c a se , th e su b ject a n d verb are in v erted ( h a d he taken, were I, should y o u a rrive).
STRUCTURE

T h e fo llo w in g e x a m p le sh o w s h o w th is s e n te n c e p a ttern c o u ld b e te ste d in th e S tr u c


ture se c tio n o f th e T O E F L test.

Exam ple
The report would have been accepted in checking its accuracy.
(A) if more care
(B) more care had been taken
(C) had taken more care
(D) had more care been taken

In th is e x a m p le a c o n n e c to r ( if) a n d a su b je ct a n d verb are n e e d e d , b u t /c o u ld b e o m it


ted a n d th e su b je ct a n d verb in v e r te d . A n sw er (A) is in c o r r e c t b e c a u se it c o n ta in s th e
c o n n e c to r i /a n d th e su b je ct care b u t n o verb. A n sw er (B ) js in c o r r e c t b e c a u se it c o n ta in s
th e su b je ct care a n d th e verb had. been taken b u t d o e s n o t have a c o n n e c to r . In an sw ers (C )
an d (D ), i f h as b e e n o m itte d . B e c a u se it is c o r r e c t to in vert th e su b ject more care a n d th e
h e lp in g verb had, a n sw er (D ) is c o r r e c t.

T h e fo llo w in g c h a r t lists th e c o n d itio n a l verb s th a t m ay in vert a n d th e s e n te n c e p at


tern s u se d w ith them :

INVERTED SUBJECTS AND VERBS WITH CONDITIONALS

had should were

When the verb in the conditional clause is had, should, or were, it is possible
to omit if and invert the subject and verb.
(omitted if) V S
W ere he here, he would help.

It is also possible to keep if! Then the subject and verb are not inverted.
if S V
If he were here, he would help.

EX ER C ISE 18: E ach o f th e fo llo w in g s e n te n c e s c o n ta in s a c o n d itio n a l (w ith a sta te d o r


im p lie d i f ) . C ircle th e c o n d itio n a ls , o r p u t an asterisk (*) w h er e / h a s b e e n o m itte d .
L ook at th e c la u se s th a t fo llo w an d u n d e r lin e th e su b jects o n c e a n d th e verb s tw ice. T h e n
in d ic a te if th e s e n te n c e s are c o r r e c t (C ) o r in c o r r e c t (I).

^L_ 1. *Were our neighbors a bit m ore friendly, it would be somewhat easier to get to know
them .

! 2. There are plenty o f blankets in the closet if should you get cold during the night.

-------- 3. Has he en ou gh vacation days left this year, he will take two full weeks o f f in
December.

4. Had we b een inform ed o f the decision, we m ight have had som ething to say about it.
STRUCTURE AND WRITTEN EXPRESSION

5. I would like to know could you help m e pack these boxes.

6. H e would have b een in big trouble had n ot he rem em bered the assignm ent at the
last m inute.

7. If your friends com e to visit, will they stay in a hotel or at your house?

8. H e m ight be a little m ore successful today was he a little m ore willing to do som e
hard work.

9. Should you ever visit this town again, I would be delighted to show you around.

10. D o you think that she would give the speech were she asked to do so?

S k ill 19: IN VER T T H E SUBJECT A N D VERB W IT H COMPARISONS

A n in v e r te d su b je c t a n d verb m ay o c c u r also a fter a c o m p a r iso n . T h e in v e rsio n o f a su b


j e c t a n d verb a fter a c o m p a r is o n is o p tio n a l, ra th er th an r e q u ir e d , an d it is a ra th er for
m al str u c tu r e. T h e r e h ave b e e n a n u m b e r o f in v e rte d c o m p a r iso n s o n r e c e n t T O E FL
tests, so y o u s h o u ld b e fa m ilia r w ith th is stru ctu re.

My sister spends more hours in the office <AanJohn.


My sister spends more hours in the office </mreJohn d o e s.
My sister spends more hours in the office than does John.

A ll th r e e o f th e se e x a m p le s c o n ta in th e c o m p a r iso n more . . . th a n , an d all th r e e are cor


r ec t in E n g lish . It is p o ssib le to h ave th e n o u n J o h n a lo n e , as in th e first e x a m p le; it is p os
sib le th a t th e c o m p a r is o n is fo llo w e d by th e su b je ct a n d verb J o h n does, as in th e se c o n d
e x a m p le ; it is a lso p o s sib le th at th e c o m p a r iso n is fo llo w e d by th e in v e rte d su b je ct an d
verb does Jo h n , as in th e th ird e x a m p le .

T h e fo llo w in g e x a m p le sh ow s h o w th is s e n te n c e p a tte rn c o u ld b e te sted in th e Struc


tu re s e c tio n o f t h e T O E F L test.

Exam ple
The results of the current experiment appear to be more consistent
than_____ the results of any previous tests.
(A) them
(B) were
(C) they were
(D) were they

In th is e x a m p le y o u sh o u ld n o tic e th e c o m p a r iso n more consistent than, a n d y o u sh o u ld


also u n d e r s ta n d th a t the results o jth e current experiment is b e in g c o m p a r e d w ith the results o f
an y p revious tests. B e ca u se the results o f any previous tests is th e su b ject, o n ly a verb is n e e d e d ;
th e b e st an sw ei to th is q u e s tio n is th e r e fo r e answ er ( B ) . W e k n ow that it is p o ssib le for a
su b je ct a n d a v e rb to b e in v e rte d after a c o m p a r iso n , a n d in this case th e su b ject the results
o f any previous tests c o m e s after th e verb were.
STRUCTURE

T h e fo llo w in g c h a r t lists th e s e n te n c e p a tte rn s u s e d w ith c o m p a r iso n s:

INVERTED SUBJECTS ANDVERBS WITH COMPARISONS

The subject and verb may Invert after a comparison.The following structures are
both possible.

S V (comparison) S V

We were more prepared than th e v th e r perform ers were.

S V (comparison)
>----- 1------ ' V S

We were m ore prepared than were the other perform ers.

NOTE: A subject-verb inversion after a comparison sounds rather formal.

EX E R C ISE 19: E ach o f th e fo llo w in g s e n te n c e s c o n ta in s a c o m p a r is o n . C ir cle th e c o m


p a riso n s. L o o k at th e c la u se s th at fo llo w a n d u n d e r lin e th e su b jects o n c e a n d th e verb s
tw ice. T h e n in d ic a te i f th e s e n te n c e s are c o r r e c t (C ) o r in c o r r e c t (I).

C 1. This candidate has received (m ore votes than) has any other candidate in previous
years.

I 2. Obviously we were much (m ore impressed with the perform ance than) did the other
m em bers o f the audience.

3. T h e film that we saw last night at the festival was far better than any o f the other
films.

4. T h e vegetables at the market this m orning were far fresher than were those at the
market yesterday.

5. I am afraid that is the condition o f these tires as bad as the condition o f the others.

6. We firmly believed that our team could achieve a m uch faster tim e than any o f the
others.

7. This apple pie is not as good as the last one that you made.

8. O n the fishing trip, Bobby caught twice as many fish as anyone else did.

9. T h e final speaker gave us m ore details than had any o f the previous speakers.

10. D o you know why does he n eed to sleep so many more hours than do the others?
STRUCTURE AND WRITTEN EXPRESSION

EXERCISE (Skills 15-19): E ach o f th e se s e n te n c e s c o n ta in s a stru ctu re th a t c o u ld req u ire


a n in v e rte d su b je c t a n d verb . C ircle th e stru ctu res th a t m ay r eq u ire in v e rte d su b jects an d
verb s. U n d e r lin e th e su b jects o n c e a n d th e verbs tw ice. T h e n in d ic a te i f th e s e n te n c e s are
c o r r e c t (C ) o r in c o r r e c t (I ).

_____ 1. The town council is n ot sure why have the land developers changed their plans.

_____ 2. Never in the world I believed that this would happen.

_____ 3. The day m ight have b een a little m ore enjoyable had the sun been out a little more.

_____ 4. Only on ce did the judge take the defense lawyers suggestion.

_____ 5. Down the hall to the left the offices are that need to be painted.

_____ 6. Did the scientist explain what h e put in the beaker?

_____ 7. Hardly ever it snows in this section o f the country.

8. Elijah scored m ore points in yesterdays basketball final than had any other player in
history.

9. In the state o f California, earthquakes occur regularly.

10. H e should ever call again, please tell him that I am not at hom e.

TOEFL EXERCISE (Skills 1 5 -1 9 ): C h o o se th e le tte r o f th e w o rd o r g r o u p o f w ord s that


b e st c o m p le te s th e s e n te n c e .

1. Rarely located near city lights or at 3. W ere_____ millions of dollars each year
ower e evations. replenishing eroding beaches, the coastline
(A) observatories are would be changing even more rapidly.
(B) are
(A) the U.S. Army Corps of Engineers not
(C) in the observatories
spending
(D) are observatories
(B) the U.S. Army Corps of Engineers not

2 . There aregeographic, economic, and (c) t h e S Army Corps of Engineers


cultural reasons w h y--------around the doe$ no( spend
WOr ' (D) not spending the U.S. Army Corps of
(A) diets differ Engineers
(B) do diets differ
(C) are diets different 4 - N ow here---------more skewed than in the
(D) to differ a diet auto industry.
(A) that retail trade figures
(B) retail trade figures
(C) are retail trade figures
(D) retail trade figures
STRUCTURE

5. New York Citys Central Park is nearly 8. _____ test positive for antibiotics w hen
twice as large_____ second smallest tanker trucks arrive a t a m ilk processing
country, Monaco. plant, according to federal law, the entire
truckload m ust be discarded.
(A) as
(B) is the (A) Should milk
(C) as is (B) If m ilk
(D) as is the (C) If m ilk is
(D) Milk should
6. Potassium has a valence of positive one
because it usually loses one electron when 9. Located b e h in d _____ the tw o lacrim al
with other elements. glands.
(A) does it combine (A) each eyelid
(B) it combines (B) is each eyelid
(C) in combining (C) each eyelid are
(D) combination (D) each eyelid w hich is

7. The econom ic background of labor 10. Only for a short period of tim e ______ru n
legislation will not be mentioned in this at top speed.
course,_____ bp treated.
(A) cheetahs
(A) trade unionism will not (B) do cheetahs
(B) nor trade unionism will (C) that a cheetah can
(C) nor will trade unionism (D) can
(D) neither trade unionism will

TOEFL REVIEW EXERCISE (Skills 119): C h o o se th e le tte r o f th e w o rd o r g r o u p o f


w ord s th a t b e st c o m p le te s th e s e n te n c e .
----- variety of flowers in the show, from . that Er lily Dickinson wrote, 24 were
simple carnations to the most exquisite given titles and 7 were published during
roses. her lifetiire.
(A) A wide (A) Of the 1,800 poems
(B) There was a wide (B) There were 1,800 poems
(C) Was there (C) Because the 1,800 poem s
(D) Many (D) The 1,800 poems

2. The w ed ges. . dartboard are worth Since an immediate change was needed on
from one to twenty points each. an emergency b asis,_____ by the governor
(A) they are on a to curtail railway expenditure.
(B) are on a (A) so it was proposed
(C) are they on a (B) was proposed
(D) on a (C) because of the proposal
(D) it was proposed
3. producing many new movies for
release after the new season begins. In the Morgan Library in New York City
(A) His company is --------o f medieval and renaissance
manuscripts.
(B) His companies
(C) The company (A) a collection is
(D) Why the company is (B) in a collection
(C) is a collection
(D) which is a collection
174 STRUCTURE AND WRITTEN EXPRESSION

7. Some fishing fleets might not have been so 9. Individual differences in brain-wave
inefficient in limiting their catch to target activity may shed light on why some
sp ecies_____ more strict in enforcing people are more prone to emotional
penalties. stress disorders--------

(A) the government had been (A) that others are


(B) if the government had (B) and others are
(C) had the government been (C) others are
(D) if the government (D) than are others

8. The Dewey decimal system, currently used 10 . _____ squeezed, the orange juice in a one-
in libraries throughout the w orld,--------all cup serving provides twice the minimum
written works into ten classes according to daily requirement for vitamin C.
subject. (A) It is freshly
(A) dividing (B) If freshly
(B) divides (C) You freshly
(C) it would divide (D) If it freshly
(D) was divided
WRITTEN EXPRESSION

T H E W R IT T E N EXPRESSION Q U E S T IO N S

Q u e stio n s 16 th r o u g h 4 0 in th e S tr u c tu r e a n d W ritten E x p r e ssio n s e c tio n o f th e T O E F L


test e x a m in e you r k n o w le d g e o f th e c o r r e c t way to e x p r ess y o u r se lf in E n g lish w ritin g.
E ach q u e s tio n in th is s e c tio n c o n sists o f o n e s e n te n c e in w h ic h fou r w ord s o r g r o u p s o f
w o rd s have b e e n u n d e r lin e d . You m u st c h o o s e th e le tte r o f th e w o rd o r g r o u p o f w o r d s
th at is not c o rr ec t.

Exam ple I

The final delivery of the day is the im portantest.


~A~ B C ' D

If y o u lo o k at th e u n d e r lin e d w ord s in th is e x a m p le , y o u sh o u ld n o tic e im m e d ia te ly th a t


im portantest is n o t c o r r e c t. T h e c o r r e c t su p erlative fo r m o f im portant is the m ost im p o rta n t.
T h e r e fo r e , y o u s h o u ld c h o o s e an sw er (D ) b e c a u se (D ) is n o t c o rr ec t.

Exam ple II

The books th at I read w as interesting.


~ ~B C D

I f y o u lo o k at th e u n d e r lin e d w ord s in th is e x a m p le , e a c h w o rd by it s e lf a p p e a r s to b e c o r
rect. H ow ever, th e sin g u la r v e rb w as is in c o r r e c t b e c a u se it d o e s n o t a g r e e w ith th e p lu ral
su b ject books; th e verb s h o u ld b e were in ste a d . T h e r e fo r e , y o u sh o u ld c h o o s e an sw er (C )
b e c a u se (C ) is n o t c o r r e c t.

STRATEGIES FOR THE WRITTEN EXPRESSION QUESTIONS

1. First look at th e underlined word or groups o f words. You want to see if you can spot
which of the four answer choices is not correct.
2. If you have been unable to find the error by looking only at the four underlined
expressions, then read th e co m p lete sen ten ce. Often an underlined expression is
incorrect because of something in another part of the sentence.
3. N ever leave any answers blank. Be sure to answer each question even if you are unsure
of the correct response.

T h e fo llo w in g sk ills w ill h e lp y o u to im p le m e n t th e se str a teg ie s in th e W ritten E x p r e ssio n


q u estio n s.
STRUCTURE AND WRITTEN EXPRESSION

PROBLEMS W IT H SUBJECT/VERB A GREEM ENT----------------------

S u b je c t/v e r b a g r e e m e n t is sim p le: i f th e su b je ct o f a s e n te n c e is singu lar, th e n th e verb


m u st b e sin g u la r, i f th e su b ject o f th e s e n te n c e is p lu ral, th e n th e verb m u st b e plu ral. An
s o n a verb u su a lly in d ic a te s th a t a verb is singu lar, w h ile an s o n a n o u n u su ally in d ic a tes
that th e n o u n is p lu ral. (D o n o t fo r g e t irreg u la r p lu rals o f n o u n s , su ch as women, children,
an d people.)

The boy walks to school.


The boys walk to school.

In th e first e x a m p le th e sin g u la r su b ject boy r eq u ire s a sin g u la r verb, walks. In th e se c o n d


e x a m p le th e p lu ra l su b je ct boys r eq u ire s a p lu ral verb , walk.
A lth o u g h th is m ig h t se e m q u ite sim p le , th e r e are a few situ a tio n s o n th e T O E F L test
w h e n s u b je c t/v e r b a g r e e m e n t can b e a little tricky. You sh o u ld b e c a refu l o f su b je c t/v e r b
a g r e e m e n t in th e fo llo w in g situ a tio n s: ( 1 ) a fter p r e p o sitio n a l p h rases, (2 ) a fter e x p r e s
sio n s o f q u antity, (3) a fter in v e r te d verb s, an d (4 ) a fter certa in w ords, su ch as anybody,
everything, no one, som ething, each, a n d every.

S k i l l 20: MAKE VERBS AGREE AFTER P R E P O S ITIO N A L PHRASES

S o m e tim e s p r e p o sitio n a l p h ra ses can c o m e b e tw e e n th e su b ject a n d th e verb. If th e o b


j e c t o f th e p r e p o sitio n is sin g u la r a n d th e su b je ct is p lu ral, o r i f th e o b je c t o f th e p r e p o si
tio n is plu ral a n d th e su b ject is sin gu lar, th e r e can b e c o n fu s io n in m a k in g th e su b ject an d
verb a g r e e .

T he key (to the doors) are* in the drawer.


SINGULAR PLURAL

The keys (to the door) is* in the drawer.


PLURAL SINGULAR

(* indicates an error)

In th e first e x a m p le y o u m ig h t th in k th a t doors is th e su b je ct b e c a u se it c o m e s d irectly in


fr o n t o f th e v e rb are. H ow ever, doors is n o t th e su b ject b e c a u se it is th e o b je c t o f th e p r e p o
sition to. T h e su b je ct o f th e s e n t e n c e is key, so th e verb sh o u ld b e is. In th e se c o n d e x a m
p le y o u m ig h t th in k th a t door is th e su b je ct b e c a u se it c o m e s d ir ec d y in fr o n t o f th e verb
is. You sh o u ld r e c o g n iz e in th is e x a m p le that door is n o t th e su b je ct b e c a u se it is th e o b je c t
o f th e p r e p o sitio n to. B e ca u se th e su b je ct o f th e s e n t e n c e is keys, th e verb sh o u ld b e are.
T h e fo llo w in g c h a r t o u tlin e s th e k ey in fo r m a tio n th a t y o u s h o u ld u n d e rsta n d a b o u t
su b je c t/v e r b a g r e e m e n t w ith p r e p o sitio n a l ph rases:

SUBJECT/VERB AGREEMENT WITH PREPOSITIONAL PHRASES

(prepositional phrase)

When a prepositional phrase comes between the subject and the verb, be sure that the verb
agrees with the subject.
WRITTEN EXPRESSION

EXERCISE 20: E ach o f th e fo llo w in g s e n te n c e s has o n e o r m o r e p r e p o sitio n a l p h ra ses b e


tw een th e su b je ct a n d verb. C ircle th e p r e p o sitio n a l p h rases. U n d e r lin e th e su b je cts o n c e
a n d th e verbs tw ice. T h e n in d ic a te i f th e s e n te n c e s are c o r r e c t (C ) o r in c o r r e c t (I).

C
1. T he clim bers (on the sheer face)(of the m ountain) need to be rescued.

2- T he interrogation, conducted (by three police officer^) have lasted for severalhours.

3. T he tenants in the apartm ent next to m ine is giving a party this evening.

4. T he president, surrounded by Secret Service agents, is trying to m ake his way to the
podium .

5. T he buildings destroyed during the fire are being rebuilt at the taxpayers expense.

6. Because o f the seriousness o f the com panys financial problem s, the board o f
directors have called an em ergency m eeting.

7. M anufacture o f the item s that you requested have been discontinued because o f lack
o f profit o n those items.

8. Further develop m en t o f any new ideas for future products has to be approved in
advance.

9. T h e scheduled departure time o f the trams, posted on panels throughout the


term inal buildings, are goin g to be updated.

10. Any houses built in that developm ent before 1970 have to be upgraded to m eet
current standards.

S k i l l 2 1: MAKE VERBS AGREE AFTER EXPRESSIONS OF Q U A N T IT Y

A p a rticu la r a g r e e m e n t p r o b le m o c c u r s w h en th e su b ject is an e x p r e s sio n o f qu antity,


su ch as all, most, o r some, fo llo w e d by th e p r e p o sitio n of. In th is situ a tio n , th e su b je c t [all,
most, o r some) c a n b e sin g u la r o r p lu ra l, d e p e n d in g o n w h at fo llo w s th e p r e p o s itio n of.

All (o f the book) was interesting.


SINCULAR

All (o f the books) were interesting.


PLURAL

All (o f the information) was interesting.


UNCOUNTABLE

In th e first e x a m p le th e su b je ct a ll r efe rs to th e sin g u la r n o u n book, so th e c o r r e c t verb is


th e r e fo r e th e sin g u la r verb was. In th e s e c o n d e x a m p le th e su b je c t a ll r efe rs to th e p lu ral
n o u n books, so th e c o r r e c t v erb is th e p lu ral verb were. In th e th ird e x a m p le th e su b je c t all
r efe rs to th e u n c o u n ta b le n o u n inform ation, so th e c o r r e c t verb is th e r e fo r e th e sin g id a r
verb was.
STRUCTURE AND WRITTEN EXPRESSION

T h e fa llo w in g c h a r t o u tlin e s th e key in fo r m a tio n th a t you sh o u ld u n d e r sta n d a b o u t


su b je c t/v e r b a g r e e m e n t after e x p r e s sio n s o f quantity:

SU BJEC T /V ER B A G R EEM EN T A FT ER E X P R E S S IO N S O F Q U A N T IT Y

oil
most OF THE (OBJECT) V
some
half
------------------------------------------------------------ -

W h e n an expression c f quantity is the subject, the ve rb agrees w ith th e object.

EXERCISE 21: E ach o f th e fo llo w in g s e n te n c e s h as a q u an tity e x p r e s sio n as th e subject.


U n d e r lin e th e su b jects o n c e a n d th e verb s tw ice. C ircle th e o b je c ts th a t th e verb s agree
w ith. T h e n in d ic a te i f th e s e n te n c e s are c o r r e c t (C ) o r in c o r r e c t (I).

C ] T h e witnesses saw that m ost o f the (fire) in the hills was e xlni^uis) ud .

I 2. Som e o f the (animals) from the zoo was released into the animal preserve.

_____ 3 . All o f the students in the class taught by Professor Roberts is required to turn in
their term papers next Monday.

_____ 4. H alf o f the food that we are serving to the guests are still in the refrigerator.

_____ 5. We believe that som e o f the time o f the em ployees is going to be devoted to quality
control.

_____ 6. All o f the witnesses in the jury trial, which lasted m ore than two weeks, have
indicated that they believed that the defendant was guilty.

_____ 7. She did not know where m ost o f the p eople in the room was from.

_____ 8. In spite o f what was decid ed at the m eeting, h alf o f die procedures was n ot changed.

_____ 9. I was sure that all o f the questions on the test were correct.

_____ 10. M ost o f the trouble that the em ployees discussed at the series o f m eetings was
resolved within a few weeks.

S k i l l 22: MAKE IN VER TED VERBS AGREE

W e h a v e s e e n th a t so m e tim e s in E n g lish th e su b je c t c o m e s a fter th e verb . T h is ca n o c cu r


after q u e s tio n w ord s (Skill 1 5 ), after p la c e e x p r e s sio n s (Skill 1 6 ), after n e g a tiv e ex p r es
sio n s (Sk ill 1 7 ), after o m itte d c o n d itio n a ls (Sk ill 1 8 ), a n d after so m e c o m p a r iso n s (Skill
1 9 ). W h e n th e su b ject an d verb are in v e rte d , it c a n b e d iffic u lt to lo c a te th em , a n d it can
th e r e fo r e b e a p r o b le m to m ak e th e m a gree.

(Behind the house) was* the bicycles I wanted.


(Behind the houses) were* the bicycle I wanted.
WRITTEN EXPRESSION

In th e first e x a m p le it is easy to th in k th a t house is th e su b ject, b e c a u se it c o m e s d ir ec tly in


fro n t o f th e verb was. H ouse is n o t th e su b ject, h ow ever, b e c a u se it is th e o b je c t o f th e
p r e p o sitio n behind. T h e su b ject o f th e s e n te n c e is bicycles, an d th e su b je ct bicycles c o m e s af
ter th e verb b e c a u se o f th e p la c e e x p r e s sio n behind the house. B e c a u se th e su b je c t bicycles is
plu ral, th e verb sh o u ld b e c h a n g e d to th e p lu ral were. In th e s e c o n d e x a m p le th e su b ject
bicycle c o m e s after th e v e rb were b e c a u se o f th e p la c e e x p r e s sio n behind the houses. B e ca u se
th e su b je ct bicycle is sin gu lar, th e verb s h o u ld b e c h a n g e d to th e sin g u la r was.

T h e fo llo w in g c h a r t o u tlin e s th e key in fo r m a tio n th a t y o u sh o u ld u n d e r s ta n d a b o u t


s u b je c t/v e r b a g r e e m e n t a fter in v e r te d verbs:

SUBJECT/VERB AGREEMENT AFTER INVERTEDVERBS

question
negative I
place s
condition (no if)
comparison

After question words, negative expressions, place expressions, conditions without if, and
comparisons, the verb agrees with the subject, which may be after the verb.

EXERCISE 22: E ach o f th e fo llo w in g s e n t e n c e s c o n ta in s an in v e rte d su b je c t a n d verb .


C ircle th e w ord o r g r o u p o f w o rd s th at c a u se s th e su b je ct a n d verb to in v ert. F in d th e su b
j e c t a n d verb th a t fo llo w th e se w ords. U n d e r lin e th e su b ject o n c e a n d th e v e rb tw ice.
T h e n in d ic a te i f th e s e n t e n c e s are c o r r e c t (C) o r in c o r r e c t (I).

C 1. (O nly once) this m orning were the letters delivered by the cam pus mail service.

2. (A round the corn er and to the right) is the room s that have been assigned to that
program.

3. What in the world is the children trying to do?

4. John would be studying the chapters were he able to g et h old o f the book.

5. This chapter has many more exercises than do the next one.

6. T he com puter program m er was unaware that there was so many m istakes in the
program he had written.

7. Seldom in the history o f television has two new com ed ies been so successful in one
season.

8. How many huge mistakes have the teacher actually found in the research paper?

9. T h e new p h one system is able to hold far m ore messages than was the p h on e system
that had previously b een used.

10. In the parking lot south o f the stadium was the cars that were about to be towed.
STRUCTURE AND WRITTEN EXPRESSION

S k i l l 23: MAKE VERBS AGREE AFTER CERTAIN W O R D S

C ertain w ord s in E n g lish are always gra m m a tica lly singu lar, e v e n th o u g h th ey m ig h t have
p lu ral m e a n in g s.

Everybody are go in g * to the theater.

E ven th o u g h w e u n d e r s ta n d fr o m this e x a m p le th at a lo t o f p e o p le are g o in g to th e th e


ater, everybody is sin g u la r an d r eq u ire s a sin g u la r verb. T h e p lu ral verb are g oing sh o u ld be
c h a n g e d to th e sin g u la r verb is going.

T h e fo llo w in g c h a r t lists th e g ram m atically sin g u la r w ord s th at have p lu ral m ea n in g s:

SUBJECT/VERB AGREEMENT AFTER CERTAIN WORDS

These words or expressions are grammatically singular, so they take singular verbs:

anybody everybody nobody somebody each (+ noun)


anyone everyone no one someone every (+ noun)
anything everything nothing something

EXERCISE 23: E ach o f th e fo llo w in g s e n te n c e s c o n ta in s o n e o f th e w ord s that are gram


m a tically sin g u la r b u t h ave p lu ral m e a n in g s. U n d e r lin e th e se w ord s o n c e a n d u n d e r lin e
th e v e rb s tw ice. T h e n in d ic a te i f th e s e n te n c e s are c o r r e c t (C) o r in c o r r e c t (I).

__ !__ 1. It is im possible to believe that som ebody actually admire that man.

-_ 2. Each o f the doctors in the building needs to have a separate reception area.

-------- 3. T h e president felt that no o n e were better suited for the position o f ch ief staff
advisor.

_____ 4. Everybody participating in the fund-raiser are to turn in the tickets by 8:00.

-------- 5. Because o f the low num ber o f orders, nothing has to be don e now.

-------- 6. Every time som eone take unnecessary breaks, precious m om ents o f production time
are lost.

-------- 7. Anybody who goes to the top o f the Empire State Building is im pressed with the
view.

-------- 8. Every man, woman, and child in this line are required to sign the forms in order to
com plete the registration process.

-------- 9. It is nice to believe that anything is possible if a person tries hard enough.

. 10. T h e com pany reiterated to reporters that nobody have been dismissed because o f
the incident.
WRITTEN EXPRESSION

EXERCISE (Skills 2 0 -2 3 ): U n d e r lin e th e su b jects o n c e a n d th e verb s tw ice in e a c h o f th e


fo llo w in g se n te n c e s . T h e n in d ic a te i f th e s e n te n c e s are c o r r e c t (C ) o r in c o r r e c t (I).

-------- 1. The contracts signed by the com pany has been voided because som e stipulations
were not met.

-------- 2. Ten m iles beyond the river was the farmlands that they had purchased with their life
savings.

-------- 3. Each package that is n o t properly wrapped have to be returned to the sender.

-------- 4. She would not have to enter the house through the bedroom w indow were the keys
w here they were supposed to be.

-------- 5. T he proposal brought so m uch new work to the partnership that there was not
en ou gh hours to com plete all o f it.

-------- 6. The box o f cartridges for the printer have been misplaced.

-------- 7. It is disconcerting to believe that every possible candidate has been rejected for on e
reason or another.

-------- 8. Only on ce have there b een m ore excitem ent in this city about a sporting event.

-------- 9. Bobby has a bigger bicycle than does the other children in the neighborhood.

-------- 10. If nobody have bought that car from the dealer, then you should return and make
another offer.

TOEFL EXERCISE (Skills 2023): C h o o se th e le tte r o f th e w ord o r g r o u p o f w o r d s th a t


b e st c o m p le te s th e s e n te n c e .

1. Among b e e s--------------------------------------------------------------------------------- a highly elaborate form 2. _________ h


of communication. collectors on the roofs to trap sunlight.
(A) occur (A) A home is
(B) occurs (B) Homes are
(C) it occurs (C) A home
(D) they occur (D) Homes

C h o o se th e le tte r o f th e u n d e r lin e d w o rd o r g r o u p o f w o rd s th a t is n o t c o r r e c t.

-------- 3. Each number in a binary system are formed from only two symbols.
A B C D

--------4. Scientists at the medical center is trying to determine if there is a relationship


A B ~C
between saccharine and cancer.
D

-------- 5. On the rim of the Kilauea volcano in the Hawaiian Islands are a hotel called the
A B C~ D
Volcano Hotel.
182 STRUCTURE AND WRITTEN EXPRESSION

6. The great digital advances of the electronic age, such as integrated circuitry and a
------- A B
microcomputer, has been planted in tiny chips.

7. There are many frequently mentioned reasons why one out of four arrests involve a
A B C D
juvenile.

8. Keplers Laws, principles outlining planetary movement, was formulated based on


A B C
observations made without a telescope.

9 Only with atwo-thirds vote by both houses arethe U.S. Congress able to override a
A B C
presidential veto.
D

_10. Of all the evidence that has piled up since Webster's paper was published,there isno
----- A----- B
new ideas to contradict his original theory.
D

TO EFL REVIEW EXERCISE (Skills 1-23): C h o o se th e le tte r o f th e w ord o r g r o u p o f


w o rd s th at b e st c o m p le te s th e s e n te n c e .

. several unsuccessful attempts, 4. _____ of economic cycles been helpful in


Robert Peary reached the North Pole on predicting turning points in cycles, they
April 6, 1909. would have been used more consistently.

(A) After (A) Psychological theories


(B) He made (B) Psychological theories have
(C) When (C) Had psychological theories
(D) His (D) Psychologists have theories

2. The musical instrum ent_____ is six feet 5. Hospital com m ittees--------spent weeks
long. agonizing over which artificial kidney
candidate would receive the treatments
(A) is called the bass now find that the decision is out of their
(B) it is called the bass hands.
(C) called the bass
(D) calls the bass (A) once
(B) that once
3. One problem with all languages--------they (C) have
are full of irregularities. (D) once had

(A) when
(B) so
(C) is that
(D) in case
WRITTEN EXPRESSION

C h o o se th e le tte r o f th e u n d e r lin e d w o r d o r g r o u p o f w o rd s th a t is n o t c o r r e c t.

6. More than half of the children in the 1,356-member district qualifies for
A B C
reduced-price or free lunches.
D

7. Five miles beyond the hills were a fire with its flames reaching up to the sky.
A B~ C D

8. Kettledrums, what were first played on horseback, were incorporated into the
~ A ~ B C D
orchestra in the eighteenth century.

. 9. When is a flag hung upside down, it is an internationally recognized symbol of distress.


B~ ~C~ D

.10. The Museum of the Confederation in Richmond hosts an exhibition which


A
documenting the origins and history of the banner that most Americans think of as
B C D
the Confederate flag.

PROBLEMS W IT H PARALLEL S TR U C TU R E___________________

In g o o d E n g lish an a tte m p t sh o u ld b e m a d e to m a k e th e la n g u a g e as e v e n a n d b a la n c e d
as p o s sib le . T h is b a la n c e is c a lle d p a ra llel str u c tu r e . You c a n a c h ie v e p a r a lle l str u c tu r e
by m a k in g th e fo r m s o f w ord s as sim ilar as p o ssib le . T h e fo llo w in g is a n e x a m p le o f a s e n
te n c e th a t is n o t parallel:

I like to sing and dancing.*

T h e p r o b le m in th is s e n t e n c e is n o t th e e x p r e s sio n to sing, a n d th e p r o b le m is n o t th e
w ord dancing. T h e e x p r e s sio n to sin g is c o r r e c t by itse lf, a n d th e w o rd d a n c in g is c o r r e c t by
itself. B o th o f th e fo llo w in g s e n te n c e s are correct:

I like to sing.
I like dancing.

T h e p r o b le m in th e in c o r r e c t e x a m p le is th a t to s in g a n d d a n c in g are j o i n e d to g e th e r in
o n e s e n te n c e w ith a n d . T h e y a re d iffe r e n t fo r m s w h er e it is p o s sib le to h ave sim ila r form s;
th e r e fo r e th e e x a m p le is n o t p a ra llel. It can b e c o r r e c te d in tw o d iff e r e n t ways: w e c a n
m ak e th e first e x p r e s sio n lik e th e s e c o n d , o r w e can m a k e th e s e c o n d e x p r e s s io n lik e th e
first.

I like to sing and to d an ce.


I like singing and dancing.
STRUCTURE AND WRITTEN EXPRESSION

T h e r e are several situ a tio n s in w h ic h y o u s h o u ld b e p articu larly c a re fu l o f p a r a lle l stru c


tu re. P a ra llel str u c tu r es are r eq u ire d in th e fo llo w in g situ a tio n s: (1) w ith c o o r d in a te c o n
ju n c tio n s , su c h as a n d , but, or; ( 2 ) w ith p aired c o n ju n c tio n s , su ch as both . . . a n d , e ith e r. . .
or, n e ith e r. . . nor, not only . . . but also; a n d (3) w ith c o m p a r iso n s.

S kill 24: USE PARALLEL STRUCTURE W IT H C O O R D IN A TE


C O N J U N C T IO N S

T h e j o b o f th e c o o r d in a te c o n ju n c tio n s (a n d , but, or) is to j o in to g e th e r e q u a l ex p r es


sio n s. In o t h e r w ord s, w h at is o n o n e sid e o f th e se w o rd s m u st b e p a ra llel to w h a t is o n th e
o th e r sid e . T h e s e c o n ju n c tio n s can j o i n n o u n s , o r verb s, o r a d jectives, o r p h rases, or su b
o r d in a te c la u se s, o r m a in clau ses; th e y ju s t m u st j o in to g e th e r tw o o f th e sa m e th in g . H er e
are e x a m p le s o f tw o n o u n s j o in e d by a c o o r d in a te c o n ju n c tio n :

I need to talk to the manager or the assistant manager.


She is not a teacher but a lawyer.
You can choose from activities such as hiking and kayaking.

H e r e are e x a m p le s o f two verb s j o in e d by a c o o r d in a te c o n ju n c tio n :

H e eats and sleeps only when he takes a vacation.


She invites us to her h om e but never talks with us.
You can stay hom e or go to the movies with us.

H e r e are e x a m p le s o f two ad jectives j o in e d by a c o o r d in a te c o n ju n c tio n :

My boss is sincere and nice.


The exam that he gave was short but difficult.
Class can be interesting or boring.

H e r e are e x a m p le s o f two p h ra ses j o in e d by a c o o r d in a te c o n ju n c tio n :

There are students in the classroom and in front o f the building.


The papers are on my desk or in the drawer.
The checks will be ready not at n oon but at 1:00.

H e r e are e x a m p le s o f two c la u se s j o in e d by a c o o r d in a te c o n ju n c tio n :

They are not interested in what you say or what you do.
I am here because I have to be and because I want to be.
Mr. Brown likes to go hom e early, but his wife prefers to stay late.

T h e fo llo w in g c h a r t o u tlin e s th e u se o f p arallel str u c tu r es w ith c o o r d in a te c o n ju n ctio n s:

3-------------------------------------------------------------------------------------------------------- -----
y PARALLEL STRUCTURE WITH COORDINATE CONJUNCTIONS

and
(same structure) but (same structure)
or
.

and
(same structure), (same structure), but (same structure)
or
WRITTEN EXPRESSION

EXERCISE 24: E ach o f th e fo llo w in g s e n t e n c e s c o n ta in s w ords o r g r o u p s o f w o r d s th a t


sh o u ld b e p arallel. C ircle th e w o rd th a t in d ic a te s that th e s e n te n c e s h o u ld h ave p a r a lle l
parts. U n d e r lin e th e p a rts that sh o u ld b e p arallel. T h e n in d ic a te i f th e s e n te n c e s are c o r
r ec t (C ) o r in c o r r e c t (1).

1 1. She h eld jo b s as a typist, a housekeeper, (and) in a restaurant.

C_ i> T he report you are looking for could be in the file (or) on the desk.
- 3. She works very hard but usually gets below-average grades.
4. T he speaker introduced him self, told several interesting anecdotes, and finishing
with an em otional plea.
5. You should know w hen the program starts and how many units you must com plete.
. 6. T he term paper he wrote was rather short but very impressive.
7. She suggested taking the plane this evening or that we go by train tomorrow.
8 . T he dean or the assistant dean will inform you o f when and where you should apply
for your diplom a.
9. T here are papers to file, reports to type, and those letters should be answered.
10. T he m anager n eed ed a quick but thorough response.

S kill 2 5 : USE PARALLEL STR U C TU R E W IT H PAIRED C O N J U N C T IO N S

T h e p a ir e d c o n ju n c tio n s both . . . a nd, e ith e r. . . or, n e ith e r . . . nor, a n d n ot only . . . but also
req u ire p a ra llel stru ctu res.

I know both where you went and what you did.


Either Mark orSue has the book.
T h e tickets are neither ir^my pocket nor in my purse.
H e is not only an excellen t student but also an outstanding athlete.
T h e fo llo w in g is n o t p a r a lle l a n d m u st b e c o rr ec ted :

H e wants either to go by train or by plane*.

It is n o t c o r r e c t b e c a u se to go by train is n o t p a ra llel to by plane. It can b e c o r r e c te d in sev


eral ways.

H e wants either to g o by train or to g o by plane.


H e wants to go eitherby train or by plane.
H e wants to go by either train or plane.

W h en y o u are u s in g th e s e p a ir e d c o n ju n c tio n s , b e su re that th e c o r r e c t parts are u se d to


g eth er. T h e fo llo w in g a re in c o rr ec t:

I want both this book or* that one.


F.itherSam nor* Sue is taking the course.

T h e se s e n te n c e s are in c o r r e c t b e c a u se th e w r o n g parts o f th e p a ir ed c o n ju n c tio n s are


u sed to g e th er . In th e first e x a m p le , a n d sh o u ld b e u se d w ith both. In th e s e c o n d e x a m p le ,
or sh o u ld b e u s e d w ith either.
STRUCTURE AND WRITTEN EXPRESSION

T h e fo llo w in g c h a r t o u tlin e s th e u se o f p a r a lle l stru ctu re w ith p a ir e d c o n ju n ctio n s:

PARALLEL STRUCTURE WITH PAIRED CONJUNCTIONS

. :' and
both
either or (same structure)
(same structure)
neither nor
- . .-. .; . .....
.: " : not only " r but also
v.. . . . .

EX E R C ISE 25: E ach o f th e fo llo w in g s e n te n c e s c o n ta in s w ord s o r g r o u p s o f w ords that


sh o u ld b e p arallel. C ircle t h e w ord o r w ord s th a t in d ic a te th a t th e s e n te n c e sh o u ld have
p arallel parts. U n d e r lin e th e parts th a t sh o u ld b e p a ra llel. T h e n in d ic a te if th e s e n te n c e s
are c o r r e c t (C ) o r in c o r r e c t (I).

__ l _ 1. According to the syllabus, you can (either) write a paper (or)you can take an exam .

c 2. It would be (jaoth) noticed (and) appreciated if you could finish the work before you
leave.

3. She would like neither to see a movie or to go bowling.

4. Either the manager or her assistant can help you with your refund.

5. She wants not only to take a trip to Europe but she also would like to travel to Asia.

6. H e could correct neither what you said nor you wrote.

7. Both the tailor or the laundress could fix the dam age to the dress.

8. H e not only called the police departm ent but also called the fire departm ent.

_____ 9. You can graduate either at the end o f the fall sem ester or you can graduate at the
end o f the spring semester.

10. T he m ovie was neither am using nor was it interesting.

S k il l 2 6 : USE PARALLEL S TR U C TU R E W IT H COM PARISONS

W h e n y o u m a k e a c o m p a r is o n , y o u p o in t o u t th e sim ila r itie s o r d iffe r e n c e s b e tw e e n two


th in g s , a n d th o s e sim ila r itie s o r d iffe r e n c e s m u st b e in p a ra llel fo r m . You c a n r e c o g n iz e a
c o m p a r is o n sh o w in g h o w tw o th in g s are d iffe r e n t fro m th e - e r . . . th a n or th e more . . .
than.

My school is farther than your sch ool.


To be rich is better than to be poor.
W hat is written is more easily understood than what is spoken.
WRITTEN EXPRESSION

A c o m p a r iso n sh o w in g h o w tw o th in g s are th e sam e m ig h t c o n ta in as . . . as o r e x p r e s


sio n s su c h as the sam e as o r sim ila r to.

T heir car is as big as a small house.


Renting those apartments costs about the same as leasing them.
T he work that I did is similar to the work that you did.

T h e fo llo w in g c h a r t o u tlin e s th e u se o f p a ra llel str u c tu r es w ith c o m p a r iso n s:

PARALLEL STRUCTURE WITH COMPARISONS

v : - V- - V: more ... than


. . -: ' .' ' ' . ' -er... than : - . - -- - ; :
(same structure) less ... than (same structure)
; ' - -V- 'v. ! -, b as ...as
the same... as
similar ...to d --

EXERCISE 26: E a ch o f th e fo llo w in g s e n t e n c e s c o n ta in s w ord s o r g r o u p s o f w o r d s th a t


sh o u ld b e p a r a lle l. C ircle th e w o rd o r w ord s th a t in d ic a te th a t th e s e n t e n c e s h o u ld h ave
p a r a lle l parts. U n d e r lin e th e parts th a t s h o u ld b e p arallel. T h sn in d ic a te i f e a c h s e n t e n c e
is c o r r e c t (C ) o r in c o r r e c t (I).

__ 0 1. His research for the thesis was (more useful than) hers.

__ !__ 2. D inin g in a restaurant is (m ore fun than) to eat at hom e.

_____ 3. I want a new secretary who is as efficient as the previous one.

_____ 4. What you do today sh ould be the same as did yesterday.

_____ 5. This lesson is m ore difficult than we had before.

_____ 6. You have less hom ework than they do.

_____ 7. What you do has more effect than what you say.

_____ 8. Music in your country is quite similar to my country.

_____ 9. T h e collection o f foreign journals in the university library is m ore extensive than the
high school library.

_____ 10. How to buy a used car can be as difficult as buying a new car.
188 STRUCTURE AND WRITTEN EXPRESSION

EXERCISE (Skills 2 4 -2 6 ): Circle the word or words that indicate that the sentence should
have parallel parts. Underline the parts that should be parallel. Then indicate if the sen
tences are correct (C) or incorrect (I).

_____ 1. After retirem ent he plans on traveling to exotic locations, dine in the finest
restaurants, and playing a lo t o f golf.

_____ 2. She was both surprised by and pleased with the seminar.

_____ 3. What cam e after the break was even m ore boring than had com e before.

_____ 4. H e would find the missing keys neither under the bed or b ehind the sofa.

_____ 5. D epend in g on the perspective o f the viewer, the film was considered laudable,
mediocrity, or horrendous.

_____ 6. H e exercised not only in the m orning, but he also exercised every afternoon.

_____ 7. Working four days per w eek is much m ore relaxing than working five days per week.

_____ 8. Sam is always good-natured, generous, and helps you.

_____ 9. Either you have to finish the project, or the contract will be canceled.

_____ 10. T he courses that you are required to take are m ore im portant than the courses that
you choose.

T OEFL EXERCISE (Skills 24 -2 6 ): Choose the letter of the word or group of words that
best completes the sentence.
1. Truman Capotes In Cold Blood is neither 3. A babys development is influenced by both
journalistically accurate heredity and
(A) a piece of fiction (A) by environmental factors
(B) nor a fictitious work (B) environmentally
(C) or written in a fictitious way (C) the influence of the environment
(D) nor completely fictitious (D) environment

2. Vitamin C is necessary for the prevention 4. Because bone loss occurs earlier in women
a n d _____ of scurvy. th a n _____ , the effects of osteoporosis are
more apparent in women.
(A) it cures
(B) cures (A) men do
(C) cure (B) in men
(D) for curing (C) as men
(D) similar to men
WRITTEN EXPRESSION

C h o o se th e le tte r o f th e u n d e r lin e d w o rd o r g r o u p o f w ord s th a t is n o t c o r r e c t.

5. Fire extinguishers can contain liquefied gas, dry chemicals, or watery.


A B C- - D

6. The U.S. Congress consists of both the Senate as well as the House o f Representatives
A B C D

7. The prison population in this state, now at an all time high, is higher than anv state.
A B C D

8. A well-com posed baroque opera achieves a delicate balance by focusing alternately


A B C
on the aural, visual, emotional, and philosophy elements.
D

9. Manufacturers may use food additives for preserving, to color, to flavor, or to fortifv
A B C ~
foods.

-10. A bankruptcy may be either voluntary nor involuntary.


A B C W

TOEFL REVIEW EXERCISE (Skills 126): C h o o se th e lette r o f th e w ord o r g r o u p o f


w ord s th at b e st c o m p le te s th e s e n te n c e .

1. The growth of hair_____ cyclical process, 4. The legal systems of m ost countries can Ipe
with phases of activity and inactivity. classified--------com m on law or civil law.
(A) it is (A) as either
(B) is a (B) either as
(C) which is (C) either to
(D) a regular (D) to either

2. The fire. . to have started in the 5. One difference between mathematics and
furnace under the house. language is that mathematics is precise
(A) is believed
(B) that is believed (A) language is not
(C) they believe (B) while language is not
(D) that they believe (C) but language not
(D) while is language
3. In Roman numerals, . symbols for
numeric values. 6. Your criticism of the three short stories
(A) are letters of the alphabet should not be less than 2,000*words, nor
(B) letters of the alphabet are more than 3,000.
(C) which uses letters of the alphabet (A) should it be
(D) in which letters of the alphabet are (B) it should be
(C) it is
(D) should be it
STRUCTURE AND WRITTEN EXPRESSION

C h o o se th e le tte r o f th e u n d e r lin e d w o rd o r g r o u p o f w o rd s th a t is n o t co rr ec t.

7. In 1870, the attorney general was made head of the Department of Justice, given an
A
enlarged staff, and endow with clear-cut law-enforcement functions.
B C D

8. The General Sherman Tree, the largest of all the giant sequoias, are reputed to be the
------ B C
worlds largest living thing.
D

_ 9. The skeleton of a shark is made of cartilage rather than having bone.


~fiT B C D

_10 At least one sample of each of the brands contains measurable amounts of aflatoxin,
A B
and there is three which exceed the maximum.

PROBLEMS W IT H COMPARATIVES A N D SUPERLATIVES.

S e n te n c e s w ith in c o r r e c t co m p a ra tiv es a n d su p erla tiv es c a n a p p ea r o n th e T O E F L test. It


is th e r e fo r e im p o r ta n t fo r y ou to k n ow h o w to d o th e fo llo w in g : ( 1 ) fo rm th e c om p arative
a n d su p erla tiv e correctly; (2) u s e th e c o m p a r a tiv e an d su p erla tiv e correctly; a n d (3 ) u se
th e irr eg u la r -er, -er stru ctu re th a t h as b e e n a p p e a r in g fr e q u e n tly o n th e T O E F L test.

S k i l l 27: FORM COM PARATIVES A N D SUPERLATIVES CORRECTLY

T h e p r o b le m w ith so m e o f th e c o m p a r a tiv e a n d su p e rla tiv e s e n te n c e s o n th e T O E F L test


is th a t th e c o m p a r a tiv e o r su p erlative is fo r m e d in co rrectly . You sh o u ld th e r e fo r e u n d e r
sta n d h o w to fo r m th e c o m p a r a tiv e a n d su p erla tiv e to an sw er su c h q u e s tio n s correctly.
T h e co m p a r a tiv e is fo r m e d w ith e it h e r -er o r more a n d than. In th e co m p a r a tiv e , -er is
u s e d w ith sh o r t a d jectiv es su ch as tall, an d more is u s e d w ith lo n g e r a d jectiv es su ch as
beautiful.

Bob is taller than Ron.


Sally is more beautiful than Sharon.

T h e su p e rla tiv e is fo r m e d w ith the, e ith e r -est or most, a n d so m e tim e s in, of, o r a that-c la u se.
In th e su p e rla tiv e, -est is u se d w ith sh o rt ad jectives su ch as tall, a n d most is u s e d w ith lo n g e r
a d jectiv es su ch as beautiful. ,

Bob is the tall erf man in the room.


Sally is the most beautiful o f all the w om en at the party.
T he spider over there is (Aelargesi on e that I have ever seen.
The fastest runner wins the race, (no in, of, or that)
WRITTEN EXPRESSION

T h e fo llo w in g c h a r t o u tlin e s th e p o s sib le fo r m s o f co m p a ra tiv es a n d su p erlatives:

THE FORM OF COMPARATIVESAND SUPERLATIVES

COMWWATIVE more (long adjective)


than
(short adjective) + er
r -,

SUPERLATIVE

' . '
, most (long adjective)
[ (short adjective) + est J .
, . , .
maybe ,n. o f thot

EXERCISE 27: E ach o f th e fo llo w in g s e n te n c e s c o n ta in s a c o m p a r a tiv e o r su p e rla tiv e.


C ircle th e co m p a r a tiv e o r su p e rla tiv e. T h e n in d ic a te i f th e s e n te n c e s are c o r r e c t (C ) or
in c o r r e c t (I ).

I 1. O xygen is (abundanter than )nitrogen.

2. T he directions to the exercise say to choose (the m ost appropriate) response.

3. The lesson you are studying now is the most im portantest lesson that you will have.

4. Fashions this year are shorter and more colorful than they were last year.

5. T he professor indicated that A nthonys research paper was m ore lon g than the
other students papers.

6. Alaska is the cold est than all the states in the U nited States.

7. T h e workers on the day shift are m ore rested than the workers on the nigh t shift.

8. She was m ore happier this m orning than she had been yesterday.

9. T he quarterback on this years football team is more versatile than the quarterback
on last years team.

10. She always tries to do the best and m ost efficien tjob that she can do.

S k i l l 28: USE CO M PARATIVES A N D SUPERLATIVES CORRECTLY

A n o th e r p r o b le m w ith th e c o m p a r a tiv e a n d su p erlative o n th e T O E F L te st is th a t th e y can


b e u s e d in co rrectly . T h e c o m p a r a tiv e a n d su p erla tiv e have d iffe r e n t u ses, a n d y o u s h o u ld
u n d e r s ta n d th e se d iffe r e n t u ses to answ er su c h q u e s tio n s correctly. T h e c o m p a r a tiv e is
u s e d to c o m p a r e tw o e q u a l th in gs.

T h e history class is larger than the math class.


Mary is more intelligent than Sue.

In th e first e x a m p le the history class is b e in g c o m p a r e d w ith the m ath class. In th e s e c o n d


e x a m p le M a ry is b e in g c o m p a r e d w ith Sue.
STRUCTURE AND WRITTEN EXPRESSION

T h e su p erla tiv e is u s e d w h e n th e r e are m o r e th an two item s to c o m p a r e an d y o u w an t


to sh ow th e o n e th a t is th e b est, th e b ig g e st, or in so m e way th e m o st o u tsta n d in g .

T he history class is the largest in the school.


Mary is the most intelligent o f all the students in the class.

In th e first e x a m p le the history class is c o m p a r e d w ith all th e o th e r cla sses in th e s c h o o l,


a n d th e h isto r y class is la rg er th a n e a c h o f th e o th e r classes. In th e s e c o n d e x a m p le , M a ry
is c o m p a r e d w ith all th e o th e r stu d e n ts in th e class, a n d M ary is m o r e in te llig e n t th an
e a c h o f th e o th e r stu d en ts.

T h e fo llo w in g ch art o u tlin e s th e u ses o f co m p a ra tiv es a n d su perlatives:

THE USES OF COMPARATIVES AND SUPERLATIVES

The c o m p a r a t iv e Is used to compare two equal things.

The s u p e r l a t iv e is used to show which one of many is in some way the most outstanding.

EXERCISE 28: E ach o f th e fo llo w in g s e n te n c e s c o n ta in s a co m p a ra tiv e o r su p erla tiv e.


C ircle th e com p arative o r su p erla tiv e. T h e n in d ic a te i f th e s e n te n c e s are c o r r e c t (C ) or
in c o r r e c t (I).

__ ~
r_ 1. Harvard is probably (the m ost prestigious) university in the U nited States.

__ !__ 2. Rhonda is (more hard working) o f the class.


_____ 3. The engineers hired this year have more experience than those hired last year.
-------- 4. The graduate assistant inform ed us that the first exam is the most difficult o f the
two.
_____ 5. He bought the more powerful stereo speakers that he could find.
_____ 6. T he afternoon seminar was m uch more interesting than the m orning lecture.
-------- 7. The food in this restaurant is the best o f the restaurant we visited last week.
-------- 8. The plants that have been sitting in the sunny window are far healthier than the
other plants.
_____ 9. The photocopies are the darkest that they have ever been.

-------- 10. The first journal article is the longest o f the second article.

S k il l 29: USE T H E IRREGULAR -ER, -ER STRUCTURE CORRECTLY

A n irr eg u la r c o m p a r a tiv e stru ctu re th a t has b e e n a p p ea r in g fr e q u e n tly o n th e T O E FL


te st co n sists o f tw o p a ra llel c o m p a r a tiv e s in tr o d u c e d by the.

The harder he tried, the further he fell behind.


The older the children are, the more their parents expect from them.
WRITTEN EXPRESSION

T h e firs t e x a m p le c o n t a in s th e tw o p a r a lle l c o m p a r a tiv e s the harder a n d the further. T h e sec


o n d e x a m p le c o n t a in s t h e tw o p a r a lle l c o m p a r a tiv e s the older a n d the more.
I n th is ty p e o f s e n te n c e , the a n d th e c o m p a r is o n c a n b e f o llo w e d b y a n u m b e r o f d if
f e r e n t s tru c tu re s .

The more children you have, the bigger the house you need.
The harder you work, the more you accom plish.
The greater the experien ce, the higher the salary.

I n th e firs t e x a m p le , the more is f o llo w e d by th e n o u n children a n d t h e s u b je c t a n d v e r b you


have, w h ile the bigger is f o llo w e d b y th e n o u n the house a n d th e s u b je c t a n d v e r b you need. In
th e s e c o n d e x a m p le , the harder is f o llo w e d b y th e s u b je c t a n d v e rb you work, w h ile the more
is fo llo w e d b y th e s u b je c t a n d v e r b you accomplish. I n th e t h ir d e x a m p le , the greater is fo l
lo w e d o n ly b y th e n o u n the experience, w h ile the higher is f o llo w e d o n ly b y t h e n o u n the
salary. \fou s h o u ld n o te t h a t th is la s t e x a m p le d o e s n o t e v e n c o n t a in a v e r b , y e t it is a c o r
r e c t s t r u c t u r e in E n g lis h .

T h e f o llo w in g c h a r t o u tlin e s th is ir r e g u la r -er, -er s tru c tu re :

THE-ER-ER STRUCTURE

-er ....... .. .. - . . -er


THE more (same structure), the more (same structure)

This type of sentence may o r may not include a verb.

E X E R C I S E 29: E a c h o f t h e f o llo w in g s e n te n c e s c o n t a in s t h e i r r e g u la r -er, -er s tru c t u r e .


C ir c le th e tw o c o m p a r is o n s w ith the. U n d e r l in e th e p a rts th a t s h o u ld b e p a r a lle l. T h e n in
d ic a te i f th e s e n te n c e s a re c o r r e c t ( C ) o r i n c o r r e c t (I).

^ 1. (The hotter) the food is,(h ard ei) it is to eat.

C 2. (T he warmer) the weather, (the greater) the attendance at the ou td oor concert.

_____ 3. The m ore you say, the worst the situation will be.

_____ 4. T he m ore time they have to play, the happier the children are.

_____ 5. The thicker the walls, the noise that com es through is less.

_____ 6.If you run faster, the m ore quickly youll arrive.

_____ 7. T he m ore you use the p h on e, the higher the bill will be.

_____ 8. T he harder you serve, the easier it is to win the point.

_____ 9. T h e earliest you send in your tax forms, the sooner you will receive your refund.

_____ 10. The m ore p eople there are at the party, you ll have a good time.
STRUCTURE AND WRITTEN EXPRESSION

EXERCISE (Skills 27-2 9 ): C ircle th e com p a ra tiv es a n d su p erla tiv es in th e fo llo w in g se n


te n c e s. T h e n in d ic a te if th e s e n te n c e s are c o r r e c t (C ) o r in c o r r e c t (I).

_____ 1. The coffee is m ore stronger today than it was yesterday.

_____ 2. The tree that was struck by lightning had b een the tallest o f the two trees we had in
the yard.

_____ 3. H e will buy the m ost fuel-efficient car that h e can afford.

_____ 4. The closest it gets to summer, the longer the days are.

_____ 5. The business departm ent is bigger o f the departm ents in the university.

_____ 6. 1 really do not want to live in the Southeast because it is on e o f the m ost hot areas in
the U n ited States.

_____ 7. It is preferable to use the m ost efficient and m ost effective m eth od that you can.

_____ 8. T onights dinner was m ore filling than last nigh ts.

_____ 9. The soon er the exam is scheduled, the less tim e you have to prepare.

_____ 10. The hou se is now the cleanest that it has ever been.

TOEFL EXERCISE (Skills 2 7 -2 9 ): C h o o se th e le tte r o f th e w ord o r g r o u p o f w ord s that


b e st c o m p le te s th e s e n te n c e .

1. The speed of light i s _____ the speed of 3. _____ in Stevensons landscapes, the more
sound. vitality and character the paintings seem to
possess.
(A) faster
(B) much faster than (A) The brushwork is loose
(C) the fastest (B) The looser brushwork
(D) as fast (C) The loose brushwork is
(D) The looser the brushwork is
2. The use of detail i s _____ method of
developing a controlling idea, and almost
all students employ this method.
(A) more common
(B) common
(C) m ost common
(D) the most common

C h o o se t h e lette r o f th e u n d e r lin e d w ord o r g r o u p o f w ord s th at is n o t c o rr ec t.

_____ 4. Certain types of snakes have been known to survive fasts more as a year long.
A B ~C~ If

_____ 5. The grizzly bear, which can grow up to eight feet tall, has been called a more
A B C D
dangerous animal of North America.
WRITTEN EXPRESSION

_ 6. Climate, soil type, and availability o f water are the m ost criticalfactors than
A TT ~C~
selecting the best type of grass for a lawn.
D

_ 7. Peter Abelard, a logician and theologian, was the controversialest teacher o f his age.
A ~B C D

_ 8. Protein m olecules are the m ost complex than the molecules o f carbohydrates.
A B ~C~ D

_ 9. The leek, a member of the lily family, has a mildest taste than the onion.
A B ~TT D

_10. The widely used natural fiber of all is cotton.


A B~ ~TT D

TOEFL REVIEW EXERCISE (Skills 129): C h o o se th e lette r o f th e w o r d o r g r o u p o f


w ord s th a t b e st c o m p le te s th e se n te n c e .

1 . ____ , a liberal arts college specifically for 3. While the discovery that many migratory
deaf people, is located in Washington, D.C songbirds can thrive in deforested
wintering sp ots-------- , the fact remains
(A) Gallaudet College
that these birds are dying at unusual rates.
(B) Gallaudet College is
(C) About Gallaudet College (A) it is heartening
(D) Because o f Gallaudet College (B) hearten
(C) heartening
2. --------varieties o f dogs at the show, (D) is heartening
including spaniels, poodles, and collies.
(A) The several
(B) Those
(C) Several
(D) There were several

C h o o se th e le tte r o f th e u n d e r lin e d w o rd o r g r o u p o t w ord s u ia t is n o t c o r r e c t.

--------4. The coyote is somewhat smaller in size that a timber wolf.


A B ~C D

--------5. The weather reports all showed that there were a tremendous storm front m oving in.
A B ~C~ D

--------6. Seldom cactus plants are found outside o f North America.


A ~B C ~D

--------7. In a basketball game a player what is fouled receives one or two free throws.
~A BCD

-------- 8. Until recently, California was largest producer of oranges in the United States.
A B C D
STRUCTURE AND WRITTEN EXPRESSION

9. An understanding of engineering theories and problems are impossible until basic


A B
arithmetic is fully mastered.
C D

.10. The earliest the CVS (chorionic villus sampling) procedure in the pregnancy, the
A B c
greater the risk to the baby.
D

PROBLEMS W IT H T H E FO R K OF T H E VERB
It is c o m m o n in th e W ritten E x p r e ssio n p art o f th e T O E F L test fo r th e verb s to b e fo r m e d
in correctly. T h e r e fo r e , you s h o u ld c h e c k th e fo r m o f th e verb carefully. You sh o u ld b e fa
m iliar w ith th e fo llo w in g verb form s: th e b ase fo r m , th e p r e se n t te n se , th e p r e se n t p a rtici
p le , a n d th e p a st p a r ticip le . T h e fo llo w in g are e x a m p le s o f e a c h o f th e se verb fo r m s as
they are u sed in this text:

BASE PRESENT PAST


FORM* PRESENT PARTICIPLE PARTICIPLE

walk walk(s) walking walked walked


hear hear(s) hearing heard heard
cook cook(s) cooking cooked cooked
sing sing(s) singing sang sung
come come(s) coming came come
begin begin(s) beginning began begun

You sh o u ld b e particu larly aw are o f th e fo llo w in g th r e e p r o b le m a tic situ a tio n s w ith verb s
b e c a u se th ey a re th e m o st c o m m o n a n d th e e a sie st to co rrect: ( 1 ) c h e c k w h at c o m e s a fter
have; (2) c h e c k w hat c o m e s a fter be; a n d (3 ) c h e c k w h at c o m e s a fter will, w ould, a n d o th e r
m od als.

NOTE: A more complete list of verb forms and an exercise to practice their use are
included at the back of the text in Appendix F. You may want to complete this exercise
before you continue with skills 30 through 32.

S k i l l 30: AFTER HAVE, U S E T H E PAST PARTICIPLE

W h e n e v e r you s e e th e verb h a v e in any o f its fo r m s (have, has, having, h a d ), b e su re th a t


th e verb that fo llo w s it is in th e p ast p a r tic ip le form .

They had walk* to school. (should be had walked)


We have see* the show. (should be have seen)
H e has took* the test. (should be has taken)
H a v in g a t^ , he went to school. (should be H aving eaten)
She should have did* the work. (should be should have done)
WRITTEN EXPRESSION

In a d d itio n , y o u sh o u ld b e su re th a t i f y o u h ave a su b ject a n d a p ast p a r tic ip le , y o u a lso


have th e verb have. T h is p r o b le m is p articu larly c o m m o n w ith th o s e v e rb s (su c h as sing,
sang, su n g ) th a t c h a n g e fr o m p r e se n t to p ast to p a st p a r ticip le by c h a n g in g o n ly th e vow el.

My friend sung* in the choir. (should be sang or has sung)


H e become* angry at his friend. (should be became o r has become)
T he boat sunk* in the ocean. (should be sank or has sunk)

T h e fo llo w in g c h a r t o u tlin e s th e u se o f verb fo r m s a fter have:

VERB FORMS AFTER HAVE________________________


_____________________ __
HAVE past participle
. ________

E X E R C ISE 30: E ach o f th e fo llo w in g s e n te n c e s c o n ta in s a v e rb in th e p a st o r a p ast p ar


tic ip le . U n d e r lin e th e verb s o r p ast p a r ticip le s tw ice. T h e n in d ic a te i f th e s e n t e n c e s are
c o r r e c t (C ) o r in c o r r e c t (I).

' 1. T h e young girl drunk a glass o f milk.

C 2. Before she left, she had asked her m other for perm ission.

_____ 3. H aving finished the term paper, he began studying for the exam .
_____ 4. T h e secretary has broke her typewriter.
_____ 5. T he installer should have com pletes the task m ore quickly.
_____ 6. H e has often becom e angry during meetings.
_____ 7. She has rarely rode her horse in the park.
_____ 8. Having saw the film, he was quite disappointed.
_____ 9. Tom has thought about taking that job.
_____ 10. You m ight have respond m ore effectively.

S k i l l 31: AFTER BE, USE T H E PRESENT PARTICIPLE O R T H E


PAST PARTICIPLE

T h e verb be in an y o f its fo r m s (a m , is, are, was, were, be, been, being) can b e fo llo w e d by a n
o th e r verb . T h is verb s h o u ld b e in th e p r e se n t p a r ticip le or th e p ast p a r tic ip le fo r m .
We are dai* our hom ework. (should be are doing)
T he hom ework was do* early. (should be was done)
Tom is take* the book. (should be is taking)
T he book was take* by Tom. (should be was taken)

T h e fo llo w in g c h a r t o u tlin e s th e u se o f verb fo r m s a fter be:

VERB FORMS AFTER BE

BE + (1) present participle


. , ' .. :
(2) past parti cple
STRUCTURE AND WRITTEN EXPRESSION

E X ER C ISE 31: E ach o f th e fo llo w in g s e n te n c e s c o n ta in s a verb fo r m e d w ith be. U n d e r lin e


th e verbs tw ice. T h e n in d ic a te if th e s e n te n c e s are c o r r e c t (C ) o r in c o r r e c t (I).

I 1. At 12:00 Sam is eat his lunch.

C 2. We are m eeting them later today.

3. The message was took by the receptionist.

4. Being heard was extrem ely im portant to him.

5. The Smiths are build their house on som e property that they own in the desert.

6. It had been noticed that som e staff m embers were late.

7. The report should have been subm it by noon-.

8. Are the two com panies m erge into one?

9. He could be taking four courses this semester.

10. The score inform ation has been duplicates on the back-up disk.

S k i l l 32: AFTER W ILL.W O ULD , O R O TH ER M ODALS, USE T H E BASE


FORM OF T H E VERB

W h e n e v e r y o u s e e a m o d a l, su c h as will, w ould, shall, should, can, could, may, m ight, o r must,


y o u sh o u ld b e su re th at th e verb th a t fo llo w s it is in its base form .

T he boat will leaving* at 3:00. (should be will leave)


T he doctor may arrivess* soon. (should be may arrive)
T he students must taken* the exam . (should be must take)

T h e fo llo w in g ch a rt o u d in e s th e u se o f verb fo r m s after m odals:

VERBS FORMS AFTER MODALS

MODAJ. + main form o f th e verb

EXER CISE 32: E a ch o f th e fo llo w in g s e n te n c e s c o n ta in s a verb fo r m e d w ith a m o d a l. U n


d e r lin e th e verbs tw ice. T h e n in d ic a te i f th e s e n te n c e s are c o r r e c t (C) o r in c o r r e c t (I).

- 1. The salesclerk m ight lower the price.

! 2. T he television movie will finishes in a few minutes.

3. Should everyone arrive by 8:00?

4. T he m ethod for organizing files can be improved.


WRITTEN EXPRESSION

_____ 5. The m achine may clicks o ff if it is overused.

_____ 6. Every m orning the plants must be watered.

_____ 7. The houses with ocean views could sell for considerably m ore.

_____ 8. Would anyone liked to see that movie?

_____ 9. I do n ot know w hen it will depart.

_____ 10. She will work on the project only if she can has a full-time secretary.

EX ER C ISE (S k ills 3 0 - 3 2 ) : U n d e r lin e th e verb s tw ice in th e fo llo w in g se n te n c e s . T h e n in


d ica te i f th e s e n te n c e s are c o r r e c t (C ) o r in c o r r e c t (I).

_____ 1. I have gave you all the m oney that I have.

_____ 2. The articles were put in the newspaper before he was able to stop production.

_____ 3. All the tickets for the concert m ight already be sold.

_____ 4. H e was so thirsty that he drunk several large glasses o f water.

_____ 5. The deposit will has to be paid before the apartm ent can be rented.

_____ 6. H e objects to being held without bail.

____ _ 7. Having com pleted the first chapter o f the manuscript, she decid ed to take a break.

_____ 8. If Steve had really wanted to pass his exam , he would has studied m uch m ore.

_____ 9. H e thought that he sh ould have be invited to attend the conference.

_____ 10. Before the speaker finished, many guests had rose from their seats and started for
the door.

T O E F L E X E R C ISE (S k ills 3 0 -3 2 ): C h o o se th e le tte r o f t h e u n d e r lin e d w o r d o r g r o u p o f


w ords th a t is n o t c o rr ec t.

_____ 1. Alice in Wonderland, first published in 1865, has since being translated into thirty
A B C D
languages.

_____ 2. The Peace Corps was establish on March 1, 1961, by then President John F. Kennedy.
A B C D

_____ 3 . The advisor told himself, while listening to the speech, that a dozen other reporters
~A~ B
would has already asked that question.
C D
STRUCTURE AND WRITTEN EXPRESSION

. 4. At the start of the American Revolution, lanterns were hung in the Old North Church
~A~ B C
as a signal that the British were came.
D
. 5. Before he died, Linus Pauling had wins two Nobel Prizes: the 1954 Nobel Prize
~A B C
in Chemistry and the 1962 Nobel Peace Prize.
D
. 6. On the huge Ferris wheel constructed for a world exhibition in Chicago in 1893,
~A~ B
each of the thirty-six cabs could held sixty people.
C HET
.7 . To overcome rejection of a skin graft, a system for matching donor and recipient
A B C
tissues has be developed.
TT
_ 8. Nails are commonly makeof steel butalso can contain substances such as aluminum
~A~ B C D
or brass.

.9 . A patient suffering from amnesia may had partial or total loss of memory.
~B C~ "D-
_10. Theidea o f using pure nicotine to help smokers stop was first tries in the mid-1980s
~A~ B ~D ~
with the nicotine-laced chewing gum Nicotette.

TO EFL REVIEW EXERCISE (Skills 1-32): C h o o se th e le tte r o f th e w o rd o r g r o u p o f


w ord s th a t b e st c o m p le te s th e s e n te n c e .

. separates Manhattans Upper East 4. W hen. _on July 4, 1789, the federal
Side from the Upper West Side. tariff, intended by the Founding Fathers to
be the governments primary source of
(A) Central Park
revenue, was remarkably evenhanded.
(B) Where Central Park
(C) Where is Central Park (A) was first enacted
(D) Central Park which (B) first enacted
(C) was enacted first
2. Bioluminescent anim als_____ the water or (D) it first
on land.
. inclined to push for such a
(A) live
reduction, it would probably not be
(B) are living either
successful.
(C) they are found in
(D) can be found in (A) The Office of Management
(B) The Office of Management was
3. The purpose of a labor union is to improve (C) In the Office of Management
the working conditions,_____ , and pay of (D) Were the Office of Management
its members.
(A) jobs are secure
(B) to be secure
(C) job security
(D) the job's security
WRITTEN EXPRESSION

C h o o se th e le tte r o f th e u n d e r lin e d w o rd o r g r o u p o f w ords th a t is n o t c o r r e c t.

-------- 6. Helium has the most low boiling point of all substances.
~A~ B " C D

--------7. There is twenty-six bones in the human foot, fourteen of them in the toes.
A B ~C~ D

-------- 8. Extension of the countdown hold to fourteen hours was order to give crews
~A~ B
more time to repair wiring and clear away equipment.
C D

--------9. The study demonstrates that neither experience or awareness will improve chances
A B ~C D
of success.

--------10. Som e of the eye movements used in reading is actually unnecessary.


~A~ B C D

PROBLEMS W IT H T H E USE O F T H E VERB___________________

M any d iffe r e n t p r o b le m s in u s in g th e c o r r e c t verb te n se are p o ssib le in E n g lish . H ow ever,


fo u r sp e c ific p r o b le m s o c c u r fr eq u e n tly o n th e T O E F L test, so y o u n e e d to pay c a r e fu l at
te n tio n to th e s e four: ( 1 ) k n o w in g w h e n to u se th e past w ith th e p r e se n t, ( 2 ) u s in g h a d
a n d have correctly, (3 ) u s in g th e c o r r e c t te n se w ith tim e e x p r e s sio n s, a n d (4 ) u s in g th e
c o r r e c t te n s e w ith w ill a n d would.

S kill 33: K N O W W H E N T O USE T H E PAST W IT H T H E PRESENT

O n e verb te n s e p r o b le m th a t is c o m m o n b o th in stu d e n t w ritin g a n d o n th e T O E F L te st is


th e sw itch fro m th e p ast te n s e to th e p r e se n t te n se fo r n o p articu lar r e a s o n . O fte n w h e n a
s e n te n c e h as b o th a p ast te n s e a n d a p r e se n t te n se , th e s e n te n c e is in c o r r e c t.

H e took the m oney w hen he wants* it.

T h is s e n te n c e says th at he took the money (in th e past) when he w a n ts it (in th e p r e s e n t). T h is


m e a n in g d o e s n o t m a k e an y se n se; it is im p o ssib le to d o s o m e th in g in th e p a st as a r esu lt
o f s o m e th in g y o u w a n t in th e p r e se n t. T h is s e n te n c e can b e c o r r e c te d in sev era l ways, d e
p e n d in g o n th e d e s ir e d m e a n in g .

H e took the m oney when he wanted it.


H e takes the m oney when he wants it.

T h e first e x a m p le m e a n s th a t he took the money (in th e past) when he w an ted it (in th e p a st).
T h is m e a n in g is lo g ic a l, a n d th e s e n t e n c e is co rr ec t. T h e s e c o n d e x a m p le m e a n s th a t he
takes the money (h a b itu a lly ) w hen he w ants it (h ab itu ally). T h is m e a n in g is a lso lo g ic a l, a n d
th e s e c o n d e x a m p le is also c o rr ec t.
202 STRUCTURE AND WRITTEN EXPRESSION

It is n e c e s sa r y to p o in t o u t, how ever, th a t it is p o ssib le fo r a lo g ic a l s e n te n c e in E n glish


to have b o th th e p a st a n d th e p r e s e n t te n se .

1 know that he took the m oney yesterday.

T h e m e a n in g o f th is s e n te n c e is lo g ica l: 1 know (r ig h t now , in th e p r e se n t) th a t he took the


money (yesterday, in th e p a st). You can se e from th is e x a m p le that it is p o s sib le fo r an E n g
lish s e n t e n c e to h ave b o th th e p ast a n d th e p r e se n t ten se. T h e err o r y o u n e e d to avoid is
th e sw itch from th e p ast to th e p r e se n t fo r n o p articu lar rea so n . T h e r e fo r e , w h en y o u see
a s e n te n c e o n th e T O E F L test w ith b o th th e p ast a n d th e p r e se n t te n se , y o u m u st c h e c k
th e m e a n in g o f th e s e n t e n c e ca refu lly to se e if it is lo g ic a l in E n glish .

T h e fo llo w in g ch a rt o u tlin e s th e u se o f th e p ast ten se w ith th e p r e se n t te n se in E nglish:

USING THE PASTWITH THE PRESENT

1. If you see a sentence with one verb in the past and one verb in the present, the sentence is
probably incorrect.
2. However, it is possible for a correct sentence to have both past and present together.
3. If you see the post and present together, you must check the meaning to determine whether
or not the sentence is correct.

E X E R C ISE 33: E ach o f th e fo llo w in g s e n te n c e s h a s at lea st o n e verb in th e p ast a n d o n e


verb in th e p r e se n t. U n d e r lin e th e verb s tw ice a n d d e c id e if th e m e a n in g s are lo g ica l.
T h e n in d ic a te i f th e s e n te n c e s are c o r r e c t (C ) o r in c o r r e c t (I).

1 1. I tell him the truth when he asked m e the question.

C 2. 1 understand that you were angry.

3. W hen he was a child, he always goes to the circus.

4. Last sem ester h e reads seven books and wrote five papers.

5. Steve wakes up early every m orning because he went to work early.


fi Mark studied at the American University when he is in W ashington, D.C.

7. H e is telling the teacher why he did not have time to finish his homework.

8. H e put som e m oney in his account when h e goes to the bank.

9. Tom keeps studying hard because he intended to go to dental school.

10 She is where she is today because she worked hard when she was a student.

S kill 3 4 : USE HAVE A N D HAD CORRECTLY

T w o te n se s th a t are o fte n c o n fu s e d are th e p r e se n t p e r fe c t (have + past p a r tic ip le ) and


th e past p e r fe c t (had + p ast p a r tic ip le ). T h e s e tw o te n se s have c o m p le te ly d iffe r e n t uses,
a n d you s h o u ld u n d e r sta n d h o w to d iffe r e n tia te th e m .
WRITTEN EXPRESSION 203

T h e p r e se n t p e r fe c t ( have + p a st p a r ticip le ) refers to th e p e r io d o f tim e from the p ast


u n til the present.

Sue has lived in Los A ngeles for ten years.

T h is s e n te n c e m e a n s th a t S u e h as liv ed in L os A n g e le s for th e te n years u p to now . A c


c o r d in g to this s e n te n c e , S u e is still liv in g in L os A n g e le s.

B e ca u se th e p r e se n t p e r fe c t r efe rs to a p e r io d o f tim e from th e p ast u n til th e p r e se n t,


it is n o t c o r r e c t in a s e n t e n c e th a t in d ic a te s p ast only.

A t the start o f the nineteenth century, Thomas Jefferson has become* president o f the U nited States.
Every tim e Jim worked on his car, he has improved* it.

In th e first e x a m p le , th e p h ra se at the start o f the nineteenth century in d ic a te s th a t th e a c tio n


o f th e verb was in th e p ast only, b u t th e verb in d ic a te s th e p e r io d o f tim e fr o m th e p ast u n
til th e p r e se n t. S in c e th is is n o t lo g ic a l, th e s e n te n c e is n o t c o r r e c t. T h e verb in th e first
e x a m p le sh o u ld b e became. T h e s e c o n d e x a m p le in d ic a te s th a tjim worked o n h is ca r in th e
past, b u t h e im p r o v e d it in th e p e r io d fr o m th e p ast u n til th e p r e se n t. T h is id e a a lso is n o t
lo g ic a l. T h e verb in th e s e c o n d e x a m p le s h o u ld b e th e sim p le p a st improved.

T h e p ast p e r fe c t ( had + past p a r tic ip le ) refers to a p e r io d o f tim e that started in the past
and ended in the past, before something else happened in the past.

Sue had lived in Los A ngeles for ten years when she moved to San D iego.

T h is s e n t e n c e m e a n s th a t S u e liv e d in L os A n g e le s fo r ten years in th e p ast b e fo r e sh e


m o v e d to San D ie g o in th e past. S h e n o lo n g e r lives in L os A n g e le s.

B e c a u se th e p ast p e r fe c t b e g in s in th e past a n d e n d s in th e past, it is g e n e r a lly n o t c o r


rect in th e sa m e s e n t e n c e w ith th e p r e se n t te n se .

Tom had finished the exam when the teacher collects* the papers.

T h is s e n t e n c e in d ic a te s th a t Tom finished the exam (in th e past) a n d th a t a c tio n e n d e d when


the teacher collects the papers (in th e p r e se n t). T h is is n o t lo g ic a l, so th e s e n t e n c e is n o t c o r
rect. T o m fin is h e d th e e x a m (in th e p a st), a n d th e a c tio n o f fin is h in g th e e x a m e n d e d
w h en th e te a c h e r c o lle c te d th e p a p ers. T h e r e fo r e , th e s e c o n d verb in th is e x a m p le
s h o u ld b e in th e p ast te n se , collected.

T h e fo llo w in g c h a r t o u tlin e s th e u ses o f th e p r e se n t p e r fe c t a n d th e p a st p e r fec t:

USING (HAVE + PAST PARTICIPLE) AND (HAD + PAST PARTICIPLE)


TENSE FORM MEANING USE

present perfect hove + past participle past up to now not with a past tense**
* ' '
past perfect had + past participle before past up to past not with a present tense

**Except when the time expression since is part of the sentence (see Skill 35).
STRUCTURE AND WRITTEN EXPRESSION

E X E R C ISE 34: E ach o f th e fo llo w in g s e n te n c e s c o n ta in s had o r have. U n d e r lin e th e verb s


tw ice a n d d e c id e i f th e m e a n in g s are lo g ica l. T h e n in d ic a te i f th e s e n te n c e s are c o r r e c t
(C ) o r in c o r r e c t (I).

C i I have always liked the designs that are on the cover.

I 2. Because her proposal had been rejected, she is depressed.

_____ 3 . T h e students have registered for classes before the sem ester started.

_____ 4. W hen she had purchased the car, she contacted the insurance agent.

_____ 5. H e said that he had finished the typing w hen you finish the reports.

_____ 6. She has enjoyed herself every time that she has gone to the zoo.

_____ 7. H e drove to the post office after he had finished preparing the package.

_____ 8. After the votes were counted, it had been determ ined that Steve was the winner.

_____ 9. Last night all the waiters and waitresses have worked overtime.

_____ 10. H e had fastened his seat belt before the airplane took off.

S k i l l 35: USE T H E CORRECT TENSE W IT H TIM E EXPRESSIONS

O fte n in s e n te n c e s in th e W ritten E x p ressio n se c tio n o f th e T O E F L te st th e r e is a tim e e x


p r e ssio n th at clearly in d ic a te s w h at verb te n se is n e e d e d in th e se n te n c e .

We m oved to New York in 1970.


We had left there by 1980.
We have lived in San Francisco since 1982.

In th e first e x a m p le , th e tim e e x p r e ssio n in 1 9 7 0 in d ic a te s th a t th e verb s h o u ld b e in th e


s im p le p ast (m oved). In th e s e c o n d e x a m p le , th e rim e e x p r e s sio n by 1 9 8 0 in d ic a te s th at
th e v e rb s h o u ld b e in th e past p e r fe c t (h a d left). In th e th ird e x a m p le , th e tim e e x p r e ssio n
since 1982 in d ic a te s th a t th e verb sh o u ld be in th e p r e se n t p e r f e c t (h a v e lived).

S o m e a d d itio n a l tim e e x p r e ssio n s that clearly in d ic a te th e c o r r e c t te n se are ago, last,


a n d lately.

She got a job two years ago.


She started working last week.
She has worked very hard lately.

In th e first e x a m p le , th e tim e e x p r e s sio n two years ago in d ic a te s th a t th e verb s h o u ld b e in


th e sim p le p a st (got). In th e se c o n d e x a m p le , th e tim e e x p r e s sio n last week in d ic a te s th at
th e verb sh o u ld b e in th e sim p le past (started). In th e third e x a m p le , th e tim e e x p r e ssio n
lately in d ic a te s th a t th e verb sh o u ld b e in th e p r e se n t p e r fe c t (h a s w orked).
WRITTEN EXPRESSION 20S

T h e fo llo w in g c h a r t lists tim e e x p r e s sio n s th a t in d ic a te th e c o r r e c t v e r b ten se:

USING CORRECT TENSES WITH TIME EXPRESSIONS


PAST PERFECT SIMPLE PAST PRESENT PERFECT

by (1920) (two years) ago since (1920)


: last (year)
' ' .r . " . .. - ... . - - .........; V . .
in (1920)

EXERCISE 35: E ach o f th e fo llo w in g s e n te n c e s c o n ta in s a tim e e x p r e s s io n . C ircle th e


tim e e x p r e s s io n s a n d u n d e r lin e th e verb s tw ice. T h e n in d ic a te i f th e s e n te n c e s are c o r
rect (C ) o r in c o r r e c t (I ).

C 1. T he p h one rang incessantly (last night.)

__ I__ 2. They have finished contacting everyone(by 4:00 yesterday)

_____ 3. T h e Pilgrims have arrived in the New World in 1612.

_____ 4. Since the new law was passed, it has been difficult to estim ate taxes.

-------- 5. T he cashier put the m oney into the account two hours ago.

_____ 6. All the votes have been counted last week.

_____ 7. T h e students are writing many com positions lately.

----- -- 8. T h e Senate votes on the law to ban cigarette sm oking in public in 1990.

_____ 9. By the time the m ain course was served, all the guests had arrived and b een seated.

_____ 10. I had not d on e m uch m ore work since I talked to you on Wednesday.

S k il l 36: USE T H E C O R R EC T TENSE W IT H W ILL A N D W O U LD

C ertain c o m b in a tio n s o f v erb s are very c o m m o n in E n g lish . O n e is th e c o m b in a tio n o f


th e sim p le p r e se n t a n d will.

I know that they will arrive soon.


It is certain that he will graduate.

A n o th e r c o m b in a tio n th a t is q u ite c o m m o n is th e c o m b in a tio n o f th e s im p le p ast a n d


would.

I knew that h e would arrive.


It was certain that h e would graduate.

It is im p o r ta n t to stress th a t in th e c o m b in a tio n d isc u s se d h e r e , th e p r e s e n t s h o u ld b e


u sed w ith w ill a n d th e p ast s h o u ld b e u s e d w ith w ould; th e y g e n e r a lly s h o u ld n o t b e m ix e d .
206 STRUCTURE AND WRITTEN EXPRESSION

T h e c o m m o n e rr o rs th a t m u st g e n e ra lly b e a v o id e d a re th e c o m b in a tio n o f th e p ast w ith


will a n d th e c o m b in a tio n o f th e p r e se n t w ith uiould.
I know that he would* arrive soon.
It was certain that h e m il* graduate.

In th e first e x a m p le , th e p r e se n t, know, is illo g ic a l w ith w ould. It can b e c o r r e c te d in tw o


d iffe r e n t ways.
I knew that he would arrive soon.
I know that he will arrive soon.

In th e s e c o n d e x a m p le , th e p ast, was, is illo g ic a l w ith wilL It c a n also b e c o r r e c te d in two


d iffe r e n t ways.
It was certain that he would graduate.
It is certain that he will graduate.

T h e fo llo w in g c h a r t o u d in e s th e u se o f te n se s w ith w ill a n d would:

USING CORRECT TENSES WITH W1U.AND W OULD

VERB MEANING USE

will after the present do not use with past

would after the past do not use with present

NOTE: There is a different modal would that Is used to make polite requests.This type of would is
often used with the present tense.
1 would like to know if you hare a pencil that 1 could borrow.

E X E R C ISE 36: E ach o f th e fo llo w in g s e n te n c e s c o n ta in s w ill o r w ould. U n d e r lin e th e


verb s tw ice a n d d e c id e i f th e m e a n in g s are lo g ic a l. T h e n in d ic a te i f th e s e n te n c e s are co r
r ec t (C ) o r in c o r r e c t (I).

I 1. H e knew that he will be able to pass the exam.

C 2. I think that I will leave tomorrow.

_____ 3. Paula did not say when she will finish the project.
_____ 4. Jake doubts that h e would have time to finish the project.
_____ 5. I know that I will go if I can afford i t
_____ 6. T h e police officer indicated that he would write a ticket if he has the time.
_____ 7. Students will often study in the library before they go to classes or before they go
hom e.
_____ 8. H e told m e that he thought h e will get the jo b in spite o f his lack o f education.
_____ 9. T h e executive vice president em phasizes at the conferences that the board would
not change its position.
_____ 10. Students will register for classes according to w ho has the highest num ber o f units.
WRITTEN EXPRESSION 207

EXERCISE (Skills 3 3 -3 6 ): U n d e r lin e th e verb s tw ice in e a c h o f th e fo llo w in g s e n te n c e s .


T h e n in d ic a te i f th e s e n te n c e s are c o r r e c t (C ) o r in c o r r e c t (I).

-------- 1. W hen he receives the m oney from the insurance com pany two days ago, he had
already rebuilt the house.

--------. 2. T h e position on the city council will be filled n ext week w hen the electorate votes.
-------- 3. T h e dentist fills the cavities every tim e the X-rays show that it was necessary.
-------- 4. W hen the bell rang, the students have left the class.
--------5. T h e space shuttle would be launched next m onth if the weather is good.
-------- 6. T h e special delivery package has arrived by n oon yesterday.
-------- 7. It is probable that the students who were tested yesterday were quite successful.
-------- 8. After forty-five students had signed up for the class, the class was closed.
-------- 9. T h e parking at the arena was inadequate for the trem endous n u m ber o f drivers who
will want to park there.

-------- 10. They have not returned to Rhode Island since they left in 1970.

TOEFL EXERCISE (Skills 3 3 -3 6 ): C h o o se th e lette r o f th e u n d e r lin e d w o r d o r g r o u p o f


w ord s th a t is n o t c o r r e c t.

-------- 1 In several of his paintings, Edward Hicks depicted the Quaker farm in Pennsylvania
A
where he spends his youth.
B C ET

-------- 2. Florida has become the twenty-seventh state in the United States on March 3, 1845.
A B C D

-------- 3. After last week's meeting, the advertising department quickly realized that the
A B
product will need a new slogan.
"C D~

-------- 4. John F. Kennedy's grandfather, John F. Fitzgerald, serves two terms as the mayor of
A B C
Boston in the beginning of the twentieth century.
D

--------5. Fort Ticonderoga, a strategically important fortification during the Revolution, had
A~
since been reconstructed and turned into a museum.
B C D

-------- 6. In making their calculations. Institute researchers assume that the least costly form
A B
o f energy would be used.
C ~D ~
STRUCTURE AND WRITTEN EXPRESSION

7. A twentv-one-vear-old man became the second casualty yesterday when he loses


A B C
control of his truck.

8. Most people had written with quill pens until pens with metal points becom e popular
A B C D
in the middle of the nineteenth century.

9. In a determined drive to pare its debt, Time Warner is launching a stock offering
A B C
plan that would potentially raise $2.8 billion.
D

_10 The formula used in the study calls for either peroxide or metaldehyde, but
~JT B C
metaldehyde was not always available.
TT

TOEFL REMEW EXERCISE (Skills 1-36): C h o o se th e lette r o f th e word o r g r o u p o f


w ord s th a t b e s t c o m p le te s th e s e n te n c e .

1 . _____ in the United States declined from 3. Bats avoid running into objects b y .
twenty million in 1910 to nine million in high-frequency sounds and listening for
the 1970s. echoes.

(A) For a number of horses (A) the emission


(B) The number of horses (B) emitted
(C) When the number of horses (C) emitting
(D) That the number of horses (D) they emit

2. Because of his reservations about the 4. It has been estimated that if we intend to
issu e ,_____ refused to vote for it. stay above the starvation level,--------the
food supply.
(A) who
(B) and (A) so we will have to double
(C) which the senator (B) and it m ust double
(D) the senator' (C) which it must be doubled
(D) w e must double

Choose the letter of the underlined word or group of words that is not correct.

5 To determine an objects force, the mass and speed of the object must be measure.
-------- A-------- B -C- D

6. The most common time for tornados to occur are in the afternoon or evening on a
A B C
hot, humid spring day.
D

7. Automakers Nissan and Ford and several aerospace research facilities in Great
A
Britain are working lately to apply active noise cancellation to entire cars and planes.
------- B------- C D
WRITTEN EXPRESSION

When a country in an early stage of development, investments in fixed capital


A B ~C~
are vital.

_ 9. John Chapman became famous in American folklore as "Johnny Appleseed after he


A
plants apple trees throughout the northeastern part of the United States.
B C D

_10. Inasmuch he kept mostly to himself, the author of The Treasure o f the Sierra Madre
A B C
was known as the mysterious B. Treuen.
TT

PROBLEMS W IT H PASSIVE VERBS___________________________

S e n te n c e s in w h ic h th e err o r is an in c o r r e c t passive a re c o m m o n in th e W ritten E x p r e s


sio n se c tio n o f th e T O E F L test. You th e r e fo r e n e e d to b e a b le to r e c o g n iz e th e c o r r e c t
form o f th e passive a n d to b e a b le to d e te r m in e w h en a passive verb r a th e r th a n an active
verb is n e e d e d in a s e n te n c e .
T h e d iffe r e n c e b e tw e e n a n active a n d a passive verb is th a t th e su b je c t in a n active s e n
te n c e does th e a c tio n o f th e verb , a n d th e su b je ct in a passive s e n te n c e receives th e a c tio n o f
th e verb. T o c o n v e r t a s e n te n c e fr o m active to passive, two c h a n g e s m u s t b e m a d e . (1 )
T h e su b ject o f th e active s e n t e n c e b e c o m e s th e o b je c t o f th e passive s e n t e n c e , w h ile th e
o b je c t o f th e active s e n te n c e b e c o m e s th e su b je ct o f th e passive s e n te n c e . (2 ) T h e verb in
th e passive s e n te n c e is fo r m e d by p u ttin g th e h e lp in g v e rb bein th e sa m e fo r m as th e v e fb
in th e active s e n te n c e a n d th e n a d d in g th e p a st p a r tic ip le o f th is verb.

SUBJECT OBJECT

T h e first e x a m p le is an active s e n te n c e . T o c o n v e r t th is active s e n te n c e to a p assive s e n


te n c e , y o u m u st first m ak e th e su b je ct o f th e active s e n te n c e , Margaret, th e o b je c t o f th e
passive s e n te n c e w ith by. T h e o b je c t o f th e active s e n te n c e , letter, b e c o m e s t h e su b je c t o f
th e passive s e n te n c e . N e x t, th e passive verb c a n b e fo r m e d . B e c a u se wrote is in th e p a st
te n se in th e active s e n te n c e , th e p a st te n se o f be (was) is u s e d in th e p assive s e n te n c e .
T h e n th e verb xvrote in th e active s e n te n c e is c h a n g e d to th e p ast p a r tic ip le written in th e
passive s e n te n c e .
It s h o u ld b e n o te d th a t in a passive s e n te n c e , by + object d o e s n o t n e e d to b e in c lu d e d
to h a v e a c o m p le te s e n te n c e . T h e fo llo w in g are b o th e x a m p le s o f c o r r e c t s e n te n c e s .

T he letter was written yesterday by Margaret.


T he letter was written yesterday.

N o tic e th a t th e s e passive s e n te n c e s a r e c o r r e c t i f by Margaret is in c lu d e d (as in th e first e x


a m p le ) o r i f by Margaret is o m itte d (as in th e s e c o n d e x a m p le ) .
STRUCTUREANDWRITTEN EXPRESSION

NOTE: Exercises to practice active and passive forms can be found in Appendix G at the
back of the text. You may want to complete these exercises before you begin Skill 37.

S k i l l 37: USE T H E CORRECT FORM OF T H E PASSIVE

O n e way th a t th e passive can b e te sted o n th e T O E F L te st is sim p ly w ith an in c o r r e c t fo rm


o f th e p assive. T h e fo llo w in g are e x a m p le s o f passive e rro rs th a t m ig h t a p p e a r o n th e
T O E F L test:
T he portrait was p a in tin g by a fam ous artist.
The project will fin is h e d by Tim.

In th e first e x a m p le , th e passive is fo r m e d in c o rr ec tly b e c a u se th e p ast p a r tic ip le p a in ted


sh o u ld b e u s e d rath er than th e p r e se n t p a rticip le p a in tin g . In th e s e c o n d e x a m p le , th e
verb be h as n o t b e e n in c lu d e d , a n d so m e fo rm o f be is n e c essa ry for a passive verb . T h e
verb in th e s e c o n d s e n te n c e sh o u ld b e w ill be fin ish ed .

T h e fo llo w in g c h a r t o u tlin e s th e way to fo r m th e p assive correcd y:

THE FORM OFTHE PASSIVE

BE + past participic (BY + object)

E X E R C ISE 37: E ach o f th e fo llo w in g s e n te n c e s has a passive m e a n in g . U n d e r lin e tw ice


th e verb s th a t sh o u ld b e passive. T h e n in d ic a te i f th e s e n te n c e s are c o r r e c t (C ) o r in cor-
r ect (I).

1 1. T h e boy had never be stung by a bee.

C 2. T h e suits were hung in the closet when they were returned from the cleaners.

3. M oney is len ding by the credit union to those w ho want to buy hom es.

4. T h e record had been chose by dancers near the jukebox.

fi T h e topic for your research paper should have been approved by your advisor.

_____ 6. T hat song has been playing over and over again by Steve.

7. T heir utility bills have been increased again and again.

8. T he patients who are too sick to sit up are being assisted by the orderlies.

9. T he offices were thoroughly clean last evening by the n igh t crew.

. .. 10. T h e car that was struck in the intersection yesterday is b ein g repaired today.
WRITTEN EXPRESSION

S k i l l 38: R E C O G N IZ E A C T IV E A N D PASSIVE M EA N IN G S

W h e n t h e r e is n o o b je c t (w it h o r w it h o u t by) a fte r a v e rb , y o u m u s t l o o k a t t h e m e a n in g o f
th e s e n te n c e to d e t e r m in e i f t h e v e r b s h o u ld b e a c tiv e o r passive. S e n t e n c e s w it h a n in
c o r r e c t p a ssive v e r b a n d n o by + object to te ll y o u th a t th e v e r b s h o u ld b e p a ssive a r e th e
m o st d if f ic u lt p a ssive e r r o r s to re c o g n iz e o n th e T O E F L test. S t u d y th e e x a m p le s :

We m ailed the package at the post office.


T h e letter was m ailed by us today before noon.
T h e letter was mailed today before noon.
T h e letter m ailed* today before noon.

T h e f ir s t th re e e x a m p le s a b o v e a r e c o r r e c t . T h e firs t e x a m p le h a s th e a c tiv e v e r b m ailed


u sed w ith th e o b je c t package, t h e s e c o n d e x a m p le h as th e passive v e r b w as m ailed u s e d w ith
by usr, t h e t h ir d s e n t e n c e h a s th e p a ssive v e r b was m ailed u s e d w it h o u t a n o b je c t.
T h e f o u r th e x a m p le is th e ty p e o f passive e r r o r th a t a p p e a rs m o s t o f te n o n th e
T O E F L test. T h is ty p e o f s e n t e n c e h as th e fo llo w in g c h a ra c te ris tic s : (1 ) a n i n c o r r e c t p a s
sive v e r b th a t lo o k s lik e a c o r r e c t a c tiv e v e rb , a n d (2 ) n o by + object to te ll y o u t h a t a p a ssive
is n e e d e d . T o c o r r e c t t h e f o u r t h e x a m p le , th e a c tiv e v e rb n e e d s to b e c h a n g e d to th e p as
sive was m ailed.
T o d e t e r m in e th a t s u c h a s e n t e n c e is in c o r r e c t , y o u m u s t s tu d y t h e m e a n in g o f th e
su b je ct a n d th e v e rb . Y o u m u s t a sk y o u r s e lf i f th e s u b je c t does t h e a c t io n o f th e v e r b (s o a n
a ctive v e r b is n e e d e d ) o r i f t h e s u b je c t receives th e a c t io n o f th e v e r b (s o a p a ssiv e v e rb is
n e e d e d ). I n th e i n c o r r e c t e x a m p le , y o u s h o u ld stu d y th e m e a n in g o f th e s u b je c t a n d v e rb ,
the letter m ailed. Y o u s h o u ld ask y o u r s e lf i f a letter m ails its e lf (t h e le t t e r does t h e a c t i o n ) o r i f
s o m e o n e m ails a letter ( t h e le t t e r receives th e a c t io n o f b e in g m a ile d ). S i n c e a le t t e r d o e s
n o t m a il itse lf, t h e p a ssive is r e q u ir e d in th is s e n te n c e .

T h e f o llo w in g c h a r t o u d in e s th e d if f e r e n c e in m e a n in g b e tw e e n a c t iv e a n d p a ssive
verbs:

ACTIVE AND PASSIVE MEANINGS


ACTIVE The subject does the action of the verb.
PASSIVE The subject receives the action of the verb.

E X E R C I S E 38: E a c h o f t h e f o llo w in g s e n te n c e s c o n t a in s a t le a st o n e a c t iv e v e r b ; h o w e v e r,
so m e o f th e v e rb s s h o u ld b e p a ssive. U n d e r l in e th e v e rb s tw ice. T h e n in d ic a t e i f t h e se n
te n ce s a r e c o r r e c t ( C ) o r i n c o r r e c t ( I ) .

! 1. T h e car parked in a no-parking zone.

_ 2. T he physics exam began just a few m inutes ago.

-------- 3. Everything to organize the picnic has already don e.

-------- 4. T he police investigated him because o f his unusual actions.

5. T he package containing the necessary sam ples has just sent.


STRUCTUREANDWRITTEN EXPRESSION

6. The vacation to Europe will plan carefully before the scheduled departure date.

7. The coffee turned bitter w hen it left on the stove for so long.

8. The soccer gam e won in the closing m inutes.

9. The clothes m ade to rival the latest fashions o f the season.

10. W hen the roads are icy, the buses do not drive.

E X E R C I S E (S k ills 3 7 - 3 8 ): U n d e r l in e th e v e rb s tw ic e in th e f o llo w in g s e n te n c e s . T h e n in
d ic a te i f th e s e n te n c e s a re c o r r e c t ( C ) o r i n c o r r e c t ( I ) .

_____ 1. After the old radiator had be replaced, the travelers continued their cross-country
trip.

_____ 2. During the lightning storm, he struck in the head by a falling tree.

_____ 3. W hile I am on vacation, the pets should be feeds every m orning and evening.

_____ 4. A book being written now by a team o f writers will be published in the fall.

_____ 5. I found out that the real estate agent had already been leased the condom inium .

_____ 6. The hou se that Mrs. Martin has always wanted to buy has just placed on the market.

_____ 7. The foundation should have been finishing by the construction workers before they
left the construction site.

_____ 8. We m ust leave that m oney in the checking account because the bills pay o n the first
o f the m onth.

_____ 9. The horses can t be taken out now because they have been rode for the past few
hours.

_____ 10. It is being announced by a presidential aide that a lawyer from Virginia has b een
nam ed attorney general.

T O E F L E X E R C I S E (S k ills 3 7 - 3 8 ): C h o o s e th e le t t e r o f th e w o r d o r g r o u p o f w o rd s th a t
b e s t c o m p le te s th e s e n te n c e .

_discussed by the board of directors 3. The X-ray tre a tm e n ts---up to the


when it was proposed again by the th at he w as dism issed from the hospital,
supervisors. (A) gavedaiIy
(A) The problem had already (B) were given daily
(B) The problem is already (C) basically have given
(C) The problem had already been (D) daily had been given
(D) The problem has already

2. M uch of the carnage of elephants, giraffes,


and big c a ts _____ uncaring hunters.
(A) m ust com m it by
(B) m ust be com m itted
(C) m ust have com m itted
(D) m ust have been com m itted by
WRITTEN EXPRESSION

C h o o s e th e le t t e r o f t h e u n d e r lin e d w o r d o r g r o u p o f w o rd s t h a t is n o t c o r r e c t .

4. Particular issues that concern teenagers were covering in the half-hour program.
A B C D

5. Electrical impulses may also picked up by the optic nerve.


A B C D

6. Workers training for a specific job have a strong possibility of being replace by a
A B C D
machine.

7. On June 30, 1992, international timekeepers in Paris were added an extra second to
~A B C
the da
D- ^

8. The report could not be turned in on time because all the needed work lost.
A B C D

. 9. In English these questions have be formed by changing the word order of a


A B C
statement, whereas in som e languages the word order remains the same.
D

.10. He was not able to define the process by which the body had protected by the
A B C D
immunologic system.

T O E F L R E V IE W E X E R C IS E (S k ills 1-3 8): C h o o s e th e le t t e r o f t h e w o r d o r g r o u p o f


w o rd s th a t b e s t c o m p le te s t h e s e n te n c e .

1. _____ Big Dipper, a seven-star 3. impressive chapter in the book was


constellation in the shape of a cup, is part the chapter on Stuarts scientific theories.
of Ursa Major.
(A) It was the most
(A) The (B) The most
(B) It is the (C) Most
(C) With the (D) Most of the
(D) That the

2. The Military Academy at West P oint--------


on the west bank o f the Hudson River,
north of New York City.
(A) located
(B) is located
(C) which is located
(D) whose location is
214 STRUCTURE AND WRITTEN EXPRESSION

C h o o s e th e le t t e r o f th e u n d e r lin e d w o r d o r g r o u p o f w o rd s th a t is n o t c o r r e c t.

--------4. The first fish have appeared on the earth approximately 500 million years ago.
A B C D

-------- 5. Only rarely sound waves are of a single frequency encountered in practice.
A B C D

--------6. Cameos can be carved not only from onyx and sardonyx or from agate.
A B ~C~ TF

-------- 7. Although most of the wild horses in the western range have already been rounded
A B
up, t h e r :O St r e m o t e t h e a r e a , t h e g r e a t e r t h e p o s s i b i l i t y t h a t w i l d h o r s e s c a n s t i l l b e
C D
found.

8. During this period, $206 was spend annually on food by families in the lower third
~~A B D
income bracket.

_ 9. The dangers of noise are, unfortunately, not as clear-cut than are those from
A ~B C
most other health hazards.
D

_10. In a recent survey of Americans, more than 75 percent expressed the view that the
A B
government it should take a more active role in health care.

PROBLEMS W IT H N O U N S .

T h e sa m e types o f p ro b le m s w ith n o u n s a p p e a r o fte n in th e W r it t e n E x p re s s io n s e c tio n o f


th e T O E F L test. Y o u s h o u ld b e f a m ilia r w ith th e s e p r o b le m s so th a t y o u w ill re c o g n iz e
th e m easily. Y o u s h o u ld b e a b le to d o th e f o llo w in g : (1 ) use th e c o r r e c t s in g u la r o r p lu r a l
n o u n , ( 2 ) d is tin g u is h c o u n ta b le a n d u n c o u n t a b le n o u n s , (3 ) re c o g n iz e i r r e g u la r s in g u la r
a n d p lu r a l n o u n s , a n d (4 ) d is tin g u is h th e p e rs o n fro m th e th in g .

S k ill 39: USE T H E CORRECT S IN G U LA R O R PLURAL N O U N

A p r o b le m th a t is c o m m o n in th e W r it t e n E x p re s s io n s e c tio n o f t h e T O E F L test is a sin


g u la r n o u n u s e d w h e r e 'a p lu r a l n o u n is n e e d e d , o r a p lu r a l n o u n u se d w h e r e a s in g u la r
n o u n is n e e d e d .

On the table there were many dish*.


- The lab assistant finished every tots*.

I n th e f ir s t e x a m p le , m any in d ic a te s th a t th e p lu r a l dishes is n e e d e d . I n t h e s e c o n d e x a m
p le , every in d ic a t e s th a t t h e s in g u la r lest is n e e d e d .
WRITTEN EXPRESSION

I n t h e W r i t t e n E x p r e s s io n s e c tio n o f t h e T O E F L test, y o u s h o u ld w a tc h v e r y c a r e f u lly


f o r k e y w o rd s , s u c h as each, every, a, one, a n d single, th a t i n d ic a t e t h a t a n o u n s h o u ld b e sin
g u la r. Y o u s h o u ld a lso w a tc h c a r e f u lly f o r s u c h k e y w o rd s a s m any, several, both, vario u s, a n d
two ( o r a n y o t h e r n u m b e r e x c e p t one) th a t in d ic a t e th a t a n o u n s h o u ld b e p lu r a l.

T h e f o llo w in g c h a r t lists th e k e y w o rd s th a t in d ic a t e to y o u w h e t h e r a n o u n s h o u ld b e
s in g u la r o r p lu r a l:

KEYWORDS FOR SINGULAR AND PLURAL NOUNS

For Singular Nouns each every single one a

For Plural Nouns both two many several various

E X E R C I S E 39: E a c h o f t h e f o llo w in g s e n te n c e s c o n t a in s a t lea st o n e k e y w o r d to te ll y o u


i f a n o u n s h o u ld b e s in g u la r o r p lu r a l. C ir c le th e k e y w o rd s . D r a w a rro w s to t h e n o u n s
th e y d e s c rib e . T h e n in d ic a t e i f th e s e n te n c e s a re c o r r e c t ( C ) o r in c o r r e c t ( I ) .
i y i y
I 1. T h e autom otive shop stocked (man)) part for the (various) types o f H ondas.

2. (Every) receipt must be rem oved from the cashiers drawer and tallied.

3. T h e salesclerk dem onstrated various additional way that the m achine could be used.

4. T h e woman fou n d it difficult to believe that both o f the piece o f jew elry had
disappeared.

5. T he unhappy man becam e m ore and m ore discouraged with each passing days.

6. An exten d ed cruise w ould be a nice way to spend a vacation on e days. ,

7. T he m anager was surprised that not a single worker was available on Tuesday.

8. T h e h ou sekeep er cleaned the room and took two o f the occupant's dress to the
laundry.

9. W hen the first bill was defeated, the Senate im m ediately began work on a different
bills.

10. T here were several b oxes in the cupboard, and each box contained a dozen glasses.

S k il l 40: D IS T IN G U IS H C O U N T A B L E A N D U N C O U N T A B L E NOUNS

I n E n g lis h n o u n s a re c la s s ifie d as c o u n ta b le o r u n c o u n t a b le . F o r c e r t a in q u e s tio n s o n th e


T O E F L test, it is n e c e s s a r y to d is tin g u is h c o u n ta b le a n d u n c o u n t a b le n o u n s in o r d e r to
u se th e c o r r e c t m o d if ie r s w ith th e m .
STRUCTURE AND WRITTEN EXPRESSION

As th e n a m e im p lie s, c o u n ta b le n o u n s are n o u n s th at c a n b e c o u n te d . C o u n ta b le
n o u n s can c o m e in q u a n titie s o f o n e , o r two, o r a h u n d r e d , e tc . T h e n o u n book is c o u n t
ab le b e c a u se y o u can have o n e b o o k o r several book s.
U n c o u n ta b le n o u n s , o n th e o th e r h a n d , are n o u n s th a t c a n n o t b e c o u n te d b e c a u se
th ey c o m e in so m e in d e te r m in a te q u an tity o r m ass. A n o u n su c h as m ilk o r happiness ca n
n o t b e c o u n te d ; you c a n n o t have o n e m ilk o r tw o m ilk s, a n d y o u c a n n o t fin d o n e h a p p i
n e ss o r two h a p p in e ss e s . U n c o u n ta b le n o u n s are o fte n liq u id item s, su c h as water, oil, o r
sham poo. U n c o u n ta b le n o u n s can also refer to abstract id ea s, su c h as security, frien d sh ip , o r
hope.
It is im p o r ta n t fo r you to r e c o g n iz e th e d iffe r e n c e b e tw e e n c o u n ta b le a n d u n c o u n t
a b le n o u n s w h e n y o u c o m e across su ch key w ord s as m u ch a n d many.

H e has seen much* foreign films.


H e d id n t have many* f u n at the movies.

In th e first e x a m p le , m uch is in c o r r e c t b e c a u se film s is c o u n ta b le . T h is s e n te n c e s h o u ld say


m any foreig n film s . In th e s e c o n d e x a m p le , m any is in c o r r e c t b e c a u se f u n is u n c o u n ta b le .
T h is s e n t e n c e sh o u ld say m uch fu n .

T h e fo llo w in g c h a r t lists th e key w ord s th a t in d ic a te to y o u w h e th e r a n o u n sh o u ld b e


c o u n ta b le o r u n c o u n ta b le :

KEYWORDS FOR COUNTABLE AND UNCOUNTABLE NOUNS

For Countable Nouns many number few fewer

For Uncountable Nouns much amount little less

E X E R C ISE 40: E ach o f th e fo llo w in g s e n te n c e s c o n ta in s at lea st o n e k ey w ord to tell y o u


i f a n o u n sh o u ld b e c o u n ta b le o r u n c o u n ta b le . C ircle th e key w ords. D raw arrow s to th e
n o u n s th e y d e sc r ib e . T h e n in d ic a te i f th e s e n te n c e s are c o r r e c t (C ) o r in c o r r e c t (I).
r . .
x1. H e received (little) notice that the bill would have to be paid in full.

2. T he police had (few) opportunities to catch the thief who had com m itted a large

(am ount) o f crimes.

-------- 3. You will have fewer problem s with your incom e taxes if you get professional help.

-------- 4. After the strike, the company dismissed many em ployees.

-------- 5. Because the bottom corner o f the pocket was torn, m uch coins fell out.

-------- 6. Since he bought the new adapter, he has had less trouble with the m achine.

--------7. T here are much new items to purchase before leaving, and there is such a short
am ount o f time.
WRITTEN EXPRESSION 217

8. The less time you take on the assignm ent, the less pages you will com plete.

9. A few soldiers w ho had been in heavy com bat were brought back for a little rest.

10. It is better to go sh opp in g in the late evening because there are less p eo p le in the
market, and you can accom plish a num ber o f tasks in a short period o f time.

S k il l 4 1: R E C O G N IZ E IRREGULAR PLURALS OF N O U N S

M any n o u n s in E n g lish h ave irreg u la r p lu rals, an d th e se irreg u la r fo r m s c a n c a u se c o n fu


sio n in th e W ritten E x p r e ssio n s e c tio n o f th e T O E F L test. T h e irr eg u la r fo r m s th a t are
th e m o st p r o b le m a tic are p lu ral fo r m s th a t d o n o t e n d in 5.

D ifferent criteria was* used to evaluate the performers.

In this e x a m p le th e plu ral n o u n criteria lo o k s sin g u la r b e c a u se it d o e s n o t e n d in s; you


m ig h t in c o r r e c tly a ssu m e th a t it is sin g u la r b e c a u se th e r e is n o fin al 5. H ow ever, criteria is a
plu ral n o u n , so th e sin g u la r verb w as used is in c o r r e c t. T h e verb sh o u ld b e th e p lu ra l fo r m
were used.

T h e fo llo w in g c h a r t lists th e irr eg u la r plu rals th at y o u sh o u ld b e c o m e fa m ilia r with:

IRREGULAR PLURALS

Vowel change man / men foot 1feet goose / geese


woman / women tooth 1teeth mouse / mice
Add -en child / children ox / oxen

Same as singular deer / deer salmon / salmon trout / trout


fish / fish sheep / sheep

-is ---- --ES analysis / analyses diagnosis / diagnoses synthesis / syntheses


axis / axes hypothesis / hypotheses thesis / theses
crisis / crises parenthesis 1parentheses
Ends in -A bacterium / bacteria datum / data criterion / criteria
curriculum 1curricula phenomenon / phenomena
-US -/ alumnus / alumni fungus / fungi stimulus / stimuli
bacillus / bacilli nucleus / nuclei syllabus / syllabi
cactus 1cacti radius / radii

NOTE: Additional exercises to practice these irregular plurals of nouns appear in


Appendix H at the back of the text. You may want to complete these exercises before you
begin Exercise 4 1.
18 STRUCTURE AND WRITTEN EXPRESSION

E X E R C ISE 41: Each o f th e fo llo w in g s e n te n c e s c o n ta in s at lea st o n e n o u n w ith an ir r e g u


lar plu ral. C ircle th e n o u n s w ith irreg u la r plurals. T h e n in d ic a te if th e s e n te n c e s are c o r
r ec t (C ) o r in c o r r e c t (1).

1. (Parentheses)is n eed ed around that expression.

< 2. He wants to go on a fishing trip this weekend because he has heard that the (fish) are
running.

3. The syllabi for the courses is included in the packet o f materials.

4. T he diagnosis that h e heard today were not very positive.

5. T he crisis is not going to be resolved until som e o f the pressure is relieved.

6. All o f the alumni are attending the reception at the presidents house.

7. A flock o f geese were seen heading south for the winter.

8. T he teeth in the back o f his m outh needs to be capped.

9. The fungi has spread throughout the garden.

10. T he sheepdog is chasing after the sheep which are heading over the hill.

S k i l l 42: D IS T IN G U IS H TH E PERSON FROM T H E T H IN G

N o u n s in E n g lish can r efe r to p e r so n s or th in g s. S o m e tim e s in th e W ritten E x p r e ssio n


s e c tio n o f th e T O E F L test th e p e r so n is u s e d in p la c e o f th e th in g , o r th e th in g is u s e d in
p la c e o f th e p e r so n .

Ralph Nader is an authorization* in the field o f consum er affairs.


T here are many jo b opportunities in accountant*.

In th e first e x a m p le , authorization is in c o r r e c t b e c a u se authorization is a th in g an d R alph


N a d e r is a p e r so n . T h e p e r so n authority sh o u ld b e u s e d in th is s e n t e n c e . In th e s e c o n d e x
a m p le , acco u n ta n t is in c o r r e c t b e c a u se accountant is a p e r so n a n d th e fie ld in w h ic h an ac
c o u n ta n t w orks is accounting. T h e th in g accounting sh o u ld b e u se d in th is s e n te n c e .

T h e fo llo w in g ch a rt o u tlin e s w h a t you sh o u ld r e m e m b e r a b o u t th e p e r s o n o r thing:

PERSON OR THING
1. It is common to confuse a person with s thing in the Written Expression section of the TOEFL
test.
2. This type of question generally appears near the end of the Written Expression section.

WRITTEN EXPRESSION 2 19

E X E R C ISE 42: S o m e o f th e fo llo w in g s e n te n c e s c o n ta in in c o rr ec tly u s e d persons o r things.


C ircle th e in c o r r e c tly u s e d w ord s. T h e n in d ic a te if th e s e n t e n c e s are c o r r e c t (C ) o r in c o r
rect (I ).

1. In the evening he relaxes in front o f the fire and writes lo n g (p o e ts)

2. Service in the restaurant was slow because o n e cook had called in sick.

3. T h e sculpture worked from sunrise until sunset o n his new project.

4. She has received several awards for her research in engineer.

5. T he econom ist's radical views were printed in a colum n in the Sunday newspaper.

6. You must have remarkable looks to work as a m odel for Vogue.

7. H e had several critics to offer about the new play.

8. T h e gardener worked feverishly after the frost to save as many plants as possible.

9. T h e com pany hired a statistic to prepare marketing studies for the new product.

10. T h e fam ous acting has appeared in m ore than fifty Broadway plays.

E X E R C ISE (S k ills 3 9 - 4 2 ) : S tu d y th e n o u n s in th e fo llo w in g s e n te n c e s . T h e n in d ic a te i f


th e s e n te n c e s are c o r r e c t (C ) o r in c o r r e c t (I).

-------- 1. T h e professor d oes not give many exam in chem istry class, but the on es sh e gives are
difficult.

-------- 2. His thesis includes an analyses o f the hypotheses.

-------- 3. It was his dream to be a musical in the New York Philharm onic.

-------- 4. For the reception, the caterers prepared a large amc unt o f food to serve a large
num ber o f people.

-------- 5. Many job opportunities exist in the field o f nurse if you will accept a low-paying
position.

-------- 6. For each business trip you make, you can choose from many d ifferen t airlines.

-------- 7. T h e stim ulus for his career change is his acknow ledgm ent that he is in a d ead-end
job .

-------- 8. She wants to undergo a series o f treatments, but she thinks it costs a little too m uch
money.

-------- 9. T he television producer that was shown last nigh t on the CBS network from 9:00 to
11:00 was o n e o f the best shows o f the season.

_____ 10. Various sight-seeing excursion were available from the tourist agency.
220 STRUCTURE AND WRITTEN EXPRESSION

T O E F L E X E R C ISE (S k ills 3 9 - 4 2 ) : C h o o se th e lette r o f th e u n d e r lin e d w ord o r g r o u p o f


w ord s th a t is n o t c o rr ec t.

_____ 1. As a compilation of useful details, a weekly magazine commends itself in several


-------- A-------- B C
respect.
D

_____ 2. Through aauaculture, or fish farming, more than 500 million tons of fish
A B
are produced each years.
C ~ET"

_____ 3 The legal system has much safeguards to protect the right o f a defendant to an
A B C
impartial jury.
D

_____ 4. The mystery bookstore was largely a phenomena of the last decade.
~A~ B C D

_____ 5 . The Song o f Hiawatha, by Longfellow, tells the story o f the Indian heroism who
A B C
married Minehaha.

. 6. Uranus is the seventh planets from the Sun.


a ! c d

. 7. The sycamore has broad leaves with a large amount of pointed teeth.
A B C D

8. The first of two such investigation requires the students to read continuously over a
A B
period of four hours.
C D

_ 9. A quantitative analysis, using both the computer and quantitative techniques,


A B
are used to optimize financial decisions.
C D

_10. To enter the FBI National Academy, an application must be between the ages of
A B C D
twenty-three and thirty-four.
WRITTEN EXPRESSION

T O E F L REV IEW E X E R C ISE (S k ills 1 -4 2 ): C h o o se th e lette r o f th e w o r d o r g r o u p o f


w ord s th a t b e st c o m p le te s th e se n te n c e .

. The population of the earth is increasing at


1. Presidential____held every four years on
a tremendous rate a n d ------------------out of
the first Tuesday after the first Monday in
control.
November.
(A) they have become
(A) electing
(B) are soon going to be
(B) elections are
(C) soon will be
(C) is elected
(D) why it will be
(D) elected and

2. Studies of carcinogenesis in animals can Starting in 1811, traders and


manufacturers were more easily able to
provide data o n --------in human
send goods upriver in ______ provided the
susceptibility.
necessary power to counteract the flow of
(A) differences are the waters.
(B) that differences are
(C) differences have (A) steamboats
(D) differences (B) which
(C) that
3. Those who favor the new law say that the (D) that steamboats
present law does not set spending limits on
lobbyists' gifts to politicians, n o r --------
statewide funds.
(A) it limits
(B) limits it
(C) does it limit
(D) does it

C h o o se th e le tte r o f th e u n d e r lin e d w o r d o r g r o u p o f w o rd s th at is n o t c o r r e c t.

_____ 6. Temperature indicates on a bimetallic thermometer by the amount that the


A B C D
bimetallic strip bends.

_____ 7. Many of the food consumed by penguins consists of fish obtained from the ocean.
A B C D

_____ 8. Before the newspaper became widespread, a town crier has walked throughout a
A "B C"
village or town singing out the news.
D

_____ 9. All of NASA's manned spacecraft project are headquartered at the Lyndon B.
A B C D
Johnson Space Center in Houston.

.10. Fungi cause more serious plant diseased than do other parasites.
A B C D
STRUCTURE AND WRITTEN EXPRESSION

PROBLEMS W IT H P R O N O U N S __________________ ____________

P r o n o u n s are w ord s, su ch as he, she, o r it, that take th e p la c e o f n o u n s. W h e n y o u s e e a


p r o n o u n in th e W ritten E x p ressio n se c tio n o f th e T O E FL test, y o u n e e d to c h e ck th a t it
serv es th e c o r r e c t fu n c tio n in th e s e n te n c e (as a su b ject o r o b ject, fo r e x a m p le ) a n d that
it a g r ee s w ith th e n o u n it is rep la cin g . T h e fo llo w in g p r o n o u n p r o b le m s are th e m o st
c o m m o n o n th e T O E FL test: (1) d istin g u ish in g su b ject a n d o b je c t p r o n o u n s , (2) d istin
g u ish in g p o sse ssiv e p r o n o u n s an d possessive adjectives, a n d (3 ) c h e c k in g p r o n o u n refer
e n c e fo r a g r e e m e n t.

S k ill 43: D IS T IN G U IS H SUBJECT A N D OBJECT PR O N O U N S

S u b ject a n d o b je c t p r o n o u n s can b e c o n fu se d o n th e T O E FL test, so y o u sh o u ld b e able


to r e c o g n iz e th e se tw o types o f p ro n o u n s:

SUBJECT OBJECT
/ me
you you
he him
she her
it it
we us
they them

A su b je ct p r o n o u n is u sed as th e su b ject o f a verb. A n o b je c t p r o n o u n can b e u sed as the


o b je c t o f a verb o r th e o b je c t o f a p r e p o sitio n . C o m p a re th e fo llo w in g two se n te n c e s .

Sally gave the book to John.


\ s'S
She gave it to him.

In th e s e c o n d s e n te n c e th e su b ject p r o n o u n she is r e p la c in g th e n o u n Sally. T h e o b je c t o f


th e verb it is r ep la c in g th e n o u n book, a n d th e o b je c t o f th e p r e p o sitio n h im is r ep la c in g
th e n o u n Jo h n . *
T h e fo llo w in g are e x a m p le s o f th e types o f su b ject o r o b je c t p r o n o u n erro rs th a t you
m ig h t se e o n th e T O E FL test.

Him* and the girl are going shopping.


T he gift was intended for you and I*.

In th e first e x a m p le , th e o b je c t p r o n o u n him is in c o r r e c t b e c a u se th is p r o n o u n serv es as


th e su b ject o f th e se n te n c e . T h e o b ject p r o n o u n him sh o u ld b e c h a n g e d to th e su b ject
p r o n o u n he. It ca n b e d ifficu lt to r e c o g n iz e that him is th e su b ject b e c a u se th e verb are has
a d o u b le su b ject, h im a n d girl. In th e se c o n d e x a m p le , th e su b ject p r o n o u n I is in c o r r e c t
b e c a u se th is p r o n o u n serv es as th e o b ject o f th e p r e p o sitio n for. T h e su b je ct p r o n o u n I
sh o u ld b e c h a n g e d to th e o b je c t p r o n o u n me. It ca n b e d ifficu lt to r e c o g n iz e th a t / is th e
o b je c t o f th e p r e p o sitio n f o r b e c a u se th e p r e p o sitio n fo r h a s two objects: th e c o r r e c t o b
j e c t you a n d th e in c o r r e c t o b je c t I.
WRITTEN EXPRESSION

EX E R C ISE 43: E a ch o f t h e fo llo w in g s e n te n c e s c o n ta in s at least o n e su b je c t o r o b je c t p r o


n o u n . C ir cle th e p r o n o u n s . T h e n in d ic a te i f th e s e n t e n c e s are c o r r e c t (C ) o r in c o r r e c t
(I).

1. T he worst problem with (it) is that (he) cannot afford (it)

! 2. (They) saw Steve and (T) at the m ovies last night after class.

-------- 3. Perhaps you would like to g o to the seminar with they and their friends.

-------- 4. T he m other took her son to the doctors office because h e was feelin g sick.

-------- 5. I did not know that you and her were working together on the project.

-------- 6. She did not buy the sweater because it had a small hole in it.

7. T he man lead in g the sem inar gave me all the inform ation I n eed ed to make a
decision.

8. T h e cards connectin g the com puter to its printer need to be replaced before them
wear down.

9. H e is goin g to the party with you and me if you do not m ind.

10. You and her ou gh t to return the books to the library because they are already
overdue.

S k ill 44: D IS T IN G U IS H POSSESSIVE ADJECTIVES A N D P R O N O U N S


P ossessive a d jectiv es a n d p r o n o u n s b o th sh ow w h o o r w h at o w n s a n o u n . H o w ev er, p o s
sessive a d jectiv es a n d p o sse ssiv e p r o n o u n s d o n o t have th e sa m e fu n c t io n , a n d t h e s e tw o
k in d s o f p o sse ssiv e s can b e c o n fu s e d o n th e T O E F L test. A p o sse ssiv e a d je c tiv e d e sc r ib e s a
n o u n : it m u st b e a c c o m p a n ie d by a n o u n . A p ossessive p r o n o u n takes th e p la c e o f a n o u n :
it c a n n p t b e a c c o m p a n ie d by a n o u n .

1 t
T hey lent m e their book.
ADJECTIVE

T hey lent m e theirs.


PRO N O U N

N o tic e th a t in th e first e x a m p le th e p o ssessiv e ad jective their is a c c o m p a n ie d by th e n o u n


book. In th e s e c o n d e x a m p le th e p o sse ssiv e p r o n o u n iheirs is n o t a c c o m p a n ie d b y a n o u n .
T h e s e e x a m p le s sh o w th e typ es o f e rro rs th a t are p o s sib le w ith p o sse ssiv e a d je c tiv e s
a n d p o sse ssiv e p r o n o u n s o n th e T O E F L test.

Each m orning they read theirs* newspapers.


C ould you give m e your*}
224 STRUCTURE AND WRITTEN EXPRESSION

In th e first e x a m p le , th e p ossessive p r o n o u n theirs is in c o r r e c t b e c a u se it is a c c o m p a n ie d


by th e n o u n newspapers, an d a p o ssessiv e p r o n o u n c a n n o t b e a c c o m p a n ie d by a n o u n . T h e
p o ssessiv e a d jectiv e their is n e e d e d in th e first e x a m p le . In th e s e c o n d e x a m p le , th e p o s
sessive a d jectiv e y o u r is in c o r r e c t b e c a u se it is n o t a c c o m p a n ie d by a n o u n , a n d a p o sses
sive a d jectiv e m u st b e a c c o m p a n ie d by a n o u n . T h e p o sse ssiv e p r o n o u n yours is n e e d e d in
th e s e c o n d e x a m p le .

T h e fo llo w in g c h a r t o u tlin e s th e p o sse ssiv e s an d th e ir uses:

POSSESSIVE POSSESSIVE
ADJECTIVES PRONOUNS

my mine
your yours
his his
her hers
its
our ours
their theirs

must be accompanied by a noun cannot be accompanied by a noun

E X E R C ISE 44: E ach o f th e fo llo w in g s e n te n c e s c o n ta in s at le a s t o n e p o ssessiv e p r o n o u n


or ad jective. C ircle th e p ossessives in th e se se n te n c e s . T h e n in d ic a te i f th e s e n te n c e s are
c o r r e c t (C ) o r in c o r r e c t (I).

I 1. If she borrows(ycmr) coat, then you should be able to borrow (her)

C 2. Each pot and pan in (h) kitchen has (its) own place on the shelf.

_____ 3. Mary and Mark invited theirs parents to see their new apartment.

_____ 4. W hen my room mate paid her half o f the rent, I paid m ine.

_____ 5. All students need to bring theirs own pencils and answer sheets to the exam.

_____ 6. All her secretaries are working late tonight to finish her report.

_____ 7. T he horse trotting around the track won its race a few m inutes ago.

_____ 8. Before the report is finalized, the inform ation in their notes and our m ust be
proofed.

_____ 9. She worked all day cooking food and making decorations for her son s birthday
party.

_____ 10. T he weather in the m ountains this w eekend will be extrem ely cold, so please take
yours heavy jackets.
WRITTEN EXPRESSION

S k ill 45: C H E C K P R O N O U N REFERENCE FOR AG R EEM EN T

A fter y o u have c h e c k e d th a t th e su b je ct a n d o b je c t p r o n o u n s a n d th e p o sse ssiv e s are u sed


correctly, y o u sh o u ld a lso c h e c k e a c h o f th e se p r o n o u n s a n d p o sse ssiv e s fo r a g r e e m e n t.
T h e fo llo w in g are e x a m p le s o f e rro rs o f th is type th at y o u m ig h t fin d o n th e T O E F L test:

T h e boys will cause trouble if you let him*.


Everyone m ust give their* nam e.

In th e first e x a m p le , th e sin g u la r p r o n o u n h im is in c o r r e c t b e c a u s e it r e fe r s to th e plu ral


n o u n boys. T h is p r o n o u n s h o u ld b e r e p la c e d w ith th e p lu ra l n o u n them. In th e s e c o n d e x
a m p le , th e p lu ra l p o ssessiv e a d jectiv e their is in c o r r e c t b e c a u s e it r efe rs to th e sin gu lar
everyone. T h is a d jectiv e s h o u ld b e r e p la c e d w ith th e sin g u la r his o r his or her.

T h e fo llo w in g c h a r t o u tlin e s w h at y o u s h o u ld r e m e m b e r a b o u t c h e c k in g p r o n o u n
r e fe r e n c e :

PRONOUN AGREEMENT
1. Be sure that every proncun and possessive agrees with the
th< noun it refers to.
A r,[l:
2. You generally check back in the sentence for agreement.

E X E R C ISE 45: E ach o f th e fo llo w in g s e n te n c e s c o n ta in s at le a s t o n e p r o n o u n o r p o sses


sive. C ir cle th e p r o n o u n s a n d p o ssessiv es. D raw arrow s to th e n o u n s th e y r efe r to. T h e n
in d ic a te i f th e s e n t e n c e s are c o r r e c t (C ) o r in c o r r e c t (I).

! 1 If a person really wants to succeed, (they) must always work hard.

C 2. If you see the students from the math class, could you return (their) exam papers to
(them?)

3. Som e friends and I w ent to see a movie, and afterwards we wrote a critique about
them.

4. If you have a problem , you are welcom e to discuss it with m e before you u y to
resolve them.

5. I know you had a terrible time last week, but you m ust try to forget about it.

6. At the start o f the program , each student n eeds to see his advisor about his schedule.

7. In spite o f its small size, these video recorders produce excellen t tapes.

8. W hatever the situation, you should reflect profoundly abou t them before com ing to
a decision.

9. T he p eop le 1 admire m ost are those who m anage to solve their own problems.

10. If anyone stops by w hile I am at the m eeting, please take a m essage from them.
STRUCTURE AND WRITTEN EXPRESSION

E X ER C ISE (Sk ills 4 3 - 4 5 ) : C ircle th e p r o n o u n s a n d p ossessives in th e fo llo w in g se n


te n c e s. T h e n in d ic a te i f th e s e n te n c e s are c o r r e c t (C ) o r in c o r r e c t ( I ) .

_____ 1. H elicopters are being used m ote and m ore in em ergency situadons because o f its
ability to reach out-of-the-way places.

_____ 2. The worker was fired by the chem ical com pany because his refused to work with
certain dangerous chemicals.

_____ 3. If you have car trouble while driving on the freeway, you should pull your car over to
the side o f the freeway and wait for help.

_____ 4. The administration will n o t install the new security system because they cost so
much.

_____ 5. Some parents prefer to send their children to private schools because they believe
the children will be better educated.

_____ 6. The air traffic controller was not blam ed for the accident because he had strictly
follow ed the correct procedures.

_____ 7. The new student has b een assigned to work on the project with you and I.

_____ 8. Many different kinds o f aspirin are on the market, but theirs effectiveness seem s to
be equal.

_____ 9. You must bring a tent and a sleep in g bag for your trip to the Sierras.

_____ 10. Each o f the team m em bers had their new uniform.

T O E F L E X ER C ISE (S k ills 4 3 - 4 5 ) : C h o o se th e le tte r o f th e u n d e r lin e d w ord or g r o u p o f


w ord s th at is n o t co rr ec t.

_____ 1. Superman made their com ic debut in 1938 in Action Comics.


A B- C D

_____ 2 Commercial letters of credit are often used to finance export trade, but them can
A B C
have other uses.
D

_____ 3 When children experience too much frustration, its behavior ceases to be integrated.
A B C D

_____ 4 On March 30, 1981, President Reagan was shot as his was leaving a Washington hotel.
A B C D

_____ 5. Although the destruction that it causes is often terrible, cyclones benefit a
-----X----- B
much wider belt than they devastate.
WRITTEN EXPRESSION

6. President Andrew Jackson had an official cabinet, but him preferred the advice o f
A ~B C
his informal advisors, the Kitchen Cabinet.
D

7. After Clarence Days book Life w ith Father was rewritten as a play, they ran for
A B ~C~
six years on Broadway.
D

8. Almost half of the Pilgrims did not survive theirs first winter in the New World.
A B ~C D-

9. There was no indication from the Senate that he would agree with the decision made
A B ~C ~D ~
in the House.

_10. A baby learns the meanings of words as they are spoken by others and later uses him
~A~ B ~C~ ~D~
in sentences.

T O E F L REVIEW E X E R C ISE (S k ills 1 - 4 5 ) : C h o o se th e le tte r o f th e w o r d o r g r o u p o f


w ord s th a t b e st c o m p le te s th e s e n t e n c e .

1. ____worst phase of the Depression, more 3- Speech consists not m erely o f sounds but
than thirteen million Americans had no -------- that follow various structural
jobs. patterns.
(A) It was in the (A) o f organized sound patterns
(B) During the (B) organized sound patterns
(C) While the (C) that sound patterns are organized
(D) The (D) *n organizing sound patterns

2. When reading a book, you must keep your


point of view separate from the point o f
view i n _____ you are studying.
(A) that
(B) the material and
(C) the materials that
(D) the materials that are

C h o o se th e le tte r o f th e u n d e r lin e d w o rd o r g r o u p o f w o rd s th a t is n o t c o r r e c t.

_____ 4. The latest medical report indicated that the patient's temperature was near normal
A B
and their lungs were partially cleared.
~C~ D-

5. Most oxygen atom s have eight neutrons, but a small amount have nine or ten .
A ~B ~ C D

6. When Paine expressed his belief in independence, he praised by the public.


_A~ B C 5
STRUCTURE AND WRITTEN EXPRESSION

____ 7. A vast quantity of radioactive material is made when does a hydrogen bomb explode.
A B C D

_____ 8. Genes have several alternative form , or alleles, which are produced by mutations.
A B C D

_____ 9. A star that has used up its energy and has lost its heat became a black dwarf.
~A~ IT C D

_____ 10. Each lines of poetry written in blank verse has ten syllables, which are alternately
A- B C D
stressed and unstressed.

PROBLEMS W IT H ADJECTIVES A N D ADVERBS______________

M any d iffe r e n t p r o b le m s w ith a d jectiv es a n d adverbs are p o ssib le in th e W ritten E xp res


sio n se c tio n o f d ie T O E F L test. To id e n tify th e s e p r o b le m s, y o u m u st first b e a b le to r e c
o g n iz e adjectives an d adverbs.
O fte n adverbs are fo r m e d by a d d in g -ly to ad jectives, a n d th e se -ly adverbs are very
easy to r e c o g n iz e . T h e fo llo w in g e x a m p le s sh o w ad verb s that are fo r m e d by a d d in g -ly to
adjectives:

ADJECTIVE ADVERB

recent recently
public publicly
evident evidently

H ow ever, th e r e are m a n y ad verb s in E n g lish th a t d o n o t e n d in -ly. T h e s e ad verb s ca n b e


r e c o g n iz e d fr o m th eir m e a n in g s. T h e y can d e sc r ib e w hen so m e th in g h a p p e n s (often, soon,
later), how s o m e th in g h a p p e n s (fa st, hard, well), or where s o m e th in g h a p p e n s (here, there,
nowhere).
T h e r e are th r e e sk ills in v o lv in g a d jectiv es a n d adverbs th a t w ill h e lp y o u o n th e W rit
te n E x p ressio n se c tio n o f th e T O E F L test: (1 ) k n o w in g w h e n to u se ad jectiv es an d ad
verbs, (2) u s in g a d jectives ra th er th a n ad verb s after lin k in g verb s, a n d (3) p o s itio n in g
ad jectives an d adverbs correctly.

S kill 46: USE BASIC ADJECTIVES A N D ADVERBS CORRECTLY

S o m e tim e s in th e W ritten E x p r e ssio n se c tio n o f th e T O E F L test, ad jectives a re u s e d in


p lace o f adverbs, o r adverbs are u s e d in p la c e o f adjectives. A d jectiv es a n d ad verb s h ave
very d iffe r e n t u ses. A d jectives h ave o n ly o n e jo b : they d e sc r ib e n o u n s o r p r o n o u n s .

She is a beautiful w om an.


ADJ. NOUN

She is beautiful.
PRO ADJ
WRITTEN EXPRESSION

In th e first e x a m p le , th e a d je c tiv e bea u tifu l d e sc r ib e s th e n o u n w om an. In th e s e c o n d e x


a m p le , th e a d je c tiv e b e a u tifu l d e sc r ib e s th e p r o n o u n she.
A d verb s d o th r e e d iffe r e n t th in g s. T h e y d e sc r ib e verb s, a d jectiv es, o r o th e r adverb s.

f 1 ,
She sings beautifully.
VERB ADV.

I \ I t
She is a beautifully dressed woman.
ADV. ADJ. N OU N

I } I ^ I ^
She is a truly beautifully dressed woman.
ADV. ADV. ADJ. NOUN

In th e first e x a m p le , th e ad verb beautifully d e sc r ib es th e verb sings. In th e s e c o n d e x a m p le ,


th e adverb beautifully d e sc r ib e s th e a d jectiv e dressed (w h ic h d e sc r ib e s th e n o u n w om an). In
th e th ird e x a m p le , th e ad verb truly d e sc r ib e s th e adverb beautifully, w h ic h d e sc r ib e s th e
ad jective dressed (w h ic h d e sc r ib e s th e n o u n woman).
T h e fo llo w in g are e x a m p le s o f in c o r r e c t s e n te n c e s as th ey m ig h t a p p e a r o n th e
T O E F L test.
. I t
They were seated at a largely* table.
NOUN
j I
T h e child talked quick* to her mother.
VERB ADJ.

I t
We read an extreme* long story.
ADJ. ADJ.

In th e first e x a m p le , th e ad verb largely is in c o r r e c t b e c a u se th e a d je c tiv e large is n e e d e d to


d e sc r ib e th e n o u n table. In th e s e c o n d e x a m p le , th e ad jective quick is in c o r r e c t b e c a u se
th e adverb quickly is n e e d e d to d e sc r ib e th e verb talked. In th e last e x a m p le , th e ad jective
extreme is in c o r r e c t b e c a u se th e adverb extremely is n e e d e d to d e sc r ib e th e a d je c tiv e long.

T h e fo llo w in g c h a r t o u tlin e s th e im p o r ta n t in fo r m a tio n th at y o u s h o u ld r e m e m b e r


a b o u t th e b a sic u se o f a d jectiv es a n d adverbs:

BA SIC U SE O F A D JEC T IV ES A N D A D VERBS

ADJECTIVES ' A d je ctive s describe nouns o r pronouns.

ACWER&S A d ve rb s d escribe verbs, adjectives, o r o th e r adverbs.


STRUCTUREANDWRITTEN EXPRESSION

EX E R C ISE 46: E ach o f th e fo llo w in g s e n te n c e s h a s at le a s t o n e a d je c tiv e o r ad verb . C ircle


th e ad jectiv es a n d adverb s, a n d la b e l th e m . D raw arrow s to th e w o r d s th e y d e sc r ib e . T h e n
in d ic a te i f th e s e n t e n c e s are c o r r e c t (C ) o r in c o r r e c t (I ).

1. T h e m other was(pleasan (surprised) w hen her daughter cam e to visit.


ADJ. ADJ.

i ^
2. T he salespeople (frequently) visit the East Coast for trade shows.
ADV. VERB

3. H e was driving an expensively sports car.

4. T here is a special program on television this evening.

5. She was chosen for the leading part because she sings so well.

6. T he car was n o t com plete ready at 3:00.

7. It was difficult to believe that what we read in the newspaper was a truly story.

8. Points will be subtracted for each incorrect answered question.

9. T he production m anager quietly requested a com pletely report o f the terribly


incident.

10. The children finished their hom ework quickly so they could watch television.

S k ill 47: USE ADJECTIVES AFTER L IN K IN G VERBS

G en er a lly an ad verb r a th e r th an an ad jective w ill c o m e d ir ec d y after a verb b e c a u se th e


adverb is d e sc r ib in g th e verb.

f
She spoke nicely.
1
VERB ADV.

In th is e x a m p le , th e verb spoke is fo llo w e d by th e adverb nicely. T h is ad verb d e sc r ib e s th e


v erb spoke.
H ow ever, y o u m u st b e v e ry c a re fu l i f th e verb is a lin k in g verb . A lin k in g v erb is fo l
lo w ed by an ad jective r a th e r th a n an adverb.

t 1
She looks nice.
SUB. ADJ.

In th is e x a m p le , th e lin k in g verb looks is fo llo w e d by th e ad jective nice. T h is ad jective d e


scrib es th e su b je c t she.
WRITTEN EXPRESSION

You sh o u ld b e su re to u se a n a d je c tiv e ra th er th an an adverb a fter a lin k in g verb . B e


c a re fu l, h ow ever, b e c a u se th e a d jectiv e th a t g o e s w ith th e lin k in g verb d o e s n o t always d i
rectly fo llo w th e lin k in g verb.

1 I tl
H e seem s unusually nice.
SUB. adv. ADJ.

In this e x a m p le , th e a d je c tiv e nice, w h ic h d e sc r ib e s th e su b je c t he, is its e lf d e s c r ib e d by th e


adverb u n u su a lly. F rom th is e x a m p le , y o u sh o u ld n o tic e th a t it is p o s sib le to h a v e an a d
verb d irectly a fter a lin k in g verb , b u t o n ly i f th e ad verb d e sc r ib e s a n a d je c tiv e th a t follow s.

T h e fo llo w in g c h a r t lists c o m m o n ly u s e d U n king verb s a n d o u tlin e s th e d iffe r e n t u ses


o f ad jectives a n d ad verb s a fter reg u la r v erb s a n d lin k in g verbs:

ADJECTIVES AND ADVERBS AFTER VERBS

f '
(subject) + (regular verb) + (adverb)

A regular verb is followed by an adverb. The adverb describes the verb.

f _ ' V
(subject) + (linking verb) + (adjective)

A linking verb is followed by an adjective. The adjective describes the subject.

f ' I t I
(subject) + (linking verb) + (adverb) + (adjective)
It is possible that a linking verb is followed by an adverb and an adjective. The adverb describes
the adjective and the adjective describes the subject.

UNKING VERBS: appear feel seem


be look smell
become prove taste

EX E R C ISE 47: E a ch o f th e fo llo w in g s e n te n c e s c o n ta in s at lea st o n e a d je c tiv e o r adverb.


C ircle th e a d jectiv es a n d ad verb s, a n d la b e l th e m . D raw arrow s to th e w o r d s th ey d e sc r ib e.
T h e n in d ic a te i f th e s e n te n c e s are c o r r e c t (C ) o r in c o r r e c t (I).

f ^ 1 ___v
J__ 1. T he parents seem (angrily) about the childs report card.
ADV.

c 2. T h e speaker talked (knowingly) about (prehistoric) fossils.


ADV. ADJ.

___ 3. After she drank the lem onade, the cake tasted too sweetly to her.

___ 4. T h ro u g h o u t d in n e r we w ere b o re d because h e spoke incessantly.

5. Sam felt terribly depressed after the accident.


STRUCTURE ANDW RITTEN EXPRESSION

6. T h e n eighb or appeared calm in spite o f the fact that his house was on fire.

7. H e looked quite unhappily at the thought o f leaving his job.

8. Marla ju m p ed up quick w hen she heard the gunshot.

9. Even though we were not really hungry, the food sm elled delicious.

10. T h e history course that I took last sem ester proved m ore difficultly than I had
expected.

S k il l 48: P O S IT IO N ADJECTIVES A N D ADVERBS CORRECTLY

A d jectiv es a n d a d v erb s can a p p ea r in in c o r r e c t p o s itio n s in th e W ritten E x p r e ssio n se c


tio n o f th e T O E F L test. T h e r e are tw o c o m m o n errors o f th is type th a t y o u sh o u ld bew are
of: (1 ) th e p o s itio n o f a d jectives w ith th e n o u n s th ey d e sc r ib e , a n d (2 ) th e p o s itio n o f ad
verb s w ith o b je c ts.
In E n g lish it is c o r r e c t to p la c e a o n e-w o rd a d jectiv e in fr o n t o f th e n o u n it d escrib es.
O n th e T O E F L test, h ow ever, an in c o r r e c t s e n te n c e m ig h t have an a d jectiv e after th e
n o u n it d e sc r ib es.

T he inform ation important* is on the first page.


N OU N ADJ.

In th is e x a m p le , th e a d jectiv e im p o rta n t sh o u ld c o m e b e fo r e th e n o u n inform ation, b e


c a u se im p o rta n t d e sc r ib e s inform ation.
A s e c o n d p r o b le m you s h o u ld b e aware o f is th e p o s itio n o f ad verb s w ith o b jects o f
verb s. W h e n a verb h a s an o b je c t, an adverb d e sc r ib in g th e v erb sh o u ld n o t c o m e b etw een
th e verb a n d its o b je c t.

H e has taken recently* an English course.


ADV. OBJECT

T h is e x a m p le is in c o r r e c t b e c a u se th e adverb recently c o m e s b e tw e e n th e verb has taken


a n d its o b je c t a n E n g lish course. T h e r e are m a n y p o ssib le c o r r e c tio n s fo r th is s e n te n c e .

Recently he has taken an English course.


He has recently taken an English course
H e has taken an English course recentiy.

You ca n s e e fr o m th e s e e x a m p le s th a t th ere are m an y p o ssib le c o r r e c t p o s itio n s fo r th e


ad verb . W h a t is im p o r ta n t fo r you to r em e m b e r is th a t an adverb th a t d e sc r ib es a verb
c a n n o t c o m e b e tw e e n th e verb a n d its ob ject.
WRITTEN EXPRESSION

T h e fo llo w in g c h a r t o u tlin e s th e key p o in ts th a t y o u s h o u ld r e m e m b e r a b o u t th e p o si


tio n o f a d jectiv es a n d adverbs: p

: ,

THE POSITION OF ADJECTIVES AND ADVERBS
ADJECTIVES
A one-word adjecVve comes before the noun it describes. It does not come
directly after.

ADVERBS
An adverb can appear in many positions. It cannot be used between a verb and
its object.

E X E R C ISE R S: E a ch o f th e fo llo w in g s e n te n c e s c o n ta in s at least o n e a d je c tiv e o r adverb.


C ircle th e a d jectiv es an d adverb s, a n d la b el th em . D raw arrow s to th e w ord s th ey d e sc r ib e .
T h e n in d ic a te i f th e s e n t e n c e s are c o r r e c t (C ) o r in c o r r e c t (I).

I f ~ I ^
-------- 1. The store op en ed with a sale (fantastic)
ADJ.

2. The pharm acist has (S l w ^ ) filled our order (quickly)


ADV. ADV.

-------- 3. The political candidates expressed their opposing views.

-------- 4. T he lawyer has selected carefully a new case.

-------- 5. Frequently the coffee has tasted bitter.

-------- 6. The w edding reception was held at a restaurant expensive.

-------- 7. The salesclerk has often traveled to New York.

-------- 8. Following the failure o f the first set o f plans, the manager has altered subsequently

9. The students had to study many hours daily during the program intensive.

-------- 10. The naval officer was asked to transfer to a foreign country.

EX ER C ISE (S k ills 4 6 - 4 8 ) : C ircle th e ad jectives a n d adverbs in th e fo llo w in g s e n te n c e s

hT cT rre(I) ^ W rdS * * * d eSC db e' T h e n in d ic a te i f t h e s e n t e n c e s are c o r r e c t (C ) or

They wer= unable to see where their friends were sitting in the theater because o f
the lights dim.

2. After the com prehensive exam , she looked exhaustedly by the experience.

3. The project was remarkable close to being finished.

4. Mark always d oes his hom ework careful.


234 STRUCTURE AND WRITTEN EXPRESSION

_____ 5 . T h e p ro g ra m proved far m o re in te re stin g th a n I h a d im a g in ed it w ould be.

_____ 6 . T h e stu d e n t h a d a tte n d e d regularly all th e lectu res in the series.

_____ 7. T h e p a tie n t b e cam e h ealth y a fte r th e o p e ratio n .

_____ 8 . T h e g ra n d p a re n ts speak p roudly a b o u t all th e ir offspring.

_____ 9 , T h e m a n a g e r seem ed certainly th a t the p ro je c t w ould b e fin ish ed u n d e r budget.

_____ 10. T h e firefig h te rs w orked feverishly, a n d they p u t o u t im m ediately the fire.

T O E F L E X E R C IS E (S k ills 4 6 - 4 8 ) : C h o o se th e le tte r o f th e u n d e r lin e d w ord or g r o u p o f


w o r d s th a t is n o t c o rr ec t.

1 M odem a rt is on display at the Guggenhein M useum , a building w ith an unusually


A------- T C D
design.

2 Bv the beginning of the 1980s fifteen states had adopted already no-fault insurance
------- A------- B C D
laws.

3. H eart attacks are fatally in 75 percent of o ccu rren ces.


A B C D

4 In spite o f a trem endous am ount of electronic gadgetry, a ir traffic control still


A B C
depends heavy on people.
D

5 Only recently have Goodens industrially designers and engineers been able to
-------- A B
optim ize W atertreds unusual tread patterns for m ass p ro d u c tio n .
C ' D

6 . A baboons arm s appear as lengthily as its legs.


A B C D

7 A serious problem is how to com m unicate reliable w ith a subm erged subm arine.
---------- A---------- B C D

8 A m ericans are destroying rapidly wetlands, faster th an a n acre every two m inutes.
------ A------- ------ B C D

- 9. The c entral banking system of the U nited States consists of twelve b a rk s d istric t.
A B C D

JO. Telegraph service across the Atlantic was successful established in 1866.
A B C D
WRITTEN EXPRESSION

TOEFL REVIEW EXERCISE (Skills 1 -4 8 ): C h o o se th e le tte r o f t h e w o rd o r g r o u p o f


w ord s th at b e st c o m p le te s th e s e n te n c e .

1. Patty Berg, the top tournament winner in


2. with about fifteen tim es its weight in
women's g o lf,-------- eighty-three golf
air does gasoline allow the carburetor to
tournaments from 1935 through 1964.
run smoothly.
(A) she won
(A) It is mixed
(B) winning
(B) To mix it
(C) won
(C) When mixed
(D) who won
(D) Only when mixed

C h o o se th e lette r o f th e u n d e r lin e d w ord o r g r o u p o f w ord s th a t is n o t c o r r e c t.

3. The Colorado River reaches their maximum height during April and May.
A B C D

4. Plant proteins tend to have few amino acids than proteins from animal sources
A B C ----------5----------

5. The Viking spacecraft has landed on Mars in July of 1976.


A B C "D

6. Admiral Byrd commanded airplane expeditions over both the Arctic or the
A B ~ o'
Antarctic.

7. The advertising campaign will be based on the recent completed study.


A B C D

_ 8. Coronary occlusion results from a disease in which fatty substances with a large
A B
amount of cholesterol is deposited in the arteries.
C D

- 9. Her money gave back as soon as she threatened to take the matter to court.
A B C D

-10. Other sites o f fossil discoveries throughout Wyoming, ranging from the fiery
A B
Tyrannosaurus rex to the milder Triceratops, have proven equally e x cite.
c D
STRUCTURE AND WRITTEN EXPRESSION

MORE PROBLEMS W IT H A D JEC TIVES -------------------------------------

T h e p reviou s se c tio n d e a lt w ith variou s p r o b le m s rela ted to b o th a d jectives a n d adverbs.


T h is se c tio n d e a ls w ith a few p r o b le m s th a t are r ela ted o n ly to adjectives: (1 ) -ly a d jectiv es,
(2) p r e d ica te ad jectives, an d (3 ) - a n d - in g adjectives.

S kill 49: R EC O G N IZE -LY ADJECTIVES

G en erally w h en a w ord e n d s in -ly in E n g lish , it is an adverb. H ow ever, th e r e are a few


w ords e n d in g in -ly th at are ad jectives, a n d th e se -ly ad jectives c a n cau se c o n fu s io n in th e
W ritten E x p r e ssio n se c tio n o f th e T O E F L test.
i *
T he manager turned in his weekly report.
ADJ. NOU N

T h is e x a m p le is co rr ec t, b u t it ap p ears to b e in correct; it ap p ears that th e r e is an -ly adverb


in fro n t o f th e n o u n report. H ow ever, weekly is an adjective th at d e sc r ib es th e n o u n report.

T h e fo llo w in g ch a rt lists c o m m o n -ly ad jectives th at can a p p ea r in E nglish :

-LY ADJECTIVES

costly likely daily quarterly northerly


early lively hourly weekly easterly^
friendly lonely monthly yearly southerly
kindly manly nightly lo v e ly ___________ westerly

E X ER C ISE 49: E ach o f th e fo llo w in g s e n te n c e s c o n ta in s at least o n e ad jective o f adverb


e n d in g in -ly. C ircle th e -ly w ords, a n d la b e l th e m as e ith e r ad jectives o r adverbs. D raw ar
rows to th e w ord s they d e sc r ib e. T h e n in d ic a te if th e s e n te n c e s are c o r r e c t (C ) o r in c o r
rect ( I ) .
i y
C i. Federal taxes are (yearly) taxes which must be paid every April.

ADV.

3. Do you want to go to the early movie or the lately movie?

4. She offered m e som e friendly advice about how to deal with the terribly problem .

5 Xhe quarterly reports need to be turned in at the next weekly m eeting.

6. He did not have a manly reaction to the negatively com m ents.

7. T he likely outcom e o f the purchase o f the costly car is that he will not be able to pay
his m onthly bills.
WRITTEN EXPRESSION 237

--------8. The days she spent at the beach house were lonely and solitarily.

-------- 9. She takes her daily m edication on a regularly schedule.

-------- 10. T he kindly neighbor paid hourly visits to her unhealthily friend.

S k ill 50: USE PREDICATE ADJECTIVES CORRECTLY

C ertain a d jectiv es a p p e a r o n ly in th e p r e d ic a te o f th e s e n te n c e ; th a t is, th e y a p p e a r after a


lin k in g verb su c h as be, a n d th ey c a n n o t a p p ea r d ir ec tly in fr o n t o f th e n o u n s th at th e y d e
scrib e.

T h e snake on the rock was alive.


T he alive* snake was lying on the rock.

In th e first e x a m p le , th e p r e d ic a te a d jectiv e alive is u sed c o rr ec tly a fter th e lin k in g verb


was to d e sc r ib e th e su b je c t snake. In th e s e c o n d e x a m p le , th e p r e d ic a te a d je c tiv e a live is
u se d in c o r r e c tly in fr o n t o f th e n o u n snake. In th is p o s itio n , th e a d jc c tiv e live s h o u ld be
u sed .

T h e fo llo w in g c h a r t lists s o m e c o m m o n p r e d ic a te a d jectiv es a n d th e c o r r e s p o n d in g


fo r m s th a t can b e u se d in fr o n t o f th e n o u n :

PREDICATE ADJECTIVES
PREDICATE ADJECTIVES FORMS USED IN FRONT OF NOUN

alike (ike, similar


alive live, living
alone lone
afraid frightened
asleep sleeping

A predicate adjective appears after a linking verb such as be. It cannot appear directly in
front of the noun that it describes.

E X E R C ISE 50: E a ch o f th e fo llo w in g s e n te n c e s c o n ta in s a p r e d ic a te a d je c tiv e o r its re


la ted fo r m . C ircle th e p r e d ic a te a d jectiv es o r r e la te d fo r m s. T h e n in d ic a te i f th e s e n
te n c e s are c o r r e c t (C ) o r in c o r r e c t (I).

0 1. T he two brothers do n ot look at all (alike)

! 2. My friend brought the (alive) lobster to my house and exp ected m e to cook it.

-------- 3. Are you g o in g to be lo n e in the house tonight?

-------- 4. T he afraid child cried for his mother.

-------- 5. Everyone else was asleep by the tim e I arrived hom e.


STRUCTURE AND WRITTEN EXPRESSION

6. We com pleted our two projects in a like manner.

7. All o f the crash victims were alive when they were found.

8. She tried to walk quietly by the asleep dogs without waking them.

9. Were you feelin g afraid w hen you heard the noise?

10. A ccording to the report, the president was shot by an alone gunm an.

S k i l l 5 1: USE -ED A N D -ING ADJECTIVES CORRECTLY

Verb fo r m s e n d in g in -ed a n d -in g c a n b e u sed as ad jectives. For e x a m p le , th e verb al ad jec


tives cleaned a n d cleaning c o m e fr o m th e verb to clean.

T he woman cleans the car.


VERB

1 *
The cfeonmgwoman u car.
worked on the
ADJECTIVE

T he woman put the cleaned car back in the garage.


ADJECTIVE

In th e first e x a m p le , cleans is th e verb o f th e se n te n c e . In th e s e c o n d e x a m p le , cleaning is a


verbal ad jective d e sc r ib in g w om an. In th e third e x a m p le , cleaned is a verb al ad jective d e
scrib in g car.
V erbal ad jectives e n d in g in -ed a n d -in g c a n b e c o n fu s e d in th e W ritten E x p r e ssio n se c
tio n o f th e T O E F L test.
T he cleaning* car . . .
T he cleaned* woman . . .

T h e d iffe r e n c e b e tw e e n an -ed a n d an -in g a d jective is sim ilar to th e d iffe r e n c e b e tw e e n


th e active a n d th e passive (se e Skills 3 7 an d 3 8 ). A n - in g a d jective (lik e th e a ctiv e) m e a n s
th at th e n o u n it d e sc r ib e s is doing-the a c tio n . T h e ab ove e x a m p le a b o u t th e cleaning ca r is
n o t c o r r e c t b e c a u se a car c a n n o t d o th e a c tio n o f c le a n in g : you c a n n o t say th a t a car cleans
itself. A n -ed a d jective (lik e th e p assive) m e a n s th a t th e n o u n it d e sc r ib e s is receiving th e ac
tio n fr o m th e verb. T h e ab ove e x a m p le a b o u t the cleaned w om an is n o t c o r r e c t b e c a u se in
th is e x a m p le a w om an c a n n o t r e c e iv e th e a c tio n o f th e verb clean: th is s e n t e n c e d o e s n o t
m e a n th a t someone cleaned the w om an.
WRITTEN EXPRESSION

T h e f o llo w in g c h a r t o u tlin e s t h e k e y i n f o r m a t io n th a t y o u s h o u ld r e m e m b e r a b o u t -ed


a n d -ing a d je c tiv e s :

-ED AND -ING ADJECTIVES


TYPE MEANING USE EXAMPLE

-. -iNC
,.
active
... -
It does the action of the verb. . . . the happily playing children . . .
; ' r a : (The children ploy.)
-D passive It receives the action of the verb. .. . the frequently played record . . .
(Someone plays the record.)

E X E R C I S E 51: E a c h o f t h e f o llo w in g se n te n c e s c o n ta in s e it h e r a n -ed o r a n -ing v e r b a l a d


je c tiv e . C ir c l e t h e v e r b a l a d je c tiv e s . D r a w a rro w s to th e w o rd s th e y d e s c rib e . T h e n in d i
c a te i f th e s e n te n c e s a re c o r r e c t ( C ) o r i n c o r r e c t ( I ) .

I I ^
-------- 1. The teacher gave a quiz o n the ju st (com pleting) lesson.
C I
x_ 2. T here is a (f'asci n atin g)m ovie at the theater tonight.

-------- 3. They thought that it had been a very satisfied dinner.

-------- 4. The em pty b otd es are to the left, and the filling bottles are to the right.

----- 5. F o r lu n c h at the restau ran t she o rd ere d a m ixed salad.

-------- 6. T h e students thought that it was an interesting assignm ent

-------- 7. T he shoppers were im pressed by the reducing prices.

-------- 8. He can t afford to take lon g vacations to exotic places because he is a worked man.

-------- 9. I recently received several annoying phone calls from the insurance agent.

-------- 10. Today the bookkeeper is working on the unpaying bills.

E X E R C I S E ( S k ills 4 9 - 5 1 ): C ir c l e th e a d je c tiv e s in e a c h o f th e f o llo w in g s e n te n c e s . D r a w


a rro w s to th e n o u n s o r p r o n o u n s th e y d e s c rib e . T h e n in d ic a t e i f t h e s e n te n c e s a r e c o r r e c t
(C ) o r in c o rre c t ( I ) .

-------- 1. H er kindly words o f thanks made m e feel appreciating.

-------- 2. After the earthquake, assistance was sent to the dam aging areas.

-------- 3. Your view has som e validity; however, we do not have alike opin ion s o n the matter.

-------- 4. It is likely that the early seminar will not be the m ost interested.

-------- 5. I prefer a live theater show to a movie.


STRUCTURE AND WRITTEN EXPRESSION

_____ 6. The thesis o f your essay was not very well developed.

_____ 7. T he asleep children were wakened by the loud sound o f the crashing thunder.

_____ 8. During the nightly news show there was a lively and fascinating debate.

_____ 9. H is car was struck by an uninsured motorist.

_____ 10. The girl was all alone and feeling lonely in the darkened, frightened house.

T O E F L E X E R C ISE (S k ills 4 9 -5 1 ): C h o o se th e le tte r o f th e u n d e r lin e d w o rd o r g r o u p o f


w o rd s th a t is n o t c o rr ec t.

_____ 1 As the only major American river that flowed in a west direction, the Ohio was the
---- A---- B C
preferred route for settlers.
D

___ 2. During the annually salmon migration from the sea to fresh water, Alaska s McNeil
A B
River becomes a gathering place for brown bears waiting eagerly to catch their fill.
------C------ D

_____ 3 Edelman stresses the mounting evidence showing that greatly variation on a
; A B C
microscopic scale is likely.
D

_____ 4. Perhaps the most welcoming and friendly of the park's wild places is the live oak
A B C
forest that surrounds the districts alone visitors center in Gulf Breeze.
D

_____ 5. Haileys com et, viewing through a telescope, was quite impressive.
A B C D

____ 6 The state of deep asleep is characterized by rapid eye movement, or REM, sleep.
~A B- C D

____ 7. Among the disputing sections of the Monteverdi opera are the sinfona, the
A B C D
prologue, and the role of Ottone.

_____ 8. Most probably because of the likable rapport between anchors, the night newscast
A B C
on the local ABC affiliate has recently moved well beyond its competitors in the
D
ratings battle.
WRITTEN EXPRESSION

_ 9. Signing at the outset of a business deal, a contract offers the participants a certain
A B C
degree of legal protection from costly mistakes.
D

_10. The story presented by Fischer is a headlong tale told so effectively that
B
its momentum carries the reader right through the live endnotes.
C T T

T O E F L R EV IEW E X E R C ISE (S k ills 1 -5 1 ): C h o o se th e le tte r o f th e w o r d o r g r o u p o f


w ord s th a t b e st c o m p le te s th e s e n te n c e .

During the early nineteenth century, the 3. The daughters of Joseph LaFlesche were
S p a n i s h m i s s i o n s i n Alta, C a l i f o r n i a bom into the generation of Omaha forced
to be an integral part of the economy and to abandon tribal traditions, on the
productive capacity of the region. reservation, and to adapt to the white
man's ways.
(A) proved
(B) they proved (A) they matured
(C) they proved it (B) to mature
(D) proved it (C) maturing
(D) to maturity
2 . Still o t h e r h u r d l e s r e m a i n b e f o r e
suitable for private cars. 4. Among the most revealing aspects of
mining towns their paucity o f public
(A) fuel cells
open space.
(B) become
(C) fuel cells become (A) was
(D) that fuel cells become (B) were
(C) it was
(D) so

C h o o se th e lette r o f th e u n d e r lin e d w ord o r g r o u p o f w ord s th a t is n o t c o r r e c t.

5. Factor analysis is used to discover how many abilities are involve in intelligence test
A B C D
performance. .

6. One of the early orders of marine mammals, manatees have evolved more than fifty
B
million years ago from land animals.
C

7. Dolphins and chimps are like in that they have been shown to have language skills.
T 1 C D

8. In the appendix at the end o f the chapter are the instructions to be used for the
TT C
completion correct of the form.
D
STRUCTURE AND WRITTEN EXPRESSION

_____ 9. Used sound that varies not only in time but in space, whales at close range may
A IT C
communicate with sonarlike "pictures.
D

_____ 10. The 1898 Trans-Mississippi International Exposition has the distinction of being the
~A~ B
last major fair which held during the Victorian period.
C D

PROBLEMS W IT H ARTICLES________________________________

A rticles are very d iffic u lt to lea r n b e c a u se th e r e are m any ru les, m a n y e x c e p tio n s , and
m a n y sp ecia l cases. It is p o s sib le , how ever, to learn a few ru les that w ill h e lp y o u to u se ar
ticles c o rrectly m u c h o f th e tim e.
N o u n s in E n g lish can b e e ith e r c o u n ta b le or u n c o u n ta b le . I f a n o u n is c o u n ta b le , it
m u st b e e ith e r sin g u la r o r p lu ral. In a d d itio n to th e s e g e n e ra l types o f n o u n s , th e r e are
two types o f articles: d e fin ite (sp e c ific ) and in d e fin ite (g e n e r a l).

COUNTABLE COUNTABLE
SINGULAR PLURAL UNCOUNTABLE
ARTICLES NOUNS NOUNS NOUNS

INDEFINITE o dollar ~ dollars mnnpy


(General) on apple
apf>leS ............... ,U'Ce
.
DEFINITE the dollar the dollars the money
(Specific) the apple the apples the juice

S k i l l 52: USE ARTICLES W IT H S IN G U LA R N O U N S

You can s e e fro m th e ch a rt th a t i f a n o u n is e ith e r c o u n ta b le plu ral o r u n c o u n ta b le , it is


p o ssib le to h ave e it h e r th e d e fin ite a rticle the o r n o a rticle ( in d e fin ite ). W ith all c o u n t
a b le sin g u la r n o u n s , h ow ever, y o u m u s t have an a r tic le (u n le ss y o u have a n o th e r d e te r
m in e r su ch as my o r each).

I have money. (uncountable n o article needed)


I have books. (countable plural no article needed)
I have a book. (countable singular article needed)

T h e fo llo w in g c h a r t o u tlin e s th e key in fo r m a tio n th a t y o u sh o u ld r e m e m b e r a b o u t ar


ticles w ith sin g u la r n o u n s:

ARTICLES WITH SINGULAR NOUNS


A singular noun m utt have an article (o, an, the) or some other determiner si'ch as my or
each. (A plural noun or an uncountable noun may or m ay not have an article.)
WRITTEN EXPRESSION

E X E R C I S E 52: T h e fo llo w in g s e n te n c e s c o n ta in d iffe r e n t types o f n o u n s . C ir cle o n ly th e


c o u n ta b le sin g u la r n o u n s . M ark w h e r e a rticles (or d e te r m in e r s ) h a v e b e e n o m itte d .
T h e n in d ic a te i f th e s e n t e n c e s are c o r r e c t (C ) o r in c o r r e c t (I).

__ L _ 1. She is taking^ trip ) with friends.

C 2. In my (yard) there are flowers, trees, and grass.

_____ 3. T he m anager sent m em o to his em ployees.

_____ 4. T here is car in front o f the building.

_____ 5. T he child and his friends are having milk and cookies.

_____ 6. She is studying to be an actress in films.

_____ 7. My neighbor was arrested for throwing rocks through windows.

_____ 8. We have m achinery that prints ten pages each minute.

_____ 9. Teacher has many students during a semester.

_____ 10. Can you heat water for tea?

S k i l l 53: D IS T IN G U IS H A A N D AN

T h e b asic d iffe r e n c e b e tw e e n a a n d a n is th a t a is u s e d in fr o n t o f c o n s o n a n ts a n d a n is
u sed in fr o n t o f vo w els (a, e, i, o, u ) :

a ook an orange
a man an illness
a /age an autom obile

In reality, th e r u le is th a t a is u s e d in fr o n t o f a w ord th a t b e g in s w ith a c o n s o n a n t so u n d


a n d th a t a n is u s e d in fr o n t o f a w o rd th a t b e g in s w ith a vow el sound. P r o n o u n c e th e f o l
lo w in g e x a m p les:

a university a hand a one-way street a euphem ism a xerox m achine


an unhappy man an hour an om en an event an x-ray m achine

T h ese e x a m p les sh ow th at certain b e g in n in g letters can have e ith er a c o n so n a n t o r a vow el


sou n d . A w ord that b e g in s w ith u can b e g in with a c o n so n a n t y so u n d as in university or w ith a
vowel so u n d as in unhappy. A w ord that b e g in s with h can b e g in w ith a c o n so n a n t h so u n d as in
hand o r with a vow el s o u n d as in hour. A w ord that b eg in s with o can b e g in w ith a c o n so n a n t w
so u n d as in one o r w ith a vow el so u n d as in omen. A w ord that b e g in s w ith e can b e g in w ith ei
ther a c o n so n a n t y so u n d as in euphemism or with a vowel sou n d as in event. A w ord that b e g in s
with x can b e g in with e ith e r a c o n so n a n t z so u n d as in xerox or w ith a vow el so u n d as in x-ray.

T h e fo llo w in g c h a r t o u d in e s th e k ey in fo r m a tio n a b o u t th e u se o f a a n d an:

. .. ...
A A N D AN

A A is used in front of a singular noun with a consonant sound.

AN An is used in front of a singular noun with a vowel sound.

Be careful of words beginning with letters such as u. o, e, x, or h. Th-.y may begin with either a vowel
or a consonant sound.
STRUCTURE AND WRITTEN EXPRESSION

E X E R C ISE 53: E ach o f th e fo llo w in g s e n te n c e s c o n ta in s a o r an. C ircle e a c h a o r an. U n


d e r lin e th e b e g in n in g o f th e w ord th at directly follow s. P r o n o u n c e th e w ord . T h e n in d i
c a te if th e s e n te n c e s are c o r r e c t (C ) o r in c o r r e c t (I).

I 1. The dishwasher quit his jo b because he was making only four dollars hour.
C 2. It was (an) unexpected disappointm ent to receive (a) rejection letter from the
university.

3. A signature is required wherever you see a X on the form.

4. He bought a half gallon o f milk and a box o f a hundred envelopes.

5. An objection was raised because it was such a unacceptable idea.

6. There are two trees in the yard, an elm tree and a eucalyptus tree.

7. The police officer was n ot wearing an uniform when she arrested the suspect.
8. If you do not give m e a hand, finishing the project on time will be an impossibility.

9. She was upset when a hon est mistake was made.

10. She opened a account at a local departm ent store during a one-day sale.

S k ill 54: MAKE ARTICLES AGREE W IT H N O U N S

T h e d e fin ite article (the) is u se d fo r b o th sin gu lar a n d p lu ral n o u n s , so a g r e e m e n t is n o t a


p r o b le m w ith th e d e fin ite article. H ow ever, b e c a u se th e u se o f th e in d e fin ite a r tic le is dif
f e r e n t for sin g u la r a n d p lu ral n o u n s , y o u m u st b e carefu l o f a g r e e m e n t b e tw e e n th e in
d e fin ite a rticle a n d th e n o u n . O n e very c o m m o n a g r e e m e n t e rr o r is to u se th e sin g u la r
in d e fin ite article (a or an) w ith a plu ral n o u n .

H e saw a* new movies.


They traveled to a* nearby mountains.
D o you have another* books?

In th e se e x a m p le s , you sh o u ld n o t have a o r a n b e c a u se th e n o u n s are p lu ra l. T h e fo llo w


in g s e n te n c e s are p o ssib le c o r r e c tio n s o f th e s e n te n c e s above.

H e saw a new m ovie. (singular)


H e saw new movies. (plural)

They traveled to a nearby mountain. (singular)


They traveled to nearby m ountains. (plural)

D o you have another book? (singular)


D o you have other books? (plural)

T h e fo llo w in g ch art states th e key p o in t fo r y o u to r e m e m b e r a b o u t th e a g r e e m e n t o f


a rticles w ith n o u n s:

' .. ' .
AGREEMENT OF ARTICLESWITH NOUNS
You should never use a or an with a plural noun. ' - i
WRITTEN EXPRESSION

E X E R C ISE 54: E a ch o f th e fo llo w in g s e n t e n c e s c o n ta in s a o r an. C ir c le e a c h a o r a n.


Draw an arrow to th e n o u n it d e sc r ib e s. T h e n in d ic a te if th e s e n t e n c e s a r e c o r r e c t (C ) o r
in c o r r e c t (I).

r
__
X
1. She w ent to school in (a) local community.
t

!__ 2. T h e doctor used (an oth er pills.

-------- 3. It is necessary to have a farm or land o f your own.

-------- 4. H e must contact a m em bers o f the club.


-------- 5. You will n e e d a pen or a pencil.

-------- 6. H e is responsible for bringing a number o f items.

-------- 7. You must write a report on a subjects o f your choice.


-------- 8. They crossed through several forests and a stream.

-------- 9. T here will be another im portant lessons tomorrow.

-------- 10. H e could n ot give m e a good reasons for what he did.

S k ill 55: D IS T IN G U IS H SPECIFIC A N D G ENERAL IDEAS

W ith c o u n ta b le sin g u la r n o u n s it is p o ssib le to u se e ith e r th e d e fin ite o r th e in d e fin ite ar


ticle, b u t th ey have d iffe r e n t m e a n in g s. T h e d e fin ite article is u s e d to r e fe r to o n e sp e c ific
noun.

Tom will bring the book tomorrow.


(There is on e specific book that Tom will bring tomorrow.)

H e will arrive on A first Tuesday in July.


(There is only o n e first Tuesday in July.)

H e sailed on the Pacific Ocean.


(There is only o n e Pacific Ocean.)

T h e d e fin ite a r tic le is u s e d w h e n th e n o u n c o u ld b e o n e o f several d iffe r e n t n o u n s .

Tom will bring a book tomorrow.


(Tom will bring any o n e book.)

H e will arrive o n a Tuesday in July.


(H e will arrive o n on e o f four Tuesdays in July.)

H e sailed on an ocean.
(H e sailed on any o n e o f the w orlds oceans.)
STRUCTURE AND WRITTEN EXPRESSION

T h e fo llo w in g c h a r t o u tlin e s th e key in fo r m a tio n th a t y o u sh o u ld u n d e r sta n d a b o u t


s p e c ific a n d g e n e r a l ideas:

SPECIFICAND GENERAL IDEAS


a r t ic l e MEANING USES

A o r AN general idea Use when there are many, and you do not know which one it is.
-, ' - Use when there re many, and you do not care which one it is.

THE specific idea Use when it is the only one.


. Use when there are many, and you know which one it is.

E X E R C I S E 55: E ach o f th e fo llo w in g s e n te n c e s c o n ta in s o n e o r m o r e a rticles. C ircle th e


articles. D raw arrow s to th e n o u n s th ey d e sc r ib e. T h e n in d ic a te i f th e s e n te n c e s are c o r
r e c t (C ) o r in c o r r e c t ( I ) .

^ 1. H e took tnp on (&) Snake lLver.

C 2. Ill m eet you at (the) library later.

_____ 3. T he ball hit a child on a head.


_____ 4. H e had a best grade in the class on the exam.

_____ 5. T h e peop le who cam e here yesterday were here again today.

_____ 6. She was a m ost beautiful girl in the room.


_____ 7. T h e trip that I took last year to the Bahamas was the only vacation I had all year.

_____ 8. I n eed a piece o f paper so that I can finish the report that I am working on.

_____ 9. A basketball player threw the ball to a center o f the court.


_____ 10. T h e sixth-grade class went on a field trip to visit a Lincoln Memorial.

E X E R C I S E ( S k ills 5 2 - 5 5 ): C ircle th e a rticles in th e fo llo w in g s e n te n c e s . T h e n in d ic a te if


th e s e n te n c e s are c o r r e c t (C) o r in c o r r e c t (I).
a
_____ 1. H e took a m oney from his wallet to pay for sweater.
_____ 2. T h e notebook that he left had an im portant assignm ent in it.

_____ 3. Because o f previous disagreem ents, they are trying to arrive at an understanding.

_____ 4. T h e appearance o f room could be improved by adding a green plants.


_____ 5. T he Senate passed law banning sm oking in public workplaces.
_____ 6. Each chem istry student should bring laboratory m anual to a n ext class.
_____ 7. She adm itted that she made mistake but said that she had m ade a h on est effort.
8. H is absence from the board m eeting was a strong indications o f his desire to leave
the company.
WRITTEN EXPRESSION 247

9. T he car n eed ed gas, so the driver stopped at a service station.

10. Anyone taking group tour to the Hawaiian Islands must pay fee before a first o f the
m onth.

T O E F L E X E R C ISE (S k ills 5 2 - 5 5 ) : C h o o se th e lette r o f th e u n d e r lin e d w o rd o r g r o u p o f


w ord s th a t is n o t c o rr ec t.

_____ 1. On a trip down to the bottom of the Grand Canyon, the equipment will in all
~A~ B C
probability be carried by a burros.
D

_____ 2. Ford designed the first large-scale assembly line at plant in


' A B ~T~
Highland Park, Michigan.
D

_____ 3. In the human body, blood flows from a heart through the arteries, and it returns
A B C -
through the veins.

_____ 4. The scholarship that Wilson received to study history at Cambridge presented an
A B C ~D"
unique opportunity.

_____ 5. Observations from Earth indicate that at the solar surface, the outward magnetic
A B
field is a strongest at the polar regions.
Z ------- B-------
_____ 6. A radar images of Venus add details about a planet dominated by
A B C
volcanoes and lava.
--------- 5---------
_____ 7. In 1863 and 1864, the U.S. Congress passed the National Bank Acts , which set up a
A B
system of privately owned banks chartered by a federal government.
C D

_____ 8. An human ear responds to a wide range o f frequencies .


T ~B~ C D

_____ 9. Bacteria that live in soil and water play a vital role in recycling carbon, nitrogen,
A B
sulfur, and another chemical elements used by living things.
C D

_____ 10. During the U.S. Civil War, an American balloonist organized a balloon corps in Army.
A B U D
248 STRUCTURE AN D WRITTEN EXPRESSION

T O E F L R EV IEW E X E R C ISE (Sk ills 1 -5 5 ): C h o o se th e le tte r o f th e w ord o r g r o u p o f


w ord s th at b e s t c o m p le te s th e s e n te n c e .

1. In econom ics, "diminishing returns"


3. R arely. _remove the entire root of a
d escribes_____ resource inputs and
dandelion because of its length and
production.
sturdiness.
(A) among
(A) can the casual gardener
(B) when it is
(B) the casual gardener
(C) among them
(C) the casual gardener will
(D) the relationship between
(D) does the casual gardeners
2. When lava reaches the surface, its
temperature can be ten tim es_____ boiling
water.
(A) the temperature
(B) that of
(C) it is
(D) more

C h o o se th e le tte r o f th e u n d e r lin e d w ord o r g r o u p o f w ord s th a t is n o t c o rr ec t.

4. Operascan be broadly classified as either com edies or they are tragedies.


a b r D

5. Tungsten has the highest melting point of all metals, and for this reason it is often
A
use in equipment that must withstand high temperatures.
TT C D

6. Whereas thereare forty-three ant species in GreatBritain, the same amount of ant
A "B~ C
species can be found in a single tree in Peru.
D

7. People voice theirs opinions first in small groups or among friends and acquaintances.
A B C ~D

8. Inside the Lincoln Memorial is a large statue of Lincoln make from white marble.
A B ~C ~ D

9. Detailed photometric data of the area just north o f Triton s equatorial region indicate
A B
the existence o f a thin, transparent layers of frost.
C D

_10. U.S. census figures indicate that people with only an elementary education can earn
A B~
just half as much as college graduations.
D
WRITTEN EXPRESSION

PROBLEMS W IT H P R E P O S IT IO N S __________________________

P r e p o s itio n s can b e u s e d in tw o ways: in a literal way a n d in an id io m a tic way. In th e literal


u s e , th e p r e p o sitio n m e a n s e x a ctly w h at y o u e x p e c t.

T he boy ran up the hill.


She w ent in the house.

In th e first e x a m p le , th e p r e p o s itio n u p m e a n s th a t th e b o y w e n t in th e d ir e c tio n u p


rath er th an down. In th e s e c o n d e x a m p le , th e p r e p o sitio n in m e a n s th a t s h e w e n t into
rath er th an out o f th e h o u se .
In th e id io m a tic u se, w h ic h is w h a t a p p ea r s m o st o fte n o n th e T O E F L test, th e p r e p o
sitio n a p p ea r s in a n id io m a tic e x p r e ssio n ; th a t is, its m e a n in g in th is e x p r e s s io n h as n o th
in g to d o w ith th e lite ra l m e a n in g .

I call up my friend.
He su cceed ed in passing the course.

In th e first e x a m p le , th e w o r d up h a s n o th in g to d o w ith th e d ir e c tio n up. To call u p some


one m e a n s to telephone s o m e o n e . In th e s e c o n d e x a m p le , th e w ord in h a s n o th in g to d o
w ith th e m e a n in g o f into o r inside; it is sim p ly id io m a tic th a t th e w o r d in is u s e d a fter th e
verb succeed.
It is im p o ss ib le to list all p o te n tia l id io m a tic e x p r e s sio n s w ith th e ir p r e p o s itio n s b e
cau se th e r e are so m a n y e x p r e s s io n s th a t c o u ld a p p ea r o n th e T O E F L test. H ow ever, in
th is se c tio n y o u ca n p r a c tic e r e c o g n iz in g p r o b le m s w ith p r e p o s itio n s in T O E FL -type
q u e stio n s. T h e n , w h e n y o u are w o r k in g in th e W ritten E x p r e ssio n s e c tio n o f th e TO EFL
test, y o u sh o u ld b e aw are th a t id io m a tic e rro rs w ith p r e p o sitio n s are c o m m o n in th a t se c
tio n . T h e r e are tw o c o m m o n types o f p r o b le m s w ith p r e p o sitio n s th a t y o u s h o u ld e x p e c t:
(1) in c o r r e c t p r e p o s itio n s a n d (2) o m itte d p r e p o sitio n s.

S k ill 56: R E C O G N IZ E IN C O R R E C T PREPOSITIONS

S o m e tim e s an in c o r r e c t p r e p o s itio n is g iv en in a s e n te n c e in th e W ritten E x p r e ssio n se c


tio n o f th e T O E F L test.

T he gam e was called on* because o f rain.


I knew I could count in* you to do a good job.

T h e first e x a m p le s h o u ld say th a t th e g a m e w as called o ff b e c a u se o f rain . T h e e x p r e s sio n


called o ff m e a n s canceled, a n d th a t is th e m e a n in g th a t m ak es se n s e in th is s e n t e n c e . To call
on someone is to v is it someone, a n d th is m e a n in g d o e s n o t m ak e se n s e in th is e x a m p le . In th e
se c o n d e x a m p le , it is n o t c o r r e c t in E n g lish to count in someone. T h e c o r r e c t e x p r e s s io n is
to count on someone.
STRUCTURE AND WRITTEN EXPRESSION

E X E R C IS E 56: E ach o f th e fo llo w in g s e n te n c e s c o n ta in s at le a s t o n e p r e p o sitio n . C ircle


th e p r e p o sitio n s. T h e n in d ic a te i f th e s e n te n c e s are c o r r e c t (C ) o r in c o r r e c t (I).

0 1. (Aftei) school many students participate (in) sports.

! 2. I know I can rely (in) you to be here (on) time.

-------- 3. If you need m ore light to read, turn on the lamp n ext to you.

-------- 4. Parents always try to bring at their children to be thoughtful.


-------- 5. Ill have to consult to my attorney before making a decision.
_____ 6. Walt has lost his keys, so he must look for them.

-------- 7. I ju st d o n t approve at your cheating o n the exam.


-------- 8. Sm oking is forbidden, so you should put out your cigarette.
-------- 9. Failure to pass the test will result to the loss o f your license.

-------- 10. It is unlawful for parolees to associate with known felons.

S k ill 57: R E C O G N IZE W H E N PR EPO SITIO NS HAVE BEEN O M IT T E D

S o m e tim e s a n e c e ssa r y p r e p o sitio n h as b e e n o m itte d fro m a s e n te n c e in th e W ritten Ex


p r e ssio n s e c tio n o f th e T O E F L test.

Can you wail* me after the game?


I plan* attending the m eeting.

T h e first e x a m p le is in c o r r e c t b e c a u se it is n e c essa ry to say w ail f o r me. T h e s e c o n d e x a m


p le is in c o r r e c t b e c a u se it is n e c e ssa r y to say p la n on attending.

EX E R C ISE 57: P r e p o s itio n s h ave b e e n o m itte d in so m e o f th e fo llo w in g se n te n c e s . Mark


w h er e p r e p o s itio n s have b e e n o m itte d . T h e n in d ic a te if th e s e n te n c e s are c o r r e c t (C) or
in c o r r e c t (I).

! 1. If you take this job , it will be necessary to deal Vother departments.


C 2. Each child took o n e cookie from the plate.

3. In the discussion, Rob sided the rest

4. T h e board turned his suggestion for the project because it was too cosdy.
5. H e can always d epend his friends.

6. W hile Mrs. Sampson w ent shopping, a baby-sitter looked the children.


7. I know Steve believes what you told him.
8. Children should beware strangers.
9. It was difficult to make a decision about buying a house.
10. Tom blam ed his brother the d en t in the car.
WRITTEN EXPRESSION

E X E R C I S E (S k ills 5 6 - 5 7 ): C ir c l e th e p re p o s itio n s in th e f o llo w in g s e n te n c e s . M a r k w h e r e


th e y h a v e b e e n o m itt e d . T h e n in d ic a t e i f t h e se n te n c e s a r e c o r r e c t ( C ) o r i n c o r r e c t ( I ) .

_____ 1. T he students m ust hand in their homework.

_____ 2. It will be difficult to forgive you o f breaking your promise.

_____ 3. Elizabeth excels math and science.

_____ 4. She insisted on goin g to work in spite o f her cold.


_____ 5. Bob rem inds m e to his father because he looks just like him.

_____ 6. If you are cold, you should put on your sweater.


_____ 7. Mr. Sanders is n o t here now, but he will call you when he returns.

_____ 8. I do not want to interfere your plans.

_____ 9. Alan waited Marie after school.

_____ 10. Bill laughs m e w henever he looks me.

T O E F L E X E R C I S E (S k ills 5 6 - 5 7 ): C h o o s e th e le t te r o f t h e u n d e r li n e d w o r d o r g r o u p o f
w o rd s th a t is n o t c o r r e c t . .

____ 1 Amelia Earhart, the first woman to fly solo across the Atlantic, disappeared on June
A B C
1937 while attempting to fly around the world.
D

_____ 2. The occurrence edem a indicates the presence of a serious illness.


A B C D

_____ 3. Atomic nuclei are believed to be composed by protons and neutrons in_ equal
A B C D
numbers for the lighter elements.

_____ 4. According legend, Betsy Ross designed and sewed the first American flag.
----------- A B C D

_____ 5. The middle ear is attached for the back of the throat by the Eustachian tube.
A TT TT D

_____ 6. Plants that sprout, grow, bloom, produce seeds, and die within one year are
A B C
classified for annuals.
TT

_____ 7. A marionette is controlled by means strings connected to wooden bars.


A B C D

_____ 8. In July of 1861, Pat Garrett killed Billy the Kid inji house close Fort Sum ner.
X "B C D
STRUCTURE AND WRITTEN EXPRESSION

_____ 9. Manv comfort heating systems using steam as a working fluid operate at the
~K~ "B C D
convection principle.

_____ 10. Mars two small moons are irregularly shaped and covered for craters.
A B 5 "IT

T O E F L REV IEW E X E R C ISE (1 -5 7 ): C h o o se th e le tte r o f th e w ord o r g r o u p o f w ord s that


b e st c o m p le te s th e se n te n c e .

1. In any matter, heat tends to flow _____ to 3. _____ Army camps near Washington, D.C.,
the cooler parts. in 1861, Julia Ward Howe wrote The
Battle Hymn of the Republic.
(A) hotter parts
(B) there are hotter parts (A) She visited
(C) from the hotter parts (B) After visiting
(D) toward the hotter parts (C) When visited
(D) When was she visiting
2. Certain authorities claim that the
costumes that people wear to parties___
into their personalities.
(A) give subtle insights
(B) they give subtle insights
(C) which give subtle insights
(D) subtle insights

C h o o se th e le tte r o f th e u n d e r lin e d w ord o r g r o u p o f w ord s th a t is n o t c o r r e c t.

_____ 4. The body depends in food as its primary source of energy.


A "B T IT

_____ 5. Regular programming was interrupted to broadcast a special news bulletins.


A B C D

_____ 6. Sulfa drugs had been used to treat bacterial infection until penicillin becom es widely
~A~ B C D
available.

_____ 7. Plans for both the International Monetary Fund or the World Bank were drawn up
A IT C" D
at the Bretton Woods Conference.

_____ 8. Seldom Antarctic icebergs will move far enough north to disturb South Pacific
A B C
shipping lanes.
D
WRITTEN EXPRESSION

_ 9. In 1958, a largest recorded wave, with a height of 500 meters, occurred in Lituya
"A B C D
Bay, Alaska.

_10. Exercise in swim ming pools is particularly helpful because of the buoyant
A B C
effect water.
D

PROBLEMS W IT H USAGE.

In E n g lish cer ta in g r o u p s o f w ord s h ave sim ila r u ses, a n d th e se w ord s are s o m e tim e s c o n
fu se d in th e W ritten E x p r e ssio n s e c tio n o f th e T O E F L test. A lth o u g h v a rio u s u sa g e p r o b
le m s are p o s sib le o n th e T O E F L test, th e fo llo w in g p r o b le m s are th e m o s t c o m m o n :
(1) w h en to u se m ake a n d do; (2 ) w h e n to u se like, unlike, a n d alike; a n d (3 ) w h e n to u se
other, another, a n d others.

S k ill 58: D IS T IN G U IS H M AKE A N D DO

M ake an d do can b e c o n fu s e d in E n g lish b e c a u se th e ir m e a n in g s are so sim ilar. S in c e th e


d iffe r e n c e b e tw e e n m ake a n d do is te ste d o n the T O E F L test, y o u s h o u ld le a r n to d istin
g u ish th em .
M ake o fte n h as th e id e a o f creating or constructing. T h e fo llo w in g e x p r e s s io n s sh o w
so m e o f th e p o s sib le u ses o f make:
She likes to make her own clothes.
Would you like to make a cake for dessert?
If you make a. mistake, you should correct it.
H e was unable to make a response to the threat.

Do o fte n h as th e id e a o f com pleting o r perform ing. T h e fo llo w in g e x p r e s s io n s sh o w s o m e o f


th e p o ssib le u se s o f do:
This m orning she did all the dishes.
T he students are doing the assignments.
T h e janitors did the work they were assigned.
You can do your laundry at the laundromat.

T h e se are o n ly s o m e o f th e u ses o f m ake a n d do. M any u s e s o f m ake a n d do are id io m a tic


a n d th e r e fo r e d iffic u lt to classify.

FXF.Rr.lSF. 58: E ach o f th e fo llo w in g s e n te n c e s c o n ta in s m ake or do. C ir cle m ake o r do.


D raw arrow s to th e n o u n s th a t c o m p le te th e e x p r e ssio n s. T h e n in d ic a te i f th e s e n t e n c e s
are c o r r e c t (C ) o r in c o r r e c t (I).

J 1. T he biology stu dent (did) several mistakes in the lab report.

r ^
L2. I hop e that you will be able to (dg) m e a favor this afternoon.
STRUCTUREAND WRITTEN EXPRESSION

3. N o matter what jo b she has, she always makes her best.

4. The runner did a strong effort to increase her speed in the m ile race.

5. It is com forting to think that your work can make a difference.

6. His grade was not very good because he had not don e his homework.

7. In this jo b you will make m ore m oney than in your previous job.

8. H e was unable to do dinner because no o n e had d on e the lunch dishes.

9. It is a pleasure to work with som eone who always makes the right thing.
10. If you make a good im pression at your jo b interview, you will get the job .

S k i l l 59: D IS T IN G U IS H LIKE, ALIKE, UNLIKE, A N D D ISLIKE

Like, alike, unlike, a n d dislike are easily c o n fu s e d b e c a u se th e y lo o k so sim ila r a n d th e y have


m an y d iffe r e n t u ses. T h e r e are several stru ctu res w ith like, alike, unlike, a n d dislike th a t you
sh o u ld b e fa m ilia r w ith.
T h e first str u c tu r es y o u sh o u ld already be fam iliar w ith are th e ad jectives alike a n d like
(se e Skill 5 0 ). S tu d y th e u se o f alike an d like in th e fo llo w in g e x a m p les.
John and Tom are alike.
John and Tom worked in a like manner.
In b o th th e se e x a m p le s , alike a n d like are ad jectiv es th a t m e a n similar. In th e first e x a m p le ,
alike is a p r e d ic a te ad jective d e sc r ib in g J o h n a n d Tom. B e ca u se alike is a p r e d ic a te adjec
tive, it c a n o n ly b e u sed a fter a lin k in g verb su ch as are. In th e s e c o n d e x a m p le , like is th e
ad jective fo r m th a t is u s e d im m e d ia te ly b e fo r e th e n o u n manner.
T h e n e x t str u c tu r es y o u sh o u ld b e fam iliar w ith are th e p r e p o sitio n s like an d unlike,
w h ic h h ave o p p o s ite m e a n in g s. B e ca u se they are p r e p o sitio n s, th e y m u st b e fo llo w e d by
ob jects.
John is (likeTom).
John is (unlikeTom).
In th e first e x a m p le , th e p r e p o sitio n like is fo llo w e d by th e o b je c t Tom. It m e a n s th a t Tom
an d J o h n are sim ilar. In th e se c o n d e x a m p le , th e p r e p o sitio n u n lik e is fo llo w e d by th e o b
j e c t Tom. It m e a n s th at T o m an d J o h n are n o t sim ilar.
T h e p r e p o s itio n s like a n d u n lik e can also b e u s e d at th e b e g in n in g o f a se n te n c e .
(LikeTom ), John is tall.
(U nlikeTom), John is tall.
In th e first e x a m p le , th e p r e p o sitio n like is fo llo w e d by th e o b je c t Tom. It m e a n s th a t T om
is tall. In th e s e c o n d e x a m p le , th e p r e p o sitio n u n lik e is fo llo w e d by th e o b je c t Tom. It
m e a n s th a t T om is n o t tall.
T h e fin a l str u c tu r es th a t y o u sh o u ld b e fa m ilia r w ith are th e verb s like a n d dislike,
w h ic h h a v e o p p o s ite m e a n in g s. B e c a u se th ey are verb s, th ey are u s e d w ith su bjects.
John and Tom like the course.
John and Tom dislike the course.
In th e first e x a m p le , th e verb like fo llo w s th e su b ject John a n d Tom. It m e a n s th at b o th m en
th in k th a t th e c o u r se is en jo y a b le . In th e se c o n d e x a m p le , th e v e rb dislike fo llo w s th a t sub
j e c t J o h n a n d Tom. It m e a n s th at b o th m e n th in k th at th e c o u r se is n o t e n jo y a b le .
WRITTEN EXPRESSION

T h e fo llo w in g c h a r t o u tlin e s th e stru ctu res a n d m e a n in g s o f s e n te n c e s w ith like, alike,


unlike, a n d dislike:

LIKE, ALIKE, UNLIKE, AND DISLIKE

GRAMMAR MEANING USE

like adjective similar As an adjective, like is used before a noun.


alike adjective similar As an adjective, alike is used after a linking verb.
like preposition similar Both prepositions are followed by objects.They can both be used
unlike preposition different in many positions, including at the beginning of the sentence.

like verb enjoy Both verbs follow subjects.


dislike verb not enjoy

E X E R C ISE 59: E ach o f th e fo llo w in g s e n te n c e s c o n ta in s like, alike, unlike, o r dislike. C ircle


th e ftfce w ords. T h e n in d ic a te i f th e s e n te n c e s are c o r r e c t (C ) o r in c o r r e c t (I ).

1 1. The iwo routes you have chosen for the trip are (like.)
C 2. T he scien ce books this sem ester are (like) the books used last sem ester.
3. Alike the restaurant where we usually eat, this new restaurant has early-bird specials.
4. U nlike the traditional red fire engines, the new fire engines are yellow.
5. T he two girls disliked the fact that they were wearing alike dresses.
6. The new piece that the pianist is preparing is unlike any she has ever played before.
7. Like the W ashington Zoo, the San D iego Zoo has several panda bears.
8. The insurance package offered by that company is exactly alike the package our
com pany offers.

9. Any further work done in a like fashion will be rejected.


10. It is unfortunate that the covers for this years and last years albums are so dislike.

S k ill 60: D IS T IN G U IS H OTHER, ANOTHER, A N D O THERS

Other, another, a n d others are very easy to c o n fu s e . T o d e c id e h o w to u se e a c h o f t h e m cor-


recdy, y o u m u st c o n s id e r th r e e th in gs: (1 ) i f it is sin g u la r o r p lu ral, (2 ) i f it is d e fin it e (the)
o r in d e fin ite (a ) , a n d (3) i f it is a n ad jective (it ap p ears w ith a n o u n ) o r i f it is a p r o n o u n
(it ap p ea rs by it s e lf ) .

SINGULAR PLURAL

INDEFINITE 1 have another book. 1 have other books.


1 have another. 1 have others.

DEFINITE 1 have the other book. 1 have the other books.


1 have the other. 1 have the others.

N o tic e th a t y o u u s e another o n ly to refer to an in d e fin ite , sin g u la r id ea . Others is u s e d o n ly


as a p lu ral p r o n o u n ( n o t a c c o m p a n ie d by a n o u n ). In all o th e r c a ses, other is c o r r e c t.
STRUCTURE AND WRITTEN EXPRESSION

E X E R C ISE 60: E ach o f th e fo llo w in g s e n te n c e s c o n ta in s other, another, o r others. C ircle


other, another, o r others. T h e n in d ic a te i f t h e s e n te n c e s are c o r r e c t (C ) o r in c o r r e c t (I).

^ 1. It is essential to com plete the first program before working on the (o th ers)

__ !__ 2. The waitress will bring you (th e another) bowl o f soup if you want.
_____ 3. You should pack another pair o f shoes in case that pair gets soaked.

_____ 4. It is difficult to find others workers who are w illing to work such lo n g hours.
_____ 5. Since the lamp you wanted is out o f stock, you m ust choose another.

____ _ 6. T he other desk clerk must have put that message in your mailbox.
_____ 7. If your identification card is lost or stolen, you cannot g et another.

_____ 8. Because they were n ot pleased with the hotel accom m odations last year, they have
decided to try a other hotel this year.

_____ 9. As som e students m oved into the registration area, others took their places in line.

_____ 10. The printer will not function unless it has another cartridges.

E X E R C ISE (S k ills 5 8 - 6 0 ) : C ircle th e w o r d s in th e fo llo w in g s e n te n c e s th a t are c o m m o n ly


c o n fu s e d o n th e T O E F L test. T h e n in d ic a te i f th e s e n te n c e s are c o r r e c t (C) o r in c o r r e c t
(I).

_____ 1. W hen the cars odom eter reached 100,000, she decided that it was time to buy
another car.
_____ 2. Every time som eon e does an error in the program, several extra hours o f work are
created.

_____ 3. Like the fashions shown in this m agazine, the fashions in the other m agazine are
quite expensive.
_____ 4. Because the main highway is crowded at this hour, the driver should try to find
another routes to the stadium.
_____ 5. Although the two signatures are supposed to be exactly the same, they are n ot at
all like.

_____ 6. T h e decorators did the shopping for the material and m ade curtains for the
windows.

_____ 7. Before the administrator reads the stack o f papers on his desk, he should sign the
others that are on the file cabinet.
_____ 8. T he com m ittee is d oin g the arrangem ents for the Saturday evening banquet.
_____ 9. W hen he m ade several other big mistakes, he did his apologies to the others in
the office.
_____ 10. Perhaps the designer could select others styles if these are inappropriate.
WRITTEN EXPRESSION 257

T O E F L E X E R C ISE (S k ills 5 8 - 6 0 ) : C h o o s e th e le tte r o f th e u n d e r lin e d w o r d o r g r o u p o f


w ord s th at is n o t c o rr ec t.

____ 1.The buffalo and the bison are like except for the size and shape of the head and
X B C TT
shoulders.

_____ 2. Other interesting aspect of tachistopic training in recent years has been the
S B C
newfound use by professional teams.
TT

_____ 3. Only about 3 percent of oil wells actually do a profit.


A B C ~D

_____ 4. Dislike sumac with red berries, sumac with white berries is poisonous.
A C D

_____ 5. Pittsburgh has reduced its sm og by requiring more complete oxidation of fuel in
X B
cars, and others cities can do the same thing.
C TT

____ 6. Alike all other mammals, dolphins have lungs.


"A B C~ ~T5~

_____ 7. Up to World War II almost all important research in physics had been made in
~ A B
universities, with only university funds for support.
C D

_____ 8. Because the plan that was made yesterday is no longer feasible, the manager had to
A B C
choose another alternatives.
D~~ '

_____ 9. Particles w ith unlike charges attract each other, while particles with alike charges
A ~B~' ~~C~
repel each other.

_10. One another surprising method of forest conservation is controlled cutting o f trees.
X" B " "C D
258 STRUCTURE AND WRITTEN EXPRESSION

T O E F L REV IEW E X ER C ISE (S k ills 1 -6 0 ): C h o o se th e le tte r o f th e w o r d or g r o u p o f


w ord s th^ t b e st c o m p le te s th e s e n te n c e .

1. Wild Bill H ickok_____ for the Union Army 2. --------was unusable as farmland and
during the Civil War by posing as a difficult to traverse, the Badlands is an
Confederate officer. area in South Dakota.
(A) spied (A) So named because it
(B) spying (B) Because of
(C) a spy (C) It
(D) was spied (D) Naming it

C h o o se th e le tte r o f th e u n d e r lin e d w ord o r g r o u p o f w ord s th at is n o t co rr ec t.

_____ 3. Titania, photographed by Voyager 2 in 1986, has significantly fewer craters than
A B C
another moons of Uranus.

4. The author Francis Scott Key Fitzgerald is better know as F. Scott Fitzgerald.
A B C- "D

5. The result of the failure to plan for the future is that a child from an urban area
A B
must be took to the country to see nature.
D

6. This machine can print on a single pieces of paper, but only if the level is facing the
B C D
front of the machine.

_ 7. The development of permanent teeth, alike that of deciduous teeth, begins before
A B~ C
birth .

.8 . A crowd o f several hundred fan watched the ceremony from behind a fence.
A 13 C D

_ 9. Unlike other architects of the early modem movement, Alvar Aalto stressed
A B-
informality, personal expression, romantic, and regionality in his w ork.
C D

_10. Color blindness may exist at birth or may occur later in life as a result for disease or
A B~ C D
in ju ry .
2 9 2 2 * 2 2 * 2 22
TOEFL POST-TEST
SECTION 2
STRUCTURE AND WRITTEN EXPRESSION
l i m e 25 m in u t e s
( i n c l u d i n g t h e r e a d in g o f t h e d ir e c t i o n s )
N o w s e t y o u r c lo c k f o r 25 m in u t e s .

This section is designed to measure your ability to recognize language that is appropriate for standard
written English. There are two types of questions in this section, with special directions for each type.

S tru c tu re

D irections: These questions are incomplete sentences. Beneath each sentence you will see four
words or phrases, marked (A), (B), (C), and (D). Choose the one word or phrase that best com pletes
the sentence. Then, on your answer sheet, find the number of the question and fill in the space that
corresponds to the letter of the answer you have chosen.
Look at the following examples.

Exam ple I Sam p le Answer

The president_____ the election by a landslide

(A) won
(B) he w on
(C) yesterday
(D) fortunately

The sentence should read, "The president won the election by a landslide." Therefore, you should
choose answer (A).

Exam ple II S am p le Answer

W hen_____ the conference?

(A) the doctor attended


(B) did the doctor attend
(C) the doctor will attend
(D) the doctors attendance

The sentence should read, When did the doctor attend the conference?" Therefore, you should
choose answer (B).

I
GO ON TO THE NEXT PAGE
TOEFL* test directions and form at are rep rin ted by perm ission r
of ETS, the copyright owner. However, ali exam ples and test
questions are provided by P earson E ducation, Inc. STRUCTURE AND WRITTEN EXPRESSION POST-TEST 259
2 * 2 * 2 * 2 * 2 * 2 * 2*2
1. The planet Mercury. . rotations during 7. Created by the dissolution of limestone, the
every two trips around the Sun. underground cave system _____ Mammoth
Cave is noted for its stalactites and
(A) three complete stalagmites.
(B) completes three
(C) the completion of three (A) is known as
(D) completing three of the (B) it is known to be
(C) known as
2. In prehistoric. . of western Utah was (D) to be known
covered by Lake Bonneville.
8. Most slang terms are simply old words
(A) times, a large part additional new meanings.
(B) times, there was a large part
(C) part of the time (A) give
(D) for large parts of time (B) given
(C) are given
3. The helicopter is able to hover in . (D) they are given
powered rotors produce lift even at zero
forward speed. 9. North Carolina's Outer Banks are a chain of
low, narrow islands_____ the mainland
(A) flight because of the from the frequent Atlantic storms in the
(B) flying the area.
(C) the flying of the
(D) flight because the (A) they buffer
(B) that buffer
The upper levels of the Sun's atmosphere (C) to buffer them
are of very lo w _____ heats the gases there (D) that they buffer
to very high temperatures.
10. It is at the age of approximately eighteen
(A) dense and solar m onth s_____children begin to make
(B) density, solar activity combinations of two or three words.
(C) density, but solar activity
(D) density and activity of the Sun is (A) when many
(B) when are many
5. Lapis lazu li,. . stone, has been valued (C) when do many
for ornamental purposes for more than (D) when have many of the
6,000 years.
11. Story o f a Bad Boy, a semiautobiographical
(A) an opaque deep blue novel by Thomas Bailey Aldrich, ranks high
(B) is an opaque deep blue among books_____ have incorporated their
(C) it is an opaque deep blue boyhood experiences.
(D) that is an opaque deep blue
(A) the American authors
6. M ountaineers. _climb Mount Everest (B) which are American authors
must make reservations to do so, often up to (C) in which American authors
seven years in advance. (D) are those which American authors

(A) want to
(B) they want to
(C) who want
(D) wanting to

260 STRUCTURE AND WRITTEN EXPRESSION POST-TEST


2 2 ^ 2 ^ 2 ^ 2 2 ^ 2 2
12. In the La Brea tar pits of Los A ngeles------- 14. Not o n ly _____ more brittle than hard
which have been preserved from the maples, but they are also less able to
Pleistocene period. withstand high winds.

(A) thousands of animals are (A) soft maples are


(B) thousands are animals (B) are soft maples
(C) the thousands of animals (C) they are soft maples
(D) are thousands of animals (D) soft maples

13. _____provided aliv in g fo rn ea rly 9 0 15. _____ become blocked so that heat and
percent of the population of the American moisture could not escape, death would
colonies. result.

(A) Farming was what (A) Were the skins pores to


(B) What farming (B) The pores of the skin were to
(C) Farming was (C) The skins pores
(D) What was farming (D) If the pores of the skin

STRUCTURE AND WRITTEN EXPRESSION POST-TEST 26 I


2 * 2 2 2 2 2 * 22
W ritten Expression

Directions: In these questions, each sentence has four underlined words or phrases. The four
underlined parts of the sentence are marked (A), (B), (C), and (D). Identify the one underlined word
or phrase that must be changed in order for the sentence to be correct. Then, on your answer sheet,
find the number of the question and fill in the space that corresponds to the letter of the answer you
have chosen.
Look at the following examples.

Exam ple I Sam ple Answer


The four string on a violin are tuned
~ B ~C D~
in fifths. (D)

The sentence should read, "The four strings on a violin are tuned in fifths." Therefore, you should
choose answer (B).

Exam ple II Sam ple Answer


The research for the book Roots taking (B)
\ B- C #
Alex Haley twelve years.
D

The sentence should read, "The research for the book Roots took Alex Haley twelve years." Therefore,
you should choose answer (C).

TOEFL* test directions and form at arc reprinted by perm ission


~ _ ____ ______ _____ ___ o f ETS, the copyright owner. However, all exam ples and test
262 STRUCTURE AND \WRITTEN EXPRESSION POST-TEST questions arc provided by Pearson E ducation. Inc.
16. The wave lengths of ultraviolet light are short than those of visible light but longer
A B C D
than those of X-rays.

17 All thoroughbreds are descended from three Arabian stallion imported into England
X B D
between 1689 and 1724.

18. By measuring the rate of decay of potassium isotopes in volcanic ash, scientists
A
can date the lavers of volcanic ash and any human remains in they.
-----B----- C D

19. Hundreds of partial to complete fossil skeletons of Triceratops have been gather in
A B C
North America from rocks of the late Cretaceous period.
D

20. By the time of the dinosaurs, turtles have already developed the hard shell
A B
into which their heads and legs could be drawn.
------ C------ D
21. A zoom lens produces an inverted real image,either on the film in a camera and on
A B C D
the light-sensitive tube of a television camera.

22 The leaves and young twigs of the henna plant are ground into a powder to produce a
A B
paste that can used as a dye.
C D
23. Thirty-one pairs of spinal nerves arepresent in humans, and each pair have two
A B C D
roots

24 William Randolph Hearst built a chain of newspapers that included 25 dailies and 11
~ B C
Sunday editions at their peak in 1937.
D
25 The electromagnetic spectrum consists in bands o f different wavelengths.
A B C D

E E *

STRUCTURE AND W RITTEN EXPRESSION POST-TEST


2 2 2 2 2 * 2 * 2 rz

26. Lemon trees are similar in longevity and appear to orange trees but have more upright
A B C
growth.
D

27. Christopher Columbus, alike many other explorers, underestimated the size of the
A T T C~
Earth and overestimated the width of Asia.
D

28. Manganese, found in trace amounts in higher animals, activates a large amount of
A B
the enzymes involved in metabolic processes.
C D

29. The remains of Homo erectus, an extinct species of early man, was first discovered
X B "TT D
on the island of Java by Dutch physician Eugene Debois.

30. The Ford Motor Company introduced the moving assembly line in 1914 so that it will
A TT
be able to meet the huge demand for its Model T.
' C "D

31. By 1830, approximately 200 steamboats had become operationally on the


A B C D
Mississippi River.

32. The huge Meteor Crater was created when a 63,000-ton iron meteorites struck the
A B C D
Earth near Winslow, Arizona.

33. Daniel Boone helped to build the Wilderness Road through the Cumberland Gap,
A B
creating a route for settlers heading westerly.
C -----D-----

34. The Appalachian Mountains extend Georgia and Alabama in the south to Canada in
A B C "D
the north.

35. Howard Hughes once did more than half a billion dollars in one day in 1966 when he
A B
received a single bank draft for $546,549,171 for his share of TWA.
C D-----

9 /
264 STRUCTURE AND WRITTEN EXPRESSION POST-TEST
2 * 2 2 * 2 * 2 * 2 22
36. The city of Tam pa, Florida, is located on peninsula across Tam pa Bay from S aint
A B C D
Petersburg.

37. The closer it gets to D ecem ber 21, the first day of winter, the short the davs becom e
A B ~ C ~ D
38. Only about a hundred o u t of an estim ating 3,000 know n m ineral species
A
have been found at least reasonably suitable for use as gems
B C------- ~ D ~

39. Most of the y ear San Miguel Island is shrouded in fog, and strong northw est w inds
A B
b a tte r relentlessly the island.
C D

40. W omen have adm itted to the United States M ilitary Academy a t West Point since

1976, and the first w om en cadets graduated in 1980.


C D

This is the end of Section 2.


If you finish before 25 minutes has ended,
check your work on Section 2 only.

(s t o p ) (s t o p) (s t o p ) (s to p ] (s t o p) 0 ( 0 )

W h e n y o u fin ish th e test, y o u m ay d o th e fo llo w in g :


* T u rn to th e D ia g n o s tic C h a r t o n p a g e s 5 8 5 -5 8 7 , a n d
c ir c le th e n u m b e r s o f th e q u e s tio n s th a t y o u m isse d .
* T u rn to S c o r in g I n fo r m a tio n o n p a g e s 5 8 1 -5 8 2 , a n d
d e te r m in e y o u r T O E F L sc o r e.
* T u rn to th e P r o g r e s s C h a r t o n p a g e 5 9 1 , a n d ad d y o u r
sc o r e to th e chart.

STRUCTURE AND WRITTEN EXPRESSION POST-TEST 26S


266
SECTION THREE

READING
COMPREHENSION
268
3 a 3 a 3 a 3 a 3 a 3 a 3 a 3
DIAGNOSTIC PRE-TEST
SECTION 3
READING COMPREHENSION
T im e 55 m in u t e s
( i n c l u d i n g t h e r e a d in g o f t h e d ir e c t i o n s )
N o w s e t y o u r c lo c k f o r 55 m in u t e s .

This section is designed to m easure y our ability to read and understand sh o rt passages sim ilar in
topic and style to those that students are likely to encounter in N orth Am erican universities and
colleges. This section contains reading passages and questions about the passages.

D irectio n s: In this section you will read several passages. Each one is followed by a n u m b er of
questions about it. You are to choose the o n e best answer, (A), (B), (C), o r (D), to each question. Then,
on your answ er sheet, find the num ber o f the question and fill in the space that corresponds to the
letter of the answ er you have chosen.
Answer all questions about the inform ation in a passage on the basis o f w hat is s ta te d o r im p lie d
in that passage.
Read the following passage:

John Quincy Adams, who served as the sixth president of the United S tates from 1825
to 1829, is today recognized for his m asterful statesm anship and diplomacy. He dedicated his
life to public service, b oth in the presidency and in the various ot.'-er political offices th at he
Line held. T hroughout hi ; political career he dem onstrated hie unsw ervi ig belief in freedom of
(5) speech, the antislave -y cause, uik 1 the l ight of A m ericans u; be fre from E u ro p ean and Asian
dom ination.

E x am p le I Sam ple Answer

To w hat did John Quincy Adams devote hi ; life? (E )


(A) Im proving his personal life
(B) Serving the public
(C) Increasing his fortune
(D) W orking on his private business

According to the passage, John Quincy Adams "dedicated his life to public service. T herefore, you
should choose answ er (B).

E x am p le II Sam ple Answer

In line 4, the w ord "unswerving is closest in m eaning to


(A) m oveable
(B) insignificant
(C) unchanging
(D) diplom atic

The passage states th a t John Quincy Adams dem onstrated his unsw erving belief thro u g h o u t his
career. This im plies th at the belief did not change. Therefore, you should choose answ er (C).

______________ k
GO ON TO THE NEXT PAGE
TOEFL test directions and form at are reprinted by perm ission
of ETS. th e copyright owner. However, all exam ples and test
questions are provided by P earson Education. Inc. READING CO M PREH ENSIO N PRE-TEST 269
3 a 3 a 3 a 3 a 3 a 3 a 3 a 3
Q u e s t io n s 1 - 1 2

Algae is a prim itive form of life, a single-celled o r sim ple m ultiple-celled organism th at is able to
conduct the process of photosynthesis. It is generally found in w ater but can also be found elsewhere,
growing on such surfaces as rocks o r trees. The various types of algae are classified accoiding to their
Line pigm entation, o r coloration.
(5) Blue-green algae, o r Cyanophyta, can grow at very high tem p eratu res and u n d e r high-intensity
light. This is a m icroscopic type of algae, a n d some species consist of only one cell. Blue-gree algae is
the oldest form of life w ith photosynthetic capabilities, and fossilized rem ains of this type of algae
m ore th an 3.4 billion years old have been found in p a rts of Africa.
Green algae, or Chlorophyta, is generally found in fresh w ater. It reproduces on the surfaces of
( 10 ) enclosed bodies of w ater such as ponds o r lakes and has the appearance of a fuzzy green coating on
the water. In large quantities, this type of algae may reproduce enough to give a green color to an
entire lake. .
Brown algae, or Phaeophyta, grows in shallow, tem perate water. This type of algae is the largest
in size and is m ost recognizable as a type of seaweed; kelp is a type of brow n algae th at has grown to
(15) lengths of up to 200 feet. Its long stalks can be enm eshed on the ocean floor, o r it can float freely on
the oceans surface.
Red algae, or Rhodophyta, is a sm all, delicate organism found in the deep w aters of the
subtropics, w here it often grows w ith coral. This type of algae has an essential role in the form ation of
coral reefs: it secretes lim e from the seaw ater to foster the form ation of lim estone deposits.

1. What is the authors main purpose? 5. Algae rem nants found in Africa are

(A) To show what color algae is (A) still flourishing


(B) To differentiate the various (B) photogenic
classifications of algae (C) extrem ely old
(C) To describe where algae is found (D) red in color
(D) To clarify the appearance of the
different types of algae 6. G reen algae is generally found
(A) on the ocean floor
2. Which of the following is NOT true about (B) on top of the w ater
algae? (C) throughout ponds and lakes
(A) All types have only one cell. (D) surrounding enclosed bodies of w ater
(B) It can be found out of water.
(C) It can use photosynthesis. 7. The word "coating in line 10 could best be
(D) It is not a relatively new form of life. replaced by
(A) clothing
3. The word "pigmentation" in line 4 means
(B) covering
(A) size (C) w arm th
(B) shape (D) sw eater
(C) composition
(D) color 8. B row n algae w ould m ost likely be found
(A) on trees
4. The word "microscopic in line 6 is closest
(B) near green algae
in meaning to
(C) on rocks
(A) mechanical (D) in the ocean
(B) tiny
(C) visual

mam
(D) bacterial

270 READING COMPREHENSION PRE-TEST


3 a 3 a 3 a 3 a 3 a 3 a 3 a 3
9. According to the passage, red algae is 11. How is the inform ation in the paragraph
(A) sturdy organized?
(B) huge (A) Various details supporting a theory
(C) fragile are explored.
(D) found in shallow w ater (B) Various classifications of a specific
life form are described.
10. It can be inferred from the passage that (C) Various stages of the chronological
lim estone deposits serve as the basis of developm ent of a life form are
(A) coral reefs presented.
(B) red algae (D) Various elem ents th a t com pose a
(C) subtropical seaw ater certain life form are outlined.
(D) secret passages
12. This passage w ould m ost probably be
assigned reading in a course on
(A) chem istry
(B) physics
(C) botany
(D) zoology

READING COMPREHENSION PRE-TEST 271


3 a 3 a 3 a 3 a 3 a 3 a 3 a 3
Q u e s t io n s 1 3 -2 1

Narcolepsy is a disease characterized by m alfunctioning sleep m echanics. It can consist of a


sudden and uncontrollable bout of sleep during daylight hours and disturbed sleep during nighttim e
hours. It occurs m ore often in m en th an in w om en, and it com m only m akes its appearance during
Line adolescence o r young adulthood. At least a half m illion Am ericans are believed to be affected by
(5)
N arcolepsy can take a n um ber of form s during daylight hours. One com m on sym ptom of the
disease during daytim e hours is a sudden attack of REM (rapid-eye m ovem ent) sleep during norm a,
waking hours. This occurs in som e people hundreds of tim es in a single day, w hile o thers only have
rare occurrences. D uring a sleep attack, narcoleptics may experience autom atic behavior; even
( 10) though asleep, they m ay continue autom atically perform ing the activity they w ere involved in p rio r to
falling asleep. They may, for exam ple, continue walking, or driving, o r stirring a pot until the activity
is interrupted by external forces. Others experience cataplexy during daytim e hours; cataplexy
involves a sudden loss of muscle tone th a t m ay cause the head to droop o r the knees to wobble m
m in o r attacks o r a total collapse in m ore serious attacks. Cataplexy seem s to occur m ost often in
(15) conjunction w ith intense em otion or excitem ent.
D uring sleep hours, narcolepsy can also m anifest itself in a variety of ways. During the
transitional phase th at precedes the onset of sleep, it is com m on for hallucinations to occur. These
hallucinations, known as hypnagogic phenom ena, consist of realistic perceptions of sights and
sounds during the sem i-conscious state betw een w akefulness and sleep. N arcoleptics m ay also suffer
(20 ) from night w akening during sleep, resulting in extremely fragm ented and restless sleep. Then, upon
waking, a narcoleptic may experience sleep paralysis, the inability to move, perhaps for several
m inutes, im m ediately after waking.

13. W hich of the following would be the m ost 16. Approxim ately how m any narcoleptics are
appropriate title for the passage? there in the United States?

(A) A G ood N ights Sleep (A) Fewer than 500,000


(B) A Cure for Narcolepsy (B) More th an 500,000
(C) An U nusual Sleep D isturbance (C) Fewer than 1,500,000
(D) H allucinations during Sleep (D) More th an ! ,500,000

14. The w ord "m alfunctioning" in line 1 is 17. The word "bout in line 2 is closest in
closest in m eaning to m eaning to

(A) im properly working (A) sym ptom


(B) regularly waking (B) lack
(C) incorrectly classifying (C ) illness
(D) harshly inte,preting (D) period

15. At w hich of the following ages w ould a 18. W hich of the following w ould be m ost
person be m ost likely to develop likely to occur during daily activities?
narcolepsy? (A) Autom atic behavior
(A) 10 (B) Hallucinations
(B) 20 (C) Night wakening
(C) 30 (D) Sleep paralysis
(D) 40

272 READING COMPREHENSION PRE-TEST


3 a 3 a 3 a 3 a 3 a 3 a 3 a 3
19. Which of the following involves a complete 21. Where in the passage does the author
collapse? describe what seems to precipitate a
sudden loss of muscle tone?
(A) Automatic behavior
(B) Cataplexy (A) Lines 12-14
(C) Hallucinations (B) Lines 1415
(D) REM sleep (C) Lines 16-17
(D) Lines 20-22
20. When would hypnagogic phenomena most
likely occur?
(A) Just after going to bed
(B) In the middle of the night
(C) Soon after waking
(D) After getting up

READING COMPREHENSION PRE-TEST 273


3 a 3 a 3 a 3 a 3 a 3 a 3 a 3
Questions 22-30
Whereas literature in the first half of the eighteenth century in America had been largely
religious and moral in tone, by the latter half of the century the revolutionary fen/or that was coming
to life in the colonies began to be reflected in the literature of the time, which in turn served to further
Line influence the population. Although not all writers of this period supported the Revolution, the two
(5) best-known and m ost influential writers, Ben Franklin and Thomas Paine, were both strongly
supportive of that cause. . . . . u
Ben Franklin first attained popular success through his writings in his brothers newspaper, the
New-England Courant. In these articles he used a simple style of language and commonsense
argumentation to defend the point of view of the farmer and the Leather Apron man. He continued
( 10) with the same commonsense practicality and appeal to the common man with his work on Poor
Richard's Almanac from 1733 until 1758. Firmly established in his popular acceptance by the people,
Franklin wrote a variety of extremely effective articles and pamphlets about the colonists
revolutionary cause against England. . . . . , . f
Thomas Paine was an Englishman working as a magazine editor in Philadelphia at the time of
(1 5 ) the Revolution. His pamphlet Common Sense, which appeared in 1776, was a force in encouraging
the colonists to declare their independence from England. Then throughout the long and desperate
war years he published a series of Crisis papers (from 1776 until 1783) to encourage the colonists to
continue on with the struggle. The effectiveness of his writing was probably due to his emotional yet
oversimplified depiction of the cause of the colonists against England as a classic struggle of good
( 20 ) and evil.

22. The paragraph preceding this passage 25. It is implied in the passage that
most likely discusses
(A) som e writers in the American colonies
(A) how literature influences the supported England during the
population Revolution
(B) religious and moral literature (B) Franklin and Paine were the only
(C) literature supporting the cause of the writers to influence the Revolution
American Revolution (C) because Thomas Paine was an
(D) what made Thomas Paines literature Englishman, he supported England
successful against the colonies
(D) authors who supported England did
The word "fervor in line 2 is closest in not remain in the colonies during
meaning to the Revolution

(A) war 26. The pronoun "he" in line 8 refers to


(B) anxiety
(C) spirit (A) Thomas Paine
(D) action (B) Ben Franklin
(C) Ben Franklins brother
The word "time in line 3 could best be (D) Poor Richard
replaced by
27. According to the passage, the tone of Poor
(A) hour R ichards Almanac is
(B) period
(C) appointment (A) pragmatic
(D) duration (B) erudite
(C) theoretical
(D) scholarly

274 READING COMPREHENSION PRE-TEST




3 a 3 a 3 a 3 a 3 a 3 a 3 a 3
The word "desperate" in line 16 could best 30. The purpose of the passage is to
be replaced by
(A) discuss American literature in the
(A) unending first half of the eighteenth century
(B) hopeless (B) give biographical data on two
(C) strategic American writers
(D) combative (C) explain which authors supported the
Revolution
Where in the passage does the author (D) describe the literary influence during
describe Thomas Paines style of writing? revolutionary America

(A) Lines 4-6


(B) Lines 8-9
(C) Lines 14-15
(D) Lines 18-20

READING COMPREHENSION PRE-TEST 275


3 a 3 a 3 a 3 a 3 a 3 a 3 a 3
Questions 3 1-4 1
Federal Express is a company that specializes in rapid overnight delivery o f high-priority
packages. The first company of its type, Federal Express was founded by the youthful Fred Smith in
1971, when he was only 28 years old. Smith had actually developed the idea for the rapid delivery
Line service in a term paper for an economics class when he was a student at Yale University. The term
(5) paper reputedly received a less-than-stellar grade because of the infeasibility of the project that Smith
had outlined. The model that Smith proposed had never been tried; it was a model that was efficient
to operate but at the same time was very difficult to institute.
Smith achieved efficiency in his model by designing a system that was separate from the
passenger system and could, therefore, focus on how to deliver packages most efficiently. His strategy
(10) was to own his own planes so that he could create his own schedules and to ship all packages through
the hub city of Memphis, a set-up which resembles the spokes on the wheel of a bicycle. With this
combination of his own planes and hub set-up, he could get packages anywhere in the United States
overnight.
What made Smiths idea difficult to institute was the fact that the entire system had to be
(15) created before the company could begin operations. He needed a fleet of aircraft to collect packages
from airports every night and deliver them to Memphis, where they were immediately sorted and
flown out to their new destinations; he needed a fleet of trucks to deliver packages to and from the
various airports; he needed facilities and trained staff all in place to handle the operation. Smith had
a $4 million inheritance from his father, and he managed to raise an additional $91 million dollars
(20) from venture capitalists to get the company operating.
When Federal Express began service in 1973 in 25 cities, the company was not an immediate
success, but success did come within a relatively short period of time. The company lost $29 million
in the first 26 months of operations. However, the tide was to turn relatively quickly. By late 1976,
Federal Express was carrying an average of 19,000 packages per night and had made a profit of $3.6
(25) million. .

31. The most appropriate title for this passage 33. What is stated in the passage about Smith's
is term paper?
(A) The Problems and Frustrations of a (A) Smith submitted it through a deliver
Business Student service.
(B) The Importance of Business Studies (B) It was written by a student of Smith'
(C) The Capitalization of Federal Express (C) Its grade was mediocre.
(D) The Implementation of a Successful (D) The professor thought it had great
Business potential.

The word "developed in line 3 could best 34. What was a key idea of Smiths?
be replaced by
(A) That he should focus on passenger
(A) come up with service
(B) come about (B) That package delivery should be
(C) come across separate from passenger service
(D) come into (C) That packages could be delivered on
other companies' planes
(D) That passenger service had to be
efficient

GO ON TO THE NEXT PAGE


7
276 READING COMPREHENSION PRE-TEST
3a 3a 3a 3a 3a 3a 3a 3
35. A hub city in line 11 is 39. How long did it take Federal Express to
become profitable?
(A) a large city with small cities as
destinations (A) TWo months
(B) a city that is the final destination for (B) One year
many routes (C) Three years
(C) a city where many bicycle routes (D) Six years
begin
(D) a centralized city with destinations 40. Which paragraph explains what made
emanating from it Sm iths model effective?
(A) The first paragraph
36. It can be inferred from the passage that
(B) The second paragraph
Smith selected Memphis as his hub city
(C) The third paragraph
because it
(D) The last paragraph
(A) was near the middle o f the country
(B) had a large number of passenger 41 The tone of the passage in describing
aircraft Smiths accom plishm ents is
(C) already had a large package delivery
(A) unflattering
service
(B) sincere
(D) was a favorite passenger airport
(C) unconvincing
(D) snobbish
37. The pronoun "they" in line 16 refers to
(A) aircraft
(B) packages
(C) airports
(D) destinations

38. It is NOT mentioned in the passage that, in


order to set up his company, Smith needed
(A) airplanes
(B) trucks
(C) personnel
(D) faculty

READING COMPREHENSION PRE-TEST 277


3 A3 A 3 a 3 a 3 a 3 a 3 a 3
Q u e s t io n s 4 2 - 5 0

Perhaps better known than the Cullman Diamond is the Hope Diamond, a valuable and rare
blue gem with a background of more than 300 years as a world traveler. The 112-carat blue stone that
later became the Hope Diamond was mined in India sometime before the middle of the seventeenth
Line century and was first known to be owned by Shah Jahan, who built the Taj Mahal in memory of his
(5) beloved wife. From India, the celebrated blue stone has changed hands often, moving from location
to location in distant com ers of the world.
In the middle of the seventeenth century, a trader from France named Jean Baptiste Tavemier
acquired the large blue diamond, which was rumored to have been illegally removed from a temple.
Tavemier returned to France with the big blue gem, where the stone was purchased by the Sun King,
(10) Louis XIV. Louis XIV had it cut down from 112 to 67 carats to make its shape symmetrical and to
maximize its sparkle. The newly cut diamond, still huge by any standards, was passed down through
the royal family of France, until it arrived in the hands of Louis XVI and Marie Antoinette. During the
French Revolution, Louis XVI and his wife met their fate on the guillotine in 1793, and the big blue
diamond disappeared from public sight.
(15) The diamond somehow managed to get from France to England, where banker Henry Hope
purchased it from a gem dealer early in the nineteenth century. The huge blue stone was cut into a
45.5-carat oval, and at this point it took on the name by which it is known today. The diamond stayed
in the Hope family for around a century, when deep indebtedness brought on by a serious gambling
habit on the part of one of Henry Hopes heirs forced the sale of the diamond.
(20) From England, the Hope Diamond may have made its way into the hands of the Sultan of
Turkey; whatever route it took to get there, it eventually went on to the United States when American
Evelyn Walsh McLean purchased it in 1911. Mrs. McLean certainly enjoyed showing the diamond off;
guests in her home were sometimes astounded to notice the huge stone embellishing the neck of Mrs.
McLean's Great Dane as the huge pet trotted around the grounds of her Washington, D.c. home. The
(25) Hope Diamond later became the property o f jeweler Harry Winston, who presented the stunning 45.5-
carat piece to the Smithsonian in 1958. The Hope Diamond is now taking a well-earned rest following
its rigorous travel itinerary and is on display at the Smithsonian Institution in Washington, D.C.,
where it has been since 1958.

42. The paragraph preceding the passage m ost 44. The pronoun "it in line 12 refers to
likely discussed
(A) its shape
(A) why gems are considered valuable (B) the newly cut diamond
(B) how the Hope Diamond was mined (C) the royal family
(C) a diamond other than the Hope (D) the French Revolution
Diamond
(D) methods for mining diamonds 45. It can be inferred from the passage that the
author is not certain
43. The main idea of this passage is that the
(A) who bought the Hope Diamond in
Hope Diamond
England
(A) came from India (B) who sold the Hope Diamond in
(B) has moved around a lot England
(C) has been cut several times (C) how the Hope Diamond went from
(D) now resides in the Smithsonian France to England
(D) how big the Hope Diamond was in the
nineteenth century

278 READING COMPREHENSION PRE-TEST


3 a 3 a 3 a 3 a 3 a 3 a 3 a 3
46. A dealer" in line 16 is most likely a 49. Which country is NOT m entioned in the
passage as a place where the Hope
(A) card player Diamond spent some time?
(B) miner
(C) cutter (A) India
(D) businessman (B) France
(C) England
47. It can be determined from the passage that (D) Denmark
Henry Hope most likely had how many
carats cut off the Hope Diamond? 50. Where in the passage does the author
describe what happened to the royal
(A) 21.5 French owners of the diamond?
(B) 45.5
(C) 66.5 (A) Lines 7-8
(D) 67 (B) Lines 10-11
(C) Lines 12-14
48. According to the passage, Mrs. McLean (D) Lines 15-16

(A) donated the Hope Diamond to the


Smithsonian
(B) let her dog wear the Hope Diamond
(C) purchased the Hope Diamond from
the French
(D) had the Hope Diamond cut to its
present size of 45.5 carats

This Is the end of the Reading Comprehension Pre-Test.

W h e n y o u fin ish th e test, y o u m ay d o th e fo llo w in g :


T u r n to th e D iagnostic Chart o n p a g e s 5 8 8 - 5 9 0 , a n d
c ir c le th e n u m b e r s o f th e q u e s tio n s th a t y o u m issed .
T u r n to Scoring In form ation o n p a g e s 5 8 1 -5 8 2 , a n d
d e te r m in e you r T O E F L sco re.
T u r n to th e P rogress Chart o n p a g e 5 9 1 , a n d ad d y o u r
sc o r e to th e chart.

READING COMPREHENSION PRE-TEST 279


280
READING
COMPREHENSION

T h e th ird se c tio n o f th e T O E F L test is th e R ea d in g C o m p r e h e n s io n s e c tio n . T h is s e c tio n


c o n sists o f fifty q u e s tio n s (so m e tests m ay b e lo n g e r ). You h ave fifty-five m in u te s to c o m
p le te th e fifty q u e s tio n s in th is se c tio n .
In this part o f th e test y o u w ill b e giv en r e a d in g p assages, a n d y o u w ill b e a sk ed tw o
types o f q u e s tio n s a b o u t th e r e a d in g passages:

1. R e a d in g C o m p r e h e n s io n q u e s tio n s ask you to answ er q u e s tio n s a b o u t th e in fo r


m a tio n giv en in th e r e a d in g passages. T h e r e w ill be a variety o f q u e s tio n s a b o u t
e a c h r e a d in g p a ssa g e , in c lu d in g m a in id e a q u e s tio n s, d irectly a n sw e r e d d e ta il
q u e stio n s, a n d im p lie d d e ta il q u estio n s.
2. V ocabu lary q u e s tio n s ask you to id e n tify the m e a n in g s o f v o c a b u la r y w o r d s in th e
r e a d in g p a ssa g es. T o a n sw e r th e se q u estio n s, you m ay have to k n o w th e m e a n in g s
o f th e w ords. You c a n also id e n tify th e m e a n in g s o f s o m e o f th e w ord s by u n d e r
sta n d in g th e c o n te x t s u r r o u n d in g th e w ords, by u sin g stru ctu ra l c lu e s to id e n tify
th e m e a n in g s o f th e w ord s, o r by b rea k in g d o w n th e u n k n o w n w o rd s in to k n o w n
w ord parts in o r d e r to id e n tify th em .

G EN ERAL STRATEGIES

1. Be fam iliar w ith th e d irectio n s. The directions on every TOEFL test are the same, so it is
not necessary to spend time reading the directions carefully when you take the test. You
should be completely familiar with the directions before the day of the test.

2. D o n o t sp en d to o m uch tim e reading th e passages! You do not have time to read each
reading passage in depth, and it is quite possible to answer the questions correctly without
first reading the passages in depth. Some students prefer to spend a minute or two on each
passage reading for the main idea before starting on the questions. Other students prefer to
move directly to the questions without reading the passages first.

3. D o n o t w orry if a reading p assage is on a top ic th a t you are unfam iliar w ith . All of
the information that you need to answer the questions is Included in the passages. You do
not need any background knowledge to answer the questions.

4. N ever leave any an sw ers blank on your answ er sh ee t. Even If you are unsure of the
correct response, you should answer each question.There is no penalty for guessing.

281
READING COMPREHENSION

__t h e r e a d in g c o m p r e h e n s io n q u e s t io n s

T h e R e a d in g C o m p r e h e n sio n se c tio n o f th e T O E FL test c o n sists o f five r ea d in g passages,


c a c h fo llo w e d by a n u m b e r o f r ea d in g c o m p r e h e n sio n a n d v o ca b u la ry q u estio n s. T o p ics
o f th e r ea d in g p a ssa g e are varied, b u t they are o fte n in fo r m a tio n a l su b jects that m ig h t be
stu d ied in an A m erica n university: A m erica n history, litera tu re, art, a rch ite ctu re , g eo lo g y ,
g e o g ra p h y , a n d astronom y, for ex a m p le.
" T im e is d e fin ite ly a factor in th e R ea d in g C o m p r e h e n s io n se c tio n . M any stu d en ts
w h o take th e T O E F L test n o te that they are u n a b le to fin ish all th e q u e stio n s in this sec
tio n . T h e r e fo r e , y o u n e e d to m ak e th e m o st e ffic ie n t u se o f y o u r tim e in this se c tio n to
g e t th e h ig h e st sc o r e. T h e fo llo w in g m e th o d is th e b e st way o f a tta ck in g a r ea d in g p assage
to g et th e m o st q u e stio n s co rr ec t in a lim ited a m o u n t o f lim e .

STRATEGIES FO RT H E READING C O M PREH EN SIO N Q UESTIO N S

I. Skim th e reading passage to d e te rm in e th e m ain idea and th e overall


o rgan ization o f ideas in th e passage. You do not need to understand every detail ,n each
passage to answer the questions correctly. It is therefore a waste of time to read the passage
with the intent of understanding every single detail before you try to answer the questions.

1. Look ahead a t th e qu estion s to d e te rm in e w h a t ty p es o f q u estio n s you m ust


answer. Each type of question is answered in a different way.
3. Find th e se c tio n o f th e passage th a t deals w ith each qu estio n .T h e question-type
tells you exactly where to look in the passage to find correct answers.

For m ain idea questions, look at the first line of each paragraph.
For directly and indirectly answered d eta il questions, choose a key word in the question,
and skim for that key word (or a related idea) in order in the passage.

For vocabulary questions, the question will tell you where the word is located in the
passage.
For overall review questions, the answers are found anywhere in the passage.

4. Read th e p art o f th e passage th a t contains th e answ er carefully. The answer will


probably be in the same sentence (or one sentence before or after) the key word or idea.

5. C h o o se th e b est answ er to each qu estion from th e four answ er ch o ices listed in


you r t e s t b ook . You can choose the best answer according to what is given in the
appropriate section of the passage, eliminate definitely wrong answers, and mark your best
guess on the answer sheet.

T h e fo llo w in g sk ills will h e lp you to im p le m e n t th e se stra teg ies in th e R ea d in g C o m p re


h e n s io n se c tio n o f th e T O E FL test.
READING COMPREHENSION

Q U E S T IO N S A B O U T T H E IDEAS O F T H E PASSAGE__________

It is very c o m m o n for r e a d in g p a ssa g es in th e R e a d in g C o m p r e h e n s io n s e c tio n o f th e


T O E F L test to have q u e s tio n s a b o u t th e overall id ea s in th e p a ssa g e. T h e m o st c o m m o n
type o f q u e s tio n asks a b o u t th e m ain id e a , to p ic , title, o r su b ject. O cc a sio n a lly , th e r e w ill
also b e a q u e s tio n a b o u t h ow th e in fo r m a tio n in the p a ssa g e is o r g a n iz e d .

S k il l I: A N S W E R M A IN ID EA Q U E S T IO N S CORRECTLY

A lm o st every r e a d in g p a ssa g e o n th e T O E F L test w ill h ave a q u e s tio n a b o u t th e m a in id e a


o f a passage. S u ch a q u e s tio n m ay be w o r d e d in a variety o f ways; you m ay, for e x a m p le , b e
ask ed to id e n tify th e topic, subject, title, prim ary idea, o r m ain idea. T h e s e q u e s tio n s are all
really ask in g w h at p rim ary p o in t th e a u th o r is tryin g to g e t across in th e p a ssa g e. S in c e
T O E F L p assages are g e n e r a lly w ritten in a tr ad ition ally o r g a n iz e d m a n n e r , it is n o t d iffi
c u lt to fin d th e m ain id e a by stu d y in g th e to p ic s e n te n c e , w h ich is m o st p r o b a b ly fo u n d at
th e b e g in n in g o f a p aragrap h .
If a p assage c o n sists o f o n ly o n e p a ra g ra p h , you s h o u ld stu d y th e b e g in n in g o f th a t
paragrap h to d e te r m in e th e m a in id ea.

Exam ple I

The passage:

In the philosophy o f John Dewey, a sharp distinction is m ade


betw een intelligence" and "reasoning. According to Dewey,
intelligence is the only absolute way to achieve a balance betw een
Line realism and idealism , betw een practicality a n d w isdom of life.
(5) Intelligence involves "interacting with o th er things a n d know ing
them ," while reasoning is merely the act o f an observer, " . . . a m ind
that beholds o r grasps objects outside the world of things. . . ."
With reasoning, a level of m ental certainty can be achieved, but it
is through intelligence th at control is taken of events that shape
(10) one's life.
The question:
W hat is the topic of this passage?
(A) The intelligence of John Dewey
(B) D istinctions m ade by John Dewey
(C) Deweys ideas on the ability to reason
(D) How intelligence differs from reasoning in Dewey's works

T h e first s e n te n c e o f th is p a ssa g e d iscu sses a d istin c tio n b e tw e en th e id e a s o f in t e llig e n c e


an d r e a s o n in g in th e p h ilo s o p h y o f J o h n Dewey, so th is is p rob ab ly th e to p ic . A q u ick
c h e c k o f th e rest o f th e s e n te n c e s in th e p assage c o n fir m s that th e to p ic is in fa c t th e d if
fe r e n c e b e tw e en in t e llig e n c e a n d r e a s o n in g . N o w you sh o u ld c h e c k e a c h o f th e a n
swers to d e te r m in e w h ic h o n e c o m e s c lo sest to th e to p ic th a t y o u h ave d e te r m in e d .
A n sw er (A) m e n tio n s o n ly in te llig e n c e , so it is n o t th e top ic. A n sw er (B ) m e n tio n s d istin c
tion s th a t J o h n D ew ey m akes, b u t it d o e s n o t say sp ecifica lly w h at type o f d istin c tio n s. A n
sw er (C ) m e n tio n s o n ly r e a s o n in g , so answ er (C ) is in c o m p le te . T h e b e st an sw er is
th e r e fo r e ( D ) ; th e id e a o f how intelligence differs fro m reasoning c o m e s fr o m th e first s e n te n c e
o f th e p assage, w h ich m e n tio n s a sharp distinction . . . between intelligence a n d "reasoning.
284 READING COMPREHENSION

I f a p assage c o n sists o f m o r e th an o n e p aragrap h , you sh o u ld stu d y th e b e g in n in g o f


e a c h paragrap h to d e te r m in e th e m ain idea.

E x am p le II

T h e p assag e:
Nitrogen fixation is a process by w hich nitrogen is
continuously fed into biological circulation. In this process, certain
algae and bacteria convert nitrogen into am m onia (NH,). This
Line newly created am m onia is then for the m ost p art absorbed by
(5) plants.
The opposite process of denitrification re tu rn s n itrogen to
the air. During the process of denitrification, bacteria cause some
o f the nitrates from the soil to convert into gaseous nitrogen or
nitrous oxide (N20). In this gaseous form the nitrogen re tu rn s to the
( 10) atm osphere.

T h e q u e stio n :
W hich of the following would be the best title for this passage?
(A) The Process of Nitrogen Fixation
(B) Two Nitrogen Processes
(C) The Return of Nitrogen to the Air
(D) The Effect of Nitrogen on Plant Life

In a p a ssa g e w ith m o r e th a n o n e p aragrap h , you s h o u ld b e su re to read th e first s e n te n c e


o f e a c h p a ra g ra p h to d e te r m in e th e su bject, title, o r m a in id ea . In E x a m p le II, th e first
s e n te n c e o f th e first paragraph in d ic a tes th at th e first paragrap h is a b o u t th e p r o c ess o f
n itr o g e n fix a tio n . I f you lo o k o n ly at th e first p aragrap h , you m ig h t c h o o s e th e in c o r r e c t
an sw er (A ), w h ic h w o u ld b e a g o o d title fo r th e first paragrap h only. T h e first s e n te n c e o f
th e s e c o n d paragrap h in d ic a tes th a t th e p ro cess o f d e n itr ific a tio n is d isc u sse d in th e se c
o n d p aragrap h . A n sw er (C ) is in c o r r e c t b e c a u se the return o f nitrogen to the air is th e
p r o c ess o f d e n itr ific a tio n , a n d this is d isc u sse d in th e s e c o n d p aragrap h only. A n sw er (D )
is in c o r r e c t b e c a u s e the effect o f nitrogen on p la n t life is n o t d isc u sse d in th is passage. T h e
b e st a n sw er to th is q u e s tio n is answ er (B ); th e two n itr o g e n p r o c e sse s are n itr o g e n fixa
tio n , w h ich is d isc u sse d in th e first p aragraph, a n d d e n itr ific a tio n , w h ic h is d isc u sse d in
th e s e c o n d p aragrap h .

T h e fo llo w in g c h a r t o u tlin e s th e key in fo r m a tio n th a t y o u sh o u ld r e m e m b e r a b o u t


m a in id e a q u estio n s:

MAIN IDEA QUESTIONS

HOW TO IDENTIFY W h a t is the t o p ic o f th e passage?


THE QUESTION W h a t is the s u b je c t o f th e passage?
W h a t is the m a in id e a o f the passage?
W h a t is the a u th o rs m a in p o i n t in the passage?
W ith w ha t is the a u th o r p r im a r i ly c o n c e r n e d ?
W h ich o f th e follow ing w ould be th e best t it le ?

WHERETO FIND The answer to this type of question can generally be


THE ANSWER determined by looking at the first sentence of each paragraph.
READING COMPREHENSION 285

HOW TO ANSWER Read the first line of each paragraph.


THE QUESTION
Look for a common theme or idea in the first lines.
Pass your eyes quickly over the rest of the passage to
check that you really have found the topic sentence(s).
Eliminate any definitely wrong answers and choose the
best answer from the remaining choices.

T O E F L E X E R C ISE 1: S tu d y e a c h o f th e p assages a n d c h o o s e th e b e s t a n sw e rs to th e
q u e stio n s that follow . In this e x e r c ise , e a c h p assage is fo llo w e d by se v e ra l m a in id e a , to p ic ,
or title q u e s tio n s so th a t th e stu d e n ts can p r a c tic e th is type o f q u e s tio n . O n th e T O E F L
test, o n e p assage w o u ld p r o b a b ly n o t h a v e two su ch q u e s tio n s b e c a u se th e y a re so sim ilar.

PASSAGE ONE (Questions 1-2)


Fort Knox, Kentucky, is the site of a U.S. arm y post, but it is even m ore renow ned for the F o rt
Knox Bullion Depository, the m assive vault that contains the bulk of the U.S. governm ent's gold
deposits. Com pleted in 1936, the vault is housed in a two-story building co n stru cted of granite, steel,
Line and concrete; the vault itself is m ade of steel and concrete and has a d oor th at w eighs m ore th an
(5) twenty tons. Naturally, the m ost up-to-date security devices available are in place a t Fort K nox, and
the arm y post nearby provides fu rth er protection.

1. W hich of the following best describes the 2. W hich of the following w ould be the best
topic of the passage? title for this passage? t
(A) The city o f F ort Knox, Kentucky (A) The Massive C oncrete Vault
(B) The federal gold depository (B) Fort Knox Security
(C) The U.S. arm y post at F ort Knox (C) W here the U.S. Keeps Its Gold
(D) Gold bullion (D) A Visit to Kentucky

PASSAGE TWO (Questions 3-4)


One identifying characteristic of m inerals is their relative hardness, w hich can be d eterm ined by
scratching one m ineral w ith another. In this type of test, a h a rd er m ineral can scratch a softer one, b u t
a softer m ineral is unable to scratch the h ard er one. The Molls' hardness scale is used to ra n k m inerals
Line according to hardness. Ten m inerals are listed in this scale, ranging from talc w ith a hardness of 1 to
(5) diam ond with a hardness of 10. On this scale, quartz (num ber 7) is h arder than feldspar (num ber 6 )
and is therefore able to scratch it; however, feldspar is unable to make a m ark on quartz.

W hich of the following best states the 4. The m ain idea of this passage is th a t
subject of this passage?
(A) the hardness of a m ineral can be
(A) The hardness of diam onds determ ined by its ability to m ake a
(B) Identifying m inerals by m eans of a m ark on o th er m inerals
scratch test (B) diam onds, w ith a hard n ess of 10 on the
(C) Feldspar on the M ohs' scale M ohs scale, can scratch all o th er
(D) Recognizing m inerals in th e ir n atural m inerals
state (C) a softer m ineral <an n o t be scratched
by a h a rd er nineral
(D) talc is the firsf m ineral listed on the
M ohs sc; ie
286 READING COMPREHENSION

PASSAGE THREE (Questions 5-6)


H urricanes generally occur in the North Atlantic from May through November, with the peak of
the hurricane season in Septem ber; only rarely will they occur from Decem ber through April ini that
part of the ocean. The m ain reason for the occurrence of hurricanes during this period is th a t he
Line tem perature on the .w aters surface
.
is at its w arm est and the hum idity of the air is at its highest.
Of the tropical storm s that occur each i
year in_ .the
I. _ Atlo n i i^
N orth Atlantic, n n K i a h n n t i i V f * HT1 t n f *
only about five, on the
(5)
average, are powerful enough to be called hurricanes. To be classified as a hurricane, a tro p ic^
m ust have w inds reaching speeds of at least 117 kilom eters per hour, but the w inds are often m uch
stronger than that; the w inds of intense hurricanes can easily surpass 240 kilom eters per hou .

6 . The best title for this passage would be


5. The passage m ainly discusses
(A) how m any hurricanes occur each year (A) The N orth Atlantic Ocean
(B) Storm s of the N orthern Atlantic
(B) the strength of hurricanes
(C) the w eather in the N orth Atlantic (C) H urricanes: The Dam age and
(D) hurricanes in one p art of the world D estruction
(D) W hat H appens from May through
Novem ber
PASSAGE FOUR (Questions 7-9)
H enry W adsworth Longfellow (1807-1882) was perhaps the best-know n American poet of the
ninetecnth'century. His clear w riting style and em phasis on the prevalent values of the period made
him popular with the general public if not always with the critics He was particularly recognized o
Line his longer narrative poem s Evangeline, The Song o f Hiawatha, and The Courtship o f Miles Standish, in
(5) w hich he told stories from American history in term s of the values of the tim e.
Evangeline was set during the French and Indian W ar (1754-1763), w hen the British forced
French settlers from Nova Scotia; tw o lovers, Gabriel and Evangeline, w ere separated by the British,
and Evangeline devoted her lifetim e to the search for Gabriel. W ith its em phasis on sentim ental,
undying love, Evangeline was imm ensely popular with the public.
( 10)
In The Song o f Hiawatha, Longfellow depicted the noble life of the Am erican Indian throug e
story of the brave H iaw atha and his beloved wife M inehaha. The tear-inspiring poem follows
H iaw atha through the tragedies and trium phs of life, ending w ith the d eath of M m ehaha an
H iaw athas departure into the sunset in his canoe.
The Courtship o f Miles Standish takes place during the early period of the settlem ent of New
(15) E ngland, a period w hich w as viewed as a tim e of honor and rom ance. In this poem centered around a
love trianele, Miles Standish asks his friend John Alden to propose to Priscilla M ullins for him ; John
Alden end's up m arrying Priscilla Mullins himself, and it takes tim e for his friendship with M es
Standish to recover. As with Longfellow's o ther narrative poems, the em phasis on high ideals and
rom ance m ade the poem extremely popular.

W hich of the following best describes the 8. The best title of the passage is
m ain idea of the passage? (A) Longfellows Popular Appeal
(A) Am erican history is often depicted in (B) H istorical N arrative Poems
poetry. (C) The Lyric, D ram atic, and Narrative
(B) Longfellow described American history Poem s of Longfellow
even though people really did not (D) Longfellow and the Critics
enjoy it.
(C) The popularity of Longfellows poem s 9. The subject of the fourth paragraph is
results from his stress on the values (A) nobility and honor in the poem s
of the people. of Longfellow
(D) Longfellow w rote long narrative (B) the love triangle involving Miles
poem s that were not always Standish
p opular with the critics. (C) the popular appeal of The Courtship o f
Miles Standish
(D) th e period of the early settlem ent of
New E ngland
READING COMPREHENSION 287

S k i l l 2: R E C O G N IZE T H E O R G A N IZ A T IO N OF IDEAS

In th e R e a d in g C o m p r e h e n s io n s e c tio n o f th e T O E F L test, th e r e w ill s o m e tim e s b e q u e s


tio n s a b o u t th e o r g a n iz a tio n o f id e a s in a p assage. In this type o f q u e s tio n , y o u w ill be
ask ed to d e te r m in e h o w th e id e a s in o n e paragrap h (o r p a ra g ra p h s) r e la te to th e id e a s in
a n o th e r p aragrap h (o r p a r a g r a p h s).

E xam ple

T he p assa g e:
If asked who invented the game of baseball, m ost A m ericans
would probably reply that it was Abner Doubleday. At the
beginning of this century, there was som e disagreem ent over how
Line the gam e of baseball had actually originated, so sporting-goods
(5) m an u factu rer Spaulding inaugurated a com m ission to research the
question. In 1908 a report was published by the com m ission in
w hich A bner Doubleday, a U.S. Army officer from Cooperstown,
New York, was given credit for the invention of the game. The
N ational Baseball Hall of Fam e was established in C ooperstow n in
(10) h o nor of Doubleday.
Today, m ost sports historians are in agreem ent that
D oubleday really did not have m uch to do with the developm ent of
baseball. Instead, baseball seems to be a close relation to the
English gam e of ro u n d ers and probably has English ra th e r than
(15) Am erican roots.
T h e q u e stio n :

The second paragraph


(A) provides exam ples to support the ideas in the first p a rag ra p h
(B) precedes the ideas in the first paragraph
(C) outlines the effect of the idea in the first paragraph
(D) refutes the idea in the first paragraph

T o answ er this q u e s tio n it is n e c e ssa r y to lo o k at th e m ain id ea s o f e a c h o f th e tw o para


grap h s. T h e m ain id e a o f th e first paragrap h is fo u n d in th e first s e n t e n c e o f th e first
p aragraph: m o st p e o p le b e lie v e th a t A b n e r D o u b le d a y in v e n te d th e g a m e o f b a seb a ll.
T h e m a in id e a o f th e s e c o n d paragrap h is fo u n d in th e first lin e o f th e s e c o n d p aragraph:
h isto r ia n s g e n e r a lly a g r ee th at D o u b le d a y d id n o t in v e n t b a seb a ll. T h e s e c o n d p aragrap h
th e r e fo r e contradicts o r refutes th e in fo r m a tio n in th e first p aragrap h . T h e b e s t a n sw er to
th is q u e s tio n is an sw er (D ).
288 READING COMPREHENSION

T h e fo llo w in g ch a rt o u tlin e s th e key in fo r m a tio n th a t y o u s h o u ld r em e m b e r a b o u t


q u e s tio n s o n th e o r g a n iz a tio n o f ideas:

O R G A N IZ A T IO N O F IDEAS

HOW TO IDENTIFY H o w is the inform a tio n in the passage o r g a n iz e d ?


THE QUESTION H o w is the inform a tio n in the second p a ra g ra ph r e la t e d to
th e inform a tio n in the firs t paragraph?

WHERETO FIND The answer to this type of question can generally be determined
THE ANSWER by looking at the first sentence of the appropriate paragraphs.

HOW TO ANSWER 1. Read the first line of each paragraph.


THE QUESTION 2. Look for words that show the relationship between the
paragraphs.
3. Choose the answer that best expresses the relationship.

T O E F L E X E R C ISE 2: Stu dy each o f th e passages a n d c h o o s e th e b e st answ ers to th e q u es


tio n s th a t follow .

PASSAGE ONE (Questions 1-2)


Conflict w ithin an organization is n ot always viewed as undesirable. In fact, various m anagers
have widely divergent ideas on the value th at conflict can have.
According to the traditional view of conflict, conflict is harm ful to an organization. Managers
Line w ith this traditional view of conflict see it as their role in an organization to rid the organization of
(5) any possible sources of conflict.
The interactionist view of conflict, on the o ther hand, holds th a t conflict can serve an im portant
function in an organization by reducing complacency am ong w orkers and causing positive changes to
occur. M anagers w ho hold an interactionist view of conflict may actually take steps to stim ulate
conflict w ithin the organization.

1. How is the inform ation in the passage 2. W hat type of inform ation is included in the
organized? third paragraph?

(A) The origin of ideas about conflict is (A) A com parison of the interactionist and
presented. traditional views of conflict
(B) C ontrasting views of conflict are (B) A discussion of the w eaknesses of the
presented. interactionist view of conflict
(C) Two theorists discuss the strengths and (C) An outline of the type of m anager who
weaknesses of th eir views on prefers th e interactionist view of
conflict. conflict
(D) Exam ples o f conflict within (D) A description of one of the opposing
organizations are presented. views of conflict
READING COMPREHENSION 289

PASSAGE TWO (Questions 3 -4 )


IQ, or Intelligence Quotient, is defined as the ratio of a persons mental age to chronological age,
with the ratio multiplied by 100 to remove the decimal. Chronological age is easily determined;
mental age is generally measured by som e kind of standard test and is not so sim ple to define.
Line In theory, a standardized IQ test is set up to measure an individual's ability to perform
(5) intellectual operations such as reasoning and problem solving. These intellectual operations are
considered to represent intelligence.
In practice, it has been impossible to arrive at consensus as to which types o f intellectual
operations demonstrate intelligence. Furthermore, it has been impossible to devise a test without
cultural bias, which is to say that any IQ tests so far proposed have been shown to reflect the culture
(10) of the test makers. Test takers from that culture would, it follows, score higher on such a test than test
takers from a different culture with equal intelligence.

3. What type of information is included in the 4. How does the information in the third
first paragraph? paragraph differ from that in the second
paragraph?
(A) An argument
(B) A definition (A) It presents a contrasting point of view.
(C) An opinion (B) It follows chronologically from the
(D) A theory ideas in the second paragraph.
(C) It presents real information rather
than a premise.
(D) It presents an example of the ideas in
the second paragraph.

PASSAGE THREE (Questions 5-6)


The largest lake in the western United States is the Great Salt Lake, an inland saltwater lake in
northwestern Utah, just outside the state capital of Salt Lake City. Rivers and stream s feed into the
Great Salt Lake, but none drain out o f it; this has a major influence on both the salt content and the
Line size of the lake.
(5) Although the Great Salt Lake is fed by freshwater streams, it is actually saltier than the oceans
of the world. The salt com es from the more than two million tons of minerals that flow into the lake
each year from the rivers and creeks that feed it. Sodium and chloridethe com ponents o f salt
comprise the large majority of the lake's mineral content.
The Great Salt Lake can vary tremendously from its normal size o f 1,700 square miles,
(10) depending on long-term weather conditions. During periods of heavy rains, the size of the lake can
swell tremendously from the huge amounts o f water flowing into the lake from its feeder rivers and
streams; in 1980 the lake even reached a size of 2,400 square miles. During periods o f dry weather, the
size of the lake decreases, som etim es drastically, due to evaporation.

5. How is the information in the passage 6. The third paragraph contains information
organized? on
(A) Two unusual characteristics of the (A) how the size of the lake affects weather
Great Salt Lake are discussed. conditions
(B) Contrasting theories about the Great (B) the effects of contrasting weather
Salt Lakes salt levels are presented. conditions on the size of the lake
(C) The process by which the Great Salt (C) the effects o f changes in the size
Lake gets its salt is outlined. of the lake
(D) The reasons for the variations in the (D) the causes of the varied weather
Great Salt Lakes size are given. conditions in the area o f the lake
290 READING COMPREHENSION

T O E F L E X E R C ISE (S k ills 1 -2 ): Study e a c h o f th e p assages a n d c h o o s e th e b e st answ ers


to th e q u e s tio n s th a t follow .

PASSAGE ONE (Questions 1-2)


Common types o f calendars can be based on the Sun or on the Moon. The solar calendar is
based on the solar year. Since the solar year is 365.2422 days long, solar calendars consist of regular
years of 365 days and have an extra day every fourth year, or leap year, to make up for the additional
Line fractional amount. In a solar calendar, the waxing and waning of the moon can take place at various
(5) stages of each month.
The lunar calendar is synchronized to the lunar month rather than the solar year. Since the
lunar month is twenty-nine and a half days long, most lunar calendars have alternating months of
twenty-nine and thirty days. A twelve-month lunar year thus has 354 days, 11 days shorter than a
solar year.

1. What is the main idea of the passage? How is the information in the passage
organized?
(A) All calendars are the same.
(B) The solar calendar is based on the Sun. (A) Characteristics of the solar calendar
(C) Different calendars have dissimilar are outlined. .
bases. (B) Two types of calendars are described.
(D) The lunar month is twenty-nine and a (C) The strengths and weakness of the
half days long. lunar calendar are described.
(D) The length o f each existing calendar is
contrasted.

PASSAGE TW O (Questions 3-4)


Vaccines are prepared from harmful viruses or bacteria and administered to patients to provide
immuni Ly to specific diseases. The various types of vaccines are classified according to the method by
w h ifh they are derived.
Line The m ost basic class of vaccines actually contains disease-causing microorganisms that have
(5) been killed with a solution containing formaldehyde. In this type of vaccine, the microorganisms are
dead and therefore cannot cause disease; however, the antigens found in and on the microorganisms
can still stimulate the formation of antibodies. Examples of this type of vaccine are the ones that fight
influenza, typhoid fever, and cholera.
A second type of vaccine contains the toxins produced by the microorganisms rather than the
(10) microorganisms themselves. This type of vaccine is prepared when the microorganism itself does
little damage but the toxin within the microorganism is extremely harmful. For example, the bacteria
that cause diphtheria can thrive in the throat without much harm, but when toxins are released from
the bacteria, muscles can become paralyzed and death can ensue.
A final type of vaccine contains living microorganisms that have been rendered harmless. With
(15) tliis type of vaccine, a large number of antigen molecules are produced and the immunity that results
is generally longer lasting than the immunity from other types of vaccines. The Sabin oral antipolio
vaccine and the BCG vaccine against tuberculosis are examples of this type of vaccine.

3. Which of the following expresses the main 4. How many types of vaccines are presented
idea of the passage? in the passage?

(A) Vaccines provide immunity to specific (A) Two


diseases. (B) Three
(B) Vaccines contain disease-causing (C) Four
microorganisms. (D) Five
(C) Vaccines are derived in different ways.
(D) New approaches in administering
vaccines are being developed.
READING COMPREHENSION

PASSAGE THREE (Questions 5-7)


A hoax, unlike an honest error, is a deliberately concocted plan to present an untruth as the
truth. It can take the form of a fraud, a fake, a swindle, or a forgery, and can be accom plished in
almost any field: successful hoaxes have been foisted on the public in fields as varied as politics,
Line religion, science, art, and literature.
(5) A famous scientific hoax occurred in 1912 when Charles Dawson claimed to have uncovered a
human skull and jawbone on the Piltdown Common in southern England. These human remains were
said to be more than 500,000 years old and were unlike any other remains from that period; as such
they represented an important discovery in the study of human evolution. These remains, popularly
known as the Piltdown Man and scientifically named Eoanthropus dawsoni after their discoverer,
(10) confounded scientists for more than forty years. Finally in 1953, a chemical analysis was used to date
the bones, and it was found that the bones were modern bones that had been skillfully aged. A further
twist to the hoax was that the skull belonged to a human and the jaws to an orangutan.

5. The topic of this passage could best be 7. The second paragraph includes
described as
(A) an illustration to support the ideas in
(A) the Piltdown Man the first paragraph
(B) Charles Dawsons discovery (B) a counterargument to the ideas in the
(C) Eoanthropus daw soni first paragraph
(D) a definition and an example of a hoax (C) an analogy to the ideas in the first
paragraph
6. The authors main point is that (D) a detailed definition o f a hoax
(A) various types of hoaxes have been
perpetrated
(B) Charles Dawson discovered a human
skull and jawbone
(C) Charles Dawson was not an honest
man
(D) the human skull and jawbone were
extremely old

DIRECTLY A N S W E R E D Q U E S TIO N S

M any q u e s tio n s in th e R e a d in g C o m p r e h e n s io n se c tio n o f th e T O E F L te s t w ill r e q u ir e a n


sw ers th a t are d irectly sta te d in th e p assage. T h is m e a n s th at y o u s h o u ld b e a b le to fin d
th e an sw er to th is type o f q u e s tio n w ith o u t h a v in g to draw a c o n c lu s io n . T h e c o m m o n
q u e s tio n s o f th is type a r e (1) sta te d d e ta il q u e stio n s, (2 ) u n s ta te d d e ta il q u e s tio n s , a n d
(3) p r o n o u n r e fe r e n t q u e s tio n s.

S k i l l 3: A N S W E R STATED DETAIL Q U E S T IO N S CO RRECTLY

A stated d e ta il q u e s tio n asks a b o u t o n e p ie c e o f in fo r m a tio n in th e p a s sa g e r a th e r th a n


th e p a ssa g e as a w h o le . T h e an sw ers to th e se q u e s tio n s are g e n e r a lly g iv e n in o r d e r in th e
p assage, a n d th e c o r r e c t an sw er is o fte n a r e sta te m e n t o f w h a t is g iv e n in th e p a ssa g e . T h is
m e a n s th a t th e c o r r e c t an sw er o ft e n e x p r e s se s th e sa m e id e a as w h a t is w r itte n in th e pas
sa g e, b u t th e w o rd s are n o t e x a c tly th e sam e.
292 READING COMPREHENSION

Exam ple

The passage:
Williamsburg is a historic city in Virginia situated on a
peninsula between two rivers, the York and the James. It was
settled by English colonists in 1633, twenty-six years after the first
Line permanent English colony in America was settled at Jamestown. In
(5) the beginning the colony at Williamsburg was named Middle
Plantation because of its location in the middle of the peninsula.
The site for Williamsburg had been selected by the colonists
because the soil drainage was better there than at the Jamestown
location, and there were fewer mosquitoes.

The questions:
1. According to the passage, Williamsburg is located
(A) on an island
(B) in the middle of a river
(C) where the York and the James meet
(D) on a piece of land with rivers on two sides

2. The passage indicates that Jamestown


(A) was settled in 1633
(B) was settled twenty-six years after Williamsburg
(C) was the first permanent English colony in America
(D) was originally named Middle Plantation

3. The passage states that the name Middle Plantation


(A) is a more recent name than Williamsburg
(B) derived from the location of the colony on the
peninsula . .
(C) refers to the middle part o f England that was home
to the colonists
(D) was given to the new colony because it was located
in the middle of several plantations

T h e an sw ers to th e q u e s tio n s are g e n e ra lly fo u n d in o r d e r in th e p assage, so y o u sh o u ld


lo o k fo r th e an sw er to th e first q u e stio n n e a r th e b e g in n in g o f th e p assage. S in ce th e first
q u e s tio n asks a b o u t w h e r e W illiam sburg is located, y o u sh o u ld s e e th a t th e first s e n te n c e in
th e p a ssa g e an sw ers th e q u e stio n b e c a u se situated m e a n s located. A n sw er (A) is a n in c o r
r ec t an sw er b e c a u se W illiam sb u rg is n o t lo c a te d o n an island ; th e p assage states th a t it is
situated on a p e n in su la . A n sw er (B ) is in c o r r e c t b e c a u se W illia m sb u rg is between two rivers,
n o t in the m iddle o f a river. A n sw er (C ) is in c o r r e c t b e c a u s e th e p a ssa g e says n o th in g a b o u t
w h e th e r o r n o t th e tw o rivers m e e t at W illiam sb u rg. T h e b e st an sw er to this q u e s tio n is
an sw er (D ); w ith rivers on two sides is c lo s e s t in m e a n in g to between two rivers.
READING COMPREHENSION

T h e an sw er to th e s e c o n d q u e s tio n w ill p rob ab ly b e lo c a te d in th e p a ssa g e a fte r th e


answ er to th e first q u e s tio n . S in c e th e s e c o n d q u e s tio n is a b o u t Jam estow n, y o u s h o u ld
skim th r o u g h th e p a ssa g e to fin d th e p art o f th e p a ssa g e th a t d isc u sse s th is to p ic . T h e a n
sw er to th is q u e s tio n is fo u n d in th e s ta te m e n t th at W illia m sb u rg w as settled by E nglish
colonists in 1633, tw enty-six years after the fir s t perm a n en t E nglish colony in A m erica was settled at
Jam estow n. A n sw er (A ) is in c o r r e c t b e c a u se it was W illia m sb u rg th a t was s e ttle d in 1633.
A n sw er (B ) is in c o r r e c t b e c a u se J a m e sto w n was se ttle d before r a th e r th a n after W illiam s
burg. A n sw er (D ) is in c o r r e c t b e c a u s e th e n a m e M id d le P la n ta tio n r e fe r r e d to W illia m s
b u rg. T h e b est an sw er to th is q u e s tio n is an sw er (C ), w h ic h is d ir e c tly sta te d in th e
p assage a b o u t J a m e sto w n .
T h e answ er to th e th ird q u e s tio n w ill p rob ab ly b e lo c a te d in th e p a ssa g e a fter th e an
sw er to th e s e c o n d q u e s tio n . B e c a u se th e th ird q u e s tio n is a b o u t the n a m e M id d le P la n ta
tion, y o u sh o u ld skim th r o u g h th e p a ssa g e to fin d th e p a rt th a t d isc u s se s th is to p ic . T h e
answ er to this q u e s tio n is fo u n d in th e sta te m e n t W illiam sburg was nam ed M id d le P la n ta tio n
because o f its location in the m iddle o f the p e n in su la . A n sw er (B ) is c o r r e c t b e c a u s e it is c lo se st
in m e a n in g to th is s ta te m e n t. A n sw er (A ) is in c o r r e c t b e c a u se it is fa lse; th e a r e a was
n a m e d M id d le P la n ta tio n in the beginning, a n d th e n a m e W illia m sb u rg is more recent. A n
sw er (C) is in c o r r e c t b e c a u s e th e p a ssa g e says n o th in g a b o u t n a m in g t h e area a fter th e
c o lo n is ts h o m e in E n g la n d . A n sw er (D ) is in c o r r e c t b e c a u se th e p a ssa g e says n o th in g
a b o u t any o th e r p la n ta tio n s in th e area o f W illiam sb u rg.

T h e fo llo w in g c h a r t o u t lin e s th e key in fo r m a tio n th a t y o u s h o u ld r e m e m b e r a b o u t


sta te d d e ta il q u estio n s:

STATED DETAIL QUESTIONS

HOW TO IDENTIFY A c c o rd in g to the passage,...


THE QUESTION It is sta te d in the passage...
The passage in d ica tes th a t..
The author m e n tio n s th a t..
Which of the following is tru e. . . ?

WHERE TO FIND The answers to these questions are found in order in


THE ANSWER the passage.

HOW TO ANSWER 1. Choose a key word in the question.


THE QUESTION 2. Skim in the appropriate part of the passage for the key
word or idea.
3. Read the sentence that contains the key word or idea
carefully.
4. Look for the answer that restates an idea in the passage.
5. Eliminate the definitely wrong answers and choose the
best answer from the remaining choices.
294 READING COMPREHENSION

T O E F L E X E R C IS E 3: Stu dy e a ch o f th e p assages a n d c h o o s e th e b e s t an sw ers to th e q u es


tio n s th a t follow .

PASSAGE ONE (Questions 1-3)


Ice ages, those periods when ice covered extensive areas of the Earth, are known to have
occurred at least six times. Past ice ages can be recognized from rock strata that show evidence of
foreign materials deposited by moving walls of ice or melting glaciers. Ice ages can also be recognized
Line from land formations that have been produced from moving walls of ice, such as U-shaped valleys,
(5) sculptured landscapes, and polished rock faces.
1. According to the passage, what happens 3. According to the passage, what in the rock
during an ice age? strata is a clue to geologists of a past ice
age?
(A) Rock strata are recognized by
geologists. (A) Ice
(B) Evidence o f foreign materials is found. (B) Melting glaciers
(C) Ice covers a large portion of the Earths (C) U-shaped valleys
surface. (D) Substances from other areas
(D) Ice melts six times.

2. The passage covers how many different


methods of recognizing past ice ages?
(A) One
(B) Two
(C) Three
(D) Four

PASSA GE I WO (Questions 46)


The human heart is divided into four chambers, each of which serves its own function in the
cycle of pumping blood. The atria are the thin-walled upper chambers that gather blood as it flows
Line from the veins between heartbeats. The ventricles are the thick-walled lower chambers that receive
blood from the atria and push it into the arteries with each contraction of the heart. The left atrium
(5)
and ventricle work separately from those on the right. The role of the chambers on the right side of
the heart is to receive oxygen-depleted blood from the body tissues and send it on to the lungs; the
chambers on the left side of the heart then receive the oxygen-enriched blood from the lungs and
send it back out to the body tissues.

4. The passage indicates that the ventricles 6. According to the passage, which part of the
heart gets blood from the body tissues and
(A) have relatively thin walls
passes it on to the lungs?
(B) send blood to the atria
(C) are above the atria (A) The atria
(D) force blood into the arteries (B) The ventricles
(C) The right atrium and ventricle
5. According to the passage, when is blood (D) The left atrium and ventricle
pushed into the arteries from the
ventricles?
(A) As the heart beats
(B) Between heartbeats
(C) Before each contraction of the heart
(D) Before it is received by the atria
READING COMPREHENSION 295

PASSAGE THREE (Questions 7-9)


The Golden Age of Railroads refers to the period from the end of the Civil War to the beginning
of World War I when railroads flourished and, in fact, maintained a near monopoly in mass
transportation in the United States. One of the significant developments during the period was the
Line notable increase in uniformity, particularly through the standardization of track gauge and time.
(5) At the end of the Civil War, only about half of the nation's railroad track was laid at what is now
the standard gauge of 1.4 meters; m uch of the rest, particularly in the southern states, had a 1.5-meter
gauge. During the postwar years, tracks were converted to the 1.4-meter gauge, and by June 1, 1886,
the standardization of tracks was completed, resulting in increased efficiency and econom y in the rail
system.
(10) A further boon to railroad efficiency was the implementation of standard time in 1883. With the
adoption of standard time, four time zones were established across the country, thus simplifying
railroad scheduling and improving the efficiency of railroad service.

7. According to the passage, the Golden Age of 9. The passage indicates that standard time
Railroads was implemented

(A) occurred prior to the Civil War (A) before the Civil War
(B) was a result of World War 1 (B) on June 1,1886
(C) was a period when m ost of U.S. mass (C) after World War I
transportation was controlled by (D) before standardized track gauge was
the railroads established throughout the United
(D) resulted in a decrease in uniformity of States
track gauge

8. The passage mentions that which o f the


following occurred as a result of uniformity
of track gauge?
(A) The Civil War
(B) Improved econom y in the
transportation system
(C) Standardization of time zones
(D) Railroad schedules
READING COMPREHENSION

S k il l 4: F IN D U N S TA TE D DETAILS

You w ill so m e tim e s b e ask ed in th e R e a d in g C o m p r e h e n s io n se c tio n o f th e T O E F L test to


fin d an an sw er th a t is not stated o r not m entioned o r not true in th e p assage. T h is type o f
q u e s tio n really m e a n s th at th r e e o f th e answ ers are stated, mentioned,, o r true in th e passage,
w h ile o n e an sw er is n o t. Your actu al j o b is to fin d th e th r e e c o r r e c t answ ers an d th en
c h o o s e th e le tte r o f th e o n e r e m a in in g answer.
You s h o u ld n o te th a t th e r e are tw o k in d s o f answ ers to th is type o f q u estio n : (1) th ere
are th r e e tr u e an sw ers a n d o n e an sw er th a t is n o t d isc u sse d in th e p a ssa g e, o r (2) th ere
are th r e e tr u e an sw ers an d o n e th a t is false a c c o r d in g to th e p assage.

Exam ple

The passage:
In English there are many different kinds of expressions that
people use to give a name to anything w hose name is unknown or
momentarily forgotten. The word gadget is one such word. It was
Line first used by British sailors in the 1850s and probably came from
(5) the French word gachette, which was a small hook. In everyday use,
the word has a more general meaning. Other words are also used to
give a name to something unnamed or unknown, and these words
tend to be somewhat imaginative. Some of the move commonly
used expressions are a w hat-dye-call-it, a whatsis, a thingamabob, a
(10) thingamajig, a doodad, or a doohickey.

The questions:
1. Which of the following is NOT true about the word gadget"?
(A) It is used to name something when the name is not
known.
(B) It was used at the beginning of the nineteenth century.
(C) It most likely came from a word in the French language.
(D) Its first known use was by British sailors.

2. Which of the following is NOT mentioned in the passage as


an expression for something that is not known?
(A) A thingamabob
(B) A gadget
(C) A doohickey
(D) A what-is-it

T h e first q u e s tio n asks for th e o n e answ er th a t is n o t true, so th r e e o f th e answ ers are true
a n d o n e an sw er is not. A n sw er (B) is th e o n e an sw er th at is n o t true: th e w ord gadget was
first u s e d in 185 0 , w h ich is th e m id d le o f th e n in e te e n th c en tu ry , so answ er (B ) is th e b est
answ er. A n sw er (A ) is tru e a c c o r d in g to th e s e c o n d lin e o f th e p aragraph; answ er (C) is
tr u e a c c o r d in g to th e fo u r th an d fifth lin e s o f th e p aragrap h ; an sw er (D ) is tru e a c co r d
in g to th e fo u r th lin e o f th e p aragraph.
T h e s e c o n d q u e s tio n asks for th e o n e answ er th a t is not m entioned, so th r e e o f th e an
sw ers are listed in th e p assage a n d o n e is n o t. S in c e a thingam abob, a gadget, a n d a doohickey
are liste d in th e p a ssa g e, answ ers (A ), (B ), a n d (C ) are in c o r r e c t. H ow ever, a what-is-it is
n o t liste d in th e p a ssa g e, so answ er (D ) is th e b e st an sw er to this q u e s tio n .
READING COMPREHENSION

T h e fo llo w in g c h a r t o u tlin e s th e key in fo r m a tio n th at y o u sh o u ld r e m e m b e r a b o u t


u n sta te d d e ta il q u estio n s:

UNSTATED" DETAIL QUESTIONS

HOW TO IDENTIFY W h ich o f the follow ing is n o t s ta te d . . .?


THE QUESTION W h ich o f the follow ing is not m e n t io n e d . . .?
W h ich o f the follow ing is n o t d is c u s s e d . .. ?
A ll o f the follow ing are tru e e x c e p t . . . .

WHERETO FIND The answers to these questions are found In order in the
THE ANSWER passage.

HOW TO ANSWER 1. Choose a key w ord in the question.


THE QUESTION 2. Scan in the appropriate place in the passage for the key
w ord (or related idea).
3. Read the sentence that contains the key w o rd or idea
carefully.
4. Look for answers that are definitely true according to the
passage. Eliminate those answers.
5. Choose the answer that is not true or n o t discussed in the
passage.

T O E F L EX ER C ISE 4: Stu dy e a c h o f th e p assages an d c h o o s e th e b est a n sw ers to th e q u e s


tio n s th at follow .

PASSAGE ONE (Questions 1-2)


Blood plasm a is a clear, alm ost colorless liquid. It consists of blood from w hich the red and
white blood cells have been removed. It is often used in transfusions because a patien t generally
needs the plasm a portion of the blood m ore than the other com ponents.
Plasma differs in several im portant ways from whole blood. First of all, plasm a can be m ixed for
all donors and does not have to be from the right blood group, as whole blood does. In addition,
plasm a can be dried and stored, while whole blood cannot.

1. All of the following are true about blood 2. W hich of the following is NOT sta te d about
plasm a EXCEPT whole blood?
(A) it is a deeply colored liquid (A) It is different from plasm a.
(B) blood cells have been taken out of it (B) It cannot be dried.
(C) patients are often transfused with it (C) It is impossible to keep it in storage for
(D) it is generally m ore im portant to the a long time.
p atient than o th er p arts of whole (D) It is a clear, colorless liquid.
blood
298 READING COMPREHENSION

PASSAGE TWO (Questions 3-4)


E lizabeth C ochrane Seam an was an American journalist at the tu rn of the century w ho w rote
for the new spaper New York World under the pen nam e Nellie Bly, a nam e w hich was taken from the
Stephen F oster song Nelly Bly. She achieved fame for her exposes a n d in particu lar for the bold and
Line adventuresom e way th a t she obtained h e r stories.
(5) She felt th a t the best way to get the real story was from the inside ra th e r than as an outside
observer w ho could be treated to a p rettified version of reality. On one occasion she pretended to be a
th ie f so th a t she w ould get arrested and see for herself how female prisoners w ere really treated. On
an o th er occasion she faked m ental illness in order to be adm itted to a m ental hospital to get the real
pictu re on the treatm en t of m ental patients.

3. W hich of the following is NOT true 4. W hich of the following is NOT


about Nellie Bly? m entioned as som ething that Nellie Bly
did to get a good story?
(A) Nellie Blys real nam e was
E lizabeth Cochrane Seam an. (A) She acted like a thief.
(B) Nellie Bly was m entally ill. (B) She got arrested by the police.
(C) The nam e Nellie Bly cam e from a (C) She pretended to be ill.
song. (D) She worked as a doctor in a
(D) The nam e Nellie Bly was used on m ental hospital.
articles that Seam an wrote.

PASSAGE THREE (Questions 5-6)


D ekanaw idas role as a suprem e lawgiver in the Iroquois tribe has given him the status of
dem igod w ithin the Indian nation. Born into the H uron tribe, Dekanaw ida caused great fear in his
parents, w ho tried to drown him in his youth after a prophecy was m ade indicating that he would
Line bring great sorrow to the H uron nation. Dekanawida was to survive this a ttem pted drow ning but later
(5) left his paren ts' hom e and tribe to live am ong the Iroquois.
One of his achievem ents with the Iroquois was the institution o f a law am ong the Iroquois that
virtually ended blood feuds am ong the nation's families. W am pum , strings of beads m ade of polished
shells, was a valued com m odity in the Iroquois culture; according to policies established by
Dekanaw ida, w am pum had to be paid to the family of a m u rd er victim by the family of the killer.
(10) Since the killer was also put to death, the family of the killer had to pay the victim's family in
w am pum for two deaths, the death of the m urder victim and the death of the killer. These strict
policies im plem ented by Dekanawida helped to establish him as a wise lawgiver and leader of the
Iroquois nation.

5. According to the passage, Dekanawida was 6 . W hich of the following is NOT m entioned
NOT in the passage about wam pum ?
(A) a law m aker (A) It w as used extensively by the Huron.
(B) a H uron by birth (B) It had a high value to the Iroquois.
(C) a n ear deity (C) It w as given to a m urder victim's
(D) drow ned w hen he was young family.
(D) It w as m ade of polished shells.
READING COMPREHENSION

S k i l l 5: F IN D P R O N O U N REFERENTS

In th e R e a d in g C o m p r e h e n s io n s e c tio n o f th e T O E F L test, y o u w ill s o m e t im e s b e a sk ed


to d e te r m in e w h ic h n o u n a p r o n o u n refers to . In this type o f q u e s tio n it is im p o r ta n t to
u n d e r sta n d th a t a n o u n is g e n e r a lly u s e d first in a p assage, a n d th e p r o n o u n th a t r e fe rs to
it c o m e s after. W h e n e v e r y o u are a sk ed w h ic h n o u n a p r o n o u n refers to , y o u s h o u ld lo o k
before th e p r o n o u n to fin d th e n o u n .

Exam ple

The passage:
C arnivorous plants, such as the sundew and the Venus
flytrap, are generally found in hum id areas w here there is an
inadequate supply of nitrogen in the soil. In order to survive, these
Line plants have developed m echanism s to trap insects w ithin their
(5) foliage. They have digestive fluids to obtain the necessary nitrogen
from the insects. These plants trap the insects in a variety of ways.
The sundew has sticky hairs o n its leaves; when an insect lands on
these leaves, it gets caught up in the sticky hairs, and the leaf w raps
itself around the insect. The leaves of the Venus flytrap function
( 10) m ore like a trap, snapping suddenly and forcefully sh u t aro u n d an
insect.

The questions:
1. The p ronoun T h ey in line 5 refers to
(A) h um id areas
(B) these plants
(C) insects
(D) digestive fluids

2. The pro n o u n "it" in line 8 refers to


(A) a variety
(B) the sundew
(C) an insect
(D) the leaf

T o answ er th e first q u e s tio n , y o u sh o u ld lo o k b e fo r e th e p r o n o u n they fo r p lu ral n o u n s th a t


th e p r o n o u n c o u ld r efe r to. H u m id areas, insects, a n d these p la n ts c o m e b e fo r e th e p r o n o u n ,
so th e y are p o ssib le answ ers; digestive flu id s c o m e s after the p r o n o u n , so it is p r o b a b ly n o t
th e c o r r e c t answer. T h e n y o u sh o u ld try th e th r e e p o ssib le answ ers in th e s e n te n c e in
p la c e o f th e p r o n o u n . You s h o u ld u n d e r sta n d fro m th e c o n te x t th at these p la n ts have diges
tive flu id s to o b ta in th e n e c e ssa r y n itr o g e n fro m th e in sects, so th e b e st an sw er to th is q u e s
tio n is answ er (B ). T o an sw er th e s e c o n d q u e stio n , y o u sh o u ld lo o k b e fo r e th e p r o n o u n it
for sin gu lar n o u n s th a t th e p r o n o u n c o u ld r efe r to. A variety, the sundew , a n d a n insect c o m e
b e fo r e th e p r o n o u n , so th e y are p o ssib le answers; the le a f c o m e s after th e p r o n o u n , so it is
probab ly n o t th e c o r r e c t answ er. N e x t y o u sh o u ld try th e th r e e p o ssib le an sw ers in th e s e n
te n c e in p la c e o f th e p r o n o u n . A n insect g e ts c a u g h t u p in th e stick\ hairs, ra th e r th a n a
variety or the sundew, so th e b e st an sw er to th is q u e s tio n is answ er (C ).
300 READING COMPREHENSION

T h e fo llo w in g ch a rt o u tlin e s th e key in fo r m a tio n th a t y o u sh o u ld r e m e m b e r a b o u t


p r o n o u n referen ts:

' PRONOUN REFERENTS

HOW TO IDENTIFY The p r o n o u n " in line X refers to which of the following?


THE QUESTION

WHERETO FIND The line where the pronoun is located is generally given in
THE ANSWER the question.The noun that the pronoun refers to is generally
found before the pronoun.

HO W TO ANSWER 1. Find the pronoun in the passage. (The line where the
THE QUESTION pronoun can be found is generally stated in the question.)
2. Look for nouns that com e before the pronoun.
3. Read the part of the passage before the pronoun carefully.
4. Eliminate any definitely wrong answers and choose the
best answer from the remaining choices.

T O E F L E X E R C ISE 5: Stu dy e a c h o f th e p assages a n d c h o o s e th e b e st answ ers to th e q u es


tio n s th at follow .

PASSAGE ONE (Questions 1-2)


The full m oon th at occurs nearest the equinox of the Sun has becom e know n as the harvest
m oon. It is a bright m oon w hich allows farm ers to work late into the night for several nights; they can
w ork when the m oon is at its brightest to bring in the fall harvest. The harvest m oon of course occurs
Line at different lim es of the year in the northern and southern hem ispheres. In the northern hem isphere,
(5) the harvest m oon occurs in Septem ber at the tim e of the autum nal equinox. In the southern
hem isphere, the harvest m oon occurs in March at the tim e of the vernal equinox.

1. The pronoun "It" in line 2 refers to 2. The pronoun "they" in line 2 refers to

(A) the equinox (A) farm ers


(B) the Sun (B) nights
(C) the harvest m oon (C) tim es of the year
(D) the night (D) no rth ern and southern hem ispheres

PASSAGE TW O (Questions 3-4)


Mardi Gras, w hich m eans Fat Tuesday in French, was introduced to A merica by French
colonists in the early eighteenth century. From that tim e it has grown in popularity, particularly in
New Orleans, and today it is actually a legal holiday in several southern states. The M ardi Gras
Line celebration in New O rleans begins well before the actual M ardi Gras Day. Parades, parties, balls, and
(5) num erous festivities take place throughout the week before M ardi G ras Day; tourists from various
countries thro u g h o u t the world flock to New O rleans for the celebration, w here they take part in a
week of nonstop activities before returning home for som e m uch-needed rest.

3. The pronoun it in line 2 refers to 4. The pronoun "they" in line 6 refers to

(A) M ardi Gras (A) num erous festivities


(B) French (B) tourists
(C) th at tim e (C) various countries
(D) New O rleans (D) nonstop activities
READING COMPREHENSION 301

PASSAGE THREE (Questions 56)


The financial firm Dow Jones and Company computes business statistics every hour on the hour
of each of the business days of the year, and these statistics are known as the Dow Jones averages.
They are based on a select group of stocks and bonds that are traded on the New York Stock
Line Exchange. The Dow Jones averages are composed of four different types of averages: the average
(5) price of the common stock of thirty industrial firms, the average price of the com m on stock prices of
twenty transportation companies, the average price of the common stock prices o f fifteen utility
companies, and an overall average of all the sixty-five stocks used to compute the first three averages.
Probably the average that is the m ost commonly used is the industrial average; it is often used by an
investor interested in checking the state o f the stock market before making an investment in an
(10) industrial stock.

5. The pronoun "They" in line 3 refers to 6. The pronoun it" in line 8 refers to
(A) the business days (A) the industrial average
(B) these statistics (B) an investor
(C) stocks and bonds (C) the state of the stock market
(D) four different types (D) an investment

T O E F L E X E R C ISE (S k ills 3 -5 ) : S tu d y e a ch o f the p assages a n d c h o o s e th e b e s t an sw ers


to th e q u e s tio n s th at follow .

PASSAGE ONE (Questions 1-4)


The United States does not have a national university, but the idea has been around for quite
some time. George Washington first recommended the idea to Congress; he even selected an actual
site in Washington, D.C., and then left an endowment for the proposed national university in his will.
Line During the century following the Revolution, the idea of a national university continued to receive the
(5) support of various U.S. presidents, and philanthropist Andrew Carnegie pursued the cause at the
beginning of the present century. Although the original idea has not yet been acted upon, it continues
to be proposed in bills before Congress.

1. According to the passage, the national 3. Which of the following is NOT mentioned
university of the United States in the passage about Andrew Carnegie?
(A) has been around for a while (A) He was interested in doing charity
(B) does not exist wor k and good deeds for the public.
(C) is a very recent idea (B) He was a member of Congress.
(D) is an idea that developed during the (C) He was interested in the idea of a
present century national university.
(D) He was active in the early twentieth
2. The passage indicates that George century.
Washington did NOT do which of the
following? 4. The pronoun "it in line 6 refers to
(A) He suggested the concept for a (A) the cause
national university to Congress. (B) the beginning o f the present century
(B) He chose a location for the national (C) the original idea
university. (D) Congress
(C) He left money in his w ill for a national
university.
(D) He succeeded in establishing a
national university.
302 READING COMPREHENSION

PASSAGE TW O (Questions 5 -9 )
The La Brea tarpits, located in Hancock Park in the Los Angeles area, have proven to be an
extremely fertile source of Ice Age fossils. Apparently, during the period of the Ice Age, the tarpits
were covered by shallow pools of water; when animals came there to drink, they got caught in the
Line sticky tar and perished. The tar not only trapped the animals, leading to their death, but also served
(5) as a remarkably effective preservant, allowing near-perfect skeletons to remain hidden until the
present era.
In 1906, the remains of a huge prehistoric bear discovered in the tarpits alerted archeologists to
the potential treasure lying within the tar. Since then thousands and thousands of well-preserved
skeletons have been uncovered, including the skeletons of camels, horses, wolves, tigers, sloths, and
(10) dinosaurs.

5. Which of the following is NOT true about 8. When did archeologists become aware of
the La Brea tarpits? the possible value of the contents of the
tarpits?
(A) They contain fossils that are quite old.
(B) They are found in Hancock Park. (A) During the Ice Age
(C) They have existed since the Ice Age. (B) Thousands and thousands of years ago
(D) They are located under a swimming (C) Early in the twentieth century
pool. (D) Within the past decade

6. The pronoun "they" in line 3 refers to 9. Which of the following is NOT mentioned
as an example of a skeleton found in the
(A) the La Brea tarpits tarpits?
(B) Ice Age fossils
(C) shallow pools of water (A) A bear
(D) animals (B) A sloth
(C) A horse
7. According to the passage, how did the Ice (D) A snake
Age animals die?
(A) The water poisoned them.
(B) They got stuck in the tar.
(C) They were attacked by other animals.
(D) They were killed by hunters.

PASSAGE THREE (Questions 10-14)


When the president of the United States wants to get away from the hectic pace in Washington,
D.C., Camp David is the place to go. Camp David, in a wooded mountain area about 70 miles from
Washington, D.C., is the official retreat of the president of the United States. It consists of living space
Line for the president, the first family, and the presidential staff as well as sporting and recreational
(5) facilities.
Camp David was established by President Franklin Delano Roosevelt in 1942. He found the site
particularly appealing in that its mountain air provided relief from the summer heat of Washington
and its remote location offered a more relaxing environment than could be achieved in the capital
city.
(10) When Roosevelt first established the retreat, he called it Shangri-La, which evoked the blissful
mountain kingdom in James Hilton's novel Lost Horizon. Later, President Dwight David Eisenhower
renamed the location Camp David after his grandson David Eisenhower.
Camp David has been used for a number of significant meetings. In 1943 during World War II,
President Roosevelt met there with Great Britain's Prime Minister Winston Churchill. In 1959 at the
(15) height o f the Cold War, President Eisenhower met there with Soviet Premier Nikita Khrushchev; in
1978 President Jimmy Carter sponsored peace talks between Israels Prime Minister Menachem Begin
and Egypts President Anwar el-Sadat at the retreat at Camp David.
READING COMPREHENSION 303

10. W hich of the following is NOT discussed 13. W hich of the following is NOT true about
about Camp David? President Eisenhow er?
(A) Its location (A) He had a grandson nam ed David.
(B) Its cost (B) He attended a conference w ith N ikita
(C) Its facilities Khrushchev.
(D) Its uses (C) He nam ed the presidential retreat
Shangri-La.
11. According to the passage, who founded (D) He visited Camp David.
Camp David?
14. K hrushchev was at Cam p David in
(A) George W ashington
(B) The first family (A) 1942
(C) Franklin Delano Roosevelt (B) 1943
(D) Dwight David E isenhow er (C) 1959
(D) 1978
12. The p ronoun "he in line 10 refers to
(A) Camp David
(B) Roosevelt
(C) Jam es Hilton
(D) President Dwight David E isenhow er

T O E F L REVIEW E X E R C ISE (S k ills 1 -5 ): Stu dy e a c h o f th e p assages a n d c h o o s e th e b e st


answ ers to the q u e s tio n s th a t follow .

PASSAGE ONE (Questions 1-4)


Hay fever is a seasonal allergy to pollens; the term "hay fever," however, is a less th an adequate
description since an attack of this allergy does not incur fever a n d since such a n a tta ck can be
brought on by sources o th er th an hay-producing grasses. Hay fever is generally caused by air-borne
Line pollens, particularly ragw eed pollen. The am ount of pollen in the a ir is largely dependent on
(5) geographical location, w eather, and season. In the eastern section of the U nited States, for exam ple,
there are generally three periods w hen pollen from various sources can cause intense hay fever
suffering: in the springtim e m onths of M arch and April w hen pollen from trees is prevalent, in the
sum m er m onths of June and July w hen grass pollen fills the air, and at the end of August w hen
ragweed pollen is a t its m ost concentrated levels.

1. W hich of the following w ould be the best 3. W hich of the following is NOT discussed in
title for the passage? the passage as a determ ining factor of the
am ount of pollen in the air?
(A) The R elationship betw een Season and
Allergies (A) Place
(B) M isconceptions and Facts a bout Hay (B) Clim ate
Fever (C) Time of year
(C) Hay Fever in the E astern United States (D) Altitude
(D) How Ragweed Causes Hay Fever
4. W hich of the following is NOT tru e about
2. According to the passage, w hich o f the hay fever in the eastern U nited States?
following helps to explain why the term "hay
fever" is som ew hat of a m isnom er? (A) Suffering from hay fever is equally
severe year-round.
(A) A strong fever o ccurs after an attack. (B) Pollen from trees causes hay fever
(B) The a m ount of pollen in the a ir suffering in the spring.
depends on geographical location. (C) G rass pollen fills the a ir e arlier in the
(C) Hay fever is often caused by ragweed year th an ragw eed pollen.
pollen. I.D ) Ragweed pollen is m ost prevalent at
(D) Grass pollen is prevalent in Ju n e and the end of the sum m er.
July.
304 READING COMPREHENSION

PASSAGE TWO (Questions 5 -9 )


Lincoln s now fam ous Gettysburg Address was not, on the occasion o f its delivery, recognized as
the m asterpiece th a t it is today. Lincoln was not even the prim ary speaker a t the cerem onies, held at
the height of the Civil W ar in 1863, to dedicate the battlefield at Gettysburg. The m ain speaker was
Line o ra to r E dw ard Everett, w hose tw o-hour speech was followed by L incolns sh o rter rem arks. Lincoln
(5) began his sm all portion of the program with the words that today are im m ediately i ecognized by
m ost Am ericans: "F our score and seven years ago our fathers b rought forth on this continent a new
nation, conceived in liberty and dedicated to the proposition th a t all m en are created equal. At the
tim e of the speech, little notice was given to w hat Lincoln had said, and Lincoln considered his
appearance at the cerem onies ra th e r unsuccessful. After his speech appeared in p rin t, appreciation
( 10) for his w ords began to grow, and today it is recognized as one of the all-tim e greatest speeches.

5. The m ain idea of this passage is that 8. W hen did L incolns Gettysburg Address
begin to receive public acclaim ?
(A) the G ettysburg A ddress has always
been regarded as a m asterpiece (A) After it h ad been published
(B) at the tim e of its delivery the (B) Im m ediately after the speech
G ettysburg A ddress was truly (C) Not until the p resent day
appreciated as a m asterpiece (D) After Lincoln received growing
(C) it was not until after 1863 th at recognition
Lincoln's speech at Gettysburg took
its place in history 9. The pronoun "it in line 10 refers to which
(D) Lincoln is better recognized today than of the following?
he w as a t the tim e of his presidency
(A) H is speech
(B) Print
6. W hich of the following is true about the
(C) A ppreciation
cerem onies a t Gettysburg during the Civil
(D) One
War?
(A) Lincoln w as the m ain speaker.
(B) Lincoln gave a tw o-hour speech.
(C) Everett w as the closing speaker of the
cerem onies.
(D) Everett's speech w as longer than
Lincolns.

7. According to the passage, when Lincoln


spoke at the G ettysburg ceremonies,
(A) his w ords were im m ediately
recognized by m ost Americans
(B) he spoke for only a short period of
lim e
(C) he was enthusiastically cheered
(D) he was extrem ely proud of his
perform ance
READING COMPREHENSION

PASSAGE THREE (Questions 10-15)


According to the theory of continental drift, the continents are n ot fixed in position but instead
move slowly across the surface of the E arth, constantly changing in position relative to one another.
This theory was first proposed in the eighteenth century w hen m apm akers noticed how closely the
Line continents of the E arth fit to gether w hen they were m atched up. It was suggested th en th a t the
(5) present-day continents h a d once been one large continent that had broken up into pieces w hich
drifted apart.
Today the m odern theory of plate tectonics has developed from the theory o f con tin en tal drift.
The theory of plate tectonics suggests that the crust of the E arth is divided into six large, a n d m any
small, tectonic plates th a t drift on the lava th a t composes the inner core of the E arth. These plates
(10) consist of ocean floor and continents that quite probably began breaking up and m oving relative to
one an o th er m ore than 2 0 0 m illion years ago.

10. The topic of this passage is 14. According to the passage, w hat co nstitutes
a tectonic plate?
(A) continental drift
(B) the theory of plate tectonics (A) Lava
(C) the development of ideas about the (B) Only the continents
movement of the Earth's surface (C) The inner core of the E arth
(D) eighteenth-century mapmakers (D) The surface of the land a n d the floor
of the oceans
11. The passage states that the theory of
continental drift developed as a result of 15. W hich of the following best describes the
organization of the passage?
(A) the fixed positions of the continents
(B) the work of mapmakers (A) Two unrelated theories are presented.
(C) the rapid movement of continents (B) Two contrasting opinions are stated.
(D) the fit of the Earths plates (C) A theory is follow ed by an exam ple.
(D) One hypothesis is developed from
12. The pronoun "they" in line 4 refers to another.
(A) mapmakers
(B) continents
(C) pieces
(D) tectonic plates

13. W hich of the following is NOT true about


the theory of plate tectonics?
(A) It is not as old as the theory of
continental drift.
(B) It evolved from the theory of
continental drift.
(C) It postulates th a t the E a rth s surface
is separated into plates.
(D) It was proposed by m apm akers.
306 READING COMPREHENSION

PASSAGE FOUR (Questions 16-24)


C harles Lutw idge Dodgson is perhaps not a nam e th a t is universally recognized, but Dodgson
did achieve enorm ous success under the pseudonym Lewis Carroll. He created this pseudonym from
th e L atinization, Carolus Ludovicus, of his real given nam es. It w as u n d e r the nam e Lewis Carroll that
Line Dodgson published the childrens books Alice's Adventures in Wonderland (1865) and its sequel
(5) Through the Looking Glass (1872). Though Dodgson achieved this success in children's literature, he
was not an a u th o r of childrens books by training or profession. His education and chosen field of
pu rsu it w ere far rem oved from the field of children's literatu re and were instead focused on
theoretical m athem atics.
Dodgson graduated w ith honors from C hrist Church, Oxford, in 1854 and then em barked on a
( 10) career in the w orld of academ ia. He worked as a lecturer in m athem atics a t Oxford and, later in his
career published a n um ber of theoretical works on m athem atics u nder his own nam e ra th e r than
un d er the pseudonym that he used for his childrens stones. He produced a n um ber of texts for
students, such as A Syllabus o f Plane Algebraical Geometry (I860), Fonnulae o f Plane Trigonometry
(1861) w hich was notable for the creativity of the sym bols th at he used to express trigonom etric
(1 5 ) functions such as sine and cosine, and A Guide for the Mathematical Student (1866). In a num ber of
m ore esoteric w orks, he cham pioned the principles of Euclid; in Euclid and His Modem Rivals (1879),
he presented his ideas on the superiority of Euclid over rival m athem aticians in a highly imaginative
fashion by devising a courtroom trial of anti-Euclid m athem aticians th at he nam ed Euclid-
w reakers and ultim ately finding the defendants guilty as charged. Curiosa Mathemalica (1888-1893)
( 20 ) m ade a furth er defense of E uclids work, focusing on E uclids definition of parallel lines. These
academ ic works never had the universal im pact of Dodgsons w orks for children using the nam e
Lewis Carroll, but they dem onstrate a solid body of w ell-regarded academ ic m aterial.

16. The topic of this passage is 19. It is NOT stated in the passage th at
Dodgson
(A) the works of Lewis Carroll
(B) Charles Dodgson and Euclid (A) attended C hrist C hurch, Oxford
(C) the story of Alice's Adventures in (B) studied c hildrens literature
Wonderland (C) w as an o utstanding student
(D) Dodgson and Carroll: m athem atics (D) w as a published a u th o r of academ ic
and childrens stories works

17. According to the passage, Dodgson 20. W hat is stated in the passage about the
w ork Formulae o f Plane Trigonometry?
(A) did not use his given nam e on his
stories for children (A) It portrayed m athem atics in a creative
(B) used the sam e nam e on all his way.
published works (B) It was w ritten by Euclid.
(C) used the nam e Carroll on his (C) It was published in 1860.
m athem atical works (D) It was one of the texts that Dodgson
(D) used a pseudonym for the work about studied at Oxford.
the courtroom trial
2 1 . All of the following are stated in the
18. W hich of the following is true, according passage about the w ork Euclid and His
to the passage? Modem Rivals EXCEPT that
(A) "Lewis is a L atin name. (A) it was published in 1879
(B) "Lutwidge" is p art of Dodgsons (B) it was a highly creative work
pseudonym . (C) it described an actual trial in which
(C) Carolus" is the Latin version of the Euclid participated
nam e "Charles. (D) it described a trial in w hich "Euclid-
(D) "Ludovicus is p art of Dodgsons given w reakers were found guilty
name.
READING COMPREHENSION

22. The passage indicates that which of the 24. What is stated in the passage about
following works was about Euclid? Dodgson's academic works?
(A) A Syllabus o f Plane Algebraical (A) They are all about Euclid.
Geometry (B) They had an impact on his works for
(B) Formulae o f Plane Trigonometry children.
(C) A Guide for the M athem atical Student (C) They were published under the name
(D) Curiosa Mathematica Lewis Carroll.
(D) They were well received in the
23. The pronoun they" in line 22 refers to academic world.
(A) parallel lines
(B) these academic works
(C) Dodgson's works for children
(D) children

IN D IR EC TLY A N S W E R E D Q U E S T IO N S ____________________

S o m e q u e s tio n s in th e R e a d in g C o m p r e h e n s io n s e c tio n o f th e T O E F L test w ill r e q u ir e


a n s w e rs th a t a re n o t d ir e c t ly sta te d in th e p assage. T o a n s w e r th e se q u e s t io n s c o r re c tly ,
y o u w ill h a v e to d ra w c o n c lu s io n s f r o m in f o r m a t io n th a t is g iv e n in t h e p a ssa g e. T w o c o m
m o n types o f in d ir e c t ly a n s w e r e d q u e s tio n s a re (1 ) im p lie d d e ta il q u e s tio n s a n d (2 ) t r a n
s itio n q u e stio n s.

S kill 6: A N S W E R IM PLIED D ETA IL Q U E S T IO N S C ORRECTLY

Y o u w ill s o m e tim e s b e a s k e d to a n s w e r a q u e s tio n b y d r a w in g a c o n c lu s io n f r o m a s p e c ific


d e ta il o r d e ta ils in th e p assag e. Q u e s t io n s o f th is ty p e c o n t a in th e w o r d s im plied, inferred,
likely, o r probably to le t y o u k n o w t h a t th e a n s w e r to th e q u e s tio n is n o t d ir e c t ly s ta te d . In
th is typ e o f q u e s tio n it is i m p o r t a n t to u n d e r s t a n d th a t y o u d o n o t h a v e to p u ll t h e a n
s w e r o u t o f th in a ir . In s t e a d , so m e in f o r m a t io n w ill b e g iv e n in th e p a ssa g e, a n d y o u w ill
d ra w a c o n c lu s io n f r o m th a t in f o r m a t io n .
READING COMPREHENSION

Exam ple
The passage:
The Hawaiian language is a melodious language in which all
words are derived from an alphabet of only twelve letters, the five
vowels a, e, i, o, u and the seven consonants h, k, I, m, n, p, w. Each
Line syllable in the language ends in a vowel, and two consonants never
(5) appear together, so vowels have a much higher frequency in the
Hawaiian language than they do in English.
This musical-sounding language can be heard regularly by
visitors to the islands. Most Hawaiians speak English, but it is
quite common to hear English that is liberally spiced with words
(10) and expressions from the traditional language of the cultui e. A
visitor may be greeted with the expression aloha and may be
referred to as a malihini because he is a newcomer to the island.
This visitor may attend an outside luau where everyone eats too
much and may be invited afterwards to dance the hula.

The questions:
1. Which of the following is probably NOT a Hawaiian word?
(A) mahalo
(B) mahim ahi
(C) meklea
(D) moana
2. It is implied that a luau is
(A) a dance
(B) a feast
(C) a concert
(D) a language

T o answ er th e first q u e s tio n , you sh o u ld refer to th e part o f th e p assage w h e r e it states


th a t in th e H aw aiian la n g u a g e two consonants never appear together. F rom this y o u can draw
th e c o n c lu s io n th at answ er (C ), meklea, is probab ly n o t a H aw aiian w ord b e c a u se th e c o n
so n a n ts k a n d / a p p ea r to g e th e r in th is w ord, so answ er (C) is th e b e st answ er to this q u e s
tio n . T o an sw er th e se c o n d q u e stio n , you sh o u ld r efe r to th e p art o f th e p assage w h er e it
sta tes th at at a lu au , everyone eats loo much. F rom th is you can draw th e c o n c lu sio n th a t a
lu a u is a fea st, w h ic h is a-very large m ea l. T h e b est an sw er is th e r e fo r e answ er (B ).
READING COMPREHENSION 309

T h e fo llo w in g c h a r t o u tlin e s th e key in fo r m a tio n th at y o u s h o u ld r e m e m b e r a b o u t


im p lie d d e ta il q u estio n s:

IMPLIED DjjTAIL QUESTIONS

HOW TO IDENTIFY It is im plied in the passage t h a t . . .


THE QUESTION It can be inferred fro m th e passage t h a t . ..
It is most likely t h a t ...
W h a t probably h a p p e n e d . . . ?

WHERE TO FIND The answers to these questions are found in order in the
THE ANSWER passage.

HOW TO ANSWER 1. Choose a key word in the question.


THE QUESTION 2. Scan the passage for the key word (or a related idea).
3. Carefully read the sentence that contains the key word.
4. Look for an answer that could be true, according to that
sentence.

T O E F L E X ER C ISE 6: S tu d y e a ch o f th e p assages a n d c h o o s e th e b e st a n sw e rs to th e q u e s
tio n s th at follow .

PASSAGE ONE (Qitestions 1-2)


Eskimos need efficient and adequate means to travel across water in that the areas where they
live are surrounded by oceans, bays, and inlets and dotted with lakes and seas. Two different types of
boats have been developed by the Eskimos, each constructed to meet specific needs.
Line The kayak is something like a canoe that has been covered by a deck. A kayak is generally
(5) constructed with one opening in the deck for one rider; however, som e kayaks are made for two.
Because the deck of a kayak is covered over except for the hole (or holes) for its rider (or riders), a
kayak can tip over in the water and roll back up without filling with water and sinking. One o f the
primary uses of the kayak is for hunting.
The umiak is not closed over, as is the kayak. Instead, it is an open boat that is built to hold ten
(10) to twelve passengers. Eskimos have numerous uses for the umiak which reflect the size of the boat;
e.g., the umiak is used to haul belongings from campsite to campsite, and it is used for hunting larger
animals that are too big to be hunted in a kayak.

1. It is implied in the passage that if a 2. It can be inferred from the passage that
kayak has two holes, then an example of the animal mentioned in
lines 11-12 might be
(A) it accommodates two riders
(B) it is less stable than a kayak with (A) a kangaroo
one hole (B) a snake
(C) it is as large as an umiak (C) a whale
(D) it cannot be used on the ocean (D) a salmon
310 READING COMPREHENSION

PASSAGE TW O (Questions 35)


Two types of trees from the same family of trees share honors in certain respects as the most
impressive of trees. Both evergreen conifers, the California redwood (Sequoia sempervirens) and the
giant sequoia (Sequoiandendron giganteum) are found growing natively only in the state of California.
Line The California redwood is found along the northern coast of the state, while the giant sequoia is
(5) found inland and at higher elevations, along the western slopes of the Sierra Nevadas.
The California redwood is the tallest living tree and is in fact the tallest living thing on the face
of the Earth; the height of the tallest redwood on record is 385 feet (120 meters). Though not quite as
tall as the California redwood, with a height of 320 feet (100 meters), the giant sequoia is nonetheless
the largest and most massive of living things; giant sequoias have been measured at more than 100
(10) feet (30 meters) around the base, with weights o f more than 6,000 tons.

3. It is implied in the passage that 5. Which of the following is implied in the


passage?
(A) the leaves of only the California
redwood turn brown in the autumn (A) The giant sequoia is taller than the
(B) the leaves of only the giant sequoia California redwood.
turn brown in the winter (B) The California redwood is not as big
(C) the leaves o f both types of trees in the around as the giant sequoia.
passage turn brown in the winter (C) The California redwood weighs more
(D) the leaves of neither type of tree in the than the giant sequoia.
passage turn brown in the winter (D) Other living things are larger than the
giant sequoia.
4. It can be inferred from the passage that the
Sierra Nevadas are
(A) a type of giant redwood
(B) a coastal community
(C) a group of lakes
(D) a mountain range
READING COMPREHENSION 3 1I

PASSAGE THREE (Questions 6-8)


Probably the m ost recognized boardgam e around the w orld is the gam e of M onopoly. In this
game, players vie for w ealth by buying, selling, and renting p roperties; the key to success in the gam e,
in addition to a bit of luck, is for a player to acquire m onopolies on clusters of pro p erties in o rd e r to
Line force opponents to pay exorbitant rents a n d fees.
(5) Although the gam e is now published in countless languages and versions, w ith foreign locations
and place nam es appropriate to the target language adorning its board, the beginnings of the gam e
were considerably m ore hum ble. The gam e was invented in 1933 by Charles Darrow, du rin g the
height of the G reat D epression. Darrow, w ho lived in Germ antow n, Pennsylvania, w as him self
unem ployed during those difficult financial tim es. He set the original gam e not as m ight be expected
(10) in his hom etow n of G erm antow n, but in Atlantic City, New Jersey, the site of n u m ero u s p re
Depression vacations, w here he walked along the Boardw alk and visited at P ark Place. D arrow m ade
the first gam es by hand and sold them locally until Parker B rothers purchased the rights to M onopoly
in 1935 and took the first steps tow ard the m ass production of today.

6. The French version of M onopoly m ight 8. P arker B rothers is probably


possibly include a piece of property entitled
(A) a real estate com pany
(A) Atlantic City, New Jersey (B) a gam e m anu factu rin g com pany
(B) G erm antow n, Pennsylvania (C) a group of C harles D arrow s friends
(C) Boardwalk (D) a toy design com pany
(D) the Eiffel Tower

It is implied that Darrow selected A tlantic


City as the setting for M onopoly because
(A) it brought back good m em ories
(B) his family cam e from Atlantic City
(C) the people of G erm antow n m ight have
been angered if he h a d used
G erm antow n
(D) Atlantic City w as larger than
G erm antow n

S k i l l 7: A N S W E R T R A N S IT IO N Q U E S T IO N S CORRECTLY

You w ill s o m e tim e s b e a sk ed to d e t e r m in e w h at p ro b a b ly c a m e b e fo r e th e r e a d in g p as


sa g e (in th e preceding p a r a g r a p h ) o r w h at p rob ab ly c o m e s a fter th e r e a d in g p a ssa g e (in
th e fo llo w in g p a ra g ra p h ). O f c o u r se , th e to p ic o f th e preceding o r fo llo w in g p a ra g ra p h is n o t
d irectly sta ted , a n d y o u m u st draw a c o n c lu s io n to d e te r m in e w h a t is p r o b a b ly in th e se
paragraphs.
T h is type o f q u e s tio n is a tra n sitio n q u e s tio n . It asks y o u to d e m o n s tr a te th a t y o u u n
d e r sta n d th a t g o o d w r itin g c o n ta in s transitions from o n e p a r a g r a p h to th e n e x t. A para
g r a p h m ay start o u t w ith th e id e a o f th e p r e v io u s p a ra g ra p h as a w ay o f lin k in g th e id e a s
in th e two p aragrap h s. A p a r a g r a p h m ay a lso e n d w ith an id e a th a t w ill b e fu r th e r d e v e l
o p e d in th e fo llo w in g p a ra g ra p h .
READING COMPREHENSION

Exam ple

The passage:
Another m yth of the oceans concerns Davy Jones, who in
folklore is the m ean-spirited sovereign of the ocean's depths. The
nam e Jones is thought by som e etym ologists to have been derived
Line from the nam e "Jonah, the H ebrew p rophet who spent three days
(5) in a w hales belly.
According to tradition, any object th a t goes overboard and
sinks to the bottom of the ocean is said to have gone to Davy
Joness locker, the ocean-sized, m ythical receptacle for anything
th a t falls into the water. Needless to say, any sailor on the seas is
(JO) not so eager to take a to u r of Davy Joness locker, although it m ight
be a rath er interesting trip considering all the treasures located
there.

The questions:
1. The paragraph preceding this passage m ost probably discusses
(A) the youth of Davy Jones
(B) Davy Joness career as a sailor
(C) a different traditional story from the sea
(D) preparing to travel on the ocean
2. The topic of the paragraph following the passage m ost likely is
(A) valuable item s located at the bottom of the ocean
(B) w here Davy Jones is found today
(C) Jonah and the whale
(D) preventing objects from falling overboard

T h e first q u e s tio n asks a b o u t th e to p ic o f th e preceding p a r a g r a p h , so y o u m u st lo o k at th e


b e g in n in g o f th e p assage a n d draw a c o n c lu sio n a b o u t w h a t p ro b a b ly ca m e before. S in ce
th e p a ssa g e b e g in s w ith th e e x p r e s sio n another m yth o f the oceans, y o u sh o u ld u n d e rsta n d
th a t th e n e w p a ssa g e is g o in g to p r e se n t a second m yth o f th e o c e a n s a n d th e p reviou s pas
sage p ro b a b ly p r e se n te d th e fir s t m yth o f th e o c e a n s. A m yth is a tra d itio n a l story, so th e
b e st an sw er to th is q u e stio n is answ er (C ). T h e s e c o n d q u e s tio n asks a b o u t th e to p ic o f
th e /o ifo z w n g p a r a g ra p h , so y o u m u st lo o k :U th e e n d o f th e p a ssa g e a n d draw a c o n c lu sio n
a b o u t w h a t p ro b a b ly c o m e s after. T h e passage e n d s w ith th e m e n tio n o f all the treasures lo
cated there, a n d there is in Davy J o n e s s lock er, o r at th e b o tto m o f th e o c ea n ; th is is p r o b a
bly g o in g to b e th e to p ic o f th e n e x t p aragrap h . T h e b e s t an sw er to th e se c o n d q u e stio n is
th e r e fo r e an sw er (A ).

T h e fo llo w in g ch a rt o u tlin e s th e key in fo r m a tio n th a t y o u s h o u ld r em e m b e r a b o u t


tra n sitio n q u estio n s:

T R A N S IT IO N Q U E S T IO N S

HOW TO IDENTIFY The paragraph p r e c e d in g the passage p robably . . .


THE. QUESTION W h a t is most likely in the para g ra ph fo llo w in g the passage?

.WHERETO FIND T h e answ er can generally be found in the firs t line o f the passage
THE ANSWER for a preceding question .T he an sw e r can generally be found in
the last line fo r a follow ing question.
READING COMPREHENSION 31

HOW TO ANSWER 1. Read the firs t line for a preceding question.


THE QUESTION
2. Read the last line for a follow ing question.
.. ..
3. Draw a conclusion about what comes b efo re or a fter.
4. Choose the answer that is reflected in the firs t or last line
of the passage.

T O E F L E X E R C ISE 7: S tu d y e a c h o f th e p assages a n d c h o o s e th e b e st a n sw ers to th e q u e s


tio n s th at follow .

PASSAGE ONE (Questions 1-2)


Another program instrumental in the popularization o f science was Cosmos. This series,
broadcast on public television, dealt with topics and issues from varied fields o f science. The
principal writer and narrator o f the program was Carl Sagan, a noted astronomer and Pulitzer Prize-
winning author.

1. The paragraph preceding this passage most 2. The paragraph following this passage most
probably discusses likely contains information on what?
(A) a different scientific television series (A) The popularity of science
(B) Carl Sagans scientific achievements (B) The program Cosmos
(C) the Pulitzer Prize won by Carl Sagan (C) The astronomer Carl Sagan
(D) public television (D) Topics and issues from various fields
of science

PASSAGE TW O (Questions 34)


When a strong earthquake occurs on the ocean floor rather than on land, a tremendous force is
exerted on the seawater and one or more large, destructive waves called tsunam is can be formed.
Tsunamis are commonly called tidal waves in the United States, but this is really an inappropriate
Line name in that the cause o f the tsunami is an underground earthquake rather than the oceans tides.
(5) Far from land, a tsunami can move through the wide open vastness of the ocean at a speed of
600 miles (900 kilometers) per hour and often can travel tremendous distances without losing height
and strength. When a tsunami reaches shallow coastal water, it can reach a height of 100 feet (30
meters) or more and can cause tremendous flooding and damage to coastal areas.

3. The paragraph preceding the passage most 4. Which of the following is m ost likely the
probably discusses topic of the paragraph following the
passage?
(A) tsunamis in various parts of the world
(B) the negative effects of tsunamis (A) The causes of tsunam is
(C) land-based earthquakes (B) The destructive effects of tsunam is on
(D) the effect of tides on tsunamis the coast
(C) The differences between tsunamis and
tidal waves
(D) The distances covered by tsunamis
3 14 READING COMPREHENSION

PASSAGE THREE (Questions 5-6)


While draft laws are federal laws, marriage laws are state laws rather than federal; marriage
regulations are therefore not uniform throughout the country. The legal marriage age serves as an
example of this lack of conformity. In most states, both the man and the woman must be at least
Line eighteen years old to marry without parental consent; however, the states of Nebraska and Wyoming
(5) require the couple to be at least nineteen, while the minimum age in Mississippi is twenty-one. If
parental permission is given, then a couple can marry at sixteen in some states, and a few states even
allow marriage before the age of sixteen, though a judge's permission, in addition to the permission of
the parents, is sometimes required in this situation. Some states which allow' couples to marry at such
a young age are now considering doing away with such early marriages because o f the numerous
(10) negative effects of these young marriages.

5. The paragraph preceding the passage most 6. The topic of the paragraph following the
probably discusses passage is most likely to be
(A) state marriage laws (A) disadvantages of youthful marriages
(B) the lack of uniformity in marriage laws (B) reasons why young people decide to
(C) federal draft laws marry
(D) the minimum legal marriage age (C) the age when parental consent for
marriage is required
(D) a discussion of why some states allow
marriages before the age of sixteen

T O E F L E X E R C ISE (S k ills 6 - 7 ) : Study e a c h o f th e p a ssa g es a n d c h o o s e th e b est answ ers


to th e q u e s tio n s th a t follow .

PASSAGE ONE (Questions 1-4)


The most conservative sect of the Mennonite Church is the Old Order Amish, with 33,000
members living mainly today in the states of Pennsylvania, Ohio, and Indiana. Their lifestyle reflects
their belief in the doctrines of separation from the world and simplicity of life. The Amish have
Line steadfastly rejected the societal changes that have occurred in the previous three hundred years,
(5) preferring instead to remain securely rooted in a seventeenth-century lifestyle. They live on farms
without radios, televisions, telephones, electric lights, and cars; they dress in plainly styled and
colored old-fashioned clothes; and they farm their lands with horses and tools rather than modem
farm equipment. They have a highly communal form of living, with bam raisings and quilting bees as
commonplace activities.

1. The paragraph preceding this passage most 2. Which of the following would probably
probably discusses NOT be found on an Amish farm?
(A) other, more liberal sects of Mennonites (A) A hammer
(B) where Mennonites live (B) A cart
(C) the communal Amish lifestyle (C) A long dress
(D) the most conservative Mennonites (D) A refrigerator
READING COMPREHENSION 3 15

3. It can be inferred from the passage that a 4. Which o f the following is m ost likely the
quilting bee topic of the paragraph following the
passage?
(A) involves a group o f people
(B) is necessary when raising bees (A) The effects of the communal lifestyle
(C) always follows a barn raising on the Old Order Amish
(D) provides needed solitude (B) How the Old Order Amish differ from
the Mennonites
(C) The effect of modern technology on the
Old Order Amish
(D) The doctrines of the Old Order Amish

PASSAGE TWO (Questions 5-8 )


Various other Indian tribes also lived on the Great Plains. The Sioux, a group of seven American
Indian tribes, are best known for the fiercely combative posture against encroaching White
civilization in the 1800s. Although they are popularly referred to as Sioux, these Indian tribes did not
Line call themselves Sioux; the name was given to them by an enemy tribe. The seven Sioux tribes called
(5) themselves by som e variation of the word "Dakota, which means "allies in their language. Four
tribes of the eastern Sioux community living in Minnesota were known by the name Dakota. The
Nakota included two tribes that left the eastern woodlands and moved out onto the plains. The Teton
Sioux, or Lakota, moved even farther west to the plains of the present-day states of North Dakota,
South Dakota, and Wyoming.

5. The paragraph preceding this passage 7. It is implied in the passage that the
most probably discusses seven Sioux tribes called each other by
some for n o f the word "Dakota"
(A) how the Sioux battled the White
becaus' they were
man
(B) one of the Plains Indian tribes (A) united in a cause
(C) where the Sioux lived (B) all living in North Dakota
(D) American Indian tribes on the (C) fiercely combative
East Coast (D) enem ies

6. Which of the following represents a 8. It can be inferred from the passage that
likely reaction of the Sioux in the 1800s the present-day states of North and
to the encroaching White civilization? South Dakota
(A) The Sioux would probably help (A) are east of Minnesota
the Whites to settle in the West. (B) are hom e to the four tribes known
(B) The Sioux would probably attack by the name Dakota
the White settlers. (C) received their nam es from the
(C) The Sioux would probably invite Indian tribes living there
the Whites to smoke a peace (D) are part of the eastern woodlands
pipe.
(>) The Sioux would probably join
together in hunting parties with
the White settlers.
316 READING COMPREHENSION

PASSAGE THREE (Questions 9-12)


The extinction o f many species of birds has undoubtedly been hastened by modern man; since
1600 it has been estimated that approximately 100 bird species have become extinct over the world.
In North America, the first species known to be annihilated was the great auk, a flightless bird that
Line served as an easy source of food and bait for Atlantic fishermen through the beginning of the
(5) nineteenth century.
Shortly after the great auk's extinction, two other North American species, the Carolina parakeet
and the passenger pigeon, began dwindling noticeably in numbers. The last Carolina parakeet and the
last passenger pigeon in captivity both died in September 1914. In addition to these extinct species,
several others such as the bald eagle, the peregrine falcon, and the California condor are today
( 10) recognized as endangered; steps are being taken to prevent their extinction.

9. The number of bird species that have 11. It can be inferred from the passage that
becom e extinct in the United States the great auk was killed because
since 1600 m ost probably is (A) it was eating the fishermens catch
(A) more than 100 (B) fishermen wanted to eat it
(B) exactly 100 (C) it flew over fishing areas
(C) less than 100 (D) it baited fishermen
(D) exactly three
12. The paragraph following this passage
10. The passage implies that the great auk most probably discusses
disappeared (A) what is being done to save
(A) before 1600 endangered birds
(B) in the 1600s (B) what the bald eagle symbolizes to
(C) in the 1800s Americans
(D) in the last fifty years (C) how several bird species became
endangered
(D) other extinct species

T O E F L REV IEW E X E R C ISE (S k ills 1 -7 ): Stu dy e a c h o f th e p assages a n d c h o o s e th e b est


an sw ers to th e q u e s tio n s th a t follow .

PASSAGE ONE (Questions 1-6)


The Mason-Dixon Line is often considered by Americans to be the demarcation between the
North and the South. It is in reality the boundary that separates the state of Pennsylvania from
Maryland and parts of West Virginia. Prior to the Civil War, this southern boundary of Pennsylvania
Line separated the nonslave states to the north from the slave states to the south.
(5) The Mason-Dixon Line was established well before the Civil War, as a result of a boundary
dispute between Pennsylvania and Maryland. Two English astronomers, Charles Mason and Jeremiah
Dixon, were called in to survey the area and officially mark the boundary between the two states. The
survey was completed in 1767, and the boundary was marked with stones, many of which remain to
this day.

1. The best title for this passage would be 2. It can be inferred from the passage that
before the Civil War
(A) Dividing the North and the South
(B) The Meaning of the Mason-Dixon (A) Pennsylvania was south of the
Line Mason-Dixon Line
(C) Two English Astronomers (B) Pennsylvania was a nonslave state
(D) The History of the Mason-Dixon (C) the states south of the Mason-
Line Dixon Line had the same
opinion about slavery as
Pennsylvania
(D) the slave states were not divided
from the nonslave states
READING COMPREHENSION 3 17

3. According to the passage, the Mason- 5. The passage indicates that the Mason-
Dixon Line was established because of Dixon Line was identified with
a disagreement
(A) pieces of rock
(A) about borders (B) fences
(B) about slaves (C) a stone wall
(C) between two astronomers (D) a border crossing
(D) over surveying techniques
6. The paragraph following the passage
4. The passage states all of the following m ost probably discusses
about Mason and Dixon EXCEPT that
(A) where the M ason-Dixon Line is
(A) they came from England located
(B) they worked as astronomers (B) the Mason-Dixon Line today
(C) they caused the boundary dispute (C) the effect of the Civil War on
between Pennsylvania and slavery
Maryland (D) what happened to Charles Mason
(D) they surveyed the area of the and Jeremiah Dixon
boundary between Pennsylvania
and Maryland

PASSAGE TWO (Questions 7-12)


Manic depression is another psychiatric illness that mainly affects mood. A patient suffering
from this disease will alternate between periods of manic excitement and extreme depression, with or
without relatively normal periods in between. The changes in m ood suffered by a manic-depressive
Line patient go far beyond the day-to-day mood changes experienced by the general population. In the
(5) period of manic excitement, the m ood elevation can become so intense that it can result in extended
insomnia, extreme irritability, and heightened aggressiveness. In the period of depression, which may
last for several weeks or m onths, a patient experiences feelings of general fatigue, uselessness, and
hopelessness and, in serious cases, may contemplate suicide.

7. The paragraph preceding this passage 10. The passage indicates that m ost people
most probably discusses
(A) never undergo m ood changes
(A) when manic depression develops (B) experience occasional shifts in
(B) a different type of mental disease mood
(C) how moods are determined (C) switch wildly from highs to lows
(D) how manic depression can result (D) become highly depressed
in suicide
11. The pronoun "it in line 5 refers to
8. The topic of this passage is
(A) the general population
(A) various psychiatric illnesses (B) the m ood elevation
(B) how depression affects the mood (C) insomnia
(C) the intense period of manic (D) heightened aggressiveness
excitement
(D) the mood changes o f manic 12. The passage implies that
depression
(A) changes from excitem ent to
depression occur frequently and
9. According to the passage, a manic-
often
depressive patient in a manic phase
(B) only manic-depressive patients
would be feeling
experience aggression
(A) highly em otional (C) the depressive phase of this
(B) unhappy disease can be more harmful
(C) listless than the m anic phase
(D) relatively normal (D) suicide is inevitable in cases of
manic depression
3 18 READING COMPREHENSION

PASSAGE THREE (Questions 13-18)


Unlike earlier campaigns, the 1960 presidential campaign featured the politically innovative and
highly influential series of televised debates in the contest between the Republicans and the
Democrats. Senator John Kennedy established an early lead among the Democratic hopefuls and was
Line nominated on the first ballot at the Los Angeles convention to be the representative of the Democratic
(5) party in the presidential elections. Richard Nixon, then serving as vice president of the United States
under Eisenhower, received the nomination of the Republican party. Both Nixon and Kennedy
campaigned vigorously throughout the country and then took the unprecedented step of appearing in
face-to-face debates on television. Political experts contend that the debates were a pivotal force in
the elections. In front o f a viewership of more than 100 million citizens, Kennedy masterfully
(10) overcame Nixons advantage as the better-known and more experienced candidate and reversed the
public perception of him as too inexperienced and immature for the presidency.

13. Which of the following best expresses 16. Which of the following is NOT
the main idea of the passage? mentioned about Richard Nixon?
(A) Kennedy defeated Nixon in the (A) He was serving as vice president.
1960 presidential election. (B) He was the Republican party's
(B) Television debates were candidate for president.
instrumental in the outcome of (C) He campaigned strongly all over
the 1960 presidential election. the country.
(C) Television debates have long been (D) He was nominated on the first
a part of campaigning. ballot.
(D) Kennedy was the leading
Democratic candidate in the 17. The passage states that in the debates
1960 presidential election. with Nixon, Kennedy demonstrated to
the American people that he was
14. The passage implies that Kennedy
(A) old enough to be president
(A) was a long shot to receive the (B) more experienced than Nixon
Democratic presidential (C) better known than Nixon
nomination (D) too inexperienced to serve as
(B) won the Democratic presidential president
nomination fairly easily
(C) was not a front runner in the 18. The pronoun him in line 11 refers to
race for the Democratic
presidential nomination (A) John Kennedy
(D) cam e from behind to win the (B) Richard Nixon
Democratic presidential (C) Eisenhower
nomination (D) the better-known and more
experienced candidate
15. The passage states that the television
debates between presidential
candidates in 1960
(A) did not influence the selection of
the president
(B) were the final televised debates
(C) were fairly usual in the history of
presidential campaigns
(D) were the first presidential
campaign debates to be
televised
READING COMPREHENSION 319

PASSAGE FOUR (Questions 19-29)


Unlike these fish, which are actually extinct, the coelacanth is a type of fish that w as believed to
be extinct. However, an unexpected twentieth-century rediscovery of living coelacanths has brought
about a reassessment of the status of this prehistoric sea creature that was believed to have long since
Line disappeared from the Earth.
(5) From fossil remains of the coelacanth, paleontologists have determined that the coelacanth was
in existence around 350 m illion years ago, during the Paleozoic Era, more than 100 m illion years
before the first dinosaurs arrived on Earth. The most recent fossilized coelacanths date from around
70 million years ago, near the end of the Mesozoic Era and near the end of the age of dinosaurs.
Because no fossilized remnants o f coelacanths from the last 70 million years have been found, the
(10) coelacanth was believed to have died out around the same time as the dinosaurs.
The prehistoric coelacanth studied by paleontologists had distinctive characteristics that
differentiated it from other fish. It was named for its hollow spine and was known to have been a
powerful carnivore because of its many sharp teeth and a special joint in the skull that allowed the
ferocious teeth to move in coordination with the lower jaw. It also had a pair of fins with unusual
(15) bony and muscular development that allowed the coelacanth to dart around the ocean floor. These
fins also enable the coelacanth to search out prey trying to hide on the ocean bottom.
In 1938, a living specimen of the coelacanth was discovered in the catch of a Ashing boat off the
coast of South Africa, and since then numerous other examples of the coelacanth have been found in
the waters of the Indian Ocean. This modern version of the coelacanth is not exactly the same as its
(20) prehistoric cousin. Todays coelacanth is larger than its prehistoric relative, measuring up to six feet
in length and weighing up to 150 pounds. However, the modern version of the coelacanth still
possesses the characteristic hollow spine and distinctive fins with their unusual bony and muscular
structure.

19. The topic of the preceding paragraph is 23. It can be inferred from the passage that the
most likely word coelacanth com es from the Greek for
(A) various extinct fish (A) extinct fish
(B) the discovery of the coelacanth (B) hollow spine
(C) a reassessment of the status of a (C) sharp teeth
number of kinds of fish (D) bony fingers
(D) a particular prehistoric sea creature
24. What is stated in the passage about the
20. This passage is about a fish prehistoric coelacanth?
(A) that is extinct (A) It was a rather feeble fish.
(B) that once was extinct (B) It lived on plants.
(C) that is becom ing extinct (C) It had few teeth.
(D) that, surprisingly, is not extinct (D) It moved its teeth in an unusual way.

21. It can be inferred from the passage that the 25. The pronoun It" in line 14 refers to
first dinosaurs m ost likely appeared on
(A) coelacanth
Earth around
(B) joint
(A) 150 million years ago (C) coordination
(B) 250 million years ago (D) jaw
(C) 350 million years ago
(D) 450 million years ago 26. According to the passage, why are
scientists sure that the prehistoric
22. Coelacanths were believed to have died out coelacanth was a flesh-eater?
after existing for
(A) Because of its hollow spine
(A) 70 million years (B) Because of the size of the skull
(B) 140 million years (C) Because of the shape and movement
(C) 280 million years of the teeth
(D) 350 million years (D) Because of its unusual bony and
muscular development
320 READING COMPREHENSION

27. How many m odem coelacanths have been 29. Which paragraph describes the earlier
found? version of the coelacanth?
(A) Only one (A) The first paragraph
(B) Only two (B) The second paragraph
(C) Only a few (C) The third paragraph
(D) Quite a few (D) The fourth paragraph

28. What is NOT true about the prehistoric


coelacanth, according to the passage?
(A) It was smaller than the modem
coelacanth.
(B) It measured as much as six feet in
length.
(C) It weighed less than 150 pounds.
(D) It had a hollow spine and distinctive
fins.

VO C A B U LA R Y Q U E S TIO N S

In th e R e a d in g C o m p r e h e n s io n se c tio n o f th e T O E F L test, th e r e w ill b e a n u m b e r o f vo


ca b u la ry q vtestions. T o answ er this type o f q u e stio n , it is o f c o u r se h e lp fu l i f y o u k n ow th e
m e a n in g o f th e w o r d th at th e the T O E F L test is te stin g . H ow ever, it is n o t always necessary
fo r y o u to k n o w th e m e a n in g o f th e word; o ften th e r e are sk ills th a t y o u can u se to h e lp
y o u fin d th e c o r r e c t answ er to th e q u estion : (1 ) fin d in g d e fin itio n s from structural clu es,
(2 ) d e te r m in in g m e a n in g s from w ord parts, an d (3) u s in g c o n te x t c lu e s to d e te r m in e
m e a n in g s.

S kill 8: F IN D D E F IN IT IO N S FROM STR U C TU R A L CLUES

W h e n y o u are a sk ed to d e te r m in e th e m e a n in g o f a w o rd in th e R e a d in g C o m p r e h e n s io n
s e c tio n o f th e T O E F L test, it is p o ssib le (1) that th e p assage p r o v id e s in fo r m a tio n a b o u t
th e m e a n in g o f th e w ord an d (2) th at th ere are stru ctu ral c lu e s to tell you th at th e d e fin i
tio n o f a w o rd is in c lu d e d in th e passage.

2-
READING COMPREHENSION

E x am p le

The passage:
One of the leading schools o f psychological thought in the
twentieth century was behaviorismthe belief that the role o f the
psychologist is to study behavior, which is observable, rather than
Line conscious or unconscious thought, which is not. Probably the best
ir,) known proponent of behaviorism is B.E Skinner, who was famous
for his research on how rewards and punishments influence
behavior. He cam e to believe that positive reinforcements such as
praise, food, or money were more effective in promoting good
behavior than negative reinforcement, or punishment.

The qu estion s:
1. In behaviorism in line 2, a psychologist is concerned with
(A) conscious thought patterns
(B) unconscious thought patterns
(C) observable actions
(D) unobservable actions
2. What is "positive reinforcement" in line 7?
(A) A gift
(B) A reward
(C) A bribe
(D) A penalty
3. What is "negative reinforcement" in line 9?
(A) A promotion
(B) A reward
(C) A surprise
(D) A punishment

T o an sw er th e first q u e s tio n , y o u s h o u ld lo o k at th e part o f th e p a ssa g e fo llo w in g th e w ord


behaviorism. T h e d a sh p u n c tu a tio n ( ) in d ic a te s th a t a d e fin itio n o r fu r th e r in fo r m a tio n
a b o u t b e h a v io r ism is g o in g to follow . In th e in fo r m a tio n fo llo w in g th e d a sh y o u s h o u ld
s e e th a t th e b e h a v io r ist is in te r e s te d in behavior, w hich is observable, so th e b e st a n sw e r to
th is q u e s tio n is an sw er (C ). T o a n sw er th e s e c o n d q u e s tio n , you s h o u ld lo o k at th e p art o f
th e p a ssage fo llo w in g th e e x p r e s s io n positive reinforcements. T h e e x p r e s s io n such as in d i
c a tes th a t e x a m p le s o f p o sitive reinforcem ent are g o in g to follow . Y our j o b is to lo o k at th e
e x a m p le s o f p o sitiv e r e in fo r c e m e n t a n d draw a c o n c lu s io n a b o u t w h at p o sitiv e r e in fo r c e
m e n t m ig h t b e . S in c e praise, fo o d , o r money m ig h t be g iv en in retu rn fo r a j o b w e ll d o n e ,
th e n positive reinforcem ent m u st b e a reward. T h e b e st an sw er to th is q u e s tio n is th e r e fo r e
an sw er (B ). T o an sw er th e th ir d q u e s tio n , you sh o u ld lo o k at th e p a rt o f th e p a ssa g e fo l
lo w in g th e e x p r e s s io n negative reinforcement. T h e w ord or fo llo w in g n egative reinforcement
tells y o u th a t th e id e a is g o in g to b e r esta te d in d iffe r e n t w ords. You can s e e in th e p a ssa g e
th a t a n o th e r w o r d fo r neg a tive reinforcem ent is p u n ish m e n t, so an sw er (D ) is th e b e s t an sw er
to th is q u e s tio n .
322 READING COMPREHENSION

T h e fo llo w in g c h a r t o u tlin e s th e key in fo r m a tio n th a t y o u sh o u ld r e m e m b e r a b o u t


stru ctu ral c lu e s to h e lp y o u u n d e r sta n d u n k n o w n vo ca b u la ry w ords:

STRUCTURAL CLUES

HOW TO IDENTIFY What is ... ?


THE QUESTION
rj
- What is the m eaning o f. .. ?
What is true about.. .?

TYPES OF CLUES Punctuation: comma, parentheses, dashes


Restatement: or, that is, in other words, i.e.
Examples: such as, for example, e.g.

WHERE TO FIND Information to help you determine what something means


THE ANSWER will generally be found after the punctuation clue, the
. ' : ' : - * : . restatement clue, or the example clue.

HOW TO ANSWER 1. Find the word in the passage.


THE QUESTION 2. Locate any structural clues.
3. Read the part of the passage after the structural clue
carefully.
4. Eliminate any definitely wrong answers and choose
the best answer from the remaining choices.

T O E F L E X E R C ISE 8: Study e a c h o f th e p assages a n d c h o o s e th e b e st answ ers to th e


q u e s tio n s th a t follow .

PASSAGE ONE (Questions 1-4)


The teddy bear is a child's toy, a nice, soft stuffed animal suitable for cuddling. It is, however, a
toy with an interesting history behind it.
Theodore Roosevelt, or Teddy as he was commonly called, was president of the United States
Line from 1901 to 1909. He was an unusually active man with varied pastimes, one of which was hunting.
(5) One day the president was invited to take part in a bear hunt; and inasmuch as Teddy was president,
his hosts wanted to ensure that he caught a bear. A bear was captured, clunked over the head to knock
it out, and tied to a tree; however, Teddy, who really wanted to actually hunt, refused to shoot the bear
and in fact demanded that the bear be extricated from the ropes; that is, he demanded that the bear
be set free.
(10) The incident attracted a lot of attention among journalists. First a cartoondrawn by Clifford
K. Berryman to make fun of this situation appeared in the Washington Post, and the cartoon was
widely distributed and reprinted throughout the country. Then toy manufacturers began producing a
toy bear which they called a "teddy bear." The teddy bear became the m ost widely recognized symbol
of Roosevelt s presidency.

1. According to line 1 of the passage, what 2. In line 4, "pastimes could best be


is a teddy bear? replaced by
(A) A ferocious animal (A) things that occurred in the past
(B) The president o f the United States (B) previous jobs
(C) A famous hunter (C) hunting trips
(D) A plaything (D) leisure activities
READING COMPREHENSION 323

3. The word extricated" in line 8 is 4. In line 10, a cartoon could best be


closest in meaning to which of the described as
following?
(A) a newspaper
(A) Released (B) a type of teddy bear
(B) Tied up (C) a drawing with a message
(C) Hunted (D) a newspaper article
(D) Shot

PASSAGE TW O (Questions 5-8 )


A supernova occurs when all of the hydrogen in the core of a huge star is transformed to iron
and explodes. All stars die after their nuclear fuel has been exhausted. Stars with little mass die
gradually, but those with relatively large mass die in a sudden explosion, a supernova. The sudden
Line flash of light can then be followed by several weeks of extremely bright light, perhaps as much light as
(5) twenty million stars.
Supemovae are not very common; they occur about once every hundred years in any galaxy, and
in 1987 a supernova that could be seen by the naked eye occurred in the Magellan Cloud, a galaxy
close to the Milky Way. Scientists periodically detect supemovae in other galaxies; however, no
supemovae have occurred in the Milky Way (the galaxy that includes the Earth) since 1604. One very
( 10 ) impressive supernova occurred in the Milky Way on July 4, 1054. There was a great explosion
followed by three months of lighted skies, and historical chronicles of the time were full o f accounts
and unusual explanations for the misunderstood phenomenonmany people believed that it meant
that the world was com ing to an end.

5. A supernova" in line 1 is which of the 7. The Milky Way" in line 9 is


following?
(A) part of the Earth
(A) The iron component of a star (B) a galaxy close to the Earth
(B) The core of a star (C) the galaxy that is hom e to the
(C) The hydrogen in a star Earth
(D) The explosion of a star (D) a creamy-colored cloud in the sky

6. According to the passage, which of the 8. Which of the following is closest in


following best describes the Magellan meaning to "phenomenon" in line 12?
Cloud" in line 7?
(A) Everyday occurrence
(A) A galaxy inside the Milky Way (B) Misunderstood event
(B) A cloud com posed of hydrogen (C) Common belief
(C) A galaxy near the Earths galaxy (D) Unusual occurrence
(D) A cloud in the sky above the Earth
READING COMPREHENSION

S k ill 9: D E T E R M IN E M EA N IN G S FROM W O R D PARTS

W h e n y o u a r e a s k e d to d e t e r m in e t h e m e a n in g o f a lo n g w o r d th a t y o u d o n o t k n o w in
th e R e a d in g C o m p r e h e n s io n s e c tio n o f th e T O E F L test, it is s o m e tim e s p o ssib le to d e t e r
m in e t h e m e a n in g o f t h e w o r d b y s tu d y in g th e w o rd p a rts.

Exam ple
The passage:
Ring Lardner himself was born into a wealthy, educated, and
cultured family. For the bulk of his career, he worked as a reporter
for newspapers in South Bend, Boston, St. Louis, and Chicago.
Line However, it is for his short stories of lower middle-class Americans
(5) that Ring Lardner is perhaps best known. In these stories, Lardner
vividly creates the language and the ambiance of this lower class,
often using the misspelled words, grammatical errors, and incorrect
diction that typified the language of the lower middle class.
The questions:
1. The word "vividly" in line 6 is closest in meaning to
(A) in a cultured way
(B) in a correct way
(C) in a lifelike way
(D) in a brief way
2. The word "misspelled in line 7 is closest in meaning to
(A) highly improper
(B) vulgar
(C) incorrectly written
(D) slang
3. The word diction in line 8 is closest in meaning to
(A) writing
(B) sentence structure
(C) form
(D) speech

In t h e f ir s t q u e s tio n , t h e w o r d v iv id ly c o n ta in s th e w o r d p a r t v iv , w h ic h m e a n s life, so th e
b e s t a n s w e r is a n s w e r ( C ) . I n th e s e c o n d q u e s tio n , th e w o rd m isspelled c o n ta in s th e w o rd
p a r t m is, w h ic h m e a n s error o r incorrect, so th e b est a n s w e r is a n s w e r ( C ) . I n th e t h ir d q u es
t io n , th e w o r d d ictio n c o n ta in s th e w o r d p a rt die, w h ic h m e a n s speak, so th e b e s t a n s w e r is
answ er (D ).

T h e f o llo w in g c h a r t c o n ta in s a fe w w o rd p a rts th a t y o u w ill n e e d to k n o w to c o m p le te


th e e x e rc is e s in th is p a r t o f t h e tex t. A m o re c o m p le te list o f w o r d p a rts a n d e x e rcise s to
p r a c t ic e t h e m c a n b e f o u n d in A p p e n d ix I a t th e b a c k o f th e tex t.

Você também pode gostar